100% found this document useful (1 vote)
3K views409 pages

Casos Clínicos de Endocrinología

Uploaded by

Dra_Marady
Copyright
© © All Rights Reserved
We take content rights seriously. If you suspect this is your content, claim it here.
Available Formats
Download as PDF, TXT or read online on Scribd
100% found this document useful (1 vote)
3K views409 pages

Casos Clínicos de Endocrinología

Uploaded by

Dra_Marady
Copyright
© © All Rights Reserved
We take content rights seriously. If you suspect this is your content, claim it here.
Available Formats
Download as PDF, TXT or read online on Scribd
You are on page 1/ 409

PEDIATRIC

ESAP 2021-2022
PEDIATRIC ENDOCRINE SELF- ASSESSMENT PROGRAM
QUESTIONS, ANSWERS, DISCUSSIONS

2 «'
*
ENDOCRINE
Reference Edition SOCIETY
REFERENCE EDITION

Pediatric ESAP™ 2021-2022


Endocrine Society’s
Pediatric Endocrine Self-Assessment Program
Questions, Answers, and Discussions

Liuska M. Pesce, MD, Program Chair (2020-present)


Clinical Associate Professor
Pediatric Thyroid Clinic Director
Stead Family Children’s Hospital
Division of Pediatric Endocrinology and Diabetes
University of Iowa Carver College of Medicine
Paola A. Palma Sisto, MD, Program Chair (2017-2021)
Associate Professor
Medical College of Wisconsin
Children’s Hospital of Wisconsin

Li Chan, MD Reema L. Habiby, MD Christine M. Trapp, MD


Reader in Molecular Associate Professor of Pediatric Endocrinology,
Endocrinology and Pediatrics Connecticut
Metabolism Northwestern University Children’s Medical Center
Honorary Consultant in Feinberg School of Medicine Associate Fellowship Director,
Paediatric Endocrinology Ann & Robert H. Lurie Pediatric Endocrinology
Centre for Endocrinology Children’s Assistant Professor of
William Harvey Research Hospital of Chicago Pediatrics,
Institute University of Connecticut
Ryan S. Miller, MD School of Medicine
Cem S. Demirci, MD Assistant Professor
Pediatric Endocrinology University of Maryland Halley Wasserman, MD
Director, T1DM Program School of Medicine Assistant Professor of
Connecticut Children’s Pediatrics
Medical Center Ron Newfield, MD Cincinnati Children’s Hospital
Assistant Professor of Clinical Professor Medical Center
Pediatrics University of California – San University of Cincinnati
Chase Family Chair in Diego College of Medicine
Juvenile Diabetes Rady Children’s Hospital San
University of Connecticut Diego Ari Wassner, MD
School of Medicine Medical Director, Thyroid
Sripriya Raman, MD Center
Oscar Escobar, MD Pediatric Endocrinologist Director, Endocrinology
Associate Professor K S Pediatrics Fellowship Training Program
University of Pittsburgh Assistant Professor of
School of Medicine Pediatrics
UPMC Children’s Hospital of Boston Children’s Hospital
Pittsburg Harvard Medical School

Abbie L. Young, MS, CGC, ELS(D)


Medical Editor

Endocrine Society
2055 L Street NW, Suite 600, Washington, DC
20036
1-888-ENDOCRINE • www.endocrine.org
The Endocrine Society is the world’s largest, oldest, and most active organization
working to advance the clinical practice of endocrinology and hormone research.
Founded in 1916, the Society now has more than 18,000 global members across a range
of disciplines. The Society has earned an international reputation for excellence in the
quality of its peer-reviewed journals, educational resources, meetings, and programs that
improve public health through the practice and science of endocrinology.

Visit us at: Other Publications:


education.endocrine.org endocrine.org/publications
endocrine.org

For updates to the professional development books between editions:


https://www.endocrine.org/bookupdates

The statements and opinions expressed in this publication are those of the individual
authors and do not necessarily reflect the views of the Endocrine Society. The Endocrine
Society is not responsible or liable in any way for the currency of the information, for any
errors, omissions, or inaccuracies, or for any consequences arising therefrom. With
respect to any drugs mentioned, the reader is advised to refer to the appropriate medical
literature and the product information currently provided by the manufacturer to verify
appropriate dosage, method and duration of administration, and other relevant
information. In all instances, it is the responsibility of the treating physician or other
health care professional, relying on independent experience and expertise, as well as
knowledge of the patient, to determine the best treatment for the patient.

PERMISSIONS: For permission to reuse material, please access


http://www.copyright.com or contact the Copyright Clearance Center, Inc. (CCC), 222
Rosewood Drive, Danvers, MA 01923, 978-750-8400. CCC is a not-for-profit organization
that provides licenses and registration for a variety of uses. For more information,
individual or institutional purchases, please contact Member Services by telephone at
202-971-3646 or 888-363-6762; e-mail, [email protected]; or visit the online store
at www.endocrine.org/store.

Copyright © 2021 by the Endocrine Society, 2055 L Street NW, Suite 600, Washington,
DC 20036. All rights reserved. No part of this publication may be reproduced, stored in a
retrieval system, posted on the Internet, or transmitted in any form, by any means,
electronic, mechanical, photocopying, recording, or otherwise, without written permission
of the publisher.

TRANSLATIONS AND LICENSING: Rights to translate and reproduce Endocrine Society


publications internationally are extended through a licensing agreement on full or partial
editions. To request rights for a local edition, please visit endocrine.org/publications or e-
mail [email protected].

ISBN: 978-1-879225-97-8
eISBN: 978-1-879225-98-5

Library of Congress Control Number: 2020947893

On the Cover: @ Freepik. Pediatrician writing prescription (by annastills).


OVERVIEW
The Pediatric Endocrine Self-Assessment Program (Pediatric
ESAP™) is a self-study curriculum specifically designed for
endocrinologists seeking a self-assessment and a broad review of
pediatric endocrinology. Pediatric ESAP consists of approximately
100 multiple-choice questions in all areas of pediatric endocrinology,
diabetes, growth, and metabolism. There is extensive discussion of
each correct answer and references.
The Pediatric ESAP reference book is intended primarily for
consultation and self-assessment of knowledge relating to
endocrinology. As a reference book, educational credits are not
available upon completion of the multiple-choice questions included.
For information on educational products that include educational
credit, please visit endocrine.org/store.

LEARNING OBJECTIVES
Pediatric ESAP 2021-2022 will allow learners to assess their
knowledge of all aspects of pediatric endocrinology. Upon
completion of this educational activity, participants will be able to:
Recognize clinical manifestations of pediatric endocrine,
growth, and metabolic disorders and select among current
options for diagnosis, management, and therapy.
Identify risk factors for endocrine and metabolic disorders
in pediatric patients and develop strategies for prevention.
Evaluate pediatric endocrine and metabolic manifestations
of systemic disorders.
Use current, evidence-based clinical guidelines and
treatment recommendations to guide diagnosis and
treatment of pediatric endocrine and metabolic disorders.

TARGET AUDIENCE
Pediatric ESAP is a self-study curriculum aimed at physicians
seeking certification or recertification in pediatric endocrinology,
program directors interested in a testing and training instrument, and
clinicians seeking a self-assessment and broad review of pediatric
endocrinology.

STATEMENT OF INDEPENDENCE
The Endocrine Society has a policy of ensuring that the content and
quality of this educational activity are balanced, independent,
objective, and scientifically rigorous. The scientific content of this
activity was developed under the supervision of the Endocrine
Society’s Pediatric ESAP Faculty.

DISCLOSURE POLICY
The faculty, committee members, and staff who are in position to
control the content of this activity are required to disclose to the
Endocrine Society and to learners any relevant financial
relationship(s) of the individual or spouse/partner that have occurred
within the last 12 months with any commercial interest(s) whose
products or services are related to the CME content. Financial
relationships are defined by remuneration in any amount from the
commercial interest(s) in the form of grants; research support;
consulting fees; salary; ownership interest (eg, stocks, stock options,
or ownership interest excluding diversified mutual funds); honoraria
or other payments for participation in speakers’ bureaus, advisory
boards, or boards of directors; or other financial benefits. The intent
of this disclosure is not to prevent CME planners with relevant
financial relationships from planning or delivering content, but rather
to provide learners with information that allows them to make their
own judgments of whether these financial relationships may have
influenced the educational activity with regard to exposition or
conclusion.
The Endocrine Society has reviewed all disclosures and resolved
or managed all identified conflicts of interest, as applicable.
The following faculty reported relevant financial relationship(s):
Ryan S. Miller, MD, is an NIH grantee. Ron Newfield, MD, is a
principal investigator for Spruce and Neurocrine for new medications;
a consultant for Spruce; a consultant and member of the
Independent Safety Committee for Ascendis; a principal investigator
for Merck; and a principal investigator for Zealand for dasiglucagon.
Ari Wassner, MD, is a topic peer reviewer for UpToDate (thyroid).
Liuska M. Pesce, MD, has a spouse who is a consultant for VIDAS
Diagnostics and GlaxoSmithKline.
The following faculty reported no relevant financial relationships:
Paola A. Palma Sisto, MD; Cem S. Demirci, MD; Oscar Escobar,
MD; Reema L. Habiby, MD; Sripriya Raman, MD; Christine M.
Trapp, MD; Halley Wasserman, MD; Li Chan, MD.
The medical editor for this program, Abbie L. Young, MS, CGC,
ELS(D), reported no relevant financial relationships.
The Endocrine Society staff associated with the development of
this program reported no relevant financial relationships.

DISCLAIMERS
The information presented in this activity represents the opinion of
the faculty and is not necessarily the official position of the Endocrine
Society.

USE OF PROFESSIONAL JUDGMENT:


The educational content in this self-assessment test relates to basic
principles of diagnosis and therapy and does not substitute for
individual patient assessment based on the health care provider’s
examination of the patient and consideration of laboratory data and
other factors unique to the patient. Standards in medicine change as
new data become available.

DRUGS AND DOSAGES:


When prescribing medications, the physician is advised to check the
product information sheet accompanying each drug to verify
conditions of use and to identify any changes in drug dosage
schedule or contraindications.

POLICY ON UNLABELED/OFF-LABEL USE


The Endocrine Society has determined that disclosure of
unlabeled/off-label or investigational use of commercial product(s) is
informative for audiences and therefore requires this information to
be disclosed to the learners at the beginning of the presentation.
Uses of specific therapeutic agents, devices, and other products
discussed in this educational activity may not be the same as those
indicated in product labeling approved by the Food and Drug
Administration (FDA). The Endocrine Society requires that any
discussions of such “off-label” use be based on scientific research
that conforms to generally accepted standards of experimental
design, data collection, and data analysis. Before recommending or
prescribing any therapeutic agent or device, learners should review
the complete prescribing information, including indications,
contraindications, warnings, precautions, and adverse events.

ACKNOWLEDGMENT OF COMMERCIAL SUPPORT


This activity is not supported by educational grant(s) or other funds
from any commercial supporter.

PUBLICATION DATE: February 2021


Common Abbreviations Used in Pediatric ESAP

ACTH --------------------------------------------------- corticotropin


ACE inhibitor---------------angiotensin-converting enzyme inhibitor
ALT ----------------------------------------alanine aminotransferase
AST ------------------------------------ aspartate aminotransferase
BMI -------------------------------------------------body mass index
CNS ------------------------------------------ central nervous system
CT -------------------------------------------- computed tomography
DHEA ---------------------------------------dehydroepiandrosterone
DHEA-S -----------------------------dehydroepiandrosterone sulfate
DNA ------------------------------------------- deoxyribonucleic acid
DPP-4 inhibitor --------------------- dipeptidyl-peptidase 4 inhibitor
DXA ------------------------------ dual-energy x-ray absorptiometry
FDA ---------------------------------- Food and Drug Administration
FGF-23 ---------------------------------- fibroblast growth factor 23
FNA ------------------------------------------- fine-needle aspiration
FSH ------------------------------------- follicle-stimulating hormone
GH -------------------------------------------------- growth hormone
GHRH -------------------------- growth hormone–releasing hormone
GLP-1 receptor agonist -----glucagonlike peptide 1 receptor agonist
GnRH ----------------------------- gonadotropin-releasing hormone
hCG --------------------------------- human chorionic gonadotropin
HDL ----------------------------------------- high-density lipoprotein
HIV-----------------------------------human immunodeficiency virus
HMG-CoA reductase inhibitor ----------------------------------------
3-hydroxy-3-methylglutaryl coenzyme A reductase inhibitor
IGF-1-------------------------------------- insulinlike growth factor 1
LDL ------------------------------------------ low-density lipoprotein
LH -----------------------------------------------luteinizing hormone
MCV -------------------------------------- mean corpuscular volume
MIBG ------------------------------------- meta-iodobenzylguanidine
MRI ------------------------------------ magnetic resonance imaging
NPH insulin --------------------- neutral protamine Hagedorn insulin
PCSK9 inhibitor ----proprotein convertase subtilisin/kexin 9 inhibitor
PET ---------------------------------- positron emission tomography
PSA ---------------------------------------- prostate-specific antigen
PTH -------------------------------------------- parathyroid hormone
PTHrP------------------------- parathyroid hormone–related protein
SGLT-2 inhibitor ---------- sodium-glucose cotransporter 2 inhibitor
SHBG ---------------------------------sex hormone–binding globulin
T3 ---------------------------------------------------- triiodothyronine
T4 ----------------------------------------------------------- thyroxine
TPO antibodies --------------------------thyroperoxidase antibodies
TRH ---------------------------------- thyrotropin-releasing hormone
TRAb ---------------------------------------TSH-receptor antibodies
TSH ------------------------------------------------------- thyrotropin
VLDL ------------------------------------ very low-density lipoprotein
Contents

Copyright

Pediatric Endocrine Self-Assessment Program 2021-2022: Part I


Pediatric Endocrine Self-Assessment Program 2021-2022: Part II

Pediatric Endocrine Self-Assessment Program 2021-2022: Part III


QUESTIONS
PEDIATRIC ENDOCRINE SELF-
ASSESSMENT PROGRAM 2021-2022
Part I
1 A 4-and-7/12-year-old boy born to Scottish parents presents to
the emergency department with recurrent bouts of vomiting and
diarrhea over the past year. While in the emergency department, he
has a seizure and is noted to be hypoglycemic (blood glucose = 14.4
mg/dL [0.8 mmol/L]). He is given a glucose and fluid bolus and is
maintained on an intravenous dextrose and saline solution.
He was born at term and was treated for neonatal jaundice. His
parents describe him as being a sickly child who “catches
everything,” but because he has always grown well, they have not
been concerned. He has no skin problems or infections.
On physical examination, the child is hyperpigmented. His
weight is at the 50th percentile, and height is greater than the 90th
percentile. His midparental range is between the 25th and 50th
percentile. Bone age is advanced to 8.2 years. Blood pressure is
normal, and he is clinically prepubertal.
Blood tests confirm that serum cortisol is undetectable and
plasma ACTH is high.

Laboratory test results:


Random serum cortisol = <1.8 µg/dL (4.3-9.4 µg/dL) (SI: <50 nmol/L [120-620
nmol/L])
Plasma ACTH = 4310 pg/mL (6-48 pg/mL) (SI: 948.2 pmol/L [1.3-10.6 pmol/L])
Sodium = 140 mEq/L (136-144 mEq/L) (SI: 140 mmol/L [136-144 mmol/L])
Potassium = 4.8 mEq/L (3.2-5.2 mEq/L) (SI: 4.8 mmol/L [3.2-5.2 mmol/L])
Plasma renin activity = 3.5 ng/mL per h (0.6-3.8 ng/mL per h)
Aldosterone = 6.7 ng/dL (3.6-16.2 ng/dL) (SI: 185 pmol/L [100-450 pmol/L])
17-Hydroxyprogesterone = 33.0 ng/dL (<92.4 nmol/L) (SI: 1.0 nmol/L [<2.8
nmol/L])
Androstenedione = <10.0 ng/dL (10.0-16.9 ng/dL) (SI: <0.35 nmol/L [0.35-0.59
nmol/L])
Calcium = 9.2 mg/dL (8.4-10.0 mg/dL) (SI: 2.3 mmol/L [2.1-2.5 mmol/L])
TSH = 4.0 mIU/L (0.5-4.8 mIU/L)
Free T4 = 1.1 ng/dL (0.9-1.6 ng/dL) (SI: 13.8 pmol/L [12.0-20.6 pmol/L])
Adrenal antibodies, negative
Very long-chain fatty acids, normal

Which of the following is the most likely diagnosis?


A. Addison disease
B. Familial glucocorticoid deficiency
C. Congenital adrenal hypoplasia
D. Congenital adrenal hyperplasia
E. X-linked adrenoleukodystrophy
2 A 2-year-old boy presents to the emergency department with left
eye esotropia 1 week after a fall from standing height. MRI shows
dilated optic nerve sheaths bilaterally, optic nerve tortuosity, and
mild flattening of the posterior globes consistent with clinical
papilledema. He has an unusual head shape. A 3-dimensional
reconstruction CT shows complete fusion of the sagittal suture with
associated dolichocephaly.
On physical examination, he has short stature (height <5th
percentile), mild genu varum deformity, and a waddling gait. His
hands and feet are normal. He has no dental abnormalities. His
medical history is otherwise unremarkable. He was breastfed
without vitamin D supplementation until 1 year of life. He started
walking at age 15 months, and his parents have noticed the
waddling gait since that time. X-rays are consistent with rachitic
changes at the knees.

These physical examination characteristics are most


concerning for which of the following underlying skeletal
disorders?
A. X-linked hypophosphatemic rickets
B. Hypophosphatasia
C. Hypochondroplasia
D. Severe vitamin D deficiency
E. Apert syndrome
3 A 13-year-old girl with type 1 diabetes mellitus treated with
multiple daily insulin injections uses a continuous glucose monitor
efficiently for her daily glycemic management. She plans to eat dry
cereal for her afternoon snack when she comes home from school.
The food label for this cereal is displayed (see image). She carefully
measures 2 servings of one-third cup (total two-thirds cup) in a
bowl. Her continuous glucose monitor reads 92 mg/dL (5.1
mmol/L) with a flat arrow. Her insulin-to-carbohydrate ratio in the
afternoon is 1:6 (1 unit for every 6 g carbohydrate).
How many units (rounded to the nearest whole unit) of rapid-
acting insulin she should inject before consuming this snack?
A. 10 units
B. 9 units
C. 8 units
D. 7 units
E. She should eat her snack and monitor her glucose via
continuous glucose monitoring to decide
4 A 12-year-old girl with primary hypothyroidism secondary to
chronic lymphocytic thyroiditis (negative thyroglobulin antibodies)
is seen for evaluation of thyroid nodules. Neck ultrasonography
documents a solid right nodule measuring 1.8 × 1 × 1 cm and a less
well-defined, smaller nodule on the left lobe measuring 1.2 × 0.8 ×
0.8 cm (see images).

Ultrasound-guided FNA reveals a benign nodule on the right lobe


and atypia of unclear significance in the left lobe. Options for
treatment are discussed, and the family decides to proceed with total
thyroidectomy. Pathologic examination documents a right follicular
adenoma and a 0.9-cm left lobe papillary thyroid carcinoma, classic
type, confined to the thyroid without lymphovascular invasion.
Lymph nodes are negative for tumor (3 on the left, 4 on the right,
and 13 in the left central neck). Staging is assigned as pT1a pN0
(where p stands for pathological examination of surgical specimen,
T1a refers to tumor ≤1 cm limited to the thyroid, and N0 refers to
nonregional lymph node metastasis).
Two weeks after surgery, the family comes for a follow-up visit to
discuss further management.

Which of the following is the best management plan?


A. Perform 123I scan and measure stimulated thyroglobulin
following levothyroxine withdrawal and low-iodine diet
B. Optimize levothyroxine treatment to maintain TSH between
0.5 and 1.0 mIU/L and follow-up with thyroglobulin
measurement (while on levothyroxine) 12 weeks after surgery
C. Treat with 131I following levothyroxine withdrawal and low-
iodine diet and perform posttreatment scan 4 to 7 days after
treatment
D. Optimize levothyroxine treatment to maintain TSH between
0.1 and 0.5 mIU/L and measure thyroglobulin (while on
levothyroxine) 6 weeks after surgery
E. Optimize levothyroxine to maintain TSH between 0.5 and 4.5
mIU/L and measure thyroglobulin (while on levothyroxine) 12
weeks after surgery
5 A 5-week-old male newborn is referred by his pediatrician for
evaluation of gynecomastia. He was born at 40 weeks’ gestation,
birth weight was 7 lb 12 oz (3520 g), and there were no complications
during pregnancy or delivery. His parents report that they first
noticed breast tissue at about 1 week of age. The tissue has been
increasing in size and getting firmer, and they have noticed milky
discharge on 2 occasions. His diet is primarily breast milk with some
formula supplementation. His parents state that he has had no
known contact with hormone preparations, and they use a standard
baby wash for baths.
On physical examination, his length is at the 60th percentile and
weight is at the 90th percentile. He appears well, and examination
findings are normal except for very firm, mobile breast tissue
bilaterally, measuring 7 to 8 cm. There are no midline defects, and
there is no discharge from the nipples.

Laboratory test results:


β-hCG = <1 mIU/mL (0-3 mIU/mL) (SI: <1 IU/L [0-3 IU/L])
TSH = 6.38 mIU/L (0.72-11.00 mIU/L)
Free T4 = 1.46 ng/dL (0.48-2.34 ng/dL) (SI: 18.8 pmol/L [6.2-30.1 pmol/L])
Prolactin = 108.5 ng/mL (≤10 ng/mL) (SI: 4.7 nmol/L [≤0.4 nmol/L])
Karyotype = 46,XY

Which of the following is the best next step in this patient’s


management?
A. Perform MRI of the brain and pituitary gland
B. Measure LH, FSH, and testosterone
C. Measure prolactin again in 2 to 4 weeks
D. Start dopamine agonist therapy
E. Order genetic testing of the MEN1 gene
6 A 17-year-old girl is followed in endocrinology clinic for thyroid
hormone and GH replacement. Medulloblastoma was diagnosed at
age 9 years. She had total tumor resection and received craniospinal
radiation therapy (2340 cGy to the craniospinal axis plus a boost of
1260 cGy to the posterior fossa and 1980 cGy to the tumor bed)
followed by 8 months of chemotherapy with cisplatin,
cyclophosphamide, and lomustine. At her initial endocrine visit at
age 11 years, her parents reported that she had not changed shoe size
in 2 to 3 years. Subsequently, TSH and GH deficiencies were
diagnosed, and she was prescribed appropriate replacement therapy.
Her linear growth improved. At age 13 years, primary ovarian
insufficiency was diagnosed, and hormone replacement therapy was
initiated. GH therapy was stopped at age 16 years when she
underwent menarche. At that time, her growth velocity was less
than 2 cm/y. An IGF-1 measurement 2 months after stopping GH
therapy was normal. She continues to take levothyroxine and an oral
contraceptive pill with very good adherence.

Laboratory test results (sample drawn 2 weeks ago):


Free T4 = 1.4 ng/dL (0.8-1.8 ng/dL) (SI: 18.0 pmol/L [10.3-23.2 pmol/L])
IGF-1 = 310 ng/mL (208-619 ng/mL) (SI: 40.6 nmol/L [27.2-81.1 nmol/L])
Cortisol (8 AM) = 17 µg/dL (SI: 469.0 nmol/L)
Hemoglobin A1c = 5.1% (4.0%-5.6%) (32 mmol/mol [20-38 mmol/mol])
Glucose = 91 mg/dL (70-100 mg/dL) (SI: 5.1 mmol/L [3.9-5.6 mmol/L])

At today’s clinic visit, the patient describes tiredness and weakness


with day-to-day activities. She participates in ballet and jazz 2 hours
per week, but she is considering dropping these classes, as her
energy level is low. She does well in school. Menses are regular. She
drinks about 2 L of water every day and wakes up 1 to 2 times each
night to urinate. She has no other symptoms. Her height and weight
charts are shown (see images).
On physical examination, her vital signs and examination findings
are normal.

Which of the following is the best next step in this patient’s


management?
A. Measure first-morning urine osmolality, serum osmolality, and
serum sodium levels to screen for diabetes insipidus
B. Perform a low-dose cosyntropin-stimulation test to screen for
central adrenal insufficiency
C. Perform an arginine/glucagon GH-stimulation test; if
abnormal, restart GH therapy at a dosage of 2.5 mg daily and
titrate based on IGF-1 levels
D. Perform a glucagon-stimulation test; if abnormal, start GH
therapy at a dosage of 1.0 mg daily and titrate based on IGF-1
levels
E. Reassure the patient and family that there is no need for
further workup since her laboratory test results are normal
7 An 11-month-old boy presents with failure to thrive. The child
was born full term at home after an uncomplicated pregnancy. He
was breastfed exclusively until age 4 months at which time cereal
and fruit juice were introduced. He continues to breastfeed every 2
to 3 hours during the day and eats cereal (3 tablespoons) and fruit
juice (4 oz) twice daily. Family history is unremarkable. He is the
first child for this family. Both parents are tall and the midparental
height is at the 90th percentile.
On physical examination, his length and weight are at the 3rd
percentile. His pulse rate is 100 beats/min, and respiratory rate is 38
breaths/min. Examination findings are remarkable for a prominent
abdomen and hepatosplenomegaly.

Laboratory test results:


Plasma glucose = 45 mg/dL (65-109 mg/dL) (SI: 2.5 mmol/L [3.61-6.05 mmol/L])
Lactate = 18.0 mg/dL (2-27 mg/dL) (SI: 2.0 mmol/L [0.22-2.98 mmol/L])
Triglycerides = 1000 mg/dL (30-104 mg/dL) (SI: 11.3 mmol/L [0.34-1.18 mmol/L])
ALT = 70 U/L (10-40 U/L) (SI: 1.17 µkat/L [0.17-0.67µkat/L])
AST = 80 U/L (10-40 U/L) (SI: 1.34 µkat/L [0.17-0.67 µkat/L])
Creatine kinase = 350 U/L (10-90 U/L) (SI: 5.85 µkat/L [0.17-1.5 µkat/L])

Which of the following is the most likely explanation for this


child’s presentation?
A. Glycogen debrancher deficiency
B. Phosphofructokinase deficiency
C. Glucose-6-phosphatase deficiency
D. Liver phosphorylase deficiency
E. Glucose transporter 2 deficiency
8 An 8-year-old girl presents for evaluation of poor growth. She
was born at 39 weeks’ gestation with a birth weight of 6 lb 1 oz (2750
g) after an uncomplicated pregnancy. Growth charts from her
pediatrician show that her length drifted to between the 5th and the
10th percentiles by 1 year of age and further decreased to below the
3rd percentile thereafter. Her weight has remained between the 3rd
and 10th percentiles with minimal fluctuations. She has been
otherwise healthy.
Her parents are first cousins. Her father’s height is 68 in (172.2
cm) (–0.58 SDS), and mother’s height is 60 in (153 cm) (–1.59 SDS).
Her 5-year-old brother also comes for evaluation because of similar
linear growth deceleration noted since age 3 years.
On physical examination, she has no evidence of major
dysmorphic features. Her height is 45.6 in (116 cm) (–2.08 SDS), and
weight is 44 lb (20 kg) (–1.67 SDS). She is prepubertal. Her arm span
is 46 in (117 cm). Lower extremities show no bowing.

Laboratory test results:


IGF-1 = 825 ng/mL (112-276 ng/mL) (SI: 108.1 nmol/L [14.7-36.2 nmol/L])
IGF-2 = 758 ng/mL (334-642 ng/mL) (SI: 758 µg/L [334-642 µg/L])
IGFBP-3 = 5.9 mg/L (2.1-4.2 mg/L)
GH-binding protein = 638 pmol/L (267-1638 pmol/L)
GH (obtained in the morning after 12-hour fast) = 18.0 ng/mL (0.7-6.0 ng/mL) (SI:
18.0 µg/L [0.7-6.0 µg/L])
Thyroid function, normal

Bone age is interpreted to be 7 years, 10 months by the method of


Greulich and Pyle.

Which of the following pathogenic variants most likely explains


this child’s growth failure and short stature?
A. Loss-of-function pathogenic variant in the GHR gene (growth
hormone receptor)
B. Gain-of-function pathogenic variant in the GHR gene (growth
hormone receptor)
C. Loss-of-function pathogenic variant in the IGF1R gene (IGF-1
receptor)
D. Loss-of-function pathogenic variant in the STAT5B gene
(signal transducer and activator of transcription 5B)
E. Loss-of-function pathogenic variant in the PAPPA2 gene
(pappalysin 2)
9 An 8-day-old male newborn is referred because of an abnormal
newborn screen for congenital hypothyroidism. His weight is 7 lb 1
oz (3200 g). His mother has no known thyroid disease. On physical
examination, he is not jaundiced. His examination findings are
normal. Newborn screening (sample drawn at 20 hours of life)
documents a TSH value of 222.7 mIU/L (<29 mIU/L).

Results of confirmatory testing:


Day of life 8 (different
Measurement Day of life 4 assay for TSH)
TSH >100 mIU/L 42.1 mIU/L
(reference range: 0.34-5.60 mIU/L)
Free T4 2.1 ng/dL (SI: 27.0 …
(reference range: 0.6-1.6 ng/dL [SI: 7.7- pmol/L)
20.6 pmol/L])
Total T4 … 12.6 µg/dL (SI: 162.2
(reference range, 4.3-12.5 µg/dL [SI: nmol/L)
55.3-160.9 nmol/L])
T3 uptake … 24%
(reference range: 25%-35%)

Technetium nuclear scanning on day of life 10 shows a normal-


appearing, eutopic thyroid gland.

Which of the following is the best next step in this patient’s


management?
A. Measure TSH and T4 in 1 week (off levothyroxine) and
measure the mother’s TSH and T4
B. Measure calcium and PTH
C. Start levothyroxine
D. Assess the mother’s thyroid antibodies
E. Order thyroid ultrasonography
10 A 15-year-old boy has been admitted to the hospital with tonic
clonic seizure, which was self-limiting (lasted 5 minutes). Afterward,
he was sweaty and confused. There were no obvious triggers for the
seizure.
He has a 2- to 3-year history of monthly bitemporal headaches,
lasting approximately 1 hour, which are eased with acetominophen.
He takes no prescription medications or illicit drugs. Over the past 6
weeks, his headaches have been much more severe and have been
associated with palpitations and significant sweating.
He was born at 30 weeks’ gestation and spent a short period in
neonatal intensive care unit without long-term health problems. His
vaccinations are up-to-date, and all developmental milestones are
normal. His paternal grandfather has hypertension, which was
diagnosed in his 50s. No other relevant family history is noted.
Findings on cardiovascular, respiratory, abdominal, and
cutaneous examination are normal. Findings on thyroid examination
are unremarkable and no goiter is palpable. His height is at the 50th
percentile, and weight is at the 90th percentile.
On the day of admission, he has 2 more seizures. The first lasts 90
seconds (self-terminating) and the second requires intravenous
lorazepam, which is given after 4 minutes. He is then intubated,
ventilated, and transferred to the intensive care unit. Blood pressure
is 165/84 mm Hg, and pulse rate is 105 beats/min.

Laboratory test results:


Sodium = 134 mEq/L (133-146 mEq/L) (SI: 134 mmol/L [133-146 mmol/L])
Potassium = 3.4 mEq/L (3.9-5.3 mEq/L) (SI: 3.4 mmol/L [3.9-5.3 mmol/L])
Serum urea nitrogen = 17.9 mg/dL (7.0-21.8 mg/dL) (SI: 6.4 mmol/L [2.5-7.8
mmol/L])
Creatinine = 0.9 mg/dL (0.5-1.0 mg/dL) (SI: 80 mmol/L [45-84 mmol/L])
Glucose = 139 mg/dL (70-99 mg/dL) (SI: 7.7 mmol/L [3.9-5.5 mmol/L])
TSH = 4.5 mIU/L (0.5-4.8 mIU/L)
Free T4 = 1.1 ng/dL (0.9-1.6 ng/dL) (SI: 14.0 pmol/L [12.0-20.6 pmol/L])

Which of the following would be the most useful diagnostic


investigation now?
A. Echocardiography
B. Head MRI
C. Measurement of urinary or plasma metanephrines
D. MIBG scan
E. Abdominal CT
11 A 14-year-old girl presents with menstrual irregularity and
occasional galactorrhea. Her parents report that she has always been
tall for her age. They note that between the ages of 11 and 11.5 years,
she grew 4 in (10.2 cm) to a height of 67 in (170.2 cm). She has no
headaches. Her midparental target height is 63 in (160 cm). Her
family history is noncontributory.
On physical examination, her blood pressure is 107/70 mm Hg
and pulse rate is 87 beats/min. Her height is 70.9 in (180 cm) (99.8th
percentile), weight is 162.4 lb (73.8 kg) (95.5th percentile), and BMI is
22.7 kg/m2. She has mild coarsening of her facial features. Her
breasts are Tanner stage 5 and there is easily expressible
galactorrhea. Findings on neurologic examination are normal.
Laboratory test results (8 AM):
Prolactin = 127 ng/mL (2-10 ng/mL) (SI: 5.5 nmol/L [0.1-0.4 nmol/L])
LH = 2.4 mIU/mL (0.02-12.0 mIU/mL) (SI: 2.4 IU/L [0.02-12.0 IU/L])
FSH = 3.5 mIU/mL (1.8-11.2 mIU/mL) (SI: 3.5 IU/L [1.8-11.2 IU/L])
Estradiol = 16 pg/mL (34-170 pg/mL) (SI: 58.7 pmol/L [124.8-624.1 pmol/L])
Free T4 = 1.1 ng/dL (0.98-1.63 ng/dL) (SI: 14.2 pmol/L [12.6-21.0 pmol/L])
TSH = 2.03 mIU/L (0.5-5.0 mIU/L)
Cortisol = 10.9 µg/dL (8.0-19.0 µg/dL) (SI: 300.7 nmol/L [220.7-524.2 nmol/L])
IGF-1 = 880 ng/mL (220-574 ng/mL) (SI: 115.3 nmol/L [28.8-75.2 nmol/L])
GH = 15.2 ng/mL (0.06-4.30 ng/mL) (SI: 15.2 µg/L [0.06-4.30 µg/L])

GH levels failed to suppress on oral glucose tolerance testing. MRI


reveals a 1.2 × 1.2-cm lobulated, peripherally enhancing cystic
sellar/suprasellar lesion causing mild mass effect and elevation of
the optic chiasm. She undergoes transsphenoidal resection of the
mass. Her cortisol and thyroid levels measured 1 month
postoperatively are normal, but oral glucose tolerance testing
following surgery shows continued failure of GH levels to suppress.
She is subsequently treated with subcutaneous octreotide. After 3
months of therapy, laboratory tests are performed (8 AM):
Free T4 = 0.97 ng/dL (0.98-1.63 ng/dL) (SI: 12.5 pmol/L [12.6-21.0 pmol/L])
TSH = 0.27 mIU/L (0.5-5.0 mIU/L)
Prolactin = 10 ng/mL (2-10 ng/mL) (SI: 0.43 nmol/L [0.09-0.43 nmol/L])
GH = 1.17 ng/mL (0.06-4.30 ng/mL) (SI: 1.17 µg/L [0.06-4.30 µg/L])
IGF-1 = 266 ng/mL (220-574 ng/mL) (SI: 34.8 nmol/L [28.8-75.2 nmol/L])

Which of the following is the most likely etiology of her


abnormal thyroid function tests?
A. Destruction of thyrotropes by the tumor
B. Effect of surgical resection
C. Effect of the somatostatin analogue
D. Late effects of hyperprolactinemia
E. GH excess
12 A 3-and-6/12-year-old boy is referred to endocrine clinic for
evaluation of short stature. He was born at 40 weeks’ gestation, with
a birth weight of 6 lb 14 oz (3120 g). He is developmentally
appropriate for his age. His height is at less than the 3rd percentile,
and weight is at the 10th percentile. His midparental target height is
at the 75th percentile.
On physical examination, you notice that he is short and has a
protruding abdomen. He is initially happy and cooperative, but he
quickly becomes irritable and agitated, making it hard for you to
finish the examination.

A fingerstick blood glucose measurement is 54 mg/dL (3.0


mmol/L). You send him to the lab right away, and the following
results are documented:
Plasma glucose = 52 mg/dL (70-120 mg/dL) (SI: 2.9 mmol/L [3.9-6.7 mmol/L])
Bicarbonate = 12 mEq/L (22-26 mEq/L) (SI: 12 mmol/L [22-26 mmol/L])
Lactate = 32.4 mg/dL (9-22 mg/dL) (SI: 3.6 mmol/L [0.9-2.4 mmol/L])
β-Hydroxybutyrate = 1.1 mg/dL (SI: 108 µmol/L)
Insulin, undetectable
GH = 1.8 ng/mL (SI: 1.8 µg/L)
Cortisol = 8.9 μg/dL (SI: 245.5 nmol/L)

Which of the following is the best next step in this patient’s


evaluation?
A. Measure uric acid, order a lipid panel, and refer to a metabolic
nutritionist
B. Perform cosyntropin-stimulation testing
C. Perform brain MRI and additional laboratory tests to screen
for hypopituitarism
D. Discuss the importance of frequent feeds and follow-up in 6
months
E. Perform GH-stimulation testing
13 A 7-year-old girl is referred for evaluation of low BMI. Her
medical history is unremarkable and she is asymptomatic.
On physical examination, her height is at the 75th percentile,
weight is at the 25th percentile, and BMI is 13.15 kg/m2 (Z-score, –
2.38). Her blood pressure is 98/61 mm Hg, and pulse rate is 93
beats/min. Examination findings are normal.
Her father has a history of bilateral pheochromocytoma
diagnosed at age 41 years, and he was found to have a pathogenic
variant in the RET proto-oncogene (C634R). You recommend RET
genetic testing and the patient is found to have the same pathogenic
variant.

Which of the following is characteristic of this patient’s


condition?
A. High risk for hyperparathyroidism; screening is recommended
now
B. High risk for pheochromocytoma; screening is recommended
now
C. History of Hirschsprung disease; screening for cutaneous
lichen amyloidosis is recommended
D. High risk for aggressive medullary thyroid carcinoma; total
thyroidectomy is recommended now
E. Marfanoid habitus; high risk for metastatic medullary thyroid
carcinoma and ophthalmologic and skeletal manifestations;
screening is recommended now
14 An 8-and-3/12-year-old girl presents for evaluation of breast
development and vaginal bleeding. Her parents first noted breast
development just before age 8 years. She began developing axillary
hair and pubic hair at age 7 years. Her pediatrician is also concerned
about short stature. The patient had an episode of vaginal bleeding 1
month ago, which lasted 6 days. Three weeks later, she had another
episode of vaginal bleeding. She has no headaches or abdominal
pain. She has not had a change in shoe or clothing size in the past
year. Her energy level is normal. She does not take any medications
or use any creams or lotions. Her midparental target height is 62 in
(157.5 cm).
On physical examination, her blood pressure is 81/57 mm Hg
and pulse rate is 59 beats/min. Her height is 45.6 in (115.8 cm) (1st
percentile; Z-score, –2.37), weight is 54.1 lb (24.6 kg) (33rd percentile;
Z-score, –0.45), and BMI is 18.3 kg/m2 (83.8th percentile, Z-score,
0.99). She has some periorbital edema. Breasts are Tanner stage 2,
and pubic hair is Tanner stage 2. The labia minora are prominent.
Her visual fields are normal to confrontation. Her thyroid gland is
small to normal in size. Her skin is dry with hypertrichosis over her
back. Findings on abdominal examination are normal.
Bone age is 6 years.
Pelvic ultrasonography shows a postmenarchal uterus. The
endometrial stripe measures 0.7 cm. The right ovary is enlarged and
measures 6.1 × 3.9 × 5.5 cm with a volume of 67 mL. It contains
multiple enlarged cysts or follicles, the largest measuring up to 4 cm.
The left ovary is normal in size and measures 3.3 × 1.2 × 2.1 cm with
a volume of 7.1 mL. Like the right ovary, the left ovary contains
multiple enlarged cysts or follicles.

Which of the following most likely represents her laboratory


results?
Answer LH TSH hCG
A. ↑ Normal Normal
B. ↓ ↑↑ Normal
C. ↓ Normal Normal
D. ↓ ↑ ↑
E. ↓ ↓ Normal
15 A 3-year-old girl with a history of congenital sodium-losing
diarrhea (also known as trichohepatoenteric syndrome)
(homozygous SPINT2 pathogenic variants) with dependence on
parenteral nutrition for 80% of her caloric needs presents to the
emergency department after 1 month of increasing fatigue and
lethargy. She has had no vomiting, changes in stool pattern, or
fevers. Laboratory test results prior to the onset of these acute
symptoms have been normal. She has a history of hypothyroidism
secondary to iodine deficiency and has been off levothyroxine for 3
months following normalization of urine iodine levels with
potassium-iodine supplementation.
On physical examination in the emergency department, she is a
tired-appearing girl with dry, brittle hair and dry skin. She has
tachycardia, muscle spasms, and cramping. Her thyroid gland is
normal on examination.

Laboratory test results:


Potassium = 3.4 mEq/L (3.3-4.7 mEq/L) (SI: 3.4 mmol/L [3.3-4.7 mmol/L])
Carbon dioxide = 29 mEq/L (17-31 mEq/L) (SI: 29 mmol/L (17-31 mmol/L])
Creatinine = 0.32 mg/dL (0.17-0.42 mg/dL) (SI: 28.3 μmol/L [15.0-37.1 μmol/L])
Calcium = 7.0 mg/dL (8.5-10.1 mg/dL) (SI: 1.8 mmol/L [2.1-2.5 mmol/L])
Phosphate = 5.4 mg/dL (3.7-6.5 mg/dL) (SI: 1.7 mmol/L [1.2-2.1 mmol/L])
Magnesium = 0.8 mg/dL (1.7-2.4 mg/dL) (SI: 0.3 mmol/L [0.7-1.0 mmol/L])
Albumin = 3.1 g/dL (3.5-4.7 g/dL) (SI: 31 g/L [35-47 g/L])
PTH = 40 pg/mL (15-87 pg/mL) (SI: 40 ng/L [15-87 ng/L])

Urinary electrolytes show low excretion of calcium and magnesium.


In the emergency department, she is given infusions of calcium
chloride and magnesium sulfate. Her symptoms improve and her
condition remains stable with ongoing magnesium supplementation
only. Review of her medical record reveals no previous episodes of
hypocalcemia or hypomagnesemia.

Which of the following is the most likely cause of her


presentation?
A. Impaired magnesium absorption due to pathogenic variants in
the TRPM6 gene
B. Calcium deficiency secondary to hypoparathyroidism
C. Iodine deficiency causing hypothyroidism
D. Magnesium deficiency secondary to inadequate
supplementation
E. Gitelman syndrome
16 A 16-year-old African American girl with obesity (BMI = 31
kg/m2) presents for evaluation of oligomenorrhea and hirsutism.
She has a family history of type 2 diabetes mellitus in her father and
maternal grandfather and similar menstrual issues in a paternal
aunt.
On physical examination, you note central obesity, facial
hirsutism, acne, and marked acanthosis nigricans. Her fasting blood
glucose concentration is 101 mg/dL (70-110 mg/dL) (SI: 5.61
mmol/L [3.89-6.11 mmol/L]). Her blood pressure is 134/86 mm Hg.

An elevated concentration of which of the following laboratory


values would be the greatest predictor of cardiovascular
disease risk in this child?
A. Insulin
B. LDL cholesterol
C. Triglycerides
D. Apolipoprotein B
E. HDL cholesterol
17 A 15-year-old girl noticed a mass in her left neck 2 months ago.
Her primary care physician ordered thyroid function tests, as well as
thyroid ultrasonography. She is euthyroid. Thyroid ultrasonography
reveals 2 nodules in the left lobe and no cervical adenopathy. One
nodule is in the lower lobe and is 3.2 cm in largest dimension,
relatively round, and has a mixed solid and cystic composition. It
has normal edges, no increased blood flow, and no visible
microcalcifications. The second nodule is in the upper pole and
measures 1.1 cm. It is solid, hypoechoic, and has a taller-than-wide
shape. There is increased blood flow and a possible
microcalcification.
Findings on physical examination are unremarkable except for a
left lower lobe thyroid nodule that is not very hard. There is no
palpable cervical adenopathy.

Which of the following is the best next step in this patient’s


management?
A. Referral to a surgeon for a left hemithyroidectomy
B. Measurement of TPO antibodies and thyroglobulin antibodies
C. FNA biopsy of the 3.2-cm nodule
D. FNA biopsy of the 1.1-cm nodule
E. FNA biopsy of both nodules
18 You are asked to evaluate a 17-year-old girl for secondary
amenorrhea. She had onset of menarche at age 13 years and is
uncertain whether her periods have ever been regular. She reports
having had no menses for the past 7 months. She states that she is
not sexually active, has never had galactorrhea, and has not
experienced hot flashes. She has mild acne and no excessive body
hair. She is concerned that she has gained a lot of weight over the
past year.
She has a significant psychiatric history. Current medications are
risperidone, benztropine, clonidine, and lithium carbonate.
On physical examination, her height is 67.3 in (171 cm) (89th
percentile), weight is 229.7 lb (104.4 kg) (>99th percentile), and BMI
is 36 kg/m2 (98th percentile). She has mild darkening and
thickening of the skin on her neck and mild hirsutism (Ferriman-
Gallwey score = 8). There is no virilization. Examination findings are
otherwise normal.
Laboratory test results:
TSH = 2.98 mIU/L (0.45-4.50 mIU/L)
Free T4 = 1.17 ng/dL (0.93-1.60 ng/dL) (SI: 15.1 pmol/L [12.0-20.6 pmol/L])
β-hCG = <1 mIU/mL (0-5 mIU/mL) (SI: <1 IU/L [0-5 IU/L])
DHEA-S = 502.2 µg/dL (110.0-433.2 µg/dL) (SI: 13.6 µmol/L [3.0-11.7 µmol/L])
FSH = 6.2 mIU/mL (1.7-21.5 mIU/mL) (SI: 6.2 IU/L [1.7-21.5 IU/L])
LH = 18.6 mIU/mL (1.0-12.6 mIU/mL) (SI: 18.6 IU/L [1.0-12.6 IU/L])
Estradiol = 43.6 pg/mL (12.5-211.0 pg/mL) (SI: 160.1 pmol/L [45.9-774.6 pmol/L])
Free testosterone = 4.2 pg/mL (0.5-3.9 pg/mL) (SI: 0.15 nmol/L [0.02-0.14 nmol/L])
Prolactin = 124.9 ng/mL (4.8-23.3 ng/mL) (SI: 5.4 nmol/L [0.2-1.0 nmol/L])

Which of the following is the best next step in this patient’s


management?
A. Perform MRI of the brain and pituitary
B. Perform cosyntropin-stimulation testing to evaluate for
nonclassic CAH
C. Perform progestin withdrawal challenge
D. Provide reassurance that risperidone is the most likely cause
and continue to monitor
E. Discuss switching from risperidone to another atypical
antipsychotic agent with her psychiatrist
19 A 16-year-old male high school athlete with type 1 diabetes
mellitus decides to lift weights in preparation for football season. He
administers multiple daily insulin injections via insulin pen devices
and monitors his blood glucose with a conventional glucose meter.
He recalls having episodes of hypoglycemia when he was cycling in
the spring. Therefore, before his weight-lifting sessions, he consumes
15 to 20 g of carbohydrate in a protein bar without any insulin
coverage because he is afraid of experiencing hypoglycemia during
exercise. When he monitors his blood glucose before and after the
weight-lifting sessions, he notices higher blood glucose values even
if he does not consume the protein bar. He wonders why this would
happen and poses the question to you during a clinic visit.

Which of the following is the best advice?


A. Suspect erroneous measurements of blood glucose; advise to
monitor before, during, and after exercise and report back in 1
week
B. Advise increasing his daily basal insulin by 20%
C. Recommend increasing the insulin bolus for the meal before
lifting weights
D. Recommend a small insulin bolus before lifting weights
E. Reassure the patient and recommend no changes in insulin
dosage
20 A 10-and-5/12-year-old boy is referred by his pediatrician for
evaluation of short stature and abnormal thyroid function test
results:
Total T4 = 3.3 µg/dL (3.8-12.0 µg/dL) (SI: 42.5 nmol/L [48.9-154.4 nmol/L])
TSH = 6.7 mIU/L (0.5-4.3 mIU/L)

The patient was born at 40 weeks’ gestation with a birth weight of 8


lb 5 oz (3770 g) after an uncomplicated pregnancy. There is a vague
history of seizures in the neonatal period. He has developmental
delay and was diagnosed with attention-deficit/hyperactivity
disorder at age 5 years. He has outbursts of aggressive behavior and
is under the care of a psychiatrist. He takes several medications,
including methylphenidate, quetiapine, and guanfacine.
Hypothyroidism was diagnosed at age 1 year, and he received
levothyroxine at a dosage of 25 mcg daily until he was 3-and-5/12
years old when his medication ran out. At a follow-up visit when he
was 3-and-10/12 years old, his thyroid function was normal after 5
months of not receiving levothyroxine. Therefore, levothyroxine was
not restarted. He lives with his grandmother (his legal guardian).
The family history is not fully obtainable. His father’s height is 67 in
(170.2 cm), and his mother’s height is 60 in (152.4 cm). Midparental
target height is 66 ± 4 in (168 ± 10 cm).
On physical examination, his height is 53 in (134.6 cm) (SDS, –
1.94), weight is 87.3 lb (39.7 kg) (SDS, +0.84), and BMI is 24.2 kg/m2
(97th percentile; SDS, +1.98). He is obese and has a stocky build.
Vital signs are normal. Six-year molars are present. No thyroid gland
enlargement is noted. Pubic hair and genitalia are Tanner stage 1.
Testicular volume is 2 to 3 mL bilaterally.
Bone age is interpreted to be markedly advanced at 15 years at a
chronologic age of 10 years 5 months with fusion of epiphyses of all
distal phalanges.

Laboratory test results:


Calcium = 8.0 mg/dL (8.8-10.8 mg/dL) (SI: 2.0 mmol/L [2.2-2.7 mmol/L])
Phosphate = 6.2 mg/dL (2.3-4.5 mg/dL) (SI: 2.0 mmol/L [0.7-1.5 mmol/L])
Total protein = 8.5 g/dL (6.0-8.0 g/dL) (SI: 85 g/L [60-80 g/L])
Albumin = 5.0 g/dL (3.8-5.4 g/dL) (SI: 50 g/L [38-54 g/L])
Alkaline phosphatase = 181 U/L (<400 IU/L) (SI: 3.0 µkat/L [<6.7 µkat/L])
AST = 46.0 U/L (<40.0 U/L) (SI: 0.77 µkat/L [<0.67 µkat/L])
ALT = 24.0 U/L (<40.0 U/L) (SI: 0.40 µkat/L [<0.67 µkat/L])
Free T4 = 0.79 ng/dL (0.8-1.7 ng/dL) (SI: 10.2 pmol/L [10.3-21.9 pmol/L])
TSH = 5.9 mIU/L (0.7-5.7 mIU/L)
Total T3 = 109.0 ng/dL (123.0-211.0 ng/dL) (SI: 1.7 nmol/L [1.9-3.2 nmol/L])
PTH = 169 pg/mL (10-65 pg/mL) (SI: 169 ng/L [10-65 ng/L])

Which additional physical findings would you expect to find on


examination?
A. Almond-shaped eyes
B. Focal dysplasia of the distal radial physis
C. Widely-spaced eyes and low-set ears
D. Hemihypertrophy
E. Shortened fourth metacarpals and metatarsals
21 A 13-year-old boy presents to the emergency department with
nonspecific chest pain, aches, and pains. His blood pressure is
documented to be 170/90 mm Hg. Initial laboratory testing
documents hypokalemia with a serum potassium concentration of
2.8 mEq/L (2.8 mmol/L). There is no family history of hypertension.
He is initially seen by a cardiologist, who prescribes
antihypertensive therapy. Despite this, he remains hypertensive with
a systolic blood pressure of 140 to 150 mm Hg and a diastolic blood
pressure of 90 to 100 mm Hg. He is referred to the pediatric
endocrine department for further assessment.

Laboratory test results:


Serum aldosterone = 54.1 ng/dL (1.4-17.3 ng/dL) (SI: 1500 pmol/L [40-480
pmol/L])
Plasma renin activity = <0.15 ng/mL per h (0.5-2.6 ng/mL per h)

Findings on adrenal CT and MRI are reportedly normal.

Which of the following is the most likely diagnosis?


A. Renal artery stenosis
B. Congenital adrenal hyperplasia
C. Liddle syndrome
D. Adrenal adenoma
E. Licorice root ingestion
22 An 11-year-old girl is referred by her pediatrician because of
poor growth. Review of her growth chart shows that she was
tracking along the 95th percentile for height and the 50th percentile
for weight from 4 to 8 years of age. In the last 3 years, her height has
crossed multiple percentile lines (downward trend), whereas she has
continued to have normal weight gain. Today, she is at the 33rd
percentile for height and the 42nd percentile for weight. Chronic
myeloid leukemia was diagnosed at age 8 years, and she started
chemotherapy at that time. She did not receive any radiation
therapy. She is followed by oncology and has been responding well
to the oral pill (dasatinib) she takes every day. She is otherwise doing
well.
On physical examination, breast development is Tanner stage 1
and pubic hair is Tanner stage 2. Her midparental target height is at
the 90th percentile.
Baseline laboratory test results (sample drawn last week)
document normal complete blood cell count, basal metabolic panel,
IGF-1, IGFBP-3, free T4, and TSH.

Which of the following is the best next step in this patient’s


management?
A. Discuss the adverse effects of her cancer therapy and have her
return for follow-up in 6 months
B. Start levothyroxine and assess growth response in 6 months
C. Order a GH-stimulation test and start GH therapy
D. Explain that her growth will continue to deteriorate and she
will most likely have short stature as an adult
E. Perform DXA to evaluate bone health
23 A 30-day-old boy born at 25 weeks’ gestation with a birth
weight of 1 lb 6 oz (640 g) is noted to have temperature instability in
the neonatal intensive care unit. Intravenous antibiotics are started,
and cultures are negative. He has a history of pulmonary
hypertension. He is intubated and his condition has otherwise been
stable. He is tolerating feedings. Results of newborn screening
(sample drawn at 48 hours of life) are normal. His mother has no
history of thyroid disorders and takes no thyroid medications.

Thyroid hormone levels obtained at day of life 30:


TSH = 22 mIU/L (0.5-6.5 mIU/L)
Free T4 = 0.9 ng/dL (0.8-2.2 ng/dL) (SI: 11.6 pmol/L [10.3-28.3 pmol/L])

Results of newborn screening (sample drawn at 48 hours of life) are


normal. His mother has no history of thyroid disorders and takes no
thyroid medications.

Which of the following is the best recommendation in this


patient’s management?
A. Repeat TSH and free T4 measurement in 4 weeks
B. Measure thyroid antibodies in the mother
C. Measure TSH, free T4 by equilibrium dialysis, and total T3 in 2
days
D. Reassure the neonatologist that the results are most likely
transient and due to recovery from nonthyroidal illness;
recommend no intervention
E. Start thyroid hormone replacement and measure TSH and free
T4 again in 2 weeks
24 An 11-year-old girl presents with headaches and vision
changes. Brain MRI shows a 3.8 × 2.9 × 3.1-cm, heterogeneous,
multicystic-appearing, heterogeneously enhancing mass within the
suprasellar region consistent with a craniopharyngioma and
hydrocephalus. She is taken to the operating room for endoscopic
fenestration of the cyst with placement of a ventricular catheter. She
does well postoperatively.
Over the next year, she has an increase in the cystic component of
the tumor and requires another surgery. Complications include a
middle cerebral artery infarct, panhypopituitarism, central diabetes
insipidus, and hypothalamic obesity. Her condition is well controlled
on levothyroxine, hydrocortisone, and twice-daily oral
desmopressin. On follow-up scans, there is further increase in the
size of the cystic component of the tumor with compression of the
optic nerve, necessitating another surgery. Her sodium and free T4
levels are normal on replacement therapies before surgery, and she
receives stress doses of hydrocortisone perioperatively and
postoperatively. She is maintained on intravenous normal saline
fluids, but by postoperative day 2 she is alert and hungry and is
allowed to eat and drink ad lib, so intravenous fluids are
discontinued. Her home desmopressin regimen of 0.2 mg orally
twice daily is then initiated.

Laboratory test results:


Measurement Postoperative day 2 Postoperative day 3 Reference range
Serum sodium 139 mEq/L 153 mEq/L 136-149 mEq/L
(SI: 139 mmol/L) (SI: 153 mmol/L) (SI: 136-149 mmol/L)
Serum potassium 4.0 mEq/L 3.8 mEq/L 3.9-5.7 mEq/L
(SI: 4.0 mmol/L) (SI: 3.8 mmol/L) (SI: 3.9-5.7 mmol/L)
Serum chloride 102 mEq/L 115 mEq/L 98-108 mEq/L
(SI: 102 mmol/L) (SI: 115 mmol/L) (SI: 98-108 mmol/L)
Serum carbon dioxide 20.5 mEq/L 18 mEq/L 23-30 mEq/L
(SI: 20.5 mmol/L) (SI: 18 mmol/L) (SI: 23-30 mmol/L)
Serum urea nitrogen 16 mg/dL 15 mg/dL 7-18 mg/dL
(SI: 5.7 mmol/L) (SI: 5.4 mmol/L) (SI: 2.5-6.4 mmol/L)
Creatinine 0.54 mg/dL 0.67 mg/dL 0.25-1.14 mg/dL
(SI: 47.7 µmol/L) (SI: 59.2 µmol/L) (SI: 22.1-100.8 µmol/L)
Plasma glucose 150 mg/dL 140 mg/dL 70-110 mg/dL
(SI: 8.3 mmol/L) (SI: 7.8 mmol/L) (SI: 3.9-6.1 mmol/L)
Free T4 1.27 ng/dL … 0.97-1.67 ng/dL
(SI: 16.3 pmol/L) (SI: 12.5-21.5 pmol/L)

Which of the following is the most likely cause of her


hypernatremia?
A. Excessive administration of normal saline fluids
B. Adipsia
C. Stress doses of hydrocortisone worsening the diabetes
insipidus
D. Hyperglycemia causing dehydration
E. Insufficient desmopressin dosage
25 A primary care physician is calling to inform you that a 14-
year-old girl with type 1 diabetes mellitus under your care has
asthma exacerbation and is being treated with prednisone, 2 mg/kg
per day divided twice daily for 5 days. When on the same
prednisone dosage in the past, she developed hyperglycemia with
glucose values in the range of 300 to 400 mg/dL (16.7-22.2 mmol/L)
and mild-to-moderate ketonuria. She will receive her first
prednisone dose tomorrow morning. She is currently on insulin
pump therapy, and her most recent hemoglobin A1c value was 7.2%
(55 mmol/mol).

What, if any, initial changes in the pump settings do you


recommend to control her blood glucose while she is taking
glucocorticoids?
A. Increase all correction/insulin sensitivity factors by 30% now
B. Increase all basal rates by 30% now
C. Increase insulin-to-carbohydrate ratios by 30% now
D. Increase all basal rates by 30% within 6 to 12 hours after
initiating prednisone
E. Check blood glucose every 3 hours and correct hyperglycemia
as needed without adjusting insulin pump settings
26 A 12-and-6/12-year-old girl is referred for evaluation of short
stature. She was originally seen by an adult endocrinologist closer to
home at age 8 years because of poor growth. At that time, laboratory
test results (comprehensive metabolic panel, complete blood cell
count, thyroid function tests) and bone age were normal.
On physical examination today, her height is 49 in (124.5 cm)
(<1st percentile, –2.9 SD), weight is 118.8 lb (54 kg) (85th percentile),
and BMI is at the 98th percentile. Her vital signs are normal. She has
macrocephaly and mild retrognathia. Her arms and legs appear
short. She has mild brachydactyly. She does not have scoliosis.
Breasts and pubic hair are Tanner stage 5.
Her mother reports that she grew 1.5 cm in the last year. Breast
development and pubic hair growth started at age 9 years, and
menarche was at age 11 years. She has spontaneous regular menses.
She has a history of frequent ear infections. There are no
developmental delays. Review of systems is otherwise normal. Her
midparental target height is 64 in (162.6 cm) (close to the 50th
percentile). There is no family history of short stature.
The patient is very anxious about her growth potential.

Which of the following is the most likely explanation for her


short stature?
A. Maternal uniparental disomy of chromosome 15
B. Mosaic Turner syndrome
C. Activating pathogenic variant in the FGFR3 gene
D. Inactivating pathogenic variant in the maternal allele of the
GNAS gene
E. Pathogenic variant in the FMR1 gene
27 You are asked to evaluate a 7-week-old newborn for
hypospadias. The baby was born at 31 weeks’ gestation to a 40-year-
old mother with chronic hypertension. Family history is notable for
chordee in the patient’s 22-year-old half-brother. The pregnancy was
notable for maternal gestational diabetes, preeclampsia, and breech
presentation. Birth weight was 2 lb 11 oz (1222 g [appropriate for
gestational age]). Prenatal karyotype was documented to be 46,XY.
FISH for SRY performed on day 2 of life was normal.
The baby’s course in the neonatal intensive care unit was
unremarkable, with normal electrolytes, no respiratory distress, and
quick transition to full oral feeds.
On physical examination, the baby appears well. Phallus length is
1.9 cm. There is perineal hypospadias, chordee, and bifid scrotum.
Gonads are palpable in the inguinal canals.

Laboratory test results (sample drawn at 5 weeks of age):


Sodium = 139 mEq/L (134-144 mEq/L) (SI: 139 mmol/L [134-144 mmol/L])
Potassium = 5.2 mEq/L (3.5-5.2 mEq/L) (SI: 5.2 mmol/L [3.5-5.2 mmol/L])
Serum urea nitrogen = 3 mg/dL (5-18 mg/dL) (SI: 1.1 mmol/L [1.8-6.4 mmol/L])
Creatinine = 0.29 mg/dL (0.30-0.59 mg/dL) (SI: 25.6 µmol/L [26.5-52.2 µmol/L])
FSH = 6.6 mIU/L (3.0-6.0 mIU/L) (SI: 6.6 IU/L [3.0-6.0 IU/L])
LH = 3.2 mIU/L (<0.02-5.0 mIU/L) (SI: 3.2 IU/L [<0.02-5.0 IU/L])
17-Hydroxypregnenolone = 325 ng/dL (229-3104 ng/dL) (SI: 9.8 nmol/L [6.9-93.4
nmol/L])
17-Hydroxyprogesterone = 179.6 ng/dL (0-200 ng/dL) (SI: 5.4 nmol/L [0-6.1
nmol/L])
Total testosterone = 169 ng/dL (≤201 ng/dL) (SI: 5.86 nmol/L [≤6.97 nmol/L])
Dihydrotestosterone = 46.6 ng/dL (12-85 ng/dL) (SI: 1.6 nmol/L [0.4-2.9 nmol/L])

Which of the following diagnoses is most consistent with this


patient’s examination and laboratory findings?
A. 5α-Reductase deficiency
B. 3β-Hydroxysteroid dehydrogenase deficiency
C. Swyer syndrome (gonadal dysgenesis)
D. Partial androgen insensitivity syndrome
E. Pathogenic variant in the LHCGR gene
28 A 17-year-old boy presents with concerns about lipid
abnormalities. His LDL-cholesterol concentration is 225 mg/dL (5.83
mmol/L). His paternal grandfather and paternal uncle died early of
cardiovascular disease. Treatment is initiated with atorvastatin, 10
mg daily, which lowers the patient’s LDL-cholesterol concentration
to 140 mg/dL (3.63 mmol/L) over a 6-month period. His mother
calls this morning because he is experiencing mild discomfort in his
calves. Yesterday he had soccer tryouts, during which he was
physically active.

Laboratory test results:


Creatine kinase = 198 U/L (15-105 U/L) (SI: 3.3 µkat/L [0.3-1.8 µkat/L])
ALT = 54 U/L (<55 U/L) (SI: 0.90 µkat/L [<0.92 µkat/L])
AST = 34 U/L (<35 U/L) (SI: 0.57 µkat/L [<0.58 µkat/L])
Creatinine = 0.7 mg/dL (0.5-1.0 mg/dL) (SI: 61.9 µmol/L [44.2-88.4 µmol/L])

Atorvastatin is continued and his symptoms resolve. One month


later, he returns with new pain in his calves. He just started soccer 1
week ago.
Results of repeated laboratory tests:
Creatine kinase = 250 U/L (15-105 U/L) (SI: 4.18 µkat/L [0.25-1.75 µkat/L])
ALT = 55 U/L (<55 U/L) (SI: 0.92 µkat/L [<0.92 µkat/L])
AST = 28 U/L (<35 U/L) (SI: 0.47 µkat/L [<0.58 µkat/L])
Creatinine = 0.7 mg/dL (0.5-1.0 mg/dL) (SI: 61.9 µmol/L [44.2-88.4 µmol/L])

Bearing in mind current recommendations regarding statin


treatment in children, which of the following is the best next
step?
A. Continue atorvastatin and measure creatine kinase in 2 days
B. Stop atorvastatin; wait 2 weeks and then reintroduce
atorvastatin and assess for symptoms
C. Stop atorvastatin and pursue nonstatin cholesterol treatment
D. Admit him to the hospital for observation and treatment of
muscle damage
E. Order MRI of his legs
29 A 17-and-4/12-year-old girl has been followed for Hashimoto
thyroiditis, diagnosed at age 14-and-9/12 years on the basis of
elevated TPO antibodies.

Laboratory test results at age 14-and-6/12 years:


TSH = <0.003 mIU/L (0.5-4.3 mIU/L)
Total T4 = 11.4 µg/dL (4.5-12.0 µg/dL) (SI: 146.7 nmol/L [57.9-154.4 nmol/L])

At age 14-and-9/12 years, she was evaluated by an endocrinologist.


She had no goiter or bruit. At that time, her TSH was no longer
suppressed (0.39 mIU/L) and the following laboratory values were
documented:
Total T4 = 7.6 µg/dL (4.5-12.0 µg/dL) (SI: 97.8 nmol/L [57.9-154.4 nmol/L])
Total T3 = 124 ng/dL (84-179 ng/dL) (SI: 1.9 nmol/L [1.3-2.8 nmol/L])
TPO antibodies = 42 IU/mL (<9 IU/mL)
Thyroid-stimulating immunoglobulin, negative
Thyroglobulin antibodies, negative

She remained euthyroid and has never required levothyroxine.


However, current routine laboratory test results (at age 17-and-4/12
years) show she is hyperthyroid:
TSH = 0.01 mIU/L (0.5-4.3 mIU/L)
Free T4 = 2.3 ng/dL (0.9-1.4 ng/dL) (SI: 29.6 pmol/L [11.6-18.0 pmol/L])
Total T4 = 17.0 µg/dL (4.5-12.0 µg/dL) (SI: 218.8 nmol/L [57.9-154.4 nmol/L])
Total T3 = 288 ng/dL (86-192 ng/dL) (SI: 4.4 nmol/L [1.3-3.0 nmol/L])

She is asymptomatic (no heat intolerance, loose stools, or


palpitations), but she has lost 7.3 lb (3.3 kg) in the previous 5 months.
She has no pain in her neck or jaw.
On physical examination today, her blood pressure is 116/73 mm
Hg and pulse rate is 112 to 116 beats/min. She has a small, diffuse
goiter (3.5 cm) without a bruit. She has no thyroid nodules, no
tenderness to palpation, no lymphadenopathy in her neck, no
proptosis, and no tremor. The rest of her examination findings are
unremarkable.
One week later, laboratory tests show that she is still
hyperthyroid with similar thyroid function. A celiac panel is
negative, she has a normal erythrocyte sedimentation rate, and
thyroid-stimulating immunoglobulin is 280% (<140%).

Which of the following is the most likely diagnosis?


A. Hashitoxicosis
B. Silent subacute thyroiditis
C. Graves disease
D. Hot nodule
E. Exogenous ingestion of levothyroxine
30 You are evaluating an 8-and-2/12-year-old boy for signs of
early pubertal development. He has a history of medulloblastoma
diagnosed at age 6 years and was treated with cranial radiation to
the hypothalamic-pituitary area. He received a total of 20 Gy of
radiation therapy. Before the tumor was diagnosed, his linear growth
had been steadily progressing along the 25th percentile. At his 8-year
well-child check, his pediatrician noted evidence of testicular and
penile enlargement and referred him to you for evaluation. His
medical history and family history are noncontributory.
On physical examination, his height is 49.6 in (126 cm), which
plots at the 25th percentile (SDS, –0.65). His weight is 58.5 lb (26.6
kg), which plots at the 8th percentile (SDS, –1.35). His arm span is 50
in (127 cm). Vital signs are normal. He has no dysmorphic features.
His thyroid gland is not enlarged. Findings on neurologic
examination are grossly intact. Pubic hair is early Tanner stage 2.
Testicular volume is 6 mL bilaterally, and his stretched penile length
is 10 cm (mean penile length for 8- to 9-year-old boys is 6.3 cm). The
rest of the physical examination findings are unremarkable.
Bone age is interpreted to be 10 years and 0 months at the
chronologic age of 8 years and 2 months according to the Greulich
and Pyle method.

Laboratory test results:


Testosterone (8 AM) (by liquid chromatography/tandem mass spectrometry = 121
ng/dL (SI: 4.2 nmol/L) (Tanner stage 1: <3-10 ng/dL [SI: <0.1-0.4 nmol/L];
Tanner stage 2: 18-150 ng/dL [SI: 0.6-5.2 nmol/L])
LH (ultrasensitive) = 1.5 mIU/mL (SI: 1.5 IU/L) (Tanner stage 1: <0.02-0.3 mIU/mL
[SI: <0.02-0.3 IU/L]; Tanner stage 2: 0.2-4.9 mIU/mL [SI: 0.2-4.9 IU/L])
IGF-1 = 125 ng/mL (SI: 16.4 nmol/L) (7- to 8-year-old males: 113-261 ng/mL [SI:
14.8-34.2 nmol/L]; 9- to 10-year-old males: 123-275 ng/mL [SI: 16.1-36.0
nmol/L])
Cortisol (8 AM) = 9 µg/dL (7-25 µg/dL) (SI: 248.3 nmol/L [193.1-689.7 nmol/L])

Which of the following is the best next step in this patient’s


evaluation?
A. GnRH-stimulation test
B. Low-dose cosyntropin-stimulation test
C. GH-stimulation test
D. Brain MRI with and without contrast
E. Testicular ultrasonography
31 A 12-year-old girl presents to the outpatient clinic with a
history of chronic fatigue, attention-deficit/hyperactivity disorder,
irritability, and depression. She had a kidney stone 1 year ago, and
laboratory testing at that time showed the following:
Serum calcium = 10.1 mg/dL (8.5-10.0 mg/dL) (SI: 2.5 mmol/L [2.1-2.5 mmol/L])
Phosphate = 2.5 mg/dL (2.8-5.1 mg/dL) (SI: 0.8 mmol/L [0.9-1.6 mmol/L])
PTH = 54 pg/mL (15-87 pg/mL) (SI: 54 ng/L [15-87 ng/L])
Urine calcium-to-creatinine ratio = 0.175

Hypercalciuria was diagnosed, and she was started on a thiazide


diuretic. DXA of the lumbar spine documented a height-adjusted Z-
score of –0.7.
In clinic today, the patient is well hydrated. She takes no other
medications. There is no family history of calcium balance disorders.
Current laboratory test results:
Serum calcium = 11.1 mg/dL (8.5-10.0 mg/dL) (SI: 2.8 mmol/L [2.1-2.5 mmol/L])
Phosphate = 2.4 mg/dL (2.8-5.1 mg/dL) (SI: 0.8 mmol/L [0.9-1.6 mmol/L])
PTH = 73 pg/mL (15-87 pg/mL) (SI: 73 ng/L [15-87 ng/L])
25-Hydroxyvitamin D = 25 ng/mL (20.0-60.0 ng/mL) (SI: 62.5 nmol/L [49.9-149.8
nmol/L])
Urinary calcium-to-creatinine ratio = 0.217

Which of the following is the best next step to determine the


etiology of her condition?
A. Order genetic testing for a pathogenic variant in the CASR
gene
B. Perform parathyroid ultrasonography
C. Perform a 99Tc sestamibi scan
D. Perform DXA of the one-third distal radius
E. Stop the thiazide diuretic and repeat the laboratory tests
32 A 12-year-old girl presents for evaluation of poor growth. She
was born full-term and had a normal birth weight. Her mother
reports that she grew normally until 1 year ago when she began to
have headaches and daily abdominal pain, feeling full after very
small meals. She underwent a complete gastroenterologic workup,
including endoscopy and colonoscopy, which were unrevealing. She
lost more than 20 lb (9.1 kg) in the past year. She was subsequently
admitted to a partial psychiatric hospitalization program.
Midparental height is 63 in (160 cm). Her mother had menarche at
age 15 years, and her father underwent puberty at age 16 years. Her
medications include cyproheptadine, risperidone, and sertraline.
On physical examination, she is a pale, thin, quiet girl. Her blood
pressure is 90/59 mm Hg, and pulse rate is 89 beats/min. Her height
is 57.5 in (149 cm) (37th percentile), and weight is 68.2 lb (31 kg) (5th
percentile) (BMI = 14 kg/m2). Her breasts are Tanner stage 1, and
pubic hair is Tanner stage 2. Findings on neurologic examination are
normal.
Her growth chart is shown (see image).
Laboratory test results:
Random GH = 8.4 ng/mL (0.06-4.3 ng/mL) (SI: 8.4 µg/L [0.06-4.3 µg/L])
IGF-1= 98 ng/mL (104-665 ng/mL) (SI: 12.8 nmol/L [13.6-87.1 nmol/L])

Bone age is interpreted to be 10 years.

Which of the following is the most likely cause of her growth


delay?
A. Craniopharyngioma
B. GH insensitivity syndrome
C. Addison disease
D. Constitutional delay of growth and maturity
E. Undernutrition
33 A 14-year-old boy with Silver-Rusell syndrome has recently
moved to the area and is presenting for continuation of care. He was
born at 37 weeks’ gestation, with a birth weight of 4 lb 8 oz (2050 g)
(small for gestational age) and birth length of 17.7 in (45 cm). His
clinical course in infancy was complicated by feeding difficulties and
recurrent hypoglycemia, particularly with illness, requiring hospital
admission. He was evaluated by a geneticist and was found to have
a methylation defect involving chromosome 11p15. He started GH
therapy at age 4 years, and he developed signs of pubarche at age 7
years. He had a histrelin implant placed 2 years ago, and it was
removed last year. His midparental target height is 70 in (177.8 cm),
and his father has a history of constitutional growth delay.
His height is 62 in (157.5 cm) (25th percentile), and weight is 100.8
lb (45.8 kg) (31st percentile). On physical examination, he has Tanner
stage 3 pubic hair and a testicular volume of 3 mL bilaterally. His
height velocity over the past year has decreased from 5.6 cm/y to 4.6
cm/y.

Laboratory test results:


IGF-1 = 546 ng/mL (148-551 ng/mL) (SI: 71.5 nmol/L [19.4-72.2 nmol/L])
TSH = 1.78 mIU/L (0.450-4.500 mIU/L)
Free T4 = 1.26 ng/dL (0.8-1.8 ng/dL) (SI: 16.2 pmol/L [10.3-23.2 pmol/L])
Total testosterone = 4.7 ng/dL (≤1000 ng/dL) (SI: 0.16 nmol/L [≤34.7 nmol/L])
LH = 0.69 mIU/mL (0.31-5.29 mIU/mL) (SI: 0.69 IU/L [0.31-5.29 IU/L])

Bone age is 13 years.


Laboratory testing 1 year ago was notable for a total testosterone
concentration of 6.0 ng/dL (0.21 nmol/L) and LH concentration of
1.1 mIU/mL (1.1 IU/L).

Which of the following is the best interpretation of these data?


A. Poor adherence to treatment resulting in slow growth
B. Gonadotropin deficiency causing failed pubertal progression
and slow growth
C. Constitutional growth delay
D. Retained portion of histrelin implant, which is continuing to
suppress puberty
E. Early epiphyseal closure causing slow growth
34 A 10-year-old boy with Down syndrome presents to the
emergency department with a history of decreased activity, excessive
sleep, increased appetite, rapid weight gain, and frequent episodes
of tonic clonic seizures. He has a history of a ventricular septal defect
repaired surgically.
On physical examination, his height is 51.6 in (131 cm) (5th
percentile; –1.2 SDS) (50th-75th percentile for Down syndrome),
weight is 121 lb (55 kg) (97th percentile; 2.21 SDS), and BMI is 32.2
kg/m2 (99th percentile; 2.5 SDS).
On arrival to the emergency department, his blood glucose
concentration is 40 mg/dL (2.2 mmol/L).
Laboratory test results (critical sample):
Insulin = 119.8 μIU/mL (2.6-24.9 μIU/mL) (SI: 832.0 pmol/L [17.8-173.0 pmol/L])
C-peptide = 7.6 ng/mL (0.8-3.1 ng/mL) (SI: 2.5 nmol/L [0.26-1.03 nmol/L])
GH = <0.1 ng/mL (0-10 ng/mL) (SI: <0.1 μg/L [0-10 μg/L])
Cortisol = 8.7 μg/dL (6.2-19.4 μg/dL) (SI: 241 nmol/L [171-536 nmol/L])
Ammonia = 41 μmol/L (0-55 μmol/L)
β-Hydroxybutyrate = <4.2 mg/dL (<4.2 mg/dL) (SI: <400 μmol/L [<400 μmol/L])
Acylcarnitine profile, negative

He requires a high-concentration intravenous dextrose infusion with


a glucose infusion rate greater than 20 mg/kg per min. Intravenous
fluid infusion is interrupted to draw a second critical sample within
35 minutes, and the patient’s blood glucose concentration is less than
30 mg/dL (<1.7 mmol/L). Other results from the second critical
sample:
Insulin = 47.4 μIU/mL (SI: 329 pmol/L)
C-peptide = 4.8 ng/mL (SI: 1.58 nmol/L)
GH = 4 ng/mL (SI: 4 μg/L)
Cortisol = 10.7 μg/dL (SI: 294 nmol/L)
Ammonia = 56 μmol/L
β-Hydroxybutyrate = <4.2 mg/dL (SI: <400 μmol/L)

Which of the following is the best next step in this patient’s


management?
A. Sulfonylurea screen
B. MRI of the pancreas
C. Diazoxide challenge
D. Another critical sample before ordering an imaging study
E. Genetic testing
35 A 4-and-9/12-year-old boy presents for initial evaluation of
poor growth and short stature. He was born out of a second-degree
consanguineous relationship at term gestation with a birth weight of
7 lb 11 oz (3500 g). Motor milestones have been delayed (started
walking without support at age 3 years), but he has been reaching
mental and social milestones appropriately. He describes leg pain
while bearing weight and has an unstable gait with frequent falls. He
has not sustained any fractures. His first tooth erupted at 18 months
of age, and his primary dentition was completed at 4 and 6/12 years
of age. He is a vegetarian. He has been taking vitamin D
supplements for the last 4 months, as suggested by his pediatrician,
with no symptomatic relief and no improvement in growth.
On physical examination, he is below the 3rd percentile for height
and is at the 5th percentile for weight. His vital signs are normal. His
physical appearance is shown in the photographs (see images).

Laboratory tests ordered by his pediatrician 2 weeks ago document


the following results:
Serum calcium = 4.6 mg/dL (8.8-10.8 mg/dL) (SI: 1.2 mmol/L [2.2-2.7 mmol/L])
Serum phosphate = 3.1 mg/dL (3.8-5.8 mg/dL) (SI: 1.0 mmol/L [1.2-1.9 mmol/L])
Alkaline phosphatase = 1685 IU/L (100-320 IU/L) (SI: 28.1 µkat/L [1.7-5.3 µkat/L])
Total 25-hydroxyvitamin D = 88 ng/mL (30-100 ng/mL) (SI: 219.6 nmol/L [74.9-
249.6 nmol/L])
25-Hydroxyvitamin D2 = <5 ng/mL (SI: <12.5 nmol/L)
25-Hydroxyvitamin D3 = 88 ng/mL (SI: 219.6 nmol/L)
1,25-Dihydroxyvitamin D = 25 pg/mL (15-90 pg/mL) (SI: 65.0 pmol/L [39-234
pmol/L])
PTH = 124 pg/mL (10-65 pg/mL) (SI: 124 ng/L [10-65 ng/L])

Which of the following is the most appropriate management for


this patient?
A. Calcium supplementation and high-dosage ergocalciferol
B. Calcium supplementation and calcitriol
C. Phosphorus supplementation and calcitriol
D. Calcium supplementation alone and reevaluation in 3 months
E. Burosumab therapy
36 A 4-and-6/12-year-old girl with a recent diagnosis of
hyperthyroidism secondary to Graves disease comes for initial
consultation. She was seen by her primary care physician about 1
month ago for poor sleep and mood swings and was noted to have a
pulse rate of 124 beats/min and blood pressure of 106/52 mm Hg.
There is no history of recent illness or infections.
On physical examination, she has a nontender, symmetric goiter.
She has no upper eyelid retraction, scleral erythema, or proptosis.
Her weight is 44 lb (20 kg).

Laboratory test results:


TSH = 0.005 mIU/L (0.27-4.2 mIU/L)
Free T4 = 2.5 ng/dL (0.9-1.7 ng/dL) (SI: 32.2 ng/dL [11.6-21.9 pmol/L])
TPO antibodies = 479.9 IU/mL (<9.0 IU/mL) (SI: 479.9 kIU/L [<9.0 kIU/L])
TSH-receptor antibodies = >40 IU/L (0-1.75 IU/L)

Laboratory test results (sample drawn at today’s visit):


TSH = <0.01 mIU/L
Free T4 = 2.2 ng/dL (0.9-1.7 ng/dL) (SI: 28.3 ng/dL [11.6-21.9 pmol/L])
Total T3 = 274 ng/dL (90-240 ng/dL) (SI: 4.2 nmol/L [1.4-3.7 nmol/L])
Absolute neutrophil count (ANC) = 2.3 × 103/µL (4.0-6.5 × 103/µL)
AST = 29 U/L (5-32 U/L) (SI: 0.48 µkat/L [0.08-0.53 µkat/L])
ALT = 37 U/L (5-33 U/L) (SI: 0.62 µkat/L [0.08-0.55 µkat/L])

Methimazole is initiated at a dosage of 5 mg daily (0.3 mg/kg per


day). Since she had abnormal ANC and liver enzymes before
starting treatment, repeated levels are obtained for follow-up.
Thyroid hormone levels improve over time and absolute neutrophil
count remains stable. Liver enzymes change as follows:
Analyte 2 Weeks Later 4 Weeks Later
AST 37 U/L 45 U/L
(SI: 0.6 µkat/L) (SI: 0.8 µkat/L)
ALT 72 U/L 144 U/L
(SI: 1.2 µkat/L) (SI: 2.4 µkat/L)
Which of the following is the best next step in this patient’s
management?
A. Discontinue methimazole and discuss thyroidectomy
B. Discontinue methimazole and plan for radioactive iodine
therapy
C. Decrease the methimazole dosage and measure liver enzymes
again in 2 weeks
D. Order a comprehensive liver panel and refer to
gastroenterology
E. Decrease the methimazole dosage and refer to
gastroenterology
37 A 16-day-old newborn is referred for endocrine evaluation
following recurrent hypoglycemic episodes. He has 1 sister who is 30
months old and is reported to be healthy. His parents are
nonconsanguineous and are of Romanian origin with no notable
family history. His mother reports that her pregnancy with him was
uneventful. He was born by vaginal delivery at 38 weeks’ gestation
with a birth weight of 6 lb 3 oz (2800 g). Apgar scores were 9/10, and
assessment of cord gases documented a pH value of 7.42. He had
respiratory arrest at 15 hours of life with a documented low glucose
value. Over the next 14 days, he had recurrent hypoglycemic
episodes. He required phototherapy for 72 hours because of
jaundice.
Endocrinology consult is requested. On assessment, his weight is
7 lb 1 oz (3200 g), blood pressure is 78/42 mm Hg, and pulse rate is
140 beats/min. He appears jaundiced with no skin
hyperpigmentation or hypopigmentation. No dysmorphic features
are noted. Findings on cardiovascular and respiratory examinations
are normal. He has normal male external genitalia, and both testes
are palpable in the scrotum.

Laboratory test results (blood samples taken during a hypoglycemic


episode):
Glucose = 34.2 mg/dL (63-126 mg/dL) (SI: 1.9 mmol/L [3.5-7.0 mmol/L])
Insulin = <1.0 µIU/mL (4.0-11.0 µIU/mL) (SI: <6.9 pmol/L [27.8-76.4 pmol/L]
Sodium = 136 mEq/L (133-141 mEq/L) (SI: 136 mmol/L [133-141 mmol/L])
Potassium = 5.6 mEq/L (4.3-6.2 mEq/L) (SI: 5.6 mmol/L [4.3-6.2 mmol/L])
Lactic acid = 33.5 mg/dL (6.5-20.0 mg/dL) (SI: 3.7 mmol/L [0.72-2.22 mmol/L])
Plasma ammonia = 85 µmol/L (10-150 µmol/L)
ACTH = 19.0 pg/mL (4.7-48.8 pg/mL) (SI: 4.2 pmol/L [1.0-10.7 pmol/L])
Cortisol = <1.0 μg/dL (7.8-26.2 μg/dL) (SI: <27.6 nmol/L [215.2-722.8 nmol/L])
GH = 14.4 ng/mL (5-50 ng/mL) (SI: 14.4 µg/L [5-50 µg/L])
Free T4 = 1.02 ng/dL (0.55-1.65 ng/dL) (SI: 13.1 pmol/L [7.08-21.24 pmol/L])
TSH = 6.14 mIU/L (0.55-12.5 mIU/L)

Which of the following should be the next investigation?


A. Standard-dose cosyntropin-stimulation test
B. Low-dose cosyntropin-stimulation test
C. Brain and pituitary MRI
D. 17-Hydroxyprogesterone measurement
E. Adrenal ultrasonography
38 A 4-year-old girl is referred to you by her primary care
physician for evaluation of short stature. She was born at 39 weeks’
gestation with a birth weight of 5 lb 13 oz (2637 g) and a birth length
of 18.5 in (47 cm). The pregnancy was uneventful.
On physical examination, she has no dysmorphic features, but
she does have short stature. Her height is 36.2 in (92 cm) (SDS, –
2.08), and weight is 27.5 lb (12.5 kg) (SDS, –2.05). Arm span is 34 in
(86.4 cm).
Laboratory test results are normal, including IGF-1, IGFBP-3,
thyroid function, liver function, kidney function, celiac screen, and
inflammatory markers.
Her bone age is concordant with her chronologic age.
The family recently moved from a foreign country. Family history
reveals that her parents are second cousins. Her father’s height is 64
in (162.5 cm) (SD, –2.00). Her mother’s height is 58 in (147.3 cm) (SD,
–2.46). Neither parent has any dysmorphic features and both have
normal intellect. The patient’s 8-year-old full brother, who was born
with normal proportions, has had abnormal growth noted from 1
year of age, and he now has severe short stature with a height SDS of
–6.5. He has a prominent forehead, wide and depressed nasal bridge,
prominent lips, short arms (especially fingers, hands, and forearms)
and legs (especially feet and shanks), and normal intellect.

Which of the following are the most likely explanations for the
short stature observed in this 4-year-old girl and her 8-year-old
brother?
Answer 4-year-old patient 8-year-old brother
A. Pseudoachondroplasia Achondroplasia
B. Heterozygous activating pathogenic Homozygous activating pathogenic
variant in NPR2 variants in NPR2
C. Heterozygous inactivating pathogenic Homozygous inactivating pathogenic
variant in FGFR3 variants in FGFR3
D. Turner syndrome SHOX haploinsufficiency
E. Heterozygous inactivating pathogenic Homozygous inactivating pathogenic
variant in NPR2 variants in NPR2
39 A 13-year-old adolescent boy with a BMI of 38 kg/m2 (142%
above the 95th percentile) has been unable to lose weight over the
last 2 years despite lifestyle modification efforts, including dietary
changes and increased exercise. He has had normal vital signs at
each visit. Sexual development is Tanner stage 3.

Recent laboratory test results:


Fasting glucose = 101 mg/dL (65-99 mg/dL) (SI: 5.61 mmol/L [3.6-5.5 mmol/L])
Total cholesterol = 211 mg/dL (<170 mg/dL) (SI: 5.46 mmol/L [<4.40 mmol/L])
HDL cholesterol = 45 mg/dL (>45 mg/dL) (SI: 1.16 mmol/L [>1.16 mmol/L])
Triglycerides = 199 mg/dL (<90 mg/dL) (SI: 2.25 mmol/L [<1.02 mmol/L])
Calculated LDL cholesterol = 125 mg/dL (<110 mg/dL) (SI: 3.24 mmol/L [<2.85
mmol/L])
Hemoglobin A1c = 5.7% (4.0%-5.6%) (39 mmol/mol [20-38 mmol/mol])

He asks your opinion regarding bariatric surgery, as his mother


underwent sleeve gastrectomy 2 years ago. A bariatric surgeon at
your center is experienced in performing this procedure in
adolescents.

Which of the following findings must be considered to


determine whether he qualifies for bariatric surgery?
A. Degree of dyslipidemia
B. Completion of pubertal growth
C. Family history of bariatric surgery
D. BMI percentile
E. History participating in a formal weight-loss program
40 A nearly 15-year-old girl presents for evaluation of amenorrhea.
Breast development started 2 years ago, but she thinks it has only
minimally progressed. She developed pubic hair approximately 4
years ago. She is otherwise in good health and has no galactorrhea,
dizziness, fatigue, or constipation. She is active in cross-country and
track. There is a family history of late menarche (age 14-16 years) in
her mother and other maternal relatives.
On physical examination, her height is 67.7 in (172 cm) (+1.56
SDS) and weight is 145 lb (65.9 kg) (+0.83 SDS). Midparental target
height is 55.5 in (141 cm). She has no glandular breast tissue and has
stage 4 pubic hair. There is no hirsutism or acne.

Laboratory test results:


Progesterone = 0.8 ng/mL (≤11.9 ng/mL) (SI: 2.5 nmol/L [≤37.8 nmol/L])
Estradiol = 8.2 pg/mL (≤283 pg/mL) (SI: 30.1 pmol/L [≤1038.9 pmol/L])
FSH = 102.5 mIU/mL (0.8-8.5 mIU/mL) (SI: 102.5 IU/L [0.8-8.5 IU/L])
LH = 38.3 mIU/mL (≤15.8 mIU/mL) (SI: 38.3 IU/L [≤38.3 IU/L])
Thyroid function, normal

Pelvic ultrasonography shows the presence of an echogenic structure


behind the bladder measuring 5.2 × 1.2 × 1.8 cm not definitively
identified as a uterus. Ovaries are not visualized.

Which of the following is the most likely underlying etiology?


A. >200 CGG triplet repeats in the FMR1 gene
B. Excess ovarian androgen production
C. Abnormal karyotype
D. Ovarian failure due to follicle depletion
E. Abnormally elevated prolactin levels
41 An overweight 15-year-old girl just returned from vacation in
Mexico. A few days after taking some diet pills while on her trip, she
developed a sore throat, palpitations, and a headache. She does not
have the pills with her and cannot recall the name. Her parents
brought her to the emergency department, where she was found to
be tachycardic and hypertensive.
On physical examination, her blood pressure is 140/90 mm Hg
and pulse rate is 124 beats/min. She has a fine tremor. She has no
goiter and no thyroid gland tenderness.

Laboratory test results:


TSH = <0.03 mIU/L (0.35-5.00 mIU/L)
Free T4 = 0.92 ng/dL (0.71-1.85 ng/dL) (SI: 11.8 pmol/L [9.1-23.8 pmol/L])
Total T3 = 776 ng/dL (84-179 ng/dL) (SI: 12.0 nmol/L [1.3-2.8 nmol/L])
Erythrocyte sedimentation rate = 46 mm/h (0-20 mm/h)

Thyroid iodine uptake on a nuclear scan is low at 5.7% at 22 hours.

Which of the following assessment(s) should be ordered to


determine whether she has subacute thyroiditis or exogenous
intoxication?
A. Complete blood cell count with differential
B. Thyroglobulin measurement and assessment of thyroglobulin
antibodies
C. Calcitonin measurement
D. Procalcitonin measurement
E. Thyroid ultrasonography
42 A 6-month-old infant is seen by his pediatrician for surgical
clearance for circumcision revision. He was born at 36 weeks’
gestation and was discharged home with his parents after birth. He
has since been lost to follow-up, but his mother reports exclusively
breastfeeding him until he was 4 months old. At that time, his
parents switched his diet to pureed table foods. They follow a vegan
lifestyle. His birth weight was 6 lb, 5.1 oz (2865 g), and his current
weight is 11 lb, 1.8 oz (5040 g).
On physical examination, he is alert but cachectic in appearance
with palpable rachitic rosary, widened wrists, and frontal bossing.
His blood pressure is normal, and he is breathing comfortably on
room air.

Laboratory test results:


Calcium = 5.1 mg/dL (8.0-10.5 mg/dL) (SI: 1.3 mmol/L [2.0-2.6 mmol/L])
Phosphate = 3.3 mg/dL (4.2-8.0 mg/dL) (SI: 1.1 mmol/L [1.4-2.6 mmol/L])
Magnesium = 1.7 mg/dL (1.7-2.5 mg/dL) (SI: 0.7 mmol/L [0.7-1.0 mmol/L])
Albumin = 3.9 g/dL (2.6-4.7 g/dL) (SI: 39 g/L [26-47 g/L])
Alkaline phosphatase = 434 U/L (124-383 U/L) (SI: 7.2 µkat/L [2.1-6.4 µkat/L])
25-Hydroxyvitamin D = <4.0 ng/mL (20.0-60.0 ng/mL) (SI: 10.0 nmol/L [49.9-149.8
nmol/L])
1,25-Dihydroxyvitamin D = 17.8 pg/mL (19.9-79.3 pg/mL) (SI: 46.3 pmol/L [51.7-
206.2 pmol/L])
PTH (1-84) = 312 pg/mL (15-87 pg/mL) (SI: 312 ng/L [15-87 ng/L])

Electrocardiography shows a borderline prolonged QTc interval of


430 ms.

In addition to inpatient admission for cardiac telemetry and


monitoring weight gain on formula feeds, which of the following
is the best next step in this patient’s management?
A. Vitamin D supplementation, 400 IU daily for 3 months, and
enteral calcium, at least 50 mg/kg per day
B. Calcitriol, 0.125 mcg twice daily; enteral calcium, at least 50
mg/kg per day; and enteral phosphorus, 30 mg/kg per day
C. Calcitriol, 0.125 mcg twice daily, and intravenous calcium
gluconate
D. Reintroduction of breastfeeding; maternal high-dosage
vitamin D supplementation; and enteral calcium, at least 50
mg/kg per day
E. Vitamin D supplementation, 2000 IU daily; enteral calcium, 50
mg/kg per day; and intravenous calcium gluconate as needed
43 Liraglutide injection for subcutaneous use was approved in
June 2019 for pediatric patients (10 years and older) with type 2
diabetes mellitus.

Which of the following is a “black box” warning for the product


that must be discussed with patients and families?
A. Pancreatitis: postmarketing reports, including fatal and
nonfatal hemorrhagic or necrotizing pancreatitis
B. Renal impairment: postmarketing reports, usually in
association with nausea, vomiting, diarrhea, or dehydration,
which may sometimes require hemodialysis
C. Thyroid C-cell tumors: contraindicated in patients with a
personal or family history of medullary thyroid cancer or in
patients with multiple endocrine neoplasia type 2
D. Serious hypoglycemia: risk of hypoglycemia is higher in
pediatric patients aged 10 years and older regardless of
concomitant antidiabetes therapies
E. Hypersensitivity: postmarketing reports of serious
hypersensitivity reactions (eg, anaphylactic reactions and
angioedema)
44 A 14-year-old boy with Klinefelter syndrome diagnosed by
amniocentesis presents for evaluation. He is a freshman in a
therapeutic day school. He has attention-deficit/hyperactivity
disorder and aggressive behaviors. He has regular follow-up visits
with his psychiatrist. He is not yet shaving.
On physical examination, his blood pressure is 115/74 mm Hg
and pulse rate is 74 beats/min. His height is 71 in (180.3 cm) (>95th
percentile), and weight is 194 lb (88.2 kg) (99th percentile) (BMI =
27.1 kg/m2). He is tall with long legs. He has cystic acne and a few
facial hairs. His genitals are Tanner stage 5, pubic hair is Tanner
stage 4, and testicular volume is 4 mL bilaterally.

Which of the following are the most likely expected laboratory


findings for this boy?
Answer LH FSH Testosterone Inhibin B
A. ↑ ↑ Normal or ↓ Normal
B. Normal ↑ Normal ↑
C. ↑ ↑ Normal or ↓ ↑
D. ↑ ↑ Normal or ↓ Undetectable
E. ↑ Normal ↓ Undetectable
45 A 15-year-old girl with a history of acute lymphocytic leukemia
treated with chemotherapy and bone marrow transplant (including
total body irradiation) has been diagnosed with stage I, multifocal,
tall variant papillary thyroid carcinoma. Both thyroid lobes are
involved, with the largest dimension of 3.5 cm, and there is
extrathyroidal extension into adipose tissue and soft-tissue margin,
as well as lymphatic, vascular, and perineural invasion. She has
metastasis to bilateral cervical lymph nodes in the central neck and
lateral neck, involving level VI and levels II and IV (17/32 lymph
nodes, size of metastasis >1 cm without extracapsular invasion) (T3,
N1b, MX). A 123I whole-body scan following levothyroxine
withdrawal and low-iodine diet demonstrates minimal thyroid bed
uptake with no evidence of distal metastases. Stimulated
thyroglobulin measurement is pending.

In addition to planning to optimize levothyroxine to maintain


TSH <0.1 mIU/L, which of the following is the best next step in
this patient’s management?
A. Reassure the family regarding 123I uptake and arrange for
follow-up in 6 months with neck ultrasonography and
nonstimulated thyroglobulin measurement
B. Reassure the family regarding 123I uptake and arrange for
follow-up in 12 months with neck ultrasonography and
nonstimulated thyroglobulin measurement
C. Reassure the family regarding 123I uptake and arrange for
follow-up with nonstimulated thyroglobulin measurement in 3
months
D. Treat with 131I 30 mCi (1110 MBq) and perform posttreatment
scan 4 to 7 days later
E. Treat with 131I 120 mCi (4440 MBq) and perform
posttreatment scan 4 to 7 days later
46 A 7-year-old girl is referred after a standard lipid assessment
documented the following results:
Total cholesterol = 279 mg/dL (<170 mg/dL) (SI: 7.23 mmol/L [<4.40 mmol/L])
HDL cholesterol = 51 mg/dL (30-65 mg/dL) (SI: 1.32 mmol/L [0.78-1.68 mmol/L])
LDL cholesterol = 190 mg/dL (<130 mg/dL) (SI: 4.92 mmol/L [<3.37 mmol/L])
Triglycerides = 190 mg/dL (30-140 mg/dL) (SI: 2.15 mmol/L [0.34-1.58 mmol/L])

The blood draw was performed after she had eaten breakfast. The
patient’s mother reports that they have reduced sugary drinks and
increased physical activity for the past 6 months because of her
daughter’s rising BMI. There is no family history of early
cardiovascular disease. The patient’s maternal grandmother has high
cholesterol that was diagnosed in her 50s and she is doing well on
medication. The patient’s father is in his 40s and has a normal
cholesterol level. The patient’s mother has not had her cholesterol
checked but plans to do so at her next check-up.

Which of the following is the best next step in this patient’s


management?
A. Redraw a fasting lipid panel in 3 months
B. Initiate the Cardiovascular Health Integrated Lifestyle Diet
(CHILD-2 diet)
C. Start a fibrate
D. Start a statin
E. Start fish oil
47 A 2-year-old boy with achondroplasia is referred to you by his
primary care physician at the request of his parents for possible
management of short stature. The patient was born via cesarean
delivery at 38 weeks’ gestation with a birth weight of 6 lb 0 oz (2730
g) and a birth length of 18.7 in (47.5 cm). The pregnancy was
uneventful. The patient’s father has achondroplasia and has the
typical features of this condition. His father’s height is 54.3 in (138
cm) (–5.26 SD). His mother is healthy, and her height is 59.8 in (152
cm) (–1.73 SD).
Physical examination reveals dysmorphic features consistent with
achondroplasia, including short stature, macrocephaly, prominent
forehead, midface hypoplasia, and short extremities. His height is
31.1 in (79 cm) (–2.35 SDS), and weight is 26.4 lb (12 kg) (–0.51 SDS).
His arm span is 28.7 in (73 cm), and head circumference is 20 in (50.8
cm) (+2.5 SDS).
Laboratory tests show normal levels of IGF-1 and IGFBP-3,
normal thyroid function, normal liver and kidney function, negative
celiac screen, and normal inflammatory markers.
The parents inquire about potential therapeutic interventions for
the management of their son’s short stature to avoid the father’s
final height outcome, and they are eager to participate in a clinical
study, if necessary.

Which of the following pharmacologic interventions appears


promising for this condition?
A. Recombinant human IGF-1
B. C-type natriuretic peptide analogue
C. GnRH analogues
D. Recombinant human GH
E. Aromatase inhibitors
48 A 6-and-6/12-year-old girl is being followed up in endocrine
clinic. She has a diagnosis of classic congenital adrenal hyperplasia
(CAH) secondary to 21-hydroxylase deficiency. She was born with
ambiguous genitalia and has a family history of CAH. Her parents
report that she is well with no episodes of illness necessitating
increasing her hydrocortisone dosage. The patient’s regimen consists
of hydrocortisone, 11.3 mg/m2 per day, and fludrocortisone, 100 mg
daily.
On physical examination, her height is 37.8 in (96.1 cm) (–1.46
SDS), and weight is 30 lb (13.6 kg) (–1.56 SDS) (BMI = 14.7 kg/m2 [–
0.77 SDS]). Body surface area is 0.62 m2. Her height velocity is 9.6
cm/y. Her blood pressure is 94/50 mm Hg.

Which of the following is the best next step in this patient’s


care?
A. Perform a pubertal examination
B. Determine bone age
C. Measure 17-hydroxyprogesterone
D. Increase the hydrocortisone dosage
E. Reduce the fludrocortisone dosage
49 An 11-and-3/12-year-old girl was diagnosed with Graves
disease 6 weeks ago. She had a moderately diffuse goiter, a fine
tremor, and the following laboratory test results:
TSH = <0.03 mIU/L (0.35-5.00 mIU/L)
Total T4 = 14.1 µg/dL (4.5-12.5 µg/dL) (SI: 181.5 nmol/L [57.9-160.9 nmol/L])
Total T3 = 345 ng/dL (84-179 ng/dL) (SI: 5.3 nmol/L [1.3-2.38 nmol/L])
Thyroid-stimulating immunoglobulin = 480% (<140%)

At diagnosis, her weight was 72.2 lb (32.8 kg). Methimazole, 7.5 mg


twice daily, and propranolol were prescribed. Propranolol was
stopped less than 3 weeks later because her pulse rate was 96
beats/min. Six weeks after starting methimazole, the patient comes
for a follow-up visit.
On physical examination today, her pulse rate is 84 beats/min.
Goiter is unchanged and there is no tremor.

Laboratory test results:


TSH = <0.03 mIU/L
Total T4 = 7.2 µg/dL (SI: 92.7 nmol/L)
Total T3, pending
Complete blood cell count with differential, normal
Liver enzymes, normal

Which of the following is the best next step in this patient’s


management?
A. Wait for the result of the T3 measurement before adjusting the
methimazole dosage
B. Ask the laboratory to assess T3 uptake on the sample from that
day
C. Reduce the methimazole dosage to 10 mg once daily
D. Measure thyroid-stimulating immunoglobulin again
E. Increase the methimazole dosage to 10 mg twice daily
50 A 16-and-8/12-year-old boy is referred to your clinic for
concerns of low bone density. He is an avid cross-country runner
and lacrosse player. Over the last 18 months, he has sustained 2
stress fractures and 1 growth plate fracture that have sidelined him
from participating in sports. His orthopedic surgeon ordered a DXA
scan and laboratory tests, which were done at an outside hospital.
Osteopenia was diagnosed with a lumbar spine Z-score of –1.7.
Serum 25-hydroxyvitamin D was in the insufficient range at 20.1
ng/mL (50.2 nmol/L), and alkaline phosphatase was reported as
high at 280 U/L (4.68 µkat/L). Calcium, phosphate, magnesium, and
PTH levels were normal.
He has no other history of fractures and no relevant medical
history. Family history is notable for postmenopausal osteoporosis in
his maternal grandmother. He follows a healthy diet with 3 servings
of dairy each day. His parents are concerned that he will continue to
be injured during physical activity and that his bone disease will
impair his ability to join the military upon high school graduation.
On physical examination, he is a well-appearing young man. His
height is at the 65th percentile and weight is at the 35th percentile.
BMI is 19.5 kg/m2. In the last year, he has grown 5 in (12.7 cm). His
pubertal examination shows 15-mL testes bilaterally.

Which of the following is the most appropriate recommendation


for this patient?
A. Refer to sports medicine for evaluation of running mechanics
B. Perform DXA of the total body and total hip to assess bone
mineral density at cortical and weight-bearing skeletal sites
C. Start vitamin D supplementation to address low vitamin D
and elevated alkaline phosphatase
D. Start prophylactic oral bisphosphonate therapy to increase
bone mineral density
E. Recommend avoidance of contact sports and suggest golf for
weight-bearing exercise instead of running
51 An 11-year-old boy presents for evaluation of breast
development. His parents report that for the past several months he
has had swelling under the nipples bilaterally without discharge.
This has caused him significant distress and he has started wearing
large sweatshirts to hide the appearance of breasts. He reports no
exposure to lavender or tea tree oil. He and his parents express a
strong desire for treatment.
On physical examination, the patient’s height is 60.6 in (154 cm)
(91st percentile) and weight is 103.6 lb (47.1 kg) (88th percentile). He
appears well. Examination findings are notable for 2 cm of glandular
breast tissue bilaterally, Tanner stage 2 pubic hair development, and
testicular volume of 6 mL bilaterally.

Laboratory test results:


LH = 1.7 mIU/mL (0.03-3.70 mIU/mL [Tanner stage 2]) (SI: 1.7 IU/L [0.03-3.70
IU/L])
Total testosterone (liquid chromatography–tandem mass spectrometry) = 58 ng/dL
(18-150 ng/dL [Tanner stage 2]) (SI: 2.01 nmol/L [0.63-5.20 nmol/L])
Estradiol (sensitive) = <2.5 pg/mL (5.0-16.0 pg/mL [Tanner stage 2]) (SI: <9.2
pmol/L [18.4-58.7 pmol/L])

Results from assessment of liver, kidney, and thyroid function are


normal. Bone age is 11 years. Midparental target height is 66 in (167.6
cm).

Which of the following is the best next step in the management


of this patient’s condition?
A. Order karyotype analysis
B. Order testing for serum tumor markers
C. Prescribe an aromatase inhibitor
D. Refer to surgery for mastectomy
E. Offer reassurance and follow-up in 6 months
52 A 1-year-old girl presents to clinic as a new patient regarding
concern for excess weight gain. Her mother states that the patient
was born full term after an uncomplicated pregnancy. Birth weight
was 6 lb 6 oz (3000 g). Review of her growth charts demonstrates
that her weight was already greater than the 97th percentile by 2
months of age and that it has continued to increase significantly over
time. Her length has plotted consistently just above the 97th
percentile since early infancy.
On physical examination, you observe a tall-for-age toddler with
no obvious dysmorphic features. According to her mother, she
always seems hungry and this has been true since she was a young
infant. Developmental milestones have all been met appropriately
thus far. She has generally been in good health with no history of
frequent infections. Her mother comments that she herself has
struggled with her weight all of her life and underwent sleeve
gastrectomy in the past. There is a strong family history of
significant obesity in multiple family members spanning
generations.

Given the clinical phenotype, genetic testing is being


considered. Which of the following is the most likely genetic
finding?
A. Pathogenic variant in the BBS1 gene
B. Pathogenic variant in the LEP gene
C. Deletion in 15q11.2-q13 region on the paternal chromosome
D. Pathogenic variant in the MC4R gene
E. Pathogenic variants in the ALMS1 gene
53 At 37 weeks’ gestation, a male baby is born to a 28-year-old
woman who had minimal prenatal care. The delivery is
uncomplicated, and the baby’s Apgar scores are 9 and 9. Birth
weight is 5 lb 3 oz (2360 g). Physical examination is significant for
bilateral cleft lip, partial cleft palate, and marked hypotelorism.
Brain MRI reveals lobar holoprosencephaly with absence of cleavage
of the inferior frontal lobe with characteristic fading away of the
genu of the corpus callosum. The baby is initially given intravenous
fluids on the first day of life, but he is subsequently transitioned to
enteral feedings by 4 days of age, which he is tolerating well. The
neonatal intensive care unit has asked for an endocrinology consult.

Which of the following is the most likely laboratory finding in


this newborn?
A. Low free T4
B. Elevated serum sodium
C. Low GH
D. Low glucose
E. Low serum sodium
54 A 16-year-old girl presents for outpatient evaluation of thyroid
nodules. Two weeks ago, she saw her primary care physician for
evaluation of 9 months of secondary amenorrhea and increasing
acne. Laboratory testing revealed an elevated serum testosterone
concentration (355 ng/dL [9-58 ng/dL] [SI: 12.3 nmol/L (0.3-2.0
nmol/L)]), and pelvic ultrasonography demonstrated a 5.9-cm solid
left ovarian mass. Preoperative staging CT demonstrated cysts in
both lungs (1 to 2 cm in diameter) and multiple hypoattenuating
lesions in the thyroid measuring up to 1.1 cm in diameter.
The patient was born preterm at 31 weeks’ gestation. She
underwent resection of a benign right lung cyst at age 22 months.
She has never received external irradiation or chemotherapy. Her
family history includes a maternal aunt diagnosed with breast
cancer in her 40s.
The patient reports no symptoms of hypothyroidism or
thyrotoxicosis, dysphagia, or voice changes. Her height is at the 11th
percentile, weight is at the 82nd percentile, BMI is at the 92nd
percentile (27 kg/m2), and head circumference is at the 88th
percentile. Vital signs are normal. Thyroid examination reveals a
mildly enlarged right lobe that is irregular in texture without
discrete nodules. There is no lymphadenopathy. No abnormal skin
pigmentation is present. The rest of the examination findings are
normal.
Her serum TSH concentration is 2.63 mIU/L (0.7-5.7 mIU/L).
Thyroid ultrasonography demonstrates multiple bilateral thyroid
nodules measuring 1.0 to 1.7 cm in diameter (see image).
In addition to appropriate further evaluation of the patient’s
thyroid nodules, germline genetic testing should be performed
for which of the following conditions?
A. Carney complex
B. DICER1 syndrome
C. Familial adenomatous polyposis
D. McCune-Albright syndrome
E. PTEN hamartoma tumor syndrome
55 A 4-year-old boy is diagnosed with GH deficiency via an
arginine-L-dopa stimulation test (peak GH = 0.8 ng/mL [0.8 µg/L]).
Recombinant human GH replacement therapy is started at a dosage
of 0.18 mg/kg per week. Several weeks after treatment initiation, the
child presents to the emergency department with hypotension and
hypoglycemia. In light of the recent initiation of recombinant human
GH therapy and based on current symptoms, the patient is
suspected to have adrenal insufficiency.

Which of the following mechanisms best explains the patient’s


presentation? GH-induced:
A. Inhibition of 21-hydroxylase
B. Stimulation of 11β-hydroxysteroid dehydrogenase 2
C. Inhibition of 11β-hydroxysteroid dehydrogenase 2
D. Stimulation of 11β-hydroxysteroid dehydrogenase 1
E. Inhibition of 11β-hydroxysteroid dehydrogenase 1
56 A postpubertal 15-year-old boy presents with a healing right
clavicle fracture diagnosed 4 weeks ago after he fell from standing
height and was treated appropriately by the orthopedic team. This is
his first-ever bone fracture.
Type 1 diabetes mellitus was diagnosed at age 2 and 9/12 years.
His diabetes is managed by multiple daily insulin injections. His
glycemic control is not optimal, and he has wide glycemic variation
with weekly episodes of hypoglycemia followed by significant
hyperglycemia. A hemoglobin A1c measurement 3 months ago was
10.6% (92 mmol/mol). His point-of-care hemoglobin A1c level today
is 10.8% (95 mmol/mol). His mother believes he had elevated blood
glucose values due to fracture-related pain.
He has celiac disease but is not adherent to a gluten-free diet. His
current height is 67 in (170.2 cm) with a growth velocity less than 0.5
in/y. He is shorter than expected considering the calculated
midparental height.
He has no family history of bone disease. Neither of his 2 older
siblings has a history of frequent bone fractures.

Which of the following is the best next step in the assessment


of this patient’s bone health?
A. Measurement of osteocalcin, bone-specific alkaline
phosphatase, and C-terminal telopeptide of type I collagen
B. Measurement of IGF-1, calcium, and 25-hydroxyvitamin D
C. DXA scan
D. Skeletal survey
E. No further study (unless he has a second fracture)
57 A pediatric gastroenterologist asks for an opinion regarding a
9-year-old girl with a history of ulcerative colitis, which was
diagnosed at age 8 years. Her condition has been refractory to
treatment, and she has required intermittent steroid therapy. She
underwent colectomy and ileostomy 2 months ago and was taking
prednisolone, 20 mg once daily, which has since been reduced to 10
mg daily. Her physician would like to stop prednisolone therapy and
is seeking an opinion.

Which of the following is the best course of action?


A. Stop treatment now
B. Measure 8-AM cortisol
C. Change to alternate day treatment, then stop in 2 weeks
D. Wean treatment and measure 8-AM serum cortisol
E. Wean treatment and perform a standard cosyntropin-
stimulation test
58 During a follow-up appointment for type 1 diabetes mellitus,
an 18-year-old girl reports she is 16 weeks pregnant. Prenatal
ultrasonography at 12-weeks’ gestation was normal. Results from a
first-trimester quadruple marker blood screen were normal. Cell-free
DNA screening was negative for trisomy 21, 18, and 13 but showed
monosomy X. She searched online and read about Turner syndrome.
She is considering the option of medical termination of pregnancy
and asks for an opinion.

In addition to encouraging the patient to have further


discussions with her obstetrician, which of the following is the
best next step?
A. Reassurance and follow-up in 3 months
B. Another cell-free DNA screen
C. Fetal karyotyping by amniocentesis
D. Fetal karyotyping by chorionic villus sampling
E. Referral for counseling about options regarding medical
termination of pregnancy
59 A 6-week-old male infant presents to his primary care
physician with recent-onset jaundice. On physical examination, he is
found to have hemangiomas on his skin and an enlarged liver.
Newborn screening results were normal, but his current TSH
concentration is 42 mIU/L (0.35-5.0 mIU/L). He is referred for
further evaluation.
On physical examination, his weight is 11 lb (5000 g) (50th
percentile) and length is 21.7 in (55 cm) (25th percentile). He has a
normal heart rate of 140 beats/min with low blood pressure for age
(76/33 mm Hg). He has mild jaundice. On skin examination, there is
a 3- to 4-second capillary refill and 3 hemangiomas on his trunk (the
largest is 2 cm). The liver is enlarged, 4 cm below the costal margin.
The spleen is not enlarged. Findings on heart examination are
remarkable for a 2-3/6 systolic murmur radiating to the axilla.
Abdominal ultrasonography confirms an enlarged liver. He has
multiple well-circumscribed lesions throughout the liver,
predominantly hypoechoic, measuring up to 3.2 cm in diameter.
Doppler ultrasonography reveals internal vascular flow. The lesions
are compatible with multiple hepatic hemangiomas.
Echocardiography documents an enlarged heart with systolic
dysfunction and mitral valve failure.
Another TSH measurement is 38.2 mIU/L (0.35-5.0 mIU/L).

Which of the following additional findings are likely in this


infant?

Answer Free T4 Total T3 Reverse T3 Heart failure


A. Normal Low High Low-output
B. Low Normal Low Low-output
C. Normal Low High High-output
D. Low High High Low-output
E. Low Normal Low High-output
60 A 9-year-old girl presents for a follow-up visit. She has a
history of craniopharyngioma, resected 1 year ago, that required
subsequent radiation treatment. Her grandparents, who are her legal
guardians, think that she has gained a lot of weight since her last
visit 6 months ago. They have made a number of lifestyle
modifications at home, including cutting back on portion sizes and
increasing her physical activity, but they do not think these measures
have impacted her weight gain. Review of her growth charts reveals
that she has gained 17.5 lb (8 kg) since the last clinic visit. In addition
to evaluating for pituitary deficiencies, further hormonal
assessments are ordered.

Which of the following patterns is characteristic of this patient’s


condition?
Answer Leptin Insulin Ghrelin
A. Increased leptin Increased insulin Suppressed ghrelin
B. Decreased leptin Insulin resistance Elevated ghrelin
C. Increased leptin Insulin resistance Suppressed ghrelin
D. Decreased leptin Insulin sensitive Elevated ghrelin
E. Increased leptin Increased insulin Elevated ghrelin
61 A 2-week-old newborn presents for follow-up to her
pediatrician. She was born full term to a G3 now P3 mother. Birth
weight was 7 lb 11 oz (3500 g). Upon discharge at 36 hours of life,
she weighed 7 lb 9 oz (3200 g). On examination today, her weight is 6
lb 9 oz (3000 g). The mother is breastfeeding and providing
supplemental formula (1 oz every 3 hours, waking the baby
overnight to feed). She has normal wet diapers, but bowel
movements occur only once every 3 days.
On physical examination, the baby appears lethargic but is
afebrile. She has no unusual facial features and no heart murmur. No
bony abnormalities or signs of subcutaneous fat necrosis are noted.

Pertinent laboratory test results:


Calcium = 16.5 mg/dL (7.4-11.0 mg/dL) (SI: 4.1 mmol/L [1.9-2.8 mmol/L])
Phosphate = 6.6 mg/dL (4.6-8.5 mg/dL) (SI: 2.1 mmol/L [1.5-2.7 mmol/L])
Magnesium = 3.0 mg/dL (1.6-2.6 mg/dL) (SI: 1.2 mmol/L [0.7-1.1 mmol/L])
25-Hydroxyvitamin D = 32.8 ng/dL (20.0-60.0 ng/dL) (SI: 81.9 nmol/L [49.9-149.8
nmol/L])
1,25-Dihydroxyvitamin D = 43.4 pg/mL (19.9-79.3 pg/mL) (SI: 112.8 pmol/L [51.7-
206.2 pmol/L])
Alkaline phosphatase = 161 U/L (90-273 U/L) (SI: 2.69 μkat/L [1.50-4.56 μkat/L])
PTH = 63 pg/mL (15-65 pg/mL) (SI: 63 ng/L [15-65 ng/L])

Family history is pertinent for mild hypercalcemia in the father, but


he has no history of kidney stones. A maternal great aunt had a
parathyroid adenoma removed at age 33 years. There is otherwise no
family history of endocrine disorders.

Which of the following is the most likely pathogenic variant


causing this infant’s hypercalcemia?
A. Activating variant in PTH1R
B. Inactivating variant in CASR
C. Inactivating variant in MEN1
D. Inactivating variant in CYP24A1
E. Deletion of the ELN gene on chromosome 7
62 A 14-and-10/12-year-old boy with Down syndrome (trisomy
21), obesity, and history of acute lymphoblastic leukemia diagnosed
at age 7 years is referred for evaluation. Results of annual thyroid
function testing were normal, and there is no history of goiter. He
has lost 4 lb (1.8 kg) in the past 2 months without trying to lose
weight. He has no nocturia or other signs of diabetes mellitus.

His primary care physician ordered repeated thyroid function tests:


TSH = 0.01 mIU/L (0.5-4.3 mIU/L)
Total T4 = 9.0 µg/dL (4.5-12.0 µg/dL) (SI: 115.8 nmol/L [57.9-154.4 nmol/L])

Leukemia has been in remission for more than 5 years. There is no


history of asthma. He does not have palpitations, heat intolerance, or
diarrhea, but he has been having sleep difficulties for 3 months,
waking up at 3 AM and being unable to fall back asleep.
On physical examination, his height is 61.3 in (155.7 cm) (92nd
percentile based on Down syndrome charts) and weight is 155.8 lb
(70.7 kg) (85th percentile based on Down syndrome charts) (BMI =
29.1 kg/m2 [97th percentile]). His blood pressure is 123/73 mm Hg,
and pulse rate is 80 beats/min. He has typical Down syndrome
facies. There is no lymphadenopathy. He has mild proptosis and no
lid lag. His neck is short, and there is no goiter and no bruit. He has
normal patellar reflexes and no tremor.

Laboratory tests repeated 1 month after the initial low TSH


measurement demonstrate he is mildly hyperthyroid, and his
antibodies confirm Graves disease.
TSH = <0.01 mIU/L (0.5-4.30 mIU/L)
Total T4 = 11.5 µg/dL (4.5-12.0 µg/dL) (SI: 148.0 nmol/L [57.9-154.4 nmol/L])
Free T4 = 2.2 ng/dL (0.9-1.4 ng/dL) (SI: 28.3 pmol/L [11.6-18.9 pmol/L])
Total T3 = 244 ng/dL (84-179 ng/dL) (SI: 3.8 nmol/L [1.3-2.8 nmol/L])
TPO antibodies = 441 IU/mL (<9 IU/mL) (SI: 441 kIU/L [<9 kIU/L])
Thyroid-stimulating immunoglobulin = 361% (<140%)
Which of the following is the best next step in this child’s
management?
A. Perform total thyroidectomy
B. Start oral propranolol
C. Start oral methimazole
D. Observe and repeat thyroid function tests in 1 month
E. Perform radioactive iodine (131I) ablation
63 A 6-and-6/12-year-old girl of African American descent is
evaluated in the pediatric endocrinology clinic because of a 12-
month history of body odor and acne. Over the last 6 months, her
mother has noticed that she has developed pubic hair and had a
growth spurt. She is the child of nonconsanguineous parents. She
was born full term with a birth weight of 7 lb 13 oz (3550 g). She has
been well with no hospital admissions. Her current height is at the
75th percentile. At her last check-up, her height was at the 50th
percentile. Her bone age is advanced by 1.8 years compared with her
chronologic age. There is no family history of early puberty. Her
mother had menarche at age 10 years. Her 4-year-old brother is
prepubertal.
On physical examination, breast development is Tanner stage 1,
pubic hair is Tanner stage 2, and she has axillary hair development.

Which of the following diagnostic investigations should be


ordered next?
A. Adrenal ultrasonography
B. LH measurement
C. Baseline 17-hydroxyprogesterone measurement
D. TSH measurement
E. Urinary steroid profile
64 A 17-year-old girl is referred for evaluation of secondary
amenorrhea. Breast development started at age 10 years, and
menarche occurred at age 12 years. She had menses monthly until
approximately 8 months ago at which time menstrual periods
ceased. She states that pregnancy is not a possibility and that she has
otherwise been well. She has had no recent weight loss, vision
problems, headaches, or breast discharge and takes no medications
on a daily basis. Midparental target height is 65.5 in (166.4 cm).
On physical examination, her height is 64.2 in (163.1 cm) (50th
percentile), weight is 142.6 lb (64.8 kg) (80th percentile) (BMI = 24.3
kg/m2 [80th percentile]). She appears well. Examination findings are
notable for Tanner stage 5 breast development with no nipple
discharge. She does not have hirsutism. Peripheral vision is intact.
Laboratory test results:
Qualitative hCG, negative
TSH = 1.16 mIU/L (0.45-4.50 mIU/L)
Free T4 = 1.1 ng/dL (0.8-1.8 ng/dL) (SI: 14.2 pmol/L [10.3-23.2 pmol/L])
LH = 1.2 mIU/mL (0.4-11.7 mIU/mL) (SI: 1.2 IU/L [0.4-11.7 IU/L])
FSH = 7.8 mIU/mL (0.8-8.5 mIU/mL) (SI: 7.8 IU/L [0.8-8.5 IU/L])
Estradiol = 12.2 pg/mL (12.5-211 pg/mL) (SI: 44.9 pmol/L [45.9-774.6 pmol/L])
Prolactin = 78.2 ng/mL (3.2-20.0 ng/mL) (SI: 3.4 nmol/L [0.14-0.87 nmol/L])

On brain MRI, her pituitary gland measures 5.2 mm with a normal


neurohypophysis and infundibulum and no suprasellar mass. There
is a 0.3 × 0.4 × 0.3-cm area of hypoenhancement in the left side of the
pituitary gland.

Which of the following is recommended for this patient’s


management?
A. Dynamic testing of prolactin secretion to confirm the diagnosis
B. Neuro-ophthalmology examination for visual field deficits
C. Monitoring with follow-up prolactin measurement and brain
MRI in 3 to 6 months
D. Initiation of once-daily bromocriptine and monitoring with
annual echocardiography
E. Initiation of twice-weekly cabergoline and counseling for
psychiatric disturbance
65 An 11-month-old boy is admitted to the hospital for failure to
thrive. His mother had gestational diabetes mellitus that was well
controlled with dietary modifications. Prenatal course was normal.
His birth weight and birth length were appropriate for age. He had
normal weight gain and growth until 4 months of age, after which
his weight gain declined drastically and growth slowed slightly. He
has had intermittent vomiting for the last 1 month. He is otherwise a
healthy child with normal development, appetite, and activity.
Review of his growth chart documents length at the 11th percentile
(–1.17 SDS) and weight at less than the 1st percentile (–3.89 SDS)
(weight for length, –3.95 SDS). Head circumference is at 1.9 SDS.
On physical examination, he has a very thin build with a
prominent forehead and very little subcutaneous fat on his chest and
extremities. He is happy and interactive. The rest of the examination
findings, including those from an ophthalmologic examination, are
normal.

Laboratory test results:


Complete blood cell count, normal
Basic metabolic panel, normal
Liver function, normal
Free T4, normal
TSH, normal
Random GH = 12.7 ng/mL (SI: 12.7 µg/L)
IGF-1 = 110 ng/mL (17-248 ng/mL) (SI: 14.4 nmol/L [2.2-32.5 nmol/L])

Which of the following should be recommended as the best


next step to confirm this patient’s diagnosis?
A. Brain MRI
B. Cortisol level and cosyntropin-stimulation test
C. Skeletal survey
D. Bone age x-ray
E. Refer to gastroenterology
66 A 6-week-old boy presents for outpatient evaluation of poor
feeding and abdominal distention over the preceding 7 days. He was
born full term following an uncomplicated singleton pregnancy. He
was of normal weight and length at birth. He had mild neonatal
jaundice that resolved without therapy. Results of newborn
screening (sample obtained at 60 hours of life) were normal. The
state newborn thyroid screen is TSH-based. Before the onset of
symptoms, he had been healthy since birth and had been
breastfeeding and gaining weight normally. His mother has no
history of thyroid disease.
On physical examination, his length is at the 3rd percentile,
weight is at the 10th percentile, weight for length is at the 67th
percentile, and head circumference is at the 24th percentile. Pulse
rate is 130 beats/min, respiratory rate is 24 breaths/min, and blood
pressure is 92/55 mm Hg. The infant is sleeping comfortably and
wakes easily. Fontanelles are open, soft, and normal in size. The
thyroid gland is not palpable. Findings on pulmonary examination
are normal, and there is a 2/6 systolic murmur. The abdomen is
distended and nontender, and a firm liver edge is palpable 5 cm
below the costal margin. The spleen is not palpable. A 5-mm
cutaneous hemangioma is present on the left chest. The rest of the
examination findings are normal.

Laboratory test results:


TSH = 94.5 mIU/L (0.7-5.7 mIU/L)
Free T4 = 1.34 ng/dL (0.9-2.3 ng/dL) (SI: 17.2 pmol/L [11.6-29.6 pmol/L])

Which of the following additional laboratory findings is most


likely to be present?
A. Elevated serum reverse T3
B. Low serum thyroglobulin
C. Elevated serum T3
D. Elevated serum TSH-receptor antibodies
E. Elevated urinary iodine
67 A 2-year-old boy with recently diagnosed acute lymphoblastic
leukemia is hospitalized for fever with neutropenia and is found to
have Escherichia coli bacteremia. Endocrinology is consulted for
persistent hyponatremia. He is receiving tobramycin and
piperacillin/tazobactam for the bacteremia and intravenous fluids
(0.9% normal saline plus KCl [20 mmol/L]) at a maintenance rate. In
addition to dexamethasone, he is currently on the following
chemotherapy medications: daunorubicin, methotrexate, and
vincristine.
On physical examination, his blood pressure is 90/52 mm Hg,
pulse rate is 128 beats/min, respiratory rate is 24 breaths/min, and
temperature is 98.6°F (37°C). His height is 36.5 in (92.7 cm), and
weight is 29.7 lb (13.5 kg) (BMI = 15.78 kg/m²). He has moist mucous
membranes. There is no thyromegaly. Findings on cardiac
examination are normal. He is fussy, but his neurologic examination
findings are otherwise normal. His extremities are well perfused.
There is no edema.

Laboratory test results:


Sodium = 124 mEq/L (136-149 mEq/L) (SI: 124 mmol/L [136-149 mmol/L])
Potassium = 3.9 mEq/L (3.9-5.7 mEq/L) (SI: 3.9 mmol/L [3.9-5.7 mmol/L])
Chloride = 90 mEq/L (98-108 mEq/L) (SI: 90 mmol/L [98-108 mmol/L])
Carbon dioxide = 19.3 mEq/L (23.0-30.0 mEq/L) (SI: 19.3 mmol/L [23.0-30.0
mmol/L])
Glucose = 120 mg/dL (70-110 mg/dL) (SI: 6.7 mmol/L [3.9-6.1 mmol/L])
Serum urea nitrogen = 17 mg/dL (7-18 mg/dL) (SI: 6.1 mmol/L [2.5-6.4 mmol/L])
Creatinine = 0.16 mg/dL (0.25-0.74 mg/dL) (SI: 14.1 µmol/L [22.1-64.4 µmol/L])
Uric acid = 1.6 mg/dL (2.0-7.0 mg/dL) (SI: 95.2 µmol/L [119.0-416.4 µmol/L])
Triglycerides = 92 mg/dL (30-74 mg/dL) (SI: 1.04 mmol/L [0.34-0.74 mmol/L])
Free T4 = 0.96 ng/dL (0.96-1.77 ng/dL) (SI: 12.4 pmol/L [12.4-22.8 pmol/L])
TSH = 0.9 mIU/L (0.70-5.97 mIU/L)
Serum cortisol (8 AM) = 0.3 µg/dL (2.3-15.0 µg/dL) (SI: 8.3 nmol/L [63.5-413.8
nmol/L])
Plasma osmolality = 264 mOsm/kg (285-295 mOsm/kg) (SI: 264 mmol/kg [285-295
mmol/kg])
Urine osmolality = 554 mOsm/kg (50-1400 mOsm/kg) (SI: 554 mmol/kg [50-1400
mmol/kg])
Urinary sodium (random) = 83 mEq/L (SI: 83 mmol/L)
Which of the following is the most likely primary cause of this
patient’s hyponatremia?
A. Hypertriglyceridemia
B. Cerebral/renal salt wasting
C. Excessive fluid administration
D. Syndrome of inappropriate antidiuresis
E. Cortisol insufficiency
68 A 9-and-10/12-year-old girl with Turner syndrome is being
seen for follow-up evaluation of growth. Turner syndrome was
diagnosed at age 15 months, and karyotype analysis documented a
45,X karyotype with no mosaicism. She has been receiving
recombinant human GH since age 4 years after slowed growth was
documented. Her response to GH therapy had been good (see growth
chart): the red lines are the 5th, 50th, and 95th percentiles of the
Centers for Disease Control chart for normal girls, and the green
lines are the 5th, 10th, 25th, 50th, 75th, 90th, and 95th percentiles for
girls with untreated Turner syndrome). Over the last 1.5 to 2 years,
her linear growth has decelerated. Her growth velocity over the last
10 months was poor at 3.4 cm/y.
On physical examination, there is no evidence of pubertal
development with Tanner stage 1 breast development and pubic
hair. At previous appointments, her IGF-1 values have been in the
upper reference range or slightly above. For this reason, she has been
maintained on a relatively low GH dosage. Her current dosage is 0.9
mg subcutaneously 7 days a week, which translates into a total dose
of 0.19 mg/kg per week. A bone age obtained at this visit is
interpreted to be 8 years, 10 months at a chronologic age of 9 years,
10 months. Her IGF-1 concentration is 476 ng/mL (62.4 nmol/L) (Z-
score, +2.1).
Midparental target height is 61.8 in (157 cm), which falls between
the 10th and 25th percentile for women in the standard Centers for
Disease Control growth charts.
Which of the following is the most appropriate intervention to
improve final height outcome?
A. Increase the GH dosage to 0.35 mg/kg per week
B. Start oral estradiol and maintain the current GH dosage
C. Start oxandrolone at a dosage of 0.03 mg/kg daily and
maintain the current GH dosage
D. Start an aromatase inhibitor (anastrozole) at a dosage of 1 mg
orally daily and maintain the current GH dosage
E. Continue the current GH dosage and consider starting
estradiol at age 14 to 16 years
69 A 13-year-old boy presents for evaluation of obesity. His BMI
has consistently plotted at greater than the 95th percentile for weight
since he was a young child. His family is very concerned about his
risk for medical problems due to obesity. His pediatrician has
screened him for glucose abnormalities and also ordered a fasting
lipid panel, all of which had normal results. The family asks about
screening him for liver problems because his mother, who also has
obesity, states that she has “fatty liver.”

Which of the following is the best next step in this patient’s


evaluation?
A. Abdominal ultrasonography
B. Measurement of AST
C. Measurement of ALT
D. Referral to gastroenterology
E. Abdominal MRI
70 A 10-year-old girl was recently diagnosed with high-risk acute
lymphocytic leukemia. Part of her induction chemotherapy included
high-dosage prednisone, 20 mg twice daily, for the last month. She
now frequently experiences back pain, but her parents cannot
identify a preceding traumatic event. Since being diagnosed with
acute lymphocytic leukemia, she has gained 6.6 lb (3 kg). She has not
yet started puberty.

Which of the following is the best next step regarding the


assessment and treatment of this patient’s vertebral
compression fractures?
A. Strict bed rest until pain resolves
B. Lumbar spine DXA; if results are normal, no further imaging is
indicated
C. Lumbar spine DXA and vertebral fracture analysis by DXA
D. Lumbar spine DXA and dedicated spine x-rays
E. Bisphosphonate therapy for treatment of vertebral
compression fractures
71 A 16-year-old girl has been aware of a lump in the left side of
her neck for the past few months. At her annual well-child visit, her
pediatrician orders ultrasonography, which shows a thyroid nodule
in the left lobe measuring 5 cm with features suspicious for thyroid
cancer. It is solid and hypoechoic and has a few microcalcifications.
There is no evidence of the nodule invading the capsule and no
cervical lymphadenopathy. Thyroid cancer is suspected, and she is
referred to pediatric endocrinology for ultrasound-guided FNA and
laboratory tests.
The FNA is interpreted to be papillary thyroid cancer. Given the
large nodule, neck CT is performed before total thyroidectomy, and
it shows no cervical lymph nodes. Her preoperative thyroid function
is normal and TPO and thyroglobulin antibodies are negative. Total
thyroidectomy is uneventful. No neck dissection is performed based
on her CT and ultrasonography staging. Pathologic examination
confirms the diagnosis of papillary thyroid cancer in the left lobe,
and no evidence of thyroid cancer is noted in the right lobe. The
tumor does not invade the capsule and there is no evidence of
lymphovascular invasion.
Levothyroxine, 150 mcg once daily, is initiated and the patient
returns for her first visit 1 month after surgery.

On the basis of the 2015 Pediatric Guidelines for Children with


Thyroid Nodules and Differentiated Thyroid Cancer, which of the
following is the best strategy?
A. Recommend 131I radioablation in 1 month; measure
thyroglobulin 2 months later while on levothyroxine
B. Assess thyroid function now and aim for a TSH concentration
<0.1 mIU/L; measure thyroglobulin in 2 to 3 months while on
levothyroxine
C. Assess thyroid function now and aim for a TSH concentration
0.1-0.5 mIU/L; measure thyroglobulin in 2 to 3 months while
on levothyroxine
D. Assess thyroid function now and aim for a TSH concentration
0.5-1.0 mIU/L; measure thyroglobulin in 2 to 3 months while
on levothyroxine
E. Stop levothyroxine; order a stimulated thyroglobulin
measurement and whole-body scan to be done 2 weeks later
when her TSH concentration is >30 mIU/L
72 A 3-year-old boy presents with lower-extremity bowing. He
was born at 28 weeks’ gestation, and the neonatal course was
complicated by prolonged use of total parenteral nutrition due to
necrotizing enterocolitis. He has been on full enteral feeds since
approximately 4 months of age and now consumes 24 oz of milk
daily. He also takes a chewable multivitamin.
He did not start walking until 18 months of age, which the family
attributes to his prematurity. His parents initially noted the bowing
deformity at that time and report that it appears to have worsened
over the last year. The child is now in preschool and his parents note
that compared with other children in his class, he seems to rest more
often on the playground. Growth parameters, adjusted for
prematurity, have been normal. Dentition is normal. Family history
is relevant for recurrent stress fractures in the mother who is a
marathon runner.

Laboratory test results:


Calcium = 9.8 mg/dL (8.3-10.6 mg/dL) (SI: 2.5 mmol/L [2.1-2.7 mmol/L])
Phosphate = 7.7 mg/dL (4.0-6.8 mg/dL) (SI: 2.5 mmol/L [1.3-2.2 mmol/L])
25-Hydroxyvitamin D = 73 ng/dL (20-60 ng/dL) (SI: 182.2 nmol/L [49.9-149.8
nmol/L])
Alkaline phosphatase = 76 U/L (78-300 U/L) (SI: 1.27 μkat/L [1.22-5.01 μkat/L])
PTH = 16 pg/mL (15-87 pg/mL) (SI: 16 ng/L [15-87 ng/L])

Which of the following is this child’s most likely diagnosis?


A. Vitamin D intoxication
B. Hypoparathyroidism
C. Physiologic bowing
D. Hypophosphatasia
E. Rickets of prematurity
73 A 15-year-old boy is referred for evaluation of pubertal delay.
His history is notable for cryptorchidism and orchiopexy at age 11
years. The only signs of puberty have been recent onset of pubic hair
growth. He was followed by his pediatrician from birth until 2 years
of age for poor growth, but he has grown well since then. He has no
vision problems, headaches, or fatigue, but he does report a poor
sense of smell.
His mother underwent menarche at age 13 years, and the timing
of puberty in his father is unknown. His 18-year-old brother started
puberty at 13 to 14 years of age. His midparental target height is 69.5
in (176.5 cm).
On physical examination, his height is 63.2 in (160.5 cm) (5th
percentile) and weight is 117 lb (53.2 kg) (22nd percentile). He
appears well. Pubic hair is Tanner stage 3 and testicular volume is 2
mL bilaterally.

Which of the following sets of laboratory test results would be


most likely in this patient?
Answer Inhibin B Antimullerian hormone FSH
A. 17.7 pg/mL (SI: 17.7 41 ng/mL (SI: 293 pmol/L) 0.33 mIU/mL (SI: 0.33 IU/L)
ng/L)
B. 186 pg/mL (SI: 186 ng/L) 114 ng/mL (SI: 814 pmol/L) 1.2 mIU/mL (SI: 1.2 IU/L)
C. 186 pg/mL (SI: 186 ng/L) 114 pg/mL (SI: 814 pmol/L) 34.6 mIU/mL (SI: 34.6 IU/L)
D. 17.7 pg/mL (SI: 17.7 41 ng/mL (SI: 293 pmol/L) 34.6 mIU/mL (SI: 34.6 IU/L)
ng/L)
E. 17.7 pg/mL (SI: 17.7 96 ng/mL (SI: 686 pmol/L) 0.33 mIU/mL (SI: 0.33 IU/L)
ng/L)

Reference Ranges:
Inhibin B Antimullerian hormone
21-166 pg/mL (SI: 21-166 14.0-466.0 ng/mL (SI: 100-3329
5-9.9 years <24 months
ng/L) pmol/L)
10-13.9 41-328 pg/mL (SI: 41-328 24 months-12 7.4-243.0 ng/mL (SI: 53-1736
years ng/L) years pmol/L)
14-17.9 54-295 pg/mL (SI: 54-295>12 years 0.7-19.0 ng/mL (SI: 5-136 pmol/L)
years ng/L)
47-308 pg/mL (SI: 47-308
≥18 years
ng/L)

Source: Quest Diagnostics Source: Mayo Clinic Laboratories


74 A 6-and-2/12-year-old boy with Prader-Willi syndrome
presents to the clinic for follow-up. He was born at full term. Birth
weight and length were adequate for gestational age. He was noted
to have significant hypotonia at birth and had dysmorphic features,
including narrow biparietal diameter and undescended testes. He
had issues with feeding and required a nasogastric tube and, later, a
gastrostomy tube. The suspicion of Prader-Willi syndrome was
confirmed with fluorescence in situ hybridization analysis.
At age 14 months, his length was below the 3rd percentile for age
(SDS, –2.2) and weight was at the 4th percentile for age (SDS, –1.75).
Given the reported benefits of GH therapy started early in children
with Prader-Willi syndrome, he underwent a sleep study to rule out
significant obstructive sleep apnea. The sleep study showed mild
obstructive sleep apnea, as well as a component of central apnea. GH
therapy was started at age 16 months when his length was 28.9 in
(73.4 cm) (<3rd percentile; SDS, –2.18), weight was 20.9 lb (9.5 kg)
(5th percentile; SDS, –1.64), and body surface area was 0.422 m2. The
initial GH dosage was 0.2 mg subcutaneously once daily (0.47
mg/m2 daily or 0.15 mg/kg per week). Close surveillance with
overnight pulse oximetry was recommended.
Now, at age 6-and-2/12 years, his height is 42.9 in (109 cm) (7th
percentile; SDS, –1.46), weight is 44.7 lb (20.3 kg) (39th percentile;
SDS, –0.27), and BMI is 17.1 kg/m2 (85th percentile; SDS, +1.05). His
body surface area is 0.775 m². He grew 1.5 in (3.8 cm) in the last 6
months, rendering a growth velocity of 7.6 cm/y. He has been doing
well with the GH injections without evidence of adverse effects or
worrisome signs or symptoms such as polyuria, polydipsia,
nocturia, enuresis, headaches, or vision changes. His mother reports
that since the initiation of GH therapy, the frequency of oxygen
desaturations has decreased. She has also noticed continued
improvement in his muscle tone, a finding that was corroborated in
a recent follow-up appointment in neurology. His GH dosage is 0.7
mg subcutaneously once daily (0.24 mg/kg per week).

Laboratory test results:


IGFBP-3 = 3.7 mg/L (1.5-3.4 mg/L)
IGF-1 = 215 ng/mL (60-228 ng/mL) (SI: 28.2 nmol/L [7.86-29.9 nmol/L])
IGF-1 Z-score = +2.49
Random glucose = 128 mg/dL (70-100 mg/dL) (SI: 7.1 mmol/L [3.9-5.56 mmol/L])
Hemoglobin A1c = 5.5% (4.3%-5.7%) (SI: 37 mmol/mol [23-39 mmol/mol])
TSH = 6.2 mIU/L (0.8-8.2 mIU/L)
Free T4 = 1.35 ng/dL (0.8-1.8 ng/dL) (SI: 17.4 pmol/L [10.3-23.2 pmol/L])
25-Hydroxyvitamin D = 30 ng/mL (25-100 ng/mL) (SI: 74.9 nmol/L [62.4-249.6
nmol/L])

On the basis of these results and clinical guidelines, which of


the following is the best next step in this patient’s
management?
A. Continue the current dosage of GH
B. Start thyroid hormone replacement therapy
C. Decrease the GH dosage
D. Perform an oral glucose tolerance test
E. Start vitamin D therapy
75 A 15-year-old boy has recently been diagnosed with a
pheochromocytoma. The 24-hour urine metanephrine results at the
time of diagnosis were as follows:
Urine volume = 961 mL (0-4440 mL)
Urinary normetanephrine = 1765 μg/24 h (0-366 μg/24 h) (SI: 9637 nmol/d [0-2000
nmol/d])
Urinary metanephrine = 114 μg/24 h (0-493 μg/24 h) (SI: 622 nmol/d [0-2692
nmol/d])

He undergoes unilateral adrenalectomy and has genetic testing.

A pathogenic variant in which of the following genes is most


likely?
A. RET (ret proto-oncogene)
B. VHL (von Hippel-Lindau tumor suppressor)
C. TMEM127 (transmembrane protein 127)
D. NF1 (neurofibromin 1)
E. SDHC (succinate dehydrogenase complex subunit C)
76 A postpubertal 15-year-old boy is evaluated for suboptimally
controlled type 1 diabetes mellitus diagnosed 7 years ago. He is
treated with multiple daily insulin injections with long- and rapid-
acting insulin pens.
His last 3 hemoglobin A1c measurements have fluctuated
between 8.2% and 9.7% (66-83 mmol/mol). He currently uses a
glucose meter and does not want to switch to a continuous glucose
monitoring device because he thinks it would “annoy” him. His
blood glucose values vary widely from 54 mg/dL (3.0 mmol/L) to
greater than 500 mg/dL (>27.8 mmol/L). His current 30-day glucose
average is 229 mg/dL (12.7 mmol/L) with 1 standard deviation of
102 mg/dL (5.7 mmol/L).
He describes normal daily function without any symptoms. He is
an avid soccer player and is physically active. He does not report
any sleep disruption and has no nocturia. He is dismissive when
asked about his vision.
His most recent set of laboratory results reveal normal thyroid
function, undetectable tissue transglutaminase antibodies, elevated
total cholesterol and triglycerides, normal LDL cholesterol, and
normal-range urine albumin.

Which of the following subspecialists should this patient be


referred to first to address concerns of microvascular
complications?
A. Cardiologist
B. Nephrologist
C. Neurologist
D. Ophthalmologist
E. Podiatrist
77 A 7-year-old girl is referred for evaluation of short stature. She
was born at term but was small-for-gestational-age. She has a history
of microcephaly, midface hypoplasia, hypoplastic optic nerves, mega
cisterna magna, autism spectrum disorder, and mild global
developmental delay. She has normal appetite, thirst, bowel
movements, and urination. Whole-exome sequencing done at 6
months of age (ordered by a geneticist) identified a heterozygous
pathogenic variant in the IGF1R gene (Arg739Trp). She has not had
any hospital admissions since birth. Review of her growth chart
shows that since infancy, her length and height have been at less
than the first percentile and her weight has been at the third
percentile.

Laboratory test results:


Erythrocyte sedimentation rate = 10 mm/h (0-32 mm/h)
IGF-1 = 278 ng/mL (49-267 ng/mL) (SI: 36.4 nmol/L [6.4-35.0 nmol/L])
TSH = 1.37 mIU/L (0.5-5.0 mIU/L)
Free T4 = 1.46 ng/dL (0.90-1.67 ng/dL) (SI: 18.8 pmol/L [11.6-21.5 pmol/L])
Tissue transglutaminase antibodies IgA = <2 U/mL (0-3 U/mL)
IgA = 157 mg/dL (51-220 mg/dL)

Bone age is reported to be 8 years and 10 months at the chronologic


age of 7 years and 5 months.

A problem with the function of which of the following proteins


best explains the underlying pathology of this patient’s poor
growth?
A. Cytokine receptor that activates the JAK-STAT signaling
pathway
B. Cell-surface receptor with intrinsic tyrosine kinase activity
C. Nuclear receptor that regulates gene transcription
D. Cytosolic receptor that regulates gene transcription
E. Transmembrane G-protein–coupled receptor
78 A 5-week-old girl presents for outpatient evaluation of jaundice
and decreasing oral intake over the preceding 4 days. The pregnancy
was uncomplicated until the third trimester, when intrauterine
growth restriction was observed. Delivery was induced at 38 weeks’
gestation and was uncomplicated, and birth weight was 5 lb 8 oz
(2500 g) (11th percentile). Her postnatal course was unremarkable,
and results of newborn screening were normal (sample collected at
50 hours of life). She is exclusively breastfeeding and her weight has
been increasing by 30 g daily. Her mother has no history of thyroid
disease, takes a prenatal vitamin, and consumes 2 daily servings of a
traditional Korean soup to stimulate lactation.
On physical examination, the infant’s length is at the 10th
percentile, weight is at the 15th percentile, and head circumference is
at the 18th percentile. Her pulse rate is 120 beats/min, respiratory
rate is 22 breaths/min, and blood pressure is 84/52 mm Hg. The
infant appears calm, comfortable, and sleepy. She has jaundice and
scleral icterus. Fontanelles are open, soft, and normal in size. The
thyroid gland is not palpable. The abdomen is normal with no
organomegaly. There are no skin lesions. The rest of the examination
findings are normal.

Laboratory test results:


Total bilirubin = 7.2 mg/dL (0-7.0 mg/dL) (SI: 123.1 µmol/L [0-119.7 µmol/L])
Direct bilirubin = 0.6 mg/dL (0-0.6 mg/dL) (SI: 10.3 µmol/L [0-10.3 µmol/L])
Hematocrit = 36.2% (34.1%-41.8%) (SI: 0.362 [0.341-0.418])
TSH = 108 mIU/L (1.7-9.1 mIU/L)
Free T4 = 0.39 ng/dL (0.9-2.3 ng/dL) (SI: 5.0 pmol/L [11.6-29.6 pmol/L])

Ultrasonography reveals a thyroid gland that is normal in size and


appearance.

Measurement of which of the following is most likely to confirm


the etiology of this patient’s hypothyroidism?
A. Serum reverse T3
B. Serum TPO antibodies
C. Serum T3
D. Serum TSH-receptor antibodies
E. Urinary iodine
79 A 13-year-old girl with autoimmune polyglandular syndrome
type 1 presents for routine follow-up. Her current diagnoses include
hypoparathyroidism and malabsorption secondary to chronic
diarrhea. She had the onset of thelarche approximately 3 years ago
but has not yet had menses. Approximately 1 year ago, she required
significant alteration in management of hypocalcemia with
increasing dosages of calcium and calcitriol supplementation.
Because of her malabsorption, she was eventually transitioned to
recombinant PTH given once every evening with calcium carbonate
supplementation continued once daily in the early afternoon. With
this approach, her serum calcium levels have been stable.
On physical examination, the patient is thin. She has lost 6.6 lb (3
kg) since her last visit 6 months ago. She appears mildly dehydrated
and reports fatigue, nausea, and vomiting. She has Tanner stage 4
breast development. Her pulse rate is 88 beats/min, blood pressure
is 92/56 mm Hg, and respiratory rate is 16 breaths/min.

Current laboratory test results (8-AM specimen draw):


Calcium = 10.6 mg/dL (8.4-10.2 mg/dL) (SI: 2.7 mmol/L [2.1-2.6 mmol/L])
Phosphate = 4.8 mg/dL (3.7-6.5 mg/dL) (SI: 1.6 mmol/L [1.2-2.1 mmol/L])
Magnesium = 1.8 mg/dL (1.7-2.4 mg/dL) (SI: 0.7 mmol/L [0.7-1.0 mmol/L])
Albumin = 3.7 g/dL (3.3-4.8 g/dL) (SI: 37 g/L [33-48 g/L])
PTH (1-84) = <3 pg/mL (15-87 pg/mL) (SI: <3 ng/L [15-87 ng/L])
Urinary calcium-to-creatinine ratio = 0.14 (<0.2)

Which of the following is the most important next step in this


patient’s management?
A. Screen for adrenal insufficiency
B. Screen for primary ovarian insufficiency
C. Screen for thyroid disease
D. Reduce the dosage of recombinant PTH
E. Discontinue calcium carbonate supplementation
80 A family has recently moved to the United States. Their now
20-month-old son received the diagnosis of Prader-Willi syndrome
(PWS) several months before their move to the United States, and
they would like to establish care. The parents express frustration that
the diagnosis was delayed since their son exhibited features soon
after birth that they now recognize as being common in infants with
this condition. They have been trying to learn more about PWS to
ensure the best care. They are very worried about possible
comorbidities associated with PWS, particularly the risks for
overweight/obesity, and would like to know what they might expect
in the upcoming months.

Which of the following is the most accurate description of what


to expect regarding their child’s weight status over the next 9
months?
A. Hyperphagia and lack of satiety leading to rapid weight gain
B. Increase in weight percentile without appreciable changes in
appetite or calories
C. Increase in weight percentile associated with an increased
interest in food
D. Maintenance of weight along the same percentile
E. Failure to thrive with difficulty gaining weight
81 A 6-year-old girl presents for evaluation of short stature. GH
deficiency is being considered, and the next planned step is
provocative GH-stimulation testing. The patient’s mother asks how
the testing works.

Which of the following best describes the mechanism of GH


release?
A. Arginine inhibits somatostatin
B. Glucagon stimulates α2 adrenergic receptors
C. Propranolol causes hypoglycemia
D. Clonidine inhibits α2 adrenergic receptors
E. Levodopa leads to a β2 adrenergic antagonist effect on
somatostatin
82 A 6-month-old girl presents to her pediatrician with failure to
thrive, vomiting, and seizures. Biochemical analysis reveals
hypoglycemia and primary adrenal insufficiency. She receives
treatment, and her management regimen subsequently consists of
glucocorticoid and mineralocorticoid replacement with regular
monitoring.
Over the next 2 years, she is noted to have mild
neurodevelopmental delay and develops ichthyosis and persistent
lymphopenia. Primary hypothyroidism is diagnosed, and
levothyroxine is initiated. At age 3 years, she develops nephrotic
syndrome with proteinuria and hypoalbuminemia. Renal biopsy
documents focal segmental glomerulosclerosis.

Pathogenic variants in which of the following genes are the


most likely cause of her condition?
A. SGPL1 (sphingosine-1-phosphate lyase 1)
B. MRAP (melanocortin 2 receptor accessory protein)
C. POR (cytochrome P450 oxidoreductase)
D. AIRE (autoimmune regulator)
E. AAAS (aladin WD repeat nucleoporin)
83 An otherwise healthy 4-year-old boy is referred for evaluation
of short stature. He was born at 39 weeks’ gestation via natural
vaginal delivery. Birth weight was 3 lb 15 oz (1800 g) (<3rd
percentile; SDS, –3.14) and birth length was 16.5 in (42 cm) (<3rd
percentile; SDS, –4.4). He had a few episodes of hypoglycemia in the
immediate neonatal period that resolved after 3 days, at which time
he was discharged home with his mother.
Family history is negative for disorders of growth and puberty.
Midparental target height is 68 ± 4 in (172.7 ± 10 cm). The patient has
no siblings.
On physical examination, his height is 33.5 in (85 cm) (<3rd
percentile; SDS, –4.16) and his weight is 29.7 lb (13.5 kg) (4th
percentile; SDS, –1.68). He has a prominent forehead with relative
macrocephaly and a small pointy chin. His right leg is 1 in (2.5 cm)
longer than his left leg. He has clinodactyly of the fifth digits
bilaterally. He has 2 café-au-lait spots on the right side of his lumbar
area measuring 4 × 3 cm and 2 × 1.5 cm.

Initial laboratory test results:


IGF-1 = 115 ng/mL (54-178 ng/mL) (SI: 15.1 nmol/L [7.1-23.3 nmol/L])
IGFBP-3 = 2.8 mg/L (1.4-3.0 mg/L)
Thyroid function, normal
Erythrocyte sedimentation rate, normal
Screen for celiac disease, negative

Which of the following tests is most likely to reveal the etiology


of this child’s short stature?
A. Arginine-insulin tolerance test
B. NF1 gene sequencing and deletion/duplication analysis
C. Uniparental disomy analysis of chromosome 7
D. Sequencing of the gene encoding the IGF-1 receptor
E. Methylation analysis of chromosome 11p15
84 A 17-year-old girl presents with classic symptoms of polyuria
and polydipsia, and diabetes mellitus is diagnosed. Her urinalysis
indicates no ketones. Her BMI is at the 65th percentile; the family
reports no recent weight loss. She wears bilateral hearing aids. Her
medical history is notable for bilateral sensory neural hearing loss
and a learning disability. Her family history is relevant for insulin-
requiring diabetes in her mother. Her older brother was diagnosed
with diabetes at age 19 years and he is currently on oral therapy.
The patient is started on a basal-bolus insulin regimen. She
responds well but requires much less insulin than initially prescribed
due to frequent hypoglycemia episodes. At her 2-week follow-up
visit, the following laboratory results are reviewed:
Hemoglobin A1c = 8.5% (4.0%-5.6%) (69 mmol/mol [mmol/mol]) (it was 9.8% [84
mmol/mol] at diagnosis)
Glutamic acid decarboxylase 65 antibodies = <5 IU/mL (<5 IU/mL)
Insulin autoantibodies = <0.4 U/mL (<0.4 U/mL)
Islet-cell antibodies = <1.25 JDF units (<1.25 JDF units)
Electrolytes, normal
Liver enzymes, normal

Which of the following diagnostic approaches would reveal the


etiology of this patient’s diabetes mellitus?
A. Assessment for zinc transporter-8 [ZnT8] antibodies
B. HLA typing
C. HNF1A and HNF4A genetic testing
D. WFS1 genetic testing
E. MT-TL1 genetic testing
85 A 16-year-old girl is noted to have a thyroid nodule on
examination, and ultrasonography confirms a right-sided 3-cm
nodule with features suspicious for thyroid cancer. Furthermore,
ultrasonography demonstrates a few lymph nodes in the right mid-
cervical chain, anterior to the sternocleidomastoid muscle, that show
a loss of the hilum and/or peripheral vascularity. She is euthyroid,
and TPO and thyroglobulin antibodies are negative. Results of
ultrasound-guided FNA of the nodule are interpreted as suspicious
for papillary thyroid carcinoma. She undergoes total thyroidectomy,
and based on the preoperative ultrasonography findings, the
surgeon also performs a bilateral central neck dissection and a lateral
right-sided neck dissection.
Histopathologic examination confirms the right nodule to be
papillary thyroid carcinoma, measuring 3.2 cm in largest diameter
with lymphovascular invasion. She has 2 smaller foci of papillary
thyroid carcinoma in the left lobe. She has locoregional spread: 5/7
level 6 lymph nodes and 8/10 level 3 and 4 lymph nodes are
malignant on the right side.

Which of the following is the best next step in this patient’s


management?
A. Perform chest CT to search for pulmonary metastases; decide
whether she needs 131I radioablation
B. Initiate levothyroxine to maintain TSH <0.1 mIU/L; 2 to 3
months after surgery, measure thyroglobulin
C. Initiate levothyroxine to maintain TSH <0.1 mIU/L; 2 to 3
months after surgery, measure TSH-stimulated thyroglobulin
and perform diagnostic whole-body 123I scan
D. Hold off on initiating levothyroxine postoperatively;
administer 131I once TSH is >30 mIU/L
E. Initiate levothyroxine to maintain TSH between 0.1 and 0.5
mIU/L; perform postoperative 123I whole-body scan
86 A 5-year-old boy is followed up in endocrine clinic for short
stature. He was the product of a nonconsanguineous union and was
born at 34.5 weeks gestational age with normal birth length (18.5 in
[47 cm]) and weight (6 lb 5 oz [2865 g]). In the last year, he had 4
episodes of abdominal pain, vomiting, hypoglycemia, and ketosis
requiring intravenous glucose infusions. His growth chart
demonstrates postnatal growth delay with a current height at –3.0
SD. His BMI is at the 3rd percentile. He consumes approximately
2000 calories daily (normal, 1700-1800 calories daily).
His parents and 2 younger siblings have a normal phenotype. His
mother’s height is 61.4 in (156 cm) (–1.3 SD), and his father’s height
is 67.3 in (171 cm) (–0.6 SD). His father is overweight and had
delayed puberty. His mother had normal puberty. The patient’s
father has a female first cousin who is very short (adult height 55.1 in
[140 cm]), but she has never been evaluated for short stature.
The patient has proportional short stature and no dysmorphic
features on examination. His current growth velocity is 4 cm/y. Bone
age assessment at age 4 years and 8 months was interpreted to be 1
year and 6 months.

Laboratory test results (6 months ago):


IGF-1 = 44 ng/mL (37-192 ng/mL) (SI: 5.8 nmol/L [4.8-25.2 nmol/L])
Free T4 = 1.1 ng/dL (0.9-1.7 ng/dL) (SI: 14.2 pmol/L [11.6-21.9 pmol/L])
TSH = 3.5 mIU/L (0.6-5.5 mIU/L)
Cortisol (8 AM) = 15.0 µg/dL (3.0-21.0 µg/dL) (SI: 413.8 nmol/L [82.8-579.4
nmol/L])

A dual-agent GH-stimulation test performed 2 months ago


documented a peak GH concentration of 4.0 ng/mL (4.0 µg/L).
Findings on brain MRI are normal.

Which of the following is the best next step in this patient’s


management?
A. Reassure and schedule follow-up in clinic in 6 months
B. Refer to gastrointestinal clinic for evaluation and return to
endocrine clinic once BMI has improved
C. Start GH replacement therapy at 0.2 mg/kg per week
D. Start GH replacement therapy at 0.35 mg/kg per week
E. Repeat endocrine laboratory tests, including IGF-1, free T4,
TSH, and 8-AM cortisol
87 A 3-year-old girl presents for evaluation of precocious puberty.
She was born at 31 weeks’ gestation (birth weight 3 lb 3 oz [1446 g])
and spent time in the neonatal intensive care unit because of mild
respiratory distress, anemia, and jaundice. She has reportedly had
breast tissue since infancy and it has recently increased. She has
developed some long, thick hairs in the pubic area. She has also
started outgrowing her twin sister. Development has been normal,
and she does not take any medications. She is otherwise healthy. Her
mother is concerned that her daughter has episodes of unprovoked
laughter lasting 30 to 45 seconds. Although she is able to speak
during these episodes, she is unable to stop the laughter.
On physical examination, her height is 38.2 in (97 cm) (75th
percentile). She appears well and findings on neurologic
examination are normal. She has approximately 5 cm of glandular
breast tissue bilaterally, several long, coarse pubic hairs along the
labia, and dull pink vaginal mucosa. There are several café-au-lait
macules measuring less than 5 cm each.

Laboratory test results:


Thyroid function, normal
Complete blood cell count, normal
Complete metabolic panel, normal
Estradiol = 25.2 pg/mL (<16 pg/mL) (SI: 92.5 pmol/L [<58.7 pmol/L])
LH = 9.5 mIU/mL (<0.02-0.30 mIU/mL) (SI: 9.5 IU/L [<0.02-0.30 IU/L])
FSH = 7.1 mIU/mL (0.5-6.0 mIU/mL) (SI: 7.1 IU/L [0.5-6.0 IU/L])

Which of the following is the most likely etiology of her


precocious puberty?
A. MKRN3 gene variant
B. GNAS gene variant
C. Idiopathic
D. Craniopharyngioma
E. Hypothalamic hamartoma
88 A 2-and-11/12-year-old girl is referred for evaluation of short
stature. On physical examination, her length is below the 3rd
percentile for age (SDS, –3.42), her weight is at the 8th percentile for
age (SDS, –1.38), and her head circumference is at the 4th percentile
for age (SDS, –1.74). She is a petite, nondysmorphic girl in no acute
distress. Examination findings are unremarkable. There is no
asymmetry or bowing of the lower extremities and no evidence of
scoliosis. Her teeth are in good condition with no signs of decay. She
is at Tanner stage 1 for breast development and pubic hair, and the
appearance of her external genitalia is normal for a prepubertal girl.
The growth chart available from her pediatrician is shown (see
image), and it includes measurements obtained at the present visit.
She was born via cesarean delivery at a gestational age of 38
weeks’ based on obstetric ultrasonography performed in early
pregnancy. Birth length was 16.5 in (42 cm) (<3rd percentile; SDS, –
3.55), birth weight was 4 lb 3 oz (1900 g) (<3rd percentile; SDS, –
2.72), and head circumference was 11.8 in (30 cm) (<3rd percentile;
SDS, –3.42). The obstetrician described the placenta as small.
She is the second child of otherwise healthy parents. Her father’s
height is 68 in (172.7 cm). He had normal timing of puberty and has
no history of medical issues. Her mother’s height is 62 in (157.5 cm).
She experienced menarche at age 12 years and has had no health
issues. Neither parent smokes cigarettes.
Laboratory test results:
IGF-1 = 98 ng/mL (74-202 ng/mL) (SI: 12.8 nmol/L [9.7-26.5 nmol/L])
IGFBP-3 = 1.7 mg/L (1.4-3.0 mg/L)
TSH = 5.7 mIU/L (0.6-5.5 mIU/L)
Free T4 = 1.95 ng/dL (0.8-2.2 ng/dL) (SI: 25.1 pmol/L [10.3-28.3 pmol/L])
Erythrocyte sedimentation rate = 12 mm/h (0-20 mm/h)
Complete blood cell count, no anemia and normal differential
Comprehensive chemistry panel, normal
25-Hydroxyvitamin D = 33 ng/mL (30-100 ng/mL) (SI: 82.4 nmol/L [74.9-249.6
nmol/L])
Karyotype = 46,XX

Which of the following is the most reasonable course of action?


A. Start GH therapy
B. Measure TPO and thyroglobulin antibodies
C. Perform a GH-stimulation test
D. Request a consultation with gastroenterology
E. Follow clinically until age 5 years
89 A 4-year-old boy is being seen in the outpatient setting for
ongoing care of congenital hypothyroidism. At this visit, his parents
express concern that he does not speak as clearly as their older child
did at the same age. He sometimes does not respond if they speak to
him while he is watching television. They have no concerns about
his vision or social development.
The patient was diagnosed with congenital hypothyroidism at 4
weeks of age after presenting with prolonged jaundice. He was born
at home, and no newborn screening was performed. At diagnosis,
his serum TSH concentration was 308 mIU/L and his free T4
concentration was 0.5 ng/dL (6.4 pmol/L). Levothyroxine, 50 mcg
daily (12 mcg/kg), was initiated, and 2 weeks after beginning
treatment his serum TSH concentration had decreased to 46 mIU/L
and his free T4 concentration was 2.1 ng/dL (27.0 pmol/L). TSH
normalized 1 month after treatment initiation. During the first 3
years of life, he had 5 episodes of elevated TSH (>10 mIU/L) due to
variable adherence to treatment. His current levothyroxine dosage is
75 mcg daily.
On physical examination, his length is at the 23rd percentile and
weight is at the 60th percentile. Vital signs are normal. He is alert
and energetic. He makes good eye contact and responds to direct
questions. His speech is 50% to 75% intelligible. Fontanelles are
closed. Findings on external ear and otoscopic examination are
normal. The thyroid gland is not palpable. The rest of the
examination findings are normal.
Laboratory test results:
TSH = 4.4 mIU/L (0.7-5.7 mIU/L)
Free T4 = 1.6 ng/dL (1.0-2.6 ng/dL) (SI: 20.6 pmol/L [12.9-33.5 pmol/L])

A possible hearing deficit may be contributing to the patient’s


speech delay.
Which of the following factors is most closely associated with
the risk of hearing impairment in this patient?
A. Delayed initiation of levothyroxine treatment
B. Delayed normalization of TSH levels after initiation of
treatment
C. Severity of hypothyroidism at diagnosis
D. Inadequate treatment of hypothyroidism during early
childhood
E. Male sex
90 A 16-year-old girl presents with weight loss, irritability, heat
intolerance, difficulty sleeping, and a decline in school performance
over the past several years. She is taking an oral contraceptive pill
daily. Her medical history is unremarkable and she has no history of
hearing problems. Her mother has a history of Graves disease, which
was treated with radioactive iodine at age 36 years. Her mother now
takes levothyroxine and has normal thyroid function. Her father’s
thyroid function was checked last year due to fatigue and results
were normal. The patient’s midparental height is 61 in (155 cm).
On physical examination, her pulse rate is 140 beats/min and
blood pressure is 126/52 mm Hg. Her height is 68 in (172.7 cm) (90th
percentile), and weight is 110 lb (50 kg) (25th percentile). Her skin is
moist, and there is no exophthalmos or lid lag. She has a tremor of
the outstretched hands, and the thyroid gland is diffusely enlarged
without nodules or thyroid bruit. There is a hyperactive precordium
with tachycardia and hyperactive reflexes.

Laboratory test results:


Free T4 = 4.1 ng/dL (0.98-1.63 ng/dL) (SI: 52.8 pmol/L [12.6-21.0 pmol/L])
Total T3 = 445 ng/dL (91-218 ng/dL) (SI: 6.9 nmol/L [1.4-3.3 nmol/L])
TSH = 14.0 mIU/L (0.51-4.3 mIU/L)

Which of the following is this patient’s most likely diagnosis?


A. Medication interference causing abnormal laboratory findings
B. Resistance to thyroid hormone β
C. Familial dysalbuminemic hyperthyroxinemia
D. Graves disease
E. TSH-producing adenoma
91 A 3-year-old girl with new-onset diabetes mellitus is admitted
to the hospital. Discharge is planned after her condition is stable for
24 hours following resolution of diabetic ketoacidosis. She is
responding well to the basal-bolus insulin regimen and has cleared
her ketosis. The family is interested in pursuing continuous glucose
monitoring since they are fearful of nocturnal hypoglycemia. Insulin
dosing and all safety precautions related to preventing
hypoglycemia are reviewed. The patient weighs 33 lb (15 kg).

Which of the following prescriptions is most appropriate to use


for emergency glucagon treatment in this patient?
A. Prefilled syringe, ready-to-inject glucagon, 0.5 mg
B. Prefilled syringe, ready-to-inject glucagon, 1 mg
C. Glucagon nasal powder, 3 mg
D. Glucagon nasal powder, 1.5 mg (half dose)
E. Glucagon reconstitution powder, 1 mg
92 A 10-year-old girl with myelomeningocele is referred to
endocrinology for assessment of bone health due to a fracture of her
right humerus that occurred when she fell during a school field trip.
She had a prior low-trauma tibial fracture at 2 years of age.
The patient ambulates with the assistance of lower-extremity
braces and a wheelchair for longer distances. She has a regular diet
and consumes milk at breakfast and cheese at lunch. She also has a
history of epilepsy that is well controlled with levetiracetam (she has
been seizure-free for the last 2 years).
DXA of her total body less head shows a height-adjusted Z-score
of –2.1. Metabolic bone workup reveals normal calcium, phosphate,
and magnesium; mildly elevated alkaline phosphatase for age; and a
25-hydroxyvitamin D concentration of 15.8 ng/mL (39.4 nmol/L).

Which of the following interventions is the best next step to


reduce this child’s risk for subsequent fractures?
A. Start supplemental vitamin D
B. Start supplemental vitamin D and calcium
C. Start bisphosphonate therapy
D. Induce puberty
E. Consult with the neurologist about changing seizure treatment
93 A 13-year-old boy is seen for follow-up of congenital adrenal
hyperplasia due to 21-hydroxylase deficiency. Over the last 2 years,
his control has become suboptimal on hydrocortisone replacement
therapy (14 mg/m2 per day in 3 divided dosages), with persistently
high 17-hydroxyprogesterone, androstenedione, and ACTH levels.
He was receiving vitamin D supplementation and was treated with
erythromycin for a chest infection. He has been growing along the
same percentiles for height and weight. His current height is 55.5 in
(149 cm) (50th percentile), and weight is 108 lb (49 kg) (75th
percentile). Pubertal assessment reveals Tanner stage 4 genital
development and pubic hair. Testicular volumes are 10 mL
bilaterally. He has some mild symptoms of gastritis.
Pharmacokinetic studies performed after administering 15 mg/m2
of intravenous hydrocortisone document increased cortisol clearance
with reduced half-life.

Which of the following is the most likely cause of suboptimally


controlled congenital adrenal hyperplasia in this patient?
A. Poor regimen adherence
B. Puberty
C. Gastritis
D. Obesity
E. Drug interaction
94 A 16-year-old boy presents to his pediatrician with a neck mass.
On palpation, the pediatrician notes a 3-cm, firm nodule and 2
palpable lymph nodes on the right side and lateral to the thyroid
gland. He is referred to a pediatric endocrinologist who orders
thyroid and neck ultrasonography and laboratory tests. The patient
is euthyroid and has negative thyroid antibodies. Calcitonin and
calcium levels are normal. There is no family history of thyroid
cancer.
Ultrasonography reveals a 2.8-cm, right-sided, hypoechoic, solid
nodule with microcalcifications and a smaller 1.2-cm nodule on the
left lower pole of the thyroid. Ultrasonography also identifies a
number of right-sided lymph nodes that are suspicious for
malignancy. Findings from FNA biopsy of both nodules are
suspicious for malignancy. He undergoes total thyroidectomy, with
bilateral central neck dissection and right lateral neck dissection.
Histopathologic examination reveals classic papillary thyroid cancer
involving both lobes, with spread to the central and lateral neck
compartments (T2N1bMx).

Which of the following is the most likely pathogenic genetic


alteration expected in this patient?
A. RET proto-oncogene pathogenic variant
B. RET/PTC fusion
C. PAX8/PPARG fusion
D. NTRK3/ETV6 fusion
E. BRAF V600E pathogenic variant
95 A 4-and-6/12-year-old girl is followed in clinic for short stature.
Her height is –3.5 SDS with target height at –3.0 SDS. She was born
full term with normal birth weight and length. She has no
developmental delay. In addition to short stature in her father and 2
paternal first cousins, her family history is notable for multiple
lumbar disk herniations and osteoarthritis of the knees in the third
decade of life.
On physical examination, she is overweight and has
brachydactyly and midface hypoplasia. She has no signs of puberty.
Her sitting height to standing height ratio is 0.54, and her arm span
is 1 cm more than her height. Her bone age is interpreted to be 6
years.

Which of the following is the most likely cause of this patient’s


short stature?
A. Familial short stature
B. GNAS pathogenic variant
C. 45,X karyotype
D. FGFR3 pathogenic variant
E. ACAN pathogenic variant
96 A 14-year-old boy presents for follow-up of obesity as part of
the hospital’s weight management program. He has a longstanding
history of progressive weight gain. His current BMI is greater than
140% of the 95th percentile for age and sex (class III obesity). His
parents report that they have made radical changes to their lifestyle
as a family, including cutting out sugar-sweetened beverages,
increasing fruit and vegetable intake, and cutting out fast food. His
older brother takes him to the gym at least 3 days per week, where
he uses the treadmill and stationary bike. Despite all of these efforts,
his weight has increased by 6.6 lb (3 kg) in the last 4 months. His
hemoglobin A1c value is normal at today’s visit today.
The patient and his parents would like to discuss potential
pharmacotherapeutic options to help him lose weight.

After discussing the risks and benefits of starting a weight-loss


medication, which of the following would be the most
appropriate choice for this patient?
A. Phentermine
B. Octreotide
C. Metformin
D. Liraglutide
E. Orlistat
97 A 10-year-old girl with a large craniopharyngioma is followed
for panhypopituitarism. She has diabetes insipidus and
hypothalamic obesity with impaired glucose tolerance. In addition to
pituitary hormone replacement with GH, levothyroxine,
hydrocortisone, and desmopressin, she also takes metformin. She
has tried to adhere to a strict low-carbohydrate diet, but she finds it
very difficult to maintain and she has not been successful with
weight loss. Through social media, her mother has heard of patients
with hypothalamic obesity using inhaled oxytocin as a treatment for
weight loss and she asks you about this. You inform her that use of
oxytocin in this setting is experimental.

Which of the following is a proposed mechanism of action of


oxytocin for weight loss in this setting?
A. Decreased energy consumption
B. Increased leptin resistance
C. Decreased fatty liver
D. Increased energy expenditure
E. Reduced insulin secretion
98 A 17-year-old girl is seen in the outpatient setting for follow-up
of Graves disease. Graves disease was diagnosed 3 years ago and
has been treated with methimazole, which she takes consistently.
After a recent decrease in the methimazole dosage, hyperthyroidism
recurred, and her prior methimazole dosage was resumed. Having
not achieved remission of Graves disease, she wishes to pursue
definitive therapy for hyperthyroidism. She reports no symptoms of
hypothyroidism or hyperthyroidism. She has no eye or vision
symptoms.
On physical examination, her height is at the 56th percentile,
weight is at the 25th percentile, and BMI is at the 20th percentile
(19.3 kg/m2). Her pulse rate is 74 beats/min, and blood pressure is
107/63 mm Hg. She is alert and oriented. She has normal extraocular
movements, no proptosis, and no edema or erythema of the eyelids
or conjunctivae. There is no cervical lymphadenopathy. Palpation of
the thyroid reveals a firm, asymmetric gland, with the right lobe
normal in size and the left lobe twice normal size, and no palpable
discrete nodule. This represents a change from her examination 1
year ago, at which time the left thyroid lobe was only slightly larger
than the right lobe. The rest of the examination findings are normal.
Her current TSH concentration is 1.3 mIU/L (0.7-5.7 mIU/L).
Based on the thyroid examination, thyroid ultrasonography is
performed, which reveals a 3.5-cm, solid, hypoechoic nodule in the
left thyroid lobe. The nodule contains echogenic foci consistent with
possible calcifications.

Which of the following is the most appropriate next step in this


patient’s management?
A. Continuation of methimazole and follow-up with
ultrasonography in 3 months
B. Radioactive iodine (131I) therapy
C. Thyroid scintigraphy
D. FNA of the thyroid nodule
E. Thyroidectomy
99 An 11-year-old child assigned male sex at birth who identifies
as female is referred by a pediatrician. The parents state that their
child has always identified as a girl. Starting in first grade, she
would become distressed whenever she was referred to as a boy. She
started seeing a therapist with experience in gender disorders and
was diagnosed with gender dysphoria. She socially transitioned at
age 7 years and has been well supported by her family.
Her parents are concerned that she has started showing signs of
puberty, which is causing distress. They are worried about
worsening gender dysphoria as puberty progresses. The patient is
requesting intervention to prevent developing a male body. Stage 2
testicular development is noted on physical examination. Laboratory
testing is notable for pubertal LH and testosterone levels.

Which of the following is the best advice for this patient?


A. Start gender-affirming hormone treatment with low-dosage
estradiol
B. Start a GnRH analogue to prevent further pubertal changes
C. Start medroxyprogesterone acetate to block testosterone
production
D. Refer for counseling to discuss the impact of treatment on
fertility before starting medication
E. Recommend allowing puberty to further progress to determine
whether her gender identity is permanent before pursuing
intervention
100 A 6-year-old boy diagnosed with acute lymphoblastic
leukemia starts an induction chemotherapy protocol. Therapy
includes prednisolone, vincristine, cyclophosphamide,
daunorubicin, triple intrathecal injection, and pegylated L-
asparaginase.
The treatment protocol is shown (see figure).
On day 18 of therapy, he develops fasting hypoglycemia (glucose
= 51 mg/dL [2.8 mmol/L]) with severe hunger but no other signs or
symptoms such as tremor, palpitation, anxiety, or sweating. On day
27 of therapy, he has another episode and critical blood sampling
documents the following:

Fasting serum glucose = 45 mg/dL (70-99 mg/dL) (SI: 2.5 mmol/L [3.9-5.5
mmol/L])
Insulin = 14 µIU/mL (<17 µIU/mL) (SI: 97.2 pmol/L [<118.1 pmol/L])
Free fatty acids = 13.2 mg/dL (16.9-42.3 mg/dL) (SI: 0.47 mmol/L [0.6-1.5
mmol/L])
β-Hydroxybutyrate = 1.25 mg/dL (<2.91 mg/dL) (SI: 120 µmol/L [<280 µmol/L])

He repeatedly develops fasting hypoglycemia without serious


complications (glucose ranging from 35-51 mg/dL [1.9-2.8 mmol/L])
until day 37.
His medical history is unremarkable. He was a full-term baby
(size appropriate for gestational age) with no medical complications.
His mother did not have gestational diabetes. No previous blood
glucose problems are reported. His linear growth has always been
steady. There is no family history of diabetes or hypoglycemia.

Which of the following is the most likely etiology of this


patient’s hypoglycemia?
A. Iatrogenic adrenal insufficiency
B. L-asparaginase–induced hypoglycemia
C. Inadequate nutritional intake
D. Intrathecal injection–induced GH deficiency
E. Normal hypoglycemic response during chemotherapy
ANSWERS
PEDIATRIC ENDOCRINE SELF-
ASSESSMENT PROGRAM 2021-2022
Part II
1 ANSWER: B) Familial glucocorticoid deficiency
Familial glucocorticoid deficiency (Answer B) is the most likely
diagnosis in view of this patient’s extremely high plasma ACTH
levels. He lacks biochemistry suggestive of congenital adrenal
hyperplasia (Answer D), and mineralocorticoid deficiency and
adrenal antibodies that would point towards congenital adrenal
hypoplasia (Answer C) or Addison disease (Answer A), respectively.
Very long-chain fatty acid levels were normal, thus excluding X-
linked adrenoleukodystrophy (Answer E). This child was found to
have a homozygous pathogenic variant in the gene encoding the
ACTH receptor (MC2R; S74I [serine to isoleucine at amino acid
position 74]), also known as the melanocortin 2 receptor. Many
patients with this pathogenic variant are of Scottish descent. Familial
glucocorticoid deficiency is an autosomal recessive disorder that
typically manifests during early childhood or the neonatal period
with symptoms of hypocortisolemia, the absence of
mineralocorticoid deficiency, and very high plasma ACTH, which
leads to intense hyperpigmentation. Tall stature, which normalizes
after glucocorticoid replacement, is also a feature of MC2R
pathogenic variants, probably through the action of ACTH on other
melanocortin receptors expressed in bone. The diagnosis in this case
is relatively late and demonstrates the phenotypic variability of this
condition, even in those with the same pathogenic variant.
Seven other genes associated with familial glucocorticoid
deficiency have been identified to date, including MRAP
(melanocortin 2 receptor accessory protein), STAR (steroidogenic
acute regulatory protein), MCM4 (minichromosome maintenance
complex component 4), NNT (nicotinamide nucleotide
transhydrogenase), TXNRD2 (thioredoxin reductase 2), CYP11A1
(cytochrome P450 family 11 subfamily A member 1), and SGPL1
(sphingosine-1-phosphate lyase 1). These genes are involved in
diverse pathways, and the resulting phenotypes are caused by
defective ACTH signaling, cholesterol transport, steroidogenesis,
cellular redox homeostasis, DNA replication, or sphingolipid
metabolism. There is increasing awareness of the variable
phenotypes and associated conditions seen within the familial
glucocorticoid deficiency umbrella of conditions. For example,
evidence of mineralocorticoid deficiency (transient or permanent)
has been described in those with severe pathogenic variants in
MC2R and MRAP, as well as in patients with pathogenic variants in
NNT. Later onset of symptoms has been observed in some persons
with pathogenic variants in NNT and missense mutations in MRAP
and CYP11A1. Furthermore, MCM4 and SGPL1 are associated with
other disorders such as natural killer cell deficiency and progressive
renal dysfunction, respectively.
The other causes of primary adrenal insufficiency (Addison
disease [Answer A], congenital adrenal hypoplasia [Answer C],
congenital adrenal hyperplasia [Answer D], and X-linked
adrenoleukodystrophy [Answer E]) should certainly be considered
in the differential diagnosis. Overall, autoimmune causes (Addison
disease and polyglandular syndromes) account for 80% to 90% of
primary adrenal insufficiency, and most individuals are adrenal
antibody positive. Autoimmune destruction is usually associated
with mineralocorticoid deficiency; hence, in the absence of both
features, an autoimmune cause is unlikely. Because this is a male
patient, NR0B1 (formerly DAX1) pathogenic variants as a cause of
congenital adrenal hypoplasia are also a possibility, but the late onset
plus the lack of mineralocorticoid deficiency make this a less likely
diagnosis. X-linked adrenoleukodystrophy should always be
considered and excluded by measuring very long-chain fatty acid
levels. Late-onset congenital adrenal hyperplasia is another
consideration, especially in view of the unexplained tall stature.
However, in this case, normal androgens and prepubertal findings
on examination do not support this diagnosis.

Educational Objective
Explain the pathophysiology and phenotype variability of familial
glucocorticoid deficiency.
Reference(s)
Maharaj A, Maudhoo A, Chan LF, et al. Isolated glucocorticoid deficiency: Genetic causes
and animal models. J Steroid Biochem Mol Biol. 2019;189:73-80. PMID: 30817990
Bornstein SR, Allolio B, Arlt W, et al. Diagnosis and treatment of primary adrenal
insufficiency: an Endocrine Society clinical practice guideline. J Clin Endocrinol Metab.
2016;101(2):364-389. PMID: 26760044
2 ANSWER: A) X-linked hypophosphatemic rickets
While many features of these skeletal conditions overlap, X-linked
hypophosphatemic rickets (Answer A) (XLH) is frequently
associated with progressive bowing of the lower extremities that
appears when young children learn to stand and walk, a waddling
gait, short stature, and craniosynostosis (particularly with premature
fusion of the sagittal sutures, resulting in the elongation of the skull
or dolichocephalic malformation). While the exact incidence is
unknown, a recent retrospective study suggests that craniosynostosis
may occur in more than 50% of affected patients. Dental abscesses
may develop as children get older.
Craniosynostosis can result in increased intracranial hypertension
and Chiari malformation type 1 with herniation of the cerebellar
tonsils. In some cases, craniosynostosis does not occur until later in
skull development, resulting in little morphologic changes and
sufficient room for brain growth. However, craniosynostosis can be
the presenting feature of XLH and can occur in children as young as
3 months. When cranial sutures fuse at a young age, surgical
intervention may be required to prevent complications.
Craniosynostosis syndromes can be due to pathogenic variants in
the fibroblast growth factor (FGF) receptors. FGF receptor 1 (FGFR1)
and FGF receptor 2 (FGFR2) are involved in intramembranous
osteogenesis, whereas FGF receptor 3 (FGFR3) has a primary role as
an inhibitor of endochondral ossification. The primary receptor for
FGF-23 is FGFR1, and thus this may explain the association of
craniosynostosis with this disorder.
Pathogenic variants in FGFR3 are associated with
hypochondroplasia (Answer C). Patients with this condition also
have short stature, macrocephaly, and bowed legs, but frequently
have short, broad hands and feet. However, these individuals rarely
present with craniosynostosis or radiographic signs of rickets, so
hypochondroplasia is an unlikely diagnosis in this case. One-third to
two-thirds of patients with infantile hypophosphatasia (Answer B)
also have craniosynostosis, frequently by 6 months of age. Affected
children generally have other manifestations such as bell-shaped
chest deformity, respiratory compromise, seizures, and characteristic
x-ray findings of metaphyseal tongues.
While severe vitamin D deficiency (Answer D) could explain the
radiographic findings in this patient, craniotabes is a rare skull
manifestation of this condition, not craniosynostosis. There have
been case reports of vitamin D excess causing craniosynostosis.
Apert syndrome (Answer E) is due to pathogenic variants in the
FGFR2 gene and is associated with short stature and bicoronal
synostosis with resultant widely spaced eyes, exophthalmos, and
midface hypoplasia. Affected patients also have characteristic
changes of the hands including syndactyly, most commonly of the
second through fourth fingers with a single common nail.

Educational Objective
Identify craniosynostosis as an initial presentation of X-linked
hypophosphatemic rickets.

Reference(s)
Rothenbuhler A, Fadel N, Debza Y, et al. High incidence of cranial synostosis and Chiari I
malformation in children with X-linked hypophosphatemic rickets (XLHR). J Bone
Miner Res. 2019;34(3):490-496. PMID: 30352126
Wang J, Liu S, Li J, Yi Z. The role of the fibroblast growth factor family in bone-related
diseases. Chem Biol Drug Des. 2019;94(4):1740-1749. PMID: 31260189
Vakharia JD, Matlock K, Taylor HO, Backeljauw PF, Topor LS. Craniosynostosis as the
presenting feature of X-linked hypophosphatemic rickets. Pediatrics. 2018;141(Suppl
5):S515-S519. PMID: 29610183
Whyte MP, Leung E, Wilcox WR, et al; Study 011-10 Investigators. Natural history of
perinatal and infantile hypophosphatasia: a retrospective study. J Pediatr. 2019;209:116-
124. PMID: 30979546
3 ANSWER: C) 8 units
Calculating the needs for mixed-meal insulin in children with type 1
diabetes is challenging. Both the type and the amount of
carbohydrate found in foods influence postprandial glucose levels
and can also affect overall glycemic control in individuals with
diabetes. Added sugars such as sucrose and high-fructose corn
syrup are digested, absorbed, and fully metabolized in a similar
fashion to naturally occurring monosaccharides and disaccharides.
Only about half of the carbohydrate grams from sugar alcohols and
half or less from dietary fiber are metabolized to glucose, whereas
almost all “other carbohydrate” (mainly starch such as amylose and
amylopectin) becomes blood glucose.
All food labels indicate the amount of dietary fiber in the serving
size, and newer labels provide further details about soluble and
insoluble amounts. If the total fiber amount is greater than 5 g, half
of the total amount, which usually corresponds to the insoluble
portion, should be subtracted from the total carbohydrate amount to
calculate the insulin dose. If insoluble fiber amount is listed, then it
can be used to calculate directly.
In this vignette, the patient plans to consume 54 g (27 g × 2
servings) of total carbohydrates. The insoluble fiber amount is 3 g
per serving, totaling 6 g. The amount of carbohydrate that needs to
be considered for an insulin bolus is 54 minus 6, which equals 48 g.
The correct insulin dose should be calculated by dividing 48 g by 6,
which equals 8 units (Answer C).
Ignoring the fiber content and calculating the insulin amount on
the basis of the total carbohydrate of 54 g divided by 6, which equals
9 units (Answer B), would be incorrect.
Subtracting all of the fiber content from the total carbohydrate
amount (54 minus 10, which equals 44 → divided by 6, which equals
7.3 → rounded to 7 units [Answer D]) is also incorrect.
Ten units (Answer A) provide extra insulin beyond the
carbohydrate calculation. This may be valid in some practices that
account for the fat and the protein content of the consumed food
when determining the insulin calculation. There are data to suggest
that 25% to 50% extra insulin may be needed to cover the fat and
protein load of meals and snacks, delivered in an extended fashion
that can be achieved by insulin pumps. However, in this vignette,
there is no additional information to prompt one to consider extra
insulin.
Eating her snack and monitoring her glucose to decide on the
amount of insulin (Answer E) is also wrong and goes against the
concept that all “meal” insulin should be received before food
consumption. This idea may be valid in special circumstances such
as pending severe hypoglycemia if the patient’s continuous glucose
monitoring indicated such scenario.

Educational Objective
Incorporate dietary fiber into the insulin calculation for meals and
snacks in patients with type 1 diabetes mellitus.

Reference(s)
van der Hoogt M, van Dyk JC, Dolman RC, Pieters M. Protein and fat meal content increase
insulin requirement in children with type 1 diabetes - role of duration of diabetes. J Clin
Transl Endocrinol, 2017;10:15-21. PMID: 29204367
Matteucci E, Giampietro O. Dietary strategies for adult type 1 diabetes in light of outcome
evidence. Eur J Clin Nutr. 2015;69(3):285-290. PMID: 25293432
Wheeler ML, Pi-Sunyer FX. Carbohydrate issues: type and amount. J Am Diet Assoc.
2008;108(4 Suppl 1):S34-S39. PMID: 18358253
4 ANSWER: B) Optimize levothyroxine treatment to maintain
TSH between 0.5 and 1.0 mIU/L and follow-up with thyroglobulin
measurement (while on levothyroxine) 12 weeks after surgery
Postoperative systems have not been validated in children with
papillary thyroid carcinoma (PTC). However, the American Joint
Committee on Cancer TNM classification system can describe the
extent of the disease. The inaugural management guidelines for
children with thyroid nodules and differentiated thyroid cancer
(DTC), published in 2015, use the TNM system to categorize
pediatric patients into 1 of 3 risk group categories to determine
which patients are at higher risk for persistent cervical disease or
distant metastasis.

American Thyroid Association (ATA) pediatric low-risk


category is defined as disease confined to the thyroid, with
no nodal metastasis, unknown nodal metastasis, or
incidental finding of microscopic metastasis in a small
number of central lymph nodes. This group of patients is at
low risk for distant metastasis, but may still be at risk for
residual local disease.
ATA pediatric intermediate-risk category is defined as
extensive central nodal disease (N1a) or minimal nodal
disease in other neck compartments (N1b). These patients
have increased risk of persistent local disease, although
they are still considered to be at low risk for distant
metastasis.
ATA pediatric high-risk category is defined as extensive
N1b disease or locally invasive tumors (T4 tumors that
extend beyond the thyroid capsule to invade
extrathyroidal soft tissue, fascia, carotid artery, or
mediastinal vessels).

The patient in this vignette belongs in the ATA low-risk category.


She has a classic papillary thyroid microcarcinoma (tumor less than 1
cm) confined to the thyroid, without nodal disease.
Assuming that the patient had adequate preoperative staging and
that surgery was performed by a high-volume thyroid cancer
surgeon, the current guidelines for pediatric ATA low-risk thyroid
cancer recommend (1) postoperative staging by measuring a TSH-
suppressed (on levothyroxine) thyroglobulin concentration 12 weeks
after surgery, (2) treatment with levothyroxine to maintain TSH
between 0.5 and 1.0 mIU/L (Answer B), and (3) follow-up with
measurement of TSH-suppressed thyroglobulin every 3 to 6 months
for 2 years and then annually and neck ultrasonography 6 months
after surgery and then annually.
For patients in the intermediate- and high-risk categories, TSH-
stimulated thyroglobulin measurement and whole-body scan
(Answer A) are recommended to assess for residual disease and to
determine whether the patient may benefit from 131I treatment. This
patient, however, is in the low-risk category.
For patients in the high-risk category, radioactive iodine is
generally recommended.
Universal 131I treatment for DTC (Answer C) is not
recommended for children and adolescents, as the goal for
radioactive iodine treatment is to decrease the recurrence risk and to
improve mortality, while minimizing unnecessary exposure to
radioactive iodine to prevent permanent adverse effects (lifelong
xerostomia, increased risk for salivary gland malignancy, gonadal
damage, suppression of the bone marrow, possible risk of secondary
malignancies). Interestingly, Jin et al recently described how
radioactive iodine uptake and stimulated thyroglobulin
measurement may also be useful in deciding the dose of radioactive
iodine treatment in adults with DTC. This study confirms the value
of a TSH-stimulated thyroglobulin level less than 2 ng/mL (<2
µg/L) as a predictor of the absence of DTC.
The TSH goal of 0.1 to 0.5 mIU/L (Answer D) has been
recommended for patients at intermediate risk, and less than 0.1
mIU/L has been recommended for patients at high risk. Maintaining
TSH in the normal range (Answer E) is not recommended for
patients with a history of DTC.
Of note, although this patient does not currently have
thyroglobulin antibodies, it is important to always measure
thyroglobulin antibodies when obtaining thyroglobulin levels, as the
presence of thyroglobulin antibodies makes the thyroglobulin
measurement unreliable. Presence of thyroglobulin antibodies in a
patient previously negative for antibodies is nonreassuring. In the
presence of antibodies, monitoring the antibodies using the same
assay is recommended.

Educational Objective
Explain the indications for use of 131I in treating differentiated
thyroid carcinoma and the role of levothyroxine for TSH suppression
in its management.

Reference(s)
Francis GL, Waguespack SG, Bauer AJ, et al; American Thyroid Association Guidelines Task
Force. Management guidelines for children with thyroid nodules and differentiated
thyroid cancer. Thyroid. 25(7):716-759. PMID: 25900731
Jin Y, Ruan M, Cheng L, et al. Radioiodine uptake and thyroglobulin-guided radioiodine
remnant ablation in patients with differentiated thyroid cancer: a prospective,
randomized, open-label, controlled trial. Thyroid. 2019;29(1):101-110. PMID: 30560716
5 ANSWER: C) Measure prolactin again in 2 to 4 weeks
Breast tissue is commonly found in term infants, and some milk
secretion is common as well. Transient neonatal gynecomastia is
thought to be caused by placental conversion of weak androgens to
estrogens, which enter the fetal circulation and stimulate glandular
proliferation in the breast. The patient in this case was referred
because of the unusually large amount of breast tissue.
There are few reports of hyperprolactinemia in children younger
than 5 years and most are associated with other signs of
pituitary/hypothalamic dysfunction. Defining the normal range for
prolactin levels in the newborn is challenging; laboratory reference
ranges do not identify separate ranges for infants and indicate a
normal range of less than 10 ng/mL (<0.4 nmol/L) for prepubertal
males. One study of prolactin levels in infants and children suggests
that the normal range for prolactin levels in 1-week-old neonates
may be as high as 496 ng/mL (21.6 nmol/L), but drops to 63 ng/mL
(2.7 nmol/L) in infants 2 to 12 months of age. Data on the 1-week to
2-month age range are lacking. While it is difficult to determine
whether the infant in this case has an elevated prolactin level for his
age, his value of 108.5 ng/mL (4.7 nmol/L) should prompt another
measurement (Answer C) to ensure that it is decreasing. While
pathology in such a young patient would be rare, there have been
some reports of persistently elevated prolactin levels in infants.
This infant’s prolactin level will most likely decrease over time,
and therefore performing MRI of the brain and the pituitary gland
(Answer A) would be premature. Imaging would be indicated if the
prolactin level does not decrease over time, as there would be a
greater chance of finding a hypothalamic pituitary defect. Given that
this infant most likely has an exaggerated form of nonpathologic
neonatal breast hypertrophy, dopamine agonist therapy (Answer D)
to decrease the prolactin level is not indicated. This treatment would
only be considered if the prolactin level does not decrease, and only
after CNS imaging and further evaluation of hypothalamic/pituitary
function.
LH, FSH, and testosterone levels are most likely to be detectable
due to the mini puberty of infancy and their measurement (Answer
B) would not contribute to determining whether there is an
underlying pathologic cause for this patient’s breast hypertrophy.
Multiple endocrine neoplasia type 1 is associated with tumors of
the pituitary gland. Individuals known or suspected to have
multiple endocrine neoplasia type 1 are advised to have MRI of the
brain and measurement of prolactin, IGF-1, fasting glucose, insulin,
and proinsulin every 1 to 3 years starting as early as age 5 years.
While prolactin-secreting pituitary adenomas are common in this
syndrome, such manifestations would not be expected in an infant.
Therefore, genetic testing (Answer E) is not warranted.

Educational Objective
Evaluate breast enlargement and galactorrhea in a neonate.

Reference(s)
Wiedermann G, Jonetz-Mentzel L. Establishment of reference ranges for prolactin in
neonates, infants, children and adolescents. Eur J Clin Chem Clin Biochem. 1993;31(7):447-
451. PMID: 8399785
Leung AKC, Leung AAC. Gynecomastia in infants, children, and adolescents. Recent Pat
Endocr Metab Immune Drug Discov. 2017;10(2):127-137. PMID: 28260521
Amer A, Fischer H. Images in clinical medicine: neonatal breast enlargement. N Engl J Med.
2009;360(14):1445. PMID: 19339724
6 ANSWER: D) Perform a glucagon-stimulation test; if abnormal,
start GH therapy at a dosage of 1.0 mg daily and titrate based on
IGF-1 levels
This patient has completed her growth and showed good response
to GH and levothyroxine treatment for GH and TSH deficiencies,
respectively, after craniospinal radiation for a brain tumor. She also
had appropriate pubertal development and regular menstrual cycles
on hormone replacement therapy for primary ovarian insufficiency,
which was most likely due to chemotherapy and spinal radiation.
She has had tiredness and fatigue while off GH therapy for a year
after achieving final adult height, despite maintaining a normal
thyroid hormone level and having regular menstrual cycles on
levothyroxine and estrogen/progesterone replacement therapy.
Unlike anterior pituitary hormone deficiencies, antidiuretic
hormone deficiency has not been reported after cranial radiation.
Thus, she is not at risk for diabetes insipidus. Antidiuretic hormone
deficiency can be seen due to mass effect if the tumor (eg,
germinoma) is anatomically close to the hypothalamus and/or
pituitary. In this case, patient’s fluid intake is not excessive, and her
nocturia is therefore most likely habitual. She does not need to be
screened with first-morning laboratory tests for diabetes insipidus
(Answer A).
She has no notable symptoms or signs of cortisol insufficiency
other than fatigue. Even though she is at risk for ACTH deficiency
due to cranial radiation, an 8-AM cortisol value of 17 µg/dL (469
nmol/L) is robust and does not warrant cosyntropin-stimulation
testing (Answer B).
Children receiving GH treatment for childhood GH deficiency
should be evaluated for adult GH deficiency once growth is
completed if they have a high likelihood of permanent GH
deficiency (eg, history of cranial radiation, structural lesions causing
panhypopituitarism, proven genetic causes). GH deficiency is the
most common pituitary hormone deficiency in childhood cancer
survivors. The IGF-1 level can generally be used as a surrogate
marker for GH status; however, in childhood cancer survivors, IGF-1
performs poorly, making GH-stimulation testing necessary to
diagnose GH deficiency in those patients at high risk. Although
failure to respond to 2 different GH secretagogues is recommended
to diagnose childhood GH deficiency, single-agent testing is
acceptable to diagnose adult GH deficiency using 1 of the following
3 tests based on availability and patient eligibility: (1) GHRH-
stimulation test, (2) insulin tolerance test, and (3) glucagon-
stimulation test. Several different GH cutoff values have been
proposed to diagnose adult GH deficiency based on the agent used
for testing and the age and BMI of the patient tested. More recently,
macimorelin, an orally active GH secretagogue, has been shown to
be a potential alternative agent that could be used to diagnose adult
GH deficiency.
Once a patient has been confirmed to have adult GH deficiency,
GH therapy (at an appropriate dosage) should be restarted in
adolescent and young adult patients. In patients with GH deficiency,
long gaps in GH treatment should be avoided during the transition
from childhood to adulthood to minimize the risk for compromised
bone health, cardiovascular health, and poor quality of life. Adults
younger than 30 years could be started at a GH dosage of 0.5 mg
daily, although transition-age adolescents and young adults could be
started at a slightly higher dosage such as 1.0 mg daily. Generally,
patients with adult GH deficiency are started on low-dosage GH (0.5
to 1.0 mg daily), which can be up-titrated every 1 to 2 months based
on IGF-1 levels, aiming to keep the IGF-1 level in the upper half of
the reference range (thus, Answer D is correct and Answer C is
incorrect). Reassuring a patient with a history of high-dosage cranial
radiation who has symptoms such as tiredness and fatigue and not
performing further workup for adult GH deficiency (Answer E) is
incorrect.

Educational Objective
Identify patients at risk for adult GH deficiency (eg, history of
cranial irradiation) and guide the evaluation and management of
young adult patients during transition when growth is completed.

Reference(s)
Fleseriu M, Hashim IA, Karavitaki N, et al. Hormonal replacement in hypopituitarism in
adults: an Endocrine Society clinical practice guideline. J Clin Endocrinol Metab.
2016;101(11):3888-3921. PMID: 27736313
Molitch ME, Clemmon DR, Malozowski S, Merriam GR, Vance ML; Endocrine Society.
Evaluation and treatment of adult growth hormone deficiency: an Endocrine Society
clinical practice guideline. J Clin Endocrinol Metab. 2011;96(6):1587-609. PMID: 21602453
Chemaitilly W, Li Z, Huang S, et al. Anterior hypopituitarism in adult survivors of
childhood cancers treated with cranial radiotherapy: a report from the St Jude Lifetime
Cohort study. J Clin Oncol. 2015;33(5):492-500. PMID: 25559807
Dichtel LE, Yuen KC, Bredella MA, et al. Overweight/obese adults with pituitary disorders
require lower peak growth hormone cutoff values on glucagon stimulation testing to
avoid overdiagnosis of growth hormone deficiency. J Clin Endocrinol Metab.
2014;99(12):4712-4719. PMID: 25210883
Garcia JM, Biller BMK, Korbonits M, et al. Macimorelin as a diagnostic test for adult GH
deficiency. J Clin Endocrinol Metab. 2018;103(8):3083-3093. PMID: 29860473
7 ANSWER: A) Glycogen debrancher deficiency
This patient’s presentation is consistent with glycogen storage
disease (GSD). Hepatosplenomegaly, elevated triglycerides, and
hypoglycemia are the hallmark findings of GSD that presents in
infancy. GSD types I through IV are the most common forms that
present in infancy and are associated with hepatosplenomegaly.
However, GSD type I (known as von Gierke disease) and GSD type
III (known as Cori disease, Forbes disease, and limit dextrinosis) are
the 2 forms associated with ketotic hypoglycemia,
hepatosplenomegaly, and hyperlipidemia as observed in this patient.
Because of this patient’s normal lactic acid level, the most likely
diagnosis is GSD type III, or glycogen debrancher deficiency
(Answer A). Lactic acid excess in GSD type I is due to buildup of
glucose-6-phosphate, a condition not generally present in GSD type
III.
GSD type I, or glucose-6-phosphatase deficiency (Answer C), is
classically associated with ketotic hypoglycemia as a result of
suppressed insulin secretion and increased secretion of
counterregulatory hormones in response to hypoglycemia. Elevated
glucose-6-phosphate is unable to be converted to glucose; therefore,
it is converted to pyruvate. Excess pyruvate gets converted to lactate,
especially in the presence of acidosis. These hormonal changes cause
increased glycogenolysis and gluconeogenesis, with lactic acidosis
representing increased formation and decreased use of lactate. The
hormonal changes also promote exaggerated lipolysis. This leads to
an influx of free fatty acids to the liver, where a substantial fraction is
converted to triglycerides, and increasing secretion of VLDL.
In GSD type III, complete glycogenolysis is prevented by the lack
of the glycogen debrancher enzyme, or amyloglucosidase, whose
gene is located on chromosome 1p21. There are 4 subtypes of GSD
type III as determined by variability in tissue expression of the
debrancher enzyme:
GSD IIIa represents about 85% of all GSD type III cases
(liver and muscle involvement)
GSD IIIb represents about 15% of all GSD type III cases
(only liver involvement)
GSD IIIc is very rare and is considered to be caused by a
defect in glucosidase debranching activity
GSD IIId is also very uncommon and is thought to be
caused by a defect in transferase debranching activity

During glycogenolysis and prior to the presentation to the


debrancher enzyme, phosphorylase breaks down glycogen to 2 to 4
linked glucose molecules or limit dextrins. Then, the debrancher
enzyme works through 2 processes: cleavage of a dextrin branch
from the remaining glycogen molecule (amylo-1,6-glucosidase
activity), as well as through the transfer of the dextrin to the free end
of a dextran polymer (oligo-1,4-1,4-glucanotransferase activity) to
release free glucose molecules. The defects associated with this
disorder, and thus their clinical manifestation in patients, are
variable.
GSD type III, in contrast to GSD type I, can be associated with
weakness, wasting of skeletal muscle, hypotonia, and cardiac
defects. Cardiomyopathy can be found with or without left
ventricular hypertrophy, and often presents in childhood. The
skeletal findings and weakness are often manifested in adulthood.
Osteopenia and osteoporosis commonly occur and appear to be due
to lack of good metabolic control.
Hepatomegaly and elevated transaminase levels generally
improve as patients become older and often normalize after puberty.
Adenomas are rarely found, but they have been reported to progress
to hepatocellular carcinoma.
GSD type VII (phosphofructokinase deficiency) (Answer B), GSD
type VI (liver phosphorylase deficiency) (Answer D), and glucose
transporter 2 deficiency (Answer E) do not present with the above
findings.

Educational Objective
Differentiate among the various glycogen storage disorders on the
basis of clinical findings.

Reference(s)
Kanungo S, Wells K, Tribett T, El-Gharbawy A. Glycogen metabolism and glycogen storage
disorders. Ann Transl Med. 2018;6(24):474. PMID: 30740405
Kishnani PS, Austin SL, Arn P, et al. Glycogen storage disease type III diagnosis and
management guidelines [published correction appears in Genet Med. 2010;12(9):566].
Genet Med. 2010;12(7):446-463. PMID: 20631546
8 ANSWER: E) Loss-of-function pathogenic variant in the
PAPPA2 gene (pappalysin 2)
Recently described pathogenic variants in the PAPPA2 gene
(pappalysin 2; pregnancy-associated plasma protein-A2) (Answer E)
are associated with an autosomal recessive syndrome of postnatal
growth failure and short stature. Individuals homozygous or
compound heterozygous for pathogenic variants in this gene present
with postnatal growth failure with varying degrees of proportionate
short stature. No major dysmorphic features have been described
other than mild microcephaly and long, thin bones on x-ray. This
patient’s parents are probably heterozygous and are unaffected.
Given the presence of consanguinity in this family, the risk of
autosomal recessive disorders is increased.
The IGF-1 half-life is increased by its binding to IGFBPs (IGF-
binding proteins) in circulation. Only free IGF-1 can interact with the
IGF-1 receptor to induce biologic activity. PAPP-A2 is a
metalloproteinase that releases IGF-1 from the complexes it forms
with the binding proteins (mainly IGFBP-3 and IGFBP-5).
Inactivating pathogenic variants in PAPPA2, therefore, result in
decreased levels of free IGF-1, leading to decreased linear growth.
The absence of negative feedback of free IGF-1 on GH synthesis
leads to an increase in GH production, which, in turn, induces an
increase in the hepatic production of IGF-1, IGF-2, and IGFBP-3.
Prenatal growth is dependent on IGF-1 and IGF-2. Fetal production
of IGFs is largely independent from GH regulation except during the
last few weeks of gestation. Normal maternal PAPP-A2 potentially
maintains adequate free IGF levels in the fetus; therefore, no
intrauterine growth failure occurs, which is consistent with this
patient’s history of intrauterine growth that was adequate for
gestational age based on birth weight. She has proportionate short
stature as suggested by very similar measurements of arm span and
height. Measurements of free IGF-1 would show low levels in her
case. However, this test is only available in research laboratories and
not yet in clinical practice.
Loss-of-function pathogenic variants in the GHR gene (Answer
A) are responsible for Laron-type dwarfism, also an autosomal
recessive disorder characterized by severe postnatal growth failure.
While GH levels may be elevated because of GH insensitivity, IGF-1
levels are low. Additionally, measurements of GH-binding protein,
which represents the extracellular domain of the GH receptor, are
low or undetectable. Gain-of-function pathogenic variants in the
GHR gene (Answer B) have not been described, but they would
theoretically result in an overgrowth syndrome.
Signal transducer and activator of transcription (STAT) 5b is one
of the proteins in the GH postreceptor cascade. Inactivating
pathogenic variants in the STAT5B gene (Answer D) lead to a Laron-
like syndrome with postnatal growth failure, immune deficiency
manifested by frequent respiratory infections, normal GH-binding
protein, and elevated GH levels but low IGF-1 levels.
Inactivating pathogenic variants in the gene encoding the IGF-1
receptor (Answer C) are characterized by both prenatal and
postnatal growth failure given that prenatal growth is dependent on
IGF-1 and IGF-2 to which these individuals are unresponsive.
Intrauterine growth is not dependent on GH, except for the last few
weeks of gestation. GH therapy may induce a slight improvement in
postnatal growth velocity, probably due to direct (ie, not mediated
by IGF-1) action of GH on the growth plates.

Educational Objective
Explain the importance of IGF-1 bioavailability in the normal
functioning of the GH-IGF-1 axis and identify pathogenic variants in
the PAPPA2 gene as a cause of postnatal growth failure and short
stature.

Reference(s)
Rosenfeld R. Insulin-like growth factors and the basis of growth. N Engl J Med.
2003;349(23):2184-2186. PMID: 14657423
Dauber A, Muñoz-Calvo MT, Barrios V, et al. Mutations in pregnancy-associated plasma
protein A2 cause short stature due to low IGF-I availability. EMBO Mol Med.
2016;8(4):363-374. PMID: 26902202
Argente J, Chowen JA, Pérez-Jurado LA, Frystyk J, Oxvig C. One level up: abnormal
proteolytic regulation of IGF activity plays a role in human pathophysiology. EMBO
Mol Med. 2017;9(10):1338-1345. PMID: 28801361
Kofoed EM, Hwa V, Little B, et al. Growth hormone insensitivity associated with STAT5b
mutation. N Engl J Med. 2003;349(12):1139-1147. PMID: 13679528
Abuzzahab MJ, Schneider A, Goddard A, et al; Intrauterine Growth Retardation (IUGR)
Study Group. IGF-1 receptor mutations resulting in intrauterine and postnatal growth
retardation. N Engl J Med. 2003;349(23):2211-2222. PMID: 14657428
9 ANSWER: A) Measure TSH and T4 in 1 week (off
levothyroxine) and measure the mother’s TSH and T4
With a TSH value greater than 100 mIU/L due to congenital
hypothyroidism, one would expect a much lower total T4 or free T4
level. Without a firm diagnosis of congenital hypothyroidism,
starting levothyroxine (Answer C) would be incorrect and might
prevent completion of the diagnostic evaluation, or in this case in
which free and total T4 are elevated, it may result in overtreatment.
Guidelines in Europe (European Society for Paediatric
Endocrinology) and the United States (American Academy of
Pediatrics, American Thyroid Association) do not directly address a
scenario whereby total or free T4 is elevated. Further observation
and testing is usually reserved for instances in which TSH is mildly
elevated and T4 is normal. At this patient’s visit on day of life 8, he
did not have jaundice, as might be expected in congenital
hypothyroidism. Lower T4 is also expected in resistance to TSH in
those who are homozygous or compound heterozygous for
pathogenic variants in TSHR, and TSH is often not as high as in this
case, although there is a wide range. Resistance to TSH can also be
observed in Albright hereditary osteodystrophy with PTH
resistance, but TSH is only mildly elevated in affected patients.
Therefore, measuring calcium and PTH (Answer B) is incorrect.
Mothers can transfer thyroid antibodies to the fetus, including
TSH-receptor antibodies, which can block the TSH receptor and are
expected to lead to high TSH and low or low-normal T4. In the right
clinical setting, it is recommended to measure the level of such
blocking antibodies in the baby. Therefore, assessing the mother’s
thyroid antibodies (Answer D) is incorrect.
Thyroid ultrasonography (Answer E) is helpful in distinguishing
between cases of congenital hypothyroidism due to an ectopic gland
vs a gland located in the normal position, but this is not indicated
here, as the technetium nuclear scanning showed a normal-sized
thyroid gland in the correct location. This baby had normal TSH
receptor–binding inhibiting immunoglobulins. This suggests the
possibility of abnormal TSH or assay interference. Although much
has been talked about regarding the effects of biotin, this is not
relevant in this scenario on day of life 4 and onwards, even if the
mother had been taking biotin during pregnancy. Repeated
laboratory tests using different TSH assays and methodologies have
shown normal T4 levels associated with elevated TSH. On day of life
16, this baby’s TSH value was 27.4 mIU/L, and the total T4 value
was 9.6 µg/dL (123.6 nmol/L) (same assay as the one used for
testing on day of life 8).
Testing to assess for evidence of interfering heterophile antibodies
was negative. HAMA wash yielded the same result.
Macro-TSH is a large circulating form of TSH composed of
monomeric TSH complexed with autoimmune TSH antibodies.
Macro-TSH can be detected on gel filtration chromatography with a
prevalence ranging from 0.6% to 1.6%. Unlike TSH, which is a small
bioactive hormone of 28 kDa easily filtered by the kidney, macro-
TSH is a large molecule of at least 150 kDa that most likely
accumulates in the circulation, resulting in measurements indicating
falsely increased TSH levels. Currently, none of the available 2-site
immunometric assays used for TSH testing can completely
discriminate macro-TSH from bioactive free TSH, even if some
platforms are more sensitive to its presence. Interference should be
suspected in a patient with isolated TSH elevation (typically
markedly elevated) with thyroid hormones in the upper half of the
normal range and no signs or symptoms of thyroid dysfunction. An
increased recovery of diluted samples showing nonlinearity may be
indicative of macro-TSH. It should be noted, however, that the
dilution procedure is neither specific nor sensitive. Lack of
parallelism can be encountered with other interfering antibodies (eg,
heterophilic antibodies). Gel filtration chromatography proved this
patient had macro-TSH.
Transplacental transfer of macro-TSH has been documented in
neonates as a cause of falsely elevated TSH, which is why it is
important to recheck the TSH and T4 levels a week later in the baby
and in the mother (Answer A). Laboratory studies documented the
following results in the patient’s mother:
TSH = 16.42 mIU/L (0.5-4.30 mIU/L)
Total T3 = 118 ng/dL (86-192 ng/dL) (SI: 1.8 nmol/L [1.3-3.0 nmol/L])
Total T4 = 9.0 µg/dL (4.5-12.0 µg/dL) (SI: 115.8 nmol/L [57.9-154.4 nmol/L])

At 6 weeks of life, the infant’s TSH concentration was still 21.4


mIU/L. Only at 7 months of life did it normalize, as the
transplacental macro-TSH was washed out.

Educational Objective
Diagnose macro-TSH as a confounder in newborns with elevated
TSH and normal total or free T4.

Reference(s)
Sunthornthepvarakul T, Gottschalk ME, Hayashi Y, Refetoff S. Brief report: resistance to
thyrotropin caused by mutations in the thyrotropin-receptor gene. N Engl J Med.
1995;332(3):155-160. PMID: 7528344
Favresse J, Burlacu MC, Maiter D, Gruson D. Interferences with thyroid function
immunoassays: clinical implications and detection algorithm. Endocr Rev.
2018;39(5):830-850. PMID: 29982406
American Academy of Pediatrics, Rose SR, Section on Endocrinology and Committee on
Genetics, et al. Update of newborn screening and therapy for congenital
hypothyroidism. Pediatrics. 2006;117(6):2290-2303. PMID: 16740880
Leger J, Olivieri A, Donaldson M, et al; ESPE-PSE-SLEP-JSPE-APEG-APPES-ISPAE;
Congenital Hypothyroidism Consensus Conference Group. European Society for
Paediatric Endocrinology consensus guidelines on screening, diagnosis, and
management of congenital hypothyroidism. Horm Res Paediatr. 2014;81(2):80-103. PMID:
24662106
10 ANSWER: C) Measurement of urinary or plasma
metanephrines
Urinary or plasma metanephrine measurement (Answer C) would
be the most useful test to establish the presence of a
pheochromocytoma and/or a catecholamine-secreting
paraganglioma, which is the most likely diagnosis in view of the
paroxysmal nature of the classic triad of symptoms (palpitations,
diaphoresis, and headaches). Normal thyroid function test results
and normal renal function exclude thyrotoxicosis and chronic renal
disease as causes, which are on the list of differential diagnoses.
Echocardiography (Answer A) could be useful to exclude a cardiac
cause of hypertension such as aortic coarctation or to assess end-
organ damage. Similarly, head MRI (Answer B) could be useful if
Cushing disease were suspected or to assess the consequences of
hypertension. However, neither of these options would be the next
diagnostic investigation considering the symptomatology. MIBG
scan (Answer D) and abdominal CT (Answer E) would be secondary
investigations to locate the lesion following a biochemical diagnosis
of pheochromocytoma and/or catecholamine-secreting
paraganglioma.
The 24-hour urine metanephrine results in this case are shown
(see table).
Analyte Result Reference Range
Urine volume 961 mL 0-4440 mL
Normetanephrine 1765 μg/24 h (SI: 9637 nmol/d) 0-366 μg/24 h (SI: 0-2000 nmol/d)
Metanephrine 114 μg/24 h (SI: 579 nmol/d) 0-493 μg/24 h (SI: 0-2500 nmol/d)

This patient had no phenotypic features of Cushing syndrome. MRI


brain showed multifocal cortical deep white matter and basal
ganglia edema predominantly within the basal ganglia, occipital
lobes, and brain stem, findings consistent with posterior reversible
encephalopathy syndrome. Findings on echocardiography were
normal, with an ejection fraction of 60% and no evidence of
cardiomyopathy. MIBG scan showed avid uptake within the left
adrenal gland, while abdominal CT showed a 4.1-cm lesion anterior
to the left renal pelvis displacing the renal vessels.
Phenoxybenzamine, 10 mg twice daily were initiated. He was
discharged home on phenoxybenzamine, 20 mg 4 times daily, and
propranolol, 4 times daily. He returned 2 weeks later for an elective
left adrenalectomy and genetic testing.
Pheochromocytomas and catecholamine-secreting
paragangliomas are neuroendocrine tumors derived from
chromaffin cells of the adrenal medulla or tumors that originate from
the paraganglia of the autonomic nervous system.
Pheochromocytomas and catecholamine-secreting paragangliomas
account for 0.1% to 0.6% of cases of hypertension in adults and 1% of
cases in pediatric patients. The reported incidence is 2 to 5 cases per
million per year, of which 10% occur in children.
Pheochromocytomas and catecholamine-secreting paragangliomas
secrete catecholamines and other peptides. They cause morbidity
and mortality through their secretory effects, mass effect, and/or
malignant potential. The complete classic symptom triad of
palpitations, diaphoresis, and headaches occurs in only 4% of
patients, making the diagnosis difficult. Affected individuals can
present with a wide range of symptoms, including anxiety, fatigue,
weight loss, heat intolerance, vision disturbance, pallor, tremors,
abdominal or chest pain, nausea, dizziness, change in bowel habits,
neurologic symptoms, and worsening or new-onset diabetes. The
episodic nature of symptoms with complete resolution between
episodes should trigger a high index of clinical suspicion, and the
diagnosis should be considered in all patients with sustained
hypertension (60%-90% of pediatric cases), difficult-to-control
hypertension, symptomatic hypertension, or a positive family
history.
Biochemical studies in the form of measuring 24-hour urinary
metanephrines or plasma metanephrines are the first-line
investigation to exclude pheochromocytomas and catecholamine-
secreting paragangliomas. Careful evaluation is required in those
with results in the upper-normal reference range. False-negative
results are uncommon, except in cases of incomplete 24-hour urinary
collection. Plasma metanephrines can be more reliable and
convenient for this reason. However, false-positive results do occur.
Medications including acetominophen, α- and β-adrenergic blockers,
tricyclic antidepressants, monoamine oxidase inhibitors, and
sympathomimetic agents (nicotine, ephedrine, pseudoephedrine,
cocaine) can all cause false-positive results. Circumstances of sample
collection are also important. For plasma metanephrines, the patient
should fast from midnight and be supine for 30 minutes before the
blood test. The sample must be centrifuged within 2 hours. For
urinary metanephrines, a complete 24-hour urine collection is
required for accuracy. In both cases, patients should avoid common
medications such as acetominophen and pseudoephedrine on the
day of the test.
Imaging and genetic testing are recommended once a diagnosis of
pheochromocytoma or catecholamine-secreting paraganglioma is
established. Currently, it is recognized that 40% to 50% of
pheochromocytomas and catecholamine-secreting paragangliomas
are due to underlying genetic pathogenic variants. These fall into 2
clusters of genes. Cluster 1 includes pathogenic variants in genes
encoding the von Hippel-Lindau tumor suppressor (VHL), subunits
of succinate dehydrogenase (SDHA, SDHB, SDHC, and SDHD),
succinate dehydrogenase complex assembly factor 2 (SDHAF2),
fumarate hydratase (FH), malate dehydrogenase 2 (MDH2), and egl-
9 family hypoxia inducible factors (EGLN1, EGLN2), which result in
activation of hypoxia signaling pathways. Cluster 2 includes
pathogenic variants in genes encoding neurofibromatosis type 1
(NF1), ret proto-oncogene (RET), transmembrane protein 127
(TMEM127), MYC-associated factor X (MAX), HRas proto-oncogene
GTPase (HRAS), and endothelial PAS domain protein 1 (EPAS1),
which affect kinase receptor signaling pathways.

Educational Objective
Diagnose pheochromocytoma in a child and conduct the appropriate
investigations.
Reference(s)
Pamporaki C, Hamplova B, Peitzsch M, et al. Characteristics of pediatric vs adult
pheochromocytomas and paragangliomas. J Clin Endocrinol Metab. 2017;102(4):1122-
1132. PMID: 28324046
Lenders JW, Duh QY, Eisenhofer G, et al; Endocrine Society. Pheochromocytoma and
paraganglioma: an endocrine society clinical practice guideline. J Clin Endocrinol Metab.
2014;99(6):1915-1942. PMID: 24893135
Martucci VL, Pacak K. Pheochromocytoma and paraganglioma: diagnosis, genetics,
management, and treatment. Curr Probl Cancer. 2014;38(1):7-41. PMID: 24636754
Bholah R, Bunchman TE, Review of pediatric pheochromocytoma and paraganglioma. Front
Pediatr. 2017;5:155. PMID: 28752085
11 ANSWER: C) Effect of the somatostatin analogue
This patient presented with rapid growth, tall stature, galactorrhea,
irregular menses, elevated prolactin, elevated GH, elevated IGF-1,
failure of GH to suppress with oral glucose tolerance testing, and a
sellar/suprasellar mass on MRI—findings consistent with a
GH/prolactin-secreting pituitary adenoma. First-line therapy for a
GH/prolactin-secreting macroadenoma is surgical resection.
However, this patient continued to have evidence of GH excess
postoperatively and was therefore treated with a somatostatin
analogue.
Somatostatin is an inhibitory 14–amino acid peptide hormone
present both in the hypothalamus and peripheral tissues.
Somatostatin is involved in the inhibition of the release of GH and
TSH, as well as gastrointestinal hormones, pancreatic enzymes, and
neuropeptides. Somatostatin binds to receptors belonging to the 7
transmembrane G-protein–coupled receptor superfamily. Analogues
of somatostatin, such as octreotide and lanreotide, were developed
and initially used for the treatment of acromegaly and
gastroenteropancreatic tumors and are highly effective for the
treatment of GH excess. Because somatostatin inhibits TSH release,
treatment with a somatostatin analogue (Answer C) is the most
likely cause of this patient’s low TSH level.
While destruction of thyrotrope cells can occur with a large mass
(Answer A), this girl’s thyroid function tests were normal at
presentation. Surgical resection (Answer B) could result in central
hypothyroidism, gonadotropin deficiency, ACTH deficiency, and
even diabetes insipidus. However, her thyroid function tests were
normal postoperatively. Hyperprolactinemia and GH excess were
present initially, but prolactin levels (Answer D) normalized
postoperatively. GH and IGF-1 levels (Answer E) normalized with
medical therapy.

Educational Objective
Explain the effects of somatostatin and somatostatin analogues on
TSH and GH secretion.

Reference(s)
Theodoropoulou M, Stalla GK. Somatostatin receptors: from signaling to clinical practice.
Front Neuroendocrinol. 2013;34(3):228-252. PMID: 23872332
12 ANSWER: A) Measure uric acid, order a lipid panel, and refer
to a metabolic nutritionist
Patients with hypoglycemia pose an interesting challenge to
everyday clinical practice in endocrinology. GH deficiency, cortisol
deficiency, and hyperinsulinemia could result in hypoglycemia.
However, hepatomegaly (as evidenced by the protruding abdomen
in this patient) and elevated lactate levels in the setting of
hypoglycemia are seen in patients with glycogen storage disease
(GSD). Elevated lactate levels and b-hydroxybutyrate with acidosis
at the time of hypoglycemia in a patient with hepatomegaly are
indications of possible GSD and warrant further metabolic workup
and treatment. Patients with GSD type 1 also have hyperuricemia
and hypertriglyceridemia. Therefore, measuring uric acid and
performing a lipid panel (Answer A) is the best next step.
GSD type 1 occurs in patients with homozygous or compound
heterozygous pathogenic variants in the genes encoding glucose-6-
phosphatase (GSD type 1a) or glucose-6-phosphate translocase (GSD
type 1b). Patients with GSD type 1 can present with hypoglycemia in
the neonatal period, infancy, or early childhood, but more commonly
present at 3 to 6 months of age. Clinical characteristics can mimic
those of Cushing syndrome, including doll-like facies, poor growth,
short stature, and a distended abdomen due to pronounced
hepatomegaly and nephromegaly. Biochemical manifestations
include hypoglycemia, hyperlipidemia, hypertriglyceridemia,
hyperlactatemia, and hyperuricemia. Patients with GSD type 1b also
have neutropenia and impaired neutrophil function, resulting in
recurrent bacterial infections and oral and intestinal mucosa
ulceration. Patients with GSD type 1 do not have skeletal myopathy
or increased creatine kinase levels, which are characteristic of GSD
type 3a.
Frequent feeds with complex carbohydrates and cornstarch every
3 to 5 hours, as well as close monitoring of blood glucose levels, are
essential in patients with GSD type 1. Long-acting cornstarch is
approved for overnight use in children older than 5 years. In 2018,
the US FDA approved the first gene therapy trial for patients with
GSD type 1, which, if successful, could ultimately obviate the
lifelong dependence on cornstarch for survival in these patients.
GH and cortisol measured at the time of hypoglycemia may not
be high enough to exclude GH deficiency and cortisol deficiency.
Thus, in a patient with clinical and laboratory findings suggestive of
a metabolic disorder such as GSD type 1, proceeding to expensive
diagnostic testing such as MRI that could be also be invasive if it
requires sedation given the patient’s age (Answer C) is incorrect.
Proceeding with cosyntropin-stimulation testing (Answer B) or GH-
simulation testing (Answer E) could delay the correct diagnosis or
potentially result in wrong diagnosis and neither is the best next
step. Follow-up in 6 months without further investigation and in-
depth counseling on diet (Answer D) could be life-threatening in
patients with GSD type 1 and is thus is not acceptable.

Educational Objective
Diagnose glycogen storage disease in a patient with hypoglycemia
based on their concurrent clinical and laboratory findings.

Reference(s)
Kishnani PS, Austin SL, Abdenur JE, et al. Diagnosis and management of glycogen storage
disease type I: a practice guideline of the American College of Medical Genetics and
Genomics. Genet Med. 2014;16(11):e1. PMID: 25356975
Heller S, Worona L, Consuelo A. Nutritional therapy for glycogen storage diseases. J Pediatr
Gastroenterol Nutr. 2008;47(Suppl 1):S15-S21. PMID: 18667910
Kelly A, Tang R, Becker S, Stanley CA. Poor specificity of low growth hormone and cortisol
levels during fasting hypoglycemia for the diagnoses of growth hormone deficiency
and adrenal insufficiency. Pediatrics. 2008;122(3):e522-e528. PMID: 18694902
13 ANSWER: D) High risk for aggressive medullary thyroid
carcinoma; total thyroidectomy is recommended now
The RET proto-oncogene encodes a transmembrane receptor of the
tyrosine kinase family. It has a fundamental role in the development
of sporadic or familial aggressive medullary thyroid carcinoma
(MTC) and other malignant and nonmalignant diseases. Genetic
counseling and genetic testing for germline pathogenic variants in
RET should be offered to first-degree relatives of patients with
proven MTC or known RET pathogenic variants.
In 2015, the American Thyroid Association published revised
guidelines and changed the risk categories for MTC. Detailed
information on risk classification of patients based on their RET
pathogenic variant can be found in these guidelines. The highest-risk
category includes patients with multiple endocrine neoplasia (MEN)
type 2B and the RET codon M918T pathogenic variant. The high-risk
category includes patients with RET pathogenic variants in codon
634 and the codon A883F pathogenic variant. Other common
pathogenic variants, previously labeled as A and B categories, are
now referred to as moderate-risk pathogenic variants.
Patients with MEN 2B have marfanoid body habitus, narrow,
long facies, generalized ganglioneuromatosis, skeletal malformations
(usually pes cavus, although other skeletal malformations can be
seen), and ophthalmologic abnormalities (ie, alacrima, prominent
corneal nerve fibers across corneas, plexiform subconjunctival
neuromas, conjunctival hyperemia with superficial peripheral
corneal neovascularization, and blepharitis) (Answer E). The patient
described in the vignette does not have the M918T pathogenic
variant seen in 95% of patients with MEN 2B, and she also has a
normal physical examination. Patients with a pathogenic variant in
this codon require total thyroidectomy before the age of 1 year.
For patients with the RET C634R pathogenic variant, it is
recommended to screen for MTC with annual physical examination,
cervical ultrasonography, and measurement of calcitonin starting at
age 3 years. Studies have shown no lymph node metastasis in
children with MEN 2A when the calcitonin level is below 30 or 40
pg/mL (<8.8 or 11.7 pmol/L). However, all patients with MEN 2A
require total thyroidectomy. For children in the high-risk category,
total thyroidectomy is recommended at or before age 5 years. The
patient in this vignette is at high risk to have aggressive MTC. She is
7 years old, and total thyroidectomy ideally should have been done
sooner. Therefore, this procedure is recommended now (Answer D).
Patients with RET pathogenic variants at moderate risk should have
total thyroidectomy in childhood or young adulthood, with the
timing of operation being dependent primarily on calcitonin levels.
Pheochromocytoma may develop before age 12 years in patients
with MEN 2, although it has been reported as early as 8 years.
Currently, it is recommended to start routine screening at age 11
years for patients in the high- and highest-risk categories and at the
age of 16 years for patients in the moderate-risk category, although
earlier screening could be done based on clinical judgment.
However, pheochromocytoma screening (Answer B) is not the best
answer.
Hyperparathyroidism has been reported to occur as early as 2, 6,
7, and 10 years of age in patients in the high- and moderate-risk
categories. However, current guidelines recommend screening by
age 11 years for patients at high risk and by age 16 years for patients
at moderate risk. Thus, screening for hyperparathyroidism (Answer
A) is not the best choice.
Patients with certain RET pathogenic variants are at risk for
developing Hirschsprung disease and cutaneous lichen amyloidosis
(Answer C). The RET C634R pathogenic variant is associated with
cutaneous lichen amyloidosis. In patients with MEN 2A, this
condition manifests as localized, interscapular amyloidosis, usually
preceded by pruritus. Although she may develop this condition, her
examination findings are not consistent with this diagnosis. Also, the
RET C634R mutation is not associated with Hirschsprung disease.
This patient’s calcitonin concentration was elevated at 109 pg/mL
(31.8 pg/mL). She underwent total thyroidectomy and was found to
have MTC with local invasion and angioinvasion, localized to the
thyroid. Five lymph nodes were negative for carcinoma. Although
she has a 20% probability of developing hyperparathyroidism and a
50% probability of developing pheochromocytoma, the priority now
is to proceed with total thyroidectomy. Screening for
pheochromocytoma and hyperparathyroidism is recommended by
age 11 years, although one could consider screening sooner if a
patient has symptoms or abnormal vital signs.

Educational Objective
Guide decision-making regarding prophylactic thyroidectomy in a
family with multiple endocrine neoplasia type 2A.

Reference(s)
Wells SA Jr, Asa SL, Dralle H, et al; American Thyroid Association Guidelines Task Force on
Medullary Thyroid Carcinoma. Revised American Thyroid Association guidelines for
the management of medullary thyroid carcinoma. Thyroid. 2015;25(6):567-610. PMID:
25810047
Verga U, Fugazzola L, Cambiaghi S, et al. Frequent association between MEN 2A and
cutaneous lichen amyloidosis. Clin Endocrinol (Oxf). 2003;59(2):156-161. PMID: 12864791
Castinetti F, Moley J, Mulligan L, Waguespack SG. A comprehensive review on MEN2B.
Endocr Relat Cancer. 2018;25(2):T29-T39. PMID: 28698189
14 ANSWER: B) LH, low; TSH, very high; hCG, normal
LH, FSH, TSH, and hCG are glycoprotein hormones consisting of 2
chains, the α- and β-subunits, which are noncovalently bound. The α-
subunits of LH, FSH, hCG, and TSH are identical in amino acid
sequence (92 amino acids), although each of their β chains is unique
and is responsible for the biologic specificity of the intact hormones.
However, approximately 80% of the first 115 amino acids of the β-
subunit of hCG is homologous to the sequence of the β-subunit of
LH.
This girl presented with early pubertal changes with rapid
progression to vaginal bleeding but slow linear growth. Her
examination findings are notable for short stature, normal weight,
presence of secondary sexual characteristics, periorbital edema, and
hypertrichosis. Her bone age is delayed. These findings are
consistent with primary hypothyroidism leading to the so-called
overlap or Van Wyk-Grumbach syndrome. In this scenario, TSH
would be markedly elevated, LH would be low, and hCG would be
normal (Answer B). FSH levels can be normal or elevated. One
theory to explain the mechanism that occurs in Van Wyk-Grumbach
syndrome is thought to be related to a markedly elevated TSH
concentration exerting FSH-like effects at the FSH receptor, via the
common α-subunit, leading to an increase in gonadal size and
steroidogenesis. The isolated rise in FSH is postulated to be from
slowing of the GnRH pulse frequency in primary hypothyroidism.
The low LH may be due to hyperprolactinemia from increased
thyrotropin-releasing hormone leading to decreased GnRH
secretion. In girls, the increase in estrogen from the ovaries leads to
breast development and uterine bleeding in the absence of pubic and
axillary hair development. The patient in this vignette did have
pubic hair development; however, this can be normal for age. In
boys, testicular enlargement without other signs of virilization is
seen.
Gonadotropin-dependent precocious puberty, in which LH and
FSH would be increased for age and TSH and hCG would be normal
(Answer A), would be expected to cause linear growth acceleration,
tall stature, and advanced bone age. Granulosa-cell tumors of the
ovary, in which estradiol levels would be high, with suppressed LH
and FSH levels and normal TSH and hCG levels (Answer C), usually
present with rapid onset of pubertal changes and vaginal bleeding
but would not be expected to be associated with short stature,
periorbital edema, or hypertrichosis. A germ-cell tumor of the
central nervous system, in which hCG levels are elevated and LH
levels are low, can manifest with early pubertal changes. However,
this phenomenon is only seen in boys. hCG and LH share the same
receptor, so elevations in hCG would be expected to result in
secondary sexual characteristics in boys due to stimulation of Leydig
cells, with subsequent testosterone production. However, in girls,
both LH (or hCG) and FSH are needed for ovarian estradiol
production. A germ-cell tumor of the central nervous system would
also not be likely to have an elevated TSH level as in Answer D. Low
LH, low FSH, low TSH, and normal hCG (Answer E) might be seen
in a girl with McCune-Albright syndrome who has gonadotropin-
independent precocious puberty and hyperthyroidism due to
constitutive activation of the G-protein–stimulatory subunit. Linear
growth acceleration with bone age advancement would be expected
in this scenario. Also, this patient has no evidence of café-au-lait
macules or fibrous dysplasia on examination.

Educational Objective
Explain that LH, FSH, TSH, and hCG are heterodimers composed of
a common α-subunit and a hormone-specific β-subunit.

Reference(s)
Eugster EA. Update on precocious puberty in girls. J Pediatr Adolesc Gynecol. 2019;32(5):455-
459. PMID: 31158483
Van Wyk JJ, Grumbach MM. Syndrome of precocious menstruation and galactorrhea in
juvenile hypothyroidism: an example of hormonal overlap in pituitary feedback. J
Pediatr. 1960;57:416-435.
Hachicha M, Maaloul I, Aissa K, Kamoun T, Aloulou H. Van Wyk-Grumbach syndrome: a
rare cause of precocious puberty. Presse Med. 2018;47(5):483-486. PMID: 29555166
15 ANSWER: D) Magnesium deficiency secondary to inadequate
supplementation
Magnesium is a divalent cation (as is calcium) essential for both the
secretion and function of PTH. In this clinical scenario, the patient
has symptoms of profound tetany despite a corrected calcium of 7.7
mg/dL (1.9 mmol/L). PTH is exquisitely sensitive to calcium
changes. Thus, the value of 40 pg/mL (40 ng/L) is lower than
expected. However, this is due to the profound hypomagnesemia
impairing the function of the calcium-sensing receptor. Acquired
hypoparathyroidism (Answer B) is theoretically possible, but it
would require ongoing treatment with calcium and calcitriol and
would not resolve with magnesium supplementation alone. In
addition, hypoparathyroidism is associated with
hyperphosphatemia and an increased fractional excretion of calcium
in the urine, neither of which is present in this patient.
Rare autosomal recessive pathogenic variants in the TRPM6 gene
(Answer A) on chromosome 9q22 cause hypomagnesemia with
impaired urinary magnesium excretion. Affected patients present
with symptoms of seizures and tetany in the first few weeks of life.
Despite having a congenital malabsorption syndrome, her bowel
pattern has not changed, and calcium, magnesium, and phosphate
levels have previously been normal.
Gitelman syndrome (Answer E) is due to a defect in the sodium-
chloride channel causing impaired resorption of several electrolytes
at the distal tubule of the kidney. Laboratory findings include
hypokalemic metabolic alkalosis, hypocalciuria, and
hypomagnesemia due to renal magnesium wasting. Affected
patients generally present in later childhood with symptoms of
muscle weakness, lethargy, muscle cramps, and tetany. Because most
calcium reabsorption is mediated by the calcium-sensing receptor
thick ascending loop of Henle, these patients have low urinary
calcium excretion in contrast to patients with Bartter syndrome.
This patient’s pathogenic variants in SPINT2 cause intractable
watery diarrhea and tufting of enterocytes, leading to malabsorption
and metabolic acidosis with resultant intestinal failure. Affected
patients also present with woolly hair, intrauterine growth
restriction, short stature, immunodeficiency, skin abnormalities, and
liver disease. While enteric magnesium losses are possible at
diagnosis, she has not had any changes in her bowel habits, and
prior electrolyte measurements have been normal. Shortages of
various electrolytes and micronutrients have been known to cause
significant clinical consequences if patients are not carefully
monitored. Patients on total parenteral nutrition are at risk for
hypothyroidism due to iodine deficiency, which has been a previous
concern for this patient. Iodine deficiency (Answer C) can cause
fatigue, weakness, dry brittle hair, and dry skin, but it would not be
expected to cause hypomagnesemia. Her TSH level was also checked
and was normal. At the time of this patient’s episode, there was a
national shortage of magnesium. Because she was receiving some
nutrition enterally, she was not identified as an at-risk patient
despite her history of malabsorption. This led to both calcium and
magnesium mineral supplementation being removed from her
parenteral nutrition 2 weeks before her presentation (Answer D).

Educational Objective
Diagnose magnesium deficiency, which can present similarly to
hypocalcemia, and identify total parenteral nutrition as potentially
placing a patient at risk.

Reference(s)
Walder RY, Landau D, Meyer P, et al. Mutation of TRPM6 causes familial hypomagnesemia
with secondary hypocalcemia. Nat Genet. 2002;31(2):171-174. PMID: 12032570
Seyberth HW, Weber S, Komhoff M. Bartter’s and Gitelman’s syndrome. Curr Opin Pediatr.
2017;29(2):179-186. PMID: 27906863
Ikomi C, Cole CR, Vale E, et al. Hypothyroidism and iodine deficiency in children on
chronic parenteral nutrition. Pediatrics. 2018;141(4). PMID: 29496904
Plogsted S, Adams SC, Allen K, et al; Clinical Practice Committee’s Nutrition Product
Shortage Subcommittee of the American Society for Parenteral and Enteral Nutrition.
Parenteral nutrition electrolyte and mineral product shortage considerations. Nutr Clin
Pract. 2016;31(1):132-134. PMID: 26703958
16 ANSWER: D) Apolipoprotein B
Polycystic ovary syndrome is a known metabolic disorder associated
with insulin resistance and dyslipidemia. Women with this condition
often have metabolic syndrome. Metabolic syndrome is
characterized by a group of cardiovascular risk factors, including
central obesity, high blood pressure, elevated triglycerides, low HDL
cholesterol (thus, Answer E is incorrect), and elevated fasting
glucose. In pediatric patients, metabolic syndrome is most often
diagnosed according to criteria that are adapted from adult criteria,
such as those developed by the National Cholesterol Education
Program. The most commonly used criteria specify the following
cutoff values:

Fasting blood glucose = typically >100 mg/dL (SI: >5.5


mmol/L)
HDL cholesterol = <40 mg/dL (SI: <1.04 mmol/L)
Triglycerides = >110 mg/dL (SI: >1.24 mmol/L)
Blood pressure greater than the 90th percentile for sex, age,
and height
Waist circumference greater than the 90th percentile for age
and sex
To be classified as having metabolic syndrome, an adolescent must
have abnormal values for at least 3 of these 5 criteria.
Polycystic ovary syndrome contributes to the metabolic
derangements with hyperandrogenemia, which has been associated
with enhanced lipogenesis. While patients with polycystic ovary
syndrome often have insulin resistance (Answer A), recent studies
support the idea that the presence of elevated apolipoprotein B
(Answer D), such as intestinal apolipoprotein B48 and hepatic
apolipoprotein B100, are strong predictors of future cardiovascular
disease. Increased lipid synthesis via increased activation of the
nuclear transcription factor sterol regulatory-element binding
protein-1c is also a strong predictor of future cardiovascular disease.
African American patients appear to manifest metabolic
syndrome differently than white and Hispanic patients. Black
patients with metabolic syndrome are much less likely to exhibit
elevated triglycerides. In a survey of 2456 adolescents, Johnson et al
documented that although 26% to 32% of white and Hispanic
adolescents had triglyceride concentrations above 110 mg/dL (>1.24
mmol/L), only 10% of black adolescents exhibited triglyceride
elevations (Answer C), potentially because of lower levels of
lipoprotein lipase. LDL-cholesterol elevations (Answer B) are not
significantly associated with metabolic syndrome. The mainstay of
treatment for metabolic syndrome and related abnormalities is
lifestyle modification with increased physical activity and decreased
caloric intake. However, metformin therapy has been associated with
improvements in a number of the metabolic derangements noted in
polycystic ovary syndrome in adolescent and adult women.

Educational Objective
List the lipid characteristics associated with metabolic syndrome in
the context of polycystic ovary syndrome in adolescent patients and
explain the differences observed in the manifestation of this
syndrome in various racial/ethnic groups.

Reference(s)
Vine DF, Beilin LJ, Burrows S, et al. ApoB48-lipoproteins are associated with
cardiometabolic risk in adolescents with and without polycystic ovary syndrome, J
Endocr Soc. 2020;4(8):bvaa061. PMID: 32803089
Vine DF, Wang Y, Jetha MM, Ball GD, Proctor SD. Impaired ApoB-lipoprotein and
triglyceride metabolism in obese adolescents with polycystic ovary syndrome. J Clin
Endocrinol Metab. 2017;102(3):970-982. PMID: 27997268
Johnson WD, Kroon JJ, Greenway FL, Bouchard C, Ryan D, Katzmarzyk PT. Prevalence of
risk factors for metabolic syndrome in adolescents: National Health and Nutrition
Examination Survey (NHANES), 2001-2006. Arch Pediatr Adolesc Med. 2009;163(4):371-
377. PMID: 19349567
17 ANSWER: E) FNA biopsy of both nodules
The most common form of well-differentiated thyroid cancer is
papillary thyroid cancer (PTC). Because PTC is usually multifocal, it
is crucial to determine via FNA biopsy if a thyroid nodule represents
PTC. While more adults than children have thyroid nodules, thyroid
nodules in children are more likely to be malignant. In pediatric
patients, all instances of FNA biopsy should be ultrasound-guided.
In this case, although the 2 nodules are in the left thyroid lobe,
removing the left lobe alone (Answer A) is incorrect. If this patient
has PTC, she would need completion thyroidectomy surgery.
Although evaluating whether this patient has Hashimoto
thyroiditis (Answer B) could be done, it is not the best next step.
Even if she has autoimmune thyroiditis, one would still need to
perform FNA biopsy to determine whether she has thyroid cancer.
The ultrasonography features of thyroid nodules can indicate
whether a nodule is likely to be malignant. Performing FNA biopsy
of only the largest nodule (Answer C) is incorrect, as the smaller
nodule could be malignant as well. In this case, the smaller nodule
has a few features that confer a higher risk of malignancy: solid and
hypoechoic composition, taller-than-wide shape, increased blood
flow, and a possible microcalcification. Ultrasonography images of
various thyroid nodules, illustrating the features of very low, low,
intermediate, and high suspicion for risk of malignancy in thyroid
nodules are depicted well in the adult American Thyroid
Association guidelines from 2015. The same features apply when
evaluating nodules in pediatric patients. While the smaller lesion has
some high-risk features, performing FNA biopsy on this lesion alone
(Answer D) is incorrect, as this patient requires assessment for
multifocal disease. Irregular margins—another high-risk feature—
were not present. Having cervical adenopathy also raises the
suspicion of thyroid cancer, but this was not noted on
ultrasonography or physical examination in this patient. When
performing FNA biopsy on the larger lesion in this adolescent,
ultrasound guidance should aim to sample the solid areas of the
lesion where malignancy is more likely. For a complete evaluation,
this patient should undergo ultrasound-guided FNA biopsy of both
lesions (Answer E).

Educational Objective
Identify the high-risk features for thyroid cancer on ultrasonography
and evaluate for malignancy in an adolescent with thyroid nodules.

Reference(s)
Francis GL, Waguespack SG, Bauer AJ, et al; American Thyroid Association Guidelines Task
Force. Management guidelines for children with thyroid nodules and differentiated
thyroid cancer. Thyroid. 2015;25(7):716-759. PMID: 25900731
Creo A, Alahdab F, Al Nofal A, Thomas K, Kolbe A, Pittock ST. Ultrasonography and the
American Thyroid Association ultrasound-based risk stratification tool: utility in
pediatric and adolescent thyroid nodules. Horm Res Paediatr. 2018;90(2):93-101. PMID:
30021204
Haugen BR, Alexander EK, Bible KC, et al. 2015 American Thyroid Association
management guidelines for adult patients with thyroid nodules and differentiated
thyroid cancer: the American Thyroid Association Guidelines Task Force on Thyroid
Nodules and Differentiated Thyroid Cancer. Thyroid. 2016;26(1):1-133. PMID: 26462967
18 ANSWER: E) Discuss switching from risperidone to another
atypical antipsychotic with her psychiatrist
Secondary amenorrhea is defined as the absence of menses for 3
consecutive cycles after menarche. Physiologic causes of secondary
amenorrhea include functional hypothalamic amenorrhea,
pregnancy, and lactation. An elevated prolactin level in a patient
with secondary amenorrhea raises concern for pathologic causes
such as prolactin-secreting adenomas, sellar masses, pituitary stalk
disruption, and primary hypothyroidism. Patients with psychiatric
disorders are at higher risk of amenorrhea or menstrual
irregularities, most likely related to hypothalamic-pituitary
dysfunction.
First-generation antipsychotic agents and the atypical
antipsychotic risperidone can increase prolactin levels via dopamine
receptor antagonism. Compared with risperidone, other atypical
antipsychotics (serotonin-dopamine antagonists) and partial
dopamine antagonists (ie, aripiprazole) are associated with lower
serum prolactin levels. Switching from risperidone to another
atypical antipsychotic (Answer E) may help resolve amenorrhea
without worsening psychiatric symptoms, but this should only be
undertaken after discussion with the prescribing physician. If an
elevated prolactin value is documented, current guidelines
recommend discontinuing medications that are known to increase
prolactin before performing additional testing.
While the cause of this patient’s amenorrhea and
hyperprolactinemia is most likely risperidone, this cannot be
assumed without further assessment, so simply providing
reassurance (Answer D) is incorrect. A pituitary MRI (Answer A)
would be recommended only if the antipsychotic medication cannot
be discontinued or if the prolactin level remains above 50 ng/mL
(>2.2 nmol/L) after switching medications.
This patient also has findings of mild hirsutism and a mild
elevation in free testosterone, which could be caused by polycystic
ovary syndrome or nonclassic CAH. While further workup for these
conditions should be considered (Answer B), the first priority is to
assess the elevated prolactin level. It is expected that menses would
resume if the prolactin level decreases below 50 ng/mL (<2.2
nmol/L). Thus, if menses do not resume, additional testing would be
indicated. A progestin withdrawal challenge (Answer C) would not
add useful diagnostic information at this point given that her
estrogen level is normal.
Initial laboratory test results were reviewed with this patient’s
psychiatrist, who switched her medication from risperidone to
aripiprazole. Testing 4 weeks later showed that her prolactin level
decreased to 24.6 ng/mL (1.1 nmol/L). Her hyperprolactinemia was
therefore attributed to risperidone.

Educational Objective
Differentiate among causes of secondary amenorrhea.

Reference(s)
Melmed S, Casanueva FF, Hoffman AR, et al; Endocrine Society. Diagnosis and treatment of
hyperprolactinemia: an Endocrine Society clinical practice guideline. J Clin Endocrinol
Metab. 2011;96(2):273-288. PMID: 21296991
Fourman LT, Fazeli PK. Neuroendocrine causes of amenorrhea--an update. J Clin Endocrinol
Metab. 2015;100(3):812-824. PMID: 25581597
Pritts S. Secondary amenorrhea: don’t dismiss it as ‘normal’. Curr Psychiatr. 2004;3(10):57-70.
19 ANSWER: D) Recommend a small insulin bolus before
lifting weights
Type 1 diabetes mellitus is a challenging condition to manage with
various physiologic changes of the human body. Regular exercise is
essential, but management of different forms of physical activity is
particularly challenging for both the individual with type 1 diabetes
and the health care provider. Daily exercise is highly encouraged for
patients with type 1 diabetes, as it is the best practice to recommend
for any patient. Patients with type 1 diabetes benefit from exercise by
improving glycemic control, as well as body composition and
general well-being. However, different types and intensities of
exercise have various effects on blood glucose concentrations.
Understanding the metabolic and neuroendocrine responses to
various types of exercise is essential to offer proper advice to
patients. During aerobic exercise, insulin secretion decreases and
glucagon secretion increases in the portal vein to facilitate release of
glucose from the liver to match the rate of glucose uptake into the
working muscles. During predominantly anaerobic activities such as
sprinting and during a high-intensity training session such as weight
lifting, circulating insulin concentrations do not decrease as
markedly as in purely aerobic activities, in part because the duration
of activity is typically shorter. Insulin concentrations increase above
baseline in early recovery from a high-intensity training session to
offset the rise in glucose caused by the elevations in
counterregulatory hormones and other metabolites.
Blood glucose levels tend to remain steady or drop precipitously
in patients with diabetes during aerobic forms of exercise such as
biking, hiking, walking, and rollerblading, depending on the amount
of active insulin on board (basal and/or bolus). However, in most
anaerobic forms of exercise such as weight lifting, blood glucose
rises since the insulin dose is mostly adjusted for the resting state.
This can be prevented by giving a small dose (30%-50% less) of a
blood glucose correction bolus (or half of the hourly basal rate)
before the anaerobic exercise (Answer D).
Not trusting a patient’s reporting by suspecting erroneous
glucose measurements and requesting another report in 1 week
(Answer A) could alienate the patient. Increasing the basal insulin
dose by 20% before and during exercise (temporary basal rate)
would most likely provide a similar solution if the patient is
managed via insulin pump therapy. However, increasing the daily
basal insulin by 20% (Answer B) would increase the risk of
hypoglycemia throughout the day. Increasing the insulin bolus for
the meal before lifting weights (Answer C) may cause hypoglycemia
depending on the timing of the meal and should not be
recommended. Reassurance (Answer E) will send the wrong
message to the patient in terms of tolerating hyperglycemia and
would increase the risk of ketosis during anaerobic exercise.
Image reprinted from Riddell MC, Gallen IW, Smart CE, et al. Exercise
management in type 1 diabetes: a consensus statement. Lancet Diabetes
Endocrinol. 2017;5(5):377-390.

Educational Objective
Advise patients on how to adjust their insulin regimen during
anaerobic forms of exercise.

Reference(s)
Riddell MC, Gallen IW, Smart CE, et al. Exercise management in type 1 diabetes: a
consensus statement. Lancet Diabetes Endocrinol. 2017;5(5):377-390. PMID: 28126459
Lukacs A, Barkai L. Effect of aerobic and anaerobic exercises on glycemic control in type 1
diabetic youths. World J Diabetes. 2015;6(3):534-542. PMID: 25897363
20 ANSWER: E) Shortened fourth metacarpals and metatarsals
The description of the child in this vignette most closely matches a
diagnosis of pseudohypoparathyroidism type 1A (Albright
hereditary osteodystrophy). Therefore, of the physical findings
listed, the classic finding of shortened fourth metacarpals and
metatarsals (Answer E) is the most likely to be observed. He has
additional physical manifestations consistent with this disorder,
including short stature, stocky build, obesity, and cognitive
impairment/developmental delay. His workup shows mild
hypocalcemia with slight hyperphosphatemia, as well as elevated
PTH, which suggests the presence of PTH dysfunction. The history
of seizures in infancy is most likely the result of hypocalcemia.
Manifestations of pseudohypoparathyroidism include a spectrum
of abnormal physical features, neurocognitive deficiencies, and
endocrine abnormalities that are most commonly caused by the
defective hormonal signaling for those hormones that act through G
protein–coupled receptors. Loss-of-function pathogenic variants in
the gene encoding the α subunit of the stimulatory G protein (Gsα)
lead to lack of activation of the postreceptor cascade after interaction
of the hormone (ligand) with the G protein–coupled receptor
(receptor), thus leading to hormone resistance. As such, individuals
with pseudohypoparathyroidism have primarily functional
hypoparathyroidism with elevated PTH levels because of resistance
to PTH leading to hypocalcemia and hyperphosphatemia. They may
also present with hypothyroidism due to resistance to TSH,
hypogonadism due to resistance to LH and FSH, and GH deficiency
due to resistance to GHRH. Physical manifestations of
pseudohypoparathyroidism include short stature, stocky build,
early-onset obesity, skeletal deformities (most commonly
brachydactyly), ectopic ossifications (not described in the vignette),
premature closure of growth plates, and neurocognitive
abnormalities (eg, developmental delay and loss of intellectual
function).
Almond-shaped eyes (Answer A) are commonly associated with
Prader-Willi syndrome. This condition is also associated with short
stature but not commonly with thyroid dysfunction or PTH
abnormalities.
Focal dysplasia of the distal radial physis (Answer B), or
Madelung deformity, is often seen in patients with SHOX pathogenic
variants. Affected patients uniformly have short stature, but they do
not have associated thyroid or calcium disturbances.
Noonan syndrome is associated with widely-spaced eyes, eyelid
ptosis, and low-set ears (Answer C). Patients with Noonan
syndrome commonly present with a heart murmur or
developmental delays. They do not have thyroid or calcium
abnormalities.
Silver-Russell syndrome is associated with short stature and
hemihypertrophy (Answer D). Affected patients commonly present
with failure to thrive and poor feeding. Obesity and thyroid
dysfunction are not typical findings in patients with this syndrome.

Educational Objective
Identify pseudohypoparathyroidism type 1A as a cause of short
stature and list the clinical and laboratory abnormalities associated
with this disorder.

Reference(s)
Mantovani G, Bastepe M, Monk D, et al. Diagnosis and management of
pseudohypoparathyroidism and related disorders: first international consensus
statement. Nat Rev Endocrinol. 2018;14(8):476-500. PMID: 29959430
Dauber A, Rosenfeld RG, Hirschhorn JN. Genetic evaluation of short stature. J Clin
Endocrinol Metab. 2014;99(9):3080-3092. PMID: 24915122
21 ANSWER: D) Adrenal adenoma
An adrenal adenoma (Answer D) is the most likely diagnosis in this
case, although glucocorticoid remediable aldosteronism (not given
as an option) should be excluded. Glucocorticoid remediable
aldosteronism is an autosomal dominant form of
hyperaldosteronism due a genetic defect that gives rise to fusion of
the promoter region of the CYP11A1 gene with the coding sequences
of the CYP11B2 gene (encoding aldosterone synthase), resulting in a
chimeric CYP11A1/CYP11B2 gene. As a result, aldosterone synthase
is expressed in cortisol-producing zona fasciculata cells, and excess
aldosterone is secreted in response to endogenous plasma ACTH.
Biochemically, this causes a picture of low-renin hypertension with
aldosterone excess. Treatment with dexamethasone to suppress
ACTH did not alter this patient’s serum aldosterone levels, which
rules out glucocorticoid remediable aldosteronism.
When this patient’s CT images (which were initially reported as
normal) were reviewed at a specialist center, a 10-mm nodule at the
apex of the left adrenal gland was noted. A repeated scan
demonstrated a stable lesion with no changes in size. Due to the
washout characteristics of 40%, the lesion was thought to be a lipid-
poor adenoma. The renal arteries were normal, as were the kidneys
and left adrenal gland. The patient underwent a left-sided
adrenalectomy. Macroscopically, a homogenous bright yellow
nodule measuring approximately 14 × 12 × 4 mm was present
without necrosis or hemorrhage, thus confirming a diagnosis of
adrenal cortical adenoma. Postoperatively, the patient’s blood
pressure normalized and all medications were subsequently
stopped. Genetic analysis did not reveal any germline pathogenic
variants in known genes, but analysis of the adenoma itself revealed
a heterozygous somatic gain-of-function pathogenic variant (T158A)
in the KCNJ5 gene. The KCNJ5 gene encodes the potassium channel
Kir 3.4 (potassium inwardly rectifying channel, subfamily J, member
5). Gain-of-function KCNJ5 pathogenic variants, both somatic and
germline, cluster in the selectivity filter of Kir3.4, and as such, lead to
changes in ion selectivity, increased sodium conductance, and cell
depolarization. This opens voltage-dependent calcium channels and
influx of calcium in zona glomerulosa cells, resulting in constitutive
secretion of aldosterone and cell proliferation.
Renal artery stenosis (Answer A) results in activation of the renin-
angiotensin-aldosterone system, leading to elevated renin levels. In
this case, renin is completely suppressed, which excludes renal
artery stenosis as a diagnosis.
Hypertension is a feature of congenital adrenal hyperplasia
(Answer B), in particular 11β-hydroxylase deficiency and 17α-
hydroxylase deficiency. However, this is due to the excess
production of 11-deoxycorticosterone acting on the
mineralocorticoid receptor, leading to a low renin and low
aldosterone state.
Liddle syndrome (Answer C) is an autosomal dominant condition
caused by pathogenic variants in the sodium channel epithelial 1 β
and γ subunit genes (SCNN1B and SCNN1G), which encode the
subunits that make up the epithelial sodium channel. Pathogenic
variants lead to constitutive activation of sodium reabsorption along
with water retention and increased potassium excretion. Liddle
syndrome is biochemically associated with low plasma renin activity,
low blood potassium, and normal to low levels of aldosterone.
Licorice root ingestion (Answer E) mimics true
hyperaldosteronism, but aldosterone levels are low, so this is an
incorrect diagnosis in this case.
The prevalence of hypertension (primary and secondary) in
children is reported to be between 4.5% and 13%. Secondary causes
are more common in children than in adults. In one study, secondary
causes account for 57% of all pediatric hypertension cases. Of these,
more than 50% of hypertension cases are due to renal disease, while
endocrine causes account for approximately 10%. Endocrine causes
include primary aldosteronism, pheochromocytoma, Cushing
syndrome, acromegaly, hyperparathyroidism, congenital adrenal
hyperplasia, hypothyroidism, hyperthyroidism, and renin-secreting
tumors.
The diagnosis of primary aldosteronism is made biochemically by
documenting elevated aldosterone together with a low or
undetectable plasma renin activity. The Endocrine Society guidelines
suggest 2 separate measurements to confirm the diagnosis, except in
cases where there is spontaneous hypokalemia, a plasma aldosterone
concentration greater than 19.8 ng/dL (>550 pmol/L), and plasma
renin activity below detectable limits. Adrenal venous sampling is
recommended to distinguish between unilateral or bilateral disease.
In this child, adrenal venous sampling was considered, but it was
deemed unnecessary due to marked primary aldosteronism, his
young age, and positive CT findings. Genetic testing is also
recommended in patients with confirmed primary aldosteronism
who are younger than 20 years, in those with a family history of
primary aldosteronism, or in patients who experience stroke at an
age younger than 40 years.
Unilateral laparoscopic adrenalectomy for patients with
documented unilateral primary aldosteronism is the treatment of
choice. Medical treatment with a mineralocorticoid receptor
antagonist, usually spironolactone, should be considered if surgery
is not possible or in cases of bilateral disease.

Educational Objective
Diagnose and manage primary aldosteronism due to an aldosterone-
producing adenoma.

Reference(s)
Pons Fernandez N, Moreno F, Morata J, et al. Familial hyperaldosteronism type III a novel
case and review of literature. Rev Endocr Metab Disord. 2019;20(1):27-36. PMID: 30569443
Williams TA, Reincke M. Management of endocrine disease: diagnosis and management of
primary aldosteronism: the Endocrine Society guideline 2016 revisited. Eur J Endocrinol.
2018;179(1):R19-R29. PMID: 29674485
Funder JW, Carey RM, Mantero F, et al. The management of primary aldosteronism: case
detection, diagnosis, and treatment: an Endocrine Society clinical practice guideline. J
Clin Endocrinol Metab. 2016;101(5):1889-1916. PMID: 26934393
22 ANSWER: A) Discuss the adverse effects of her cancer
therapy and have her return for follow-up in 6 months
Chronic myeloid leukemia (CML) is rare and accounts for less than
1% of all leukemias in children. Use of tyrosine kinase inhibitors
(TKIs) in the treatment of CML has significantly improved the life
expectancy of adults and children. Since the first-generation TKI
imatinib was approved for CML therapy in 2003, several new TKI
agents have emerged such as dasatinib, nilotinib, bosutinib, and
ponatinib. These potent agents are taken orally and target the BCR-
ABL1 fusion protein. Because children with CML require lifelong
TKI therapy, there is concern for potential off-target inhibition of
other kinases leading to long-term adverse effects.
TKIs may inhibit the GHRH signaling cascade, GH signaling
cascade, and IGF-1 signal transduction, resulting in GH deficiency or
resistance. TKIs may also inhibit chondrocyte activity in growth
plates. Many patients have a significant decrease in their prepubertal
growth, but experience catch-up growth during puberty. Thus, the
best course of action is to discuss the adverse effects of her cancer
therapy and have her return for follow-up in 6 months (Answer A).
Use of GH during active malignancy is contraindicated because of
concerns of neoplasia risks. Thus, further research is needed to
assess whether GH therapy (Answer C) can be safely used in
patients with CML on TKI therapy who have poor growth due to
GH deficiency or GH resistance.
TKIs are associated with subclinical and overt primary
hypothyroidism and hyperthyroidism. Mechanisms by which TKIs
result in thyroid dysfunction are unclear, but they are thought to be
related to increased clearance, decreased iodine uptake, thyroiditis,
and TPO inhibition. Thus, it is important to ensure thyroid function
is normal before starting TKIs and to periodically monitor thyroid
function—usually twice a year—while on TKI therapy. This patient
had normal free T4 and TSH levels documented 1 week before the
clinic visit. Thus, it is not necessary to prescribe levothyroxine at this
time (Answer B). Because children on TKI therapy for CML tend to
have catch-up growth during puberty, it is expected that this
patient’s growth will improve, as she is in early puberty now. She
will not necessarily have short stature as an adult (Answer D).
TKIs lead to dysregulation in bone remodeling by inhibiting the
activity of c-FMS, the macrophage colony-stimulating factor
receptor, on osteoclasts and PDGF-R (platelet-derived growth factor
receptor) and c-Abl1 on osteoblasts. TKIs may promote bone
formation in adults, but they cause bone demineralization in
children. TKIs also alter calcium and phosphate metabolism,
resulting in hypocalcemia and hypophosphatemia. It is important to
optimize bone health in these patients. However, DXA (Answer E) is
not warranted unless the patient has an unprovoked fracture or is
noted to have low bone mineral density on plain radiographs.

Educational Objective
Explain the endocrine adverse effects of tyrosine kinase inhibitor
therapy in children.

Reference(s)
Samis J, Lee P, Zimmerman D, Arceci RJ, Suttorp M, Hijiya N. Recognizing
endocrinopathies associated with tyrosine kinase inhibitor therapy in children with
chronic myelogenous leukemia. Pediatr Blood Cancer. 2016;63(8):1332-1338. PMID:
27100618
Sabnis HS, Keenum C, Lewis RW, et al. Growth disturbances in children and adolescents
receiving long-term tyrosine kinase inhibitor therapy for chronic myeloid leukaemia or
Philadelphia chromosome-positive acute lymphoblastic leukaemia. Br J Haematol.
2019;185(4):795-799. PMID: 30407613
23 ANSWER: E) Start thyroid hormone replacement and
measure TSH and free T4 again in 2 weeks
The overall incidence of congenital hypothyroidism is 1 in 3000,
although the incidence seems to be rising. Congenital
hypothyroidism is one of the most preventable causes of intellectual
disability. Therefore, newborn screening aims to identify children
with this condition to initiate treatment as soon as possible. Usually,
newborn screening identifies babies with congenital hypothyroidism
based on elevated TSH in the first 24 to 48 hours of life, with or
without a low T4 level. However, in preterm babies, there is an
atypical form of congenital hypothyroidism characterized by a
delayed rise in TSH.
The fetal thyroid gland produces thyroid hormones around the
end of the first trimester; however, the hypothalamic-pituitary-
thyroid axis is not mature until after birth, and both TSH and thyroid
hormone levels increase during gestation. Following delivery, there
is a TSH surge in term infants, to about 60 to 80 mIU/L, which
declines to about 20 mIU/L at 24 to 48 hours of life and to 6 to 10
mIU/L by 1 week of age.
In premature babies, this pattern also occurs, but the thyroid
hormone levels reached are reduced compared with those of full-
term newborns. In most preterm babies, the levels eventually
improve over time, and a rise in TSH and T4 is seen as they recover
from the several factors causing hypothyroxinemia of prematurity.
However, in some babies with congenital hypothyroidism, extreme
prematurity, and low birth weight, the TSH rise may not be seen
before the age of 30 days (delayed TSH rise). These babies will not be
diagnosed by routine TSH-based newborn screening.
Several studies have confirmed that preterm infants are at higher
risk to have congenital hypothyroidism, which may be missed by
routine newborn screening. Vigone et al found that in preterm
infants with a birth weight of 640 to 1975 g, only 5 of 24 newborns
with congenital hypothyroidism were identified by initial newborn
screening, while the other 19 were detected by second or third
screening performed between 15 and 30 days of life. This is similar
to the study of Kaluarachchi et al, which showed that of 26
premature infants found to have thyroid dysfunction, only 3 were
detected by newborn screening, while the others were detected
when TSH and free T4 were routinely obtained at 30 days of life. In a
larger study, Hunter et al also documented that 63 infants were
missed by initial newborn screening due to delayed TSH rise. A
study by Tfayli et al confirmed similar data showing a late TSH
surge in very low–birth weight infants (<1500 g) in Lebanon.
The ideal timing for second or third screening in this population
is debated, and it has been generally thought that the delayed TSH
rise can be captured at 30 days of life. Other areas of debate are the
TSH cutoff at this time and whether to consider treatment vs follow-
up—no data are available on whether lack of treatment of mildly
elevated TSH levels on the second or third screening is harmful.
Most experts agree with continuing treatment until age 2 or 3 years.
A recent systematic review of screening of congenital
hypothyroidism in preterm, low–birth weight, and very low–birth
weight neonates recommends careful interpretation of thyroid
function studies in this population. The authors recommend
screening with TSH and free T4 levels in the second, sixth, and tenth
weeks of life and treating if the TSH concentration is 20 mIU/L or
greater. They also recommend treating if any TSH value is greater
than 10 mIU/L with a free T4 value below 0.7 ng/dL (<9.0 pmol/L)
during the third to sixth weeks of life and any TSH value between 10
and 20 mIU/L with a normal free T4 level on 2 occasions.
A recent study in Ireland found that if TSH measurement had
been repeated at 2 weeks of life only, 48% of cases with congenital
hypothyroidism would have been missed. If the final screen had
occurred at 4 weeks of life, 26% cases would have been missed. The
authors recommend screening on days 3 to 5, at 1 week, at 2 weeks,
at 4 weeks, and at term-corrected gestational age. Although there is
no consensus on when to screen premature and very low-birth
weight newborns, it is important to be aware of this form of
congenital hypothyroidism with delayed TSH surge and the
importance of monitoring this fragile population to protect and
maximize their brain development.
The patient in this vignette is a premature newborn with very
low-birth weight and TSH concentration greater than 20 mIU/L at 4
weeks of life. This baby most likely has congenital hypothyroidism
with delayed TSH surge and would benefit from treatment at this
time, with close follow-up (Answer E). Although some clinicians
may want to confirm results before starting treatment, waiting 4
weeks (Answer A) would most likely delay needed treatment in this
infant. Assessing antibodies in the mother (Answer B) would not
change management. In addition, there is no maternal history of
thyroid dysfunction and determining whether the baby’s
hypothyroidism is transient could be reevaluated in the future.
Measuring TSH, free T4 by equilibrium dialysis, and total T3
(Answer C) would be more appropriate when evaluating an infant
for hypothyroxinemia with a normal TSH level to differentiate
among central hypothyroidism, nonthyroidal illness, or low
thyroxine-binding globulin. However, this would not change
management in this setting. Although recovery from nonthyroidal
illness is in the differential diagnosis for this patient, the TSH value
above 20 mIU/L with a low-normal free T4 concentration is more
suggestive of congenital hypothyroidism with delayed TSH surge.
Reassurance and no intervention and no follow-up with thyroid
function studies (Answer E) would be incorrect. If no intervention is
elected at this time, very close follow-up with repeated TSH and free
T4 measurements is recommended.

Educational Objective
Describe the clinical significance of the effect of prematurity on
thyroid function in the neonate.

Reference(s)
Hunter MK, Mandel SH, Sesser DE, et al. Follow-up of newborns with low thyroxine and
nonelevated thyroid-stimulating hormone-screening concentrations: results of the 20-
year experience in the Northwest Regional Newborn Screening Program. J Pediatr.
1998;132(1):70-74. PMID: 9470003
Mandel SJ, Hermos RJ, Larson CA, Prigozhin AB, Rojas DA, Mitchell ML. Atypical
hypothyroidism and the very low birthweight infant. Thyroid. 2000;10(8):693-695. PMID:
11014314
LaFranchi SH. Screening preterm infants for congenital hypothyroidism: better the second
time around. J Pediatr. 2014;164(6):1259-1261. PMID: 24657124
Vigone MC, Caiulo S, Di Frenna M, et al. Evolution of thyroid function in preterm infants
detected by screening for congenital hypothyroidism. J Pediatr. 2014;164(6):1296-1302.
PMID: 24518164
Kaluarachchi DC, Colaizy TT, Pesce LM, Tansey M, Klein JM. Congenital hypothyroidism
with delayed thyroid-stimulating hormone elevation in premature infants born at less
than 30 weeks gestation. J Perinatol. 2017;37(3):277-282. PMID: 27906195
Tfayli H, Charafeddine L, Tamim H, Saade J, Daher RT, Yunis K. Higher incidence rates of
hypothyroidism and late TSH rise in preterm very-low-birth-weight infants at a tertiary
care center. Horm Res Paediatr. 2018;89(4):224-232. PMID: 29642061
Hashemipour M, Hovsepian S, Ansari A, Keikha M, Khalighinejad P, Niknam N. Screening
of congenital hypothyroidism in preterm, low birth weight and very low birth weight
neonates: a systematic review. Pediatr Neonatol. 2018;59(1):3-14. PMID: 28811156
McGrath N, Hawkes CP, Mayne P, Murphy NP. Optimal timing of repeat newborn
screening for congenital hypothyroidism in preterm infants to detect delayed thyroid-
stimulating hormone elevation. J Pediatr. 2019;205:77-82. PMID: 30529133
24 ANSWER: B) Adipsia
This patient presented with recurrent cystic growth of a
craniopharyngioma. Although her pituitary function was initially
normal, with subsequent surgeries she developed
panhypopituitarism, central diabetes insipidus, and hypothalamic
obesity. Patients with lesions of the pituitary gland or hypothalamus
can present with hormone deficiencies preoperatively, during
surgery, postoperatively, or following cranial irradiation. Central
diabetes insipidus occurs due to dysfunction of the vasopressinergic
neurons of the supraoptic and paraventricular nuclei. Arginine
vasopressin (AVP) secretion is regulated by plasma and interstitial
osmotic pressure sensed by osmoreceptors in the hypothalamus, as
well as by circulating blood volume, sensed by baroreceptors.
Plasma osmolality is normally maintained in a relatively narrow
physiologic range by water intake and output regulated by AVP and
thirst. Like AVP, thirst is stimulated by hyperosmolality. The
osmoreceptors that regulate thirst are in close proximity to those
regulating AVP. Adipsia (inappropriate lack of thirst) typically
results from lesions that impair thirst and AVP secretion (eg,
structural abnormalities of the hypothalamic-pituitary region due to
congenital malformations such as optic nerve hypoplasia, septo-
optic dysplasia, holoprosencephaly, tumors such as
craniopharyngioma, trauma, and surgery). This patient initially had
normal thirst. Postoperatively, her sodium was normal at first.
However, when she was allowed to take fluids ad lib, she became
hypernatremic. This patient most likely developed unrecognized
adipsia (Answer B) as a result of the surgery.
Excess fluid administration (Answer A) is a common cause of
postoperative polyuria, but it is unlikely to cause hypernatremia if
there is normal kidney function and thirst. Her serum urea nitrogen
and creatinine remained normal, so excess fluid administration is
incorrect.
Symptoms of diabetes insipidus can be masked by glucocorticoid
deficiency until cortisol replacement is administered. However, this
patient already had a known diagnosis of ACTH deficiency and
central diabetes insipidus, and her diabetes insipidus had been well
controlled on her home desmopressin and hydrocortisone dosages.
Additionally, if her thirst mechanism were intact, she would have
been able to avoid hypernatremia by increasing her fluid intake.
Therefore, stress doses of hydrocortisone worsening the diabetes
insipidus (Answer C) is incorrect.
Hyperglycemia is a common cause of polyuria, but not
hypernatremia (Answer D). She did have some mild postoperative
hyperglycemia, but her glucose levels were not elevated enough to
cause significant increases in urine output and subsequent
dehydration.
At home she had good control on desmopressin. While her
dosage certainly could have required adjustments postoperatively,
she should have been able to maintain normal sodium levels by
increasing her oral fluid intake in response to increases in plasma
osmolality. Therefore, an insufficient desmopressin dosage (Answer
E) is incorrect.

Educational Objective
Differentiate among the conditions responsible for pathologic loss of
thirst.

Reference(s)
Di lorgi N, Morana G, Napoli F, Allegri AE, Rossi A, Maghnie M. Management of diabetes
insipidus and adipsia in the child. Best Pract Res Clin Endocrinol Metab. 2015;29(3):415-
436. PMID: 26051300
25 ANSWER: D) Increase all basal rates by 30% within 6 to 12
hours after initiating prednisone
Insulin requirements to maintain normoglycemia during
glucocorticoid therapy and stress are often difficult to estimate. The
total daily insulin requirement may increase 30% to 100% within 6 to
12 hours after the first prednisone dose and return to baseline
amounts 24 to 48 hours after therapy. Increasing the basal insulin
rate by 30% within 6 to 12 hours after initiating prednisone (Answer
D) is the best initial management strategy to prevent anticipated
hyperglycemia.
High-dosage glucocorticoids are associated with insulin
resistance and increased hepatic gluconeogenesis. These effects
markedly increase insulin requirements in patients with type 1
diabetes and may even result in ketosis. The impact of glucocorticoid
therapy on glycemic control is typically not seen immediately upon
treatment initiation and the effects may persist for several days after
discontinuing therapy. The longer the half-life of the glucocorticoid,
the longer it will affect glycemic control. The effects of
glucocorticoids on blood glucose values are influenced by the
specific glucocorticoid chosen, the dosage, and the timing of the
doses. Higher doses and more potent glucocorticoids are associated
with higher insulin requirements.
Because glucocorticoids increase insulin requirements due to
increased gluconeogenesis, these effects persist throughout the day.
Increasing the insulin-to-carbohydrate ratios by 30%, which would
be used only for mealtimes (Answer C), would not provide stable
glycemic control beyond the mealtimes, especially overnight.
Likewise, increasing the correction/sensitivity factors (Answer A)
would be helpful for controlling blood glucose after hyperglycemia
occurs, but would do little to prevent the development of
hyperglycemia. The reactive approach of just monitoring blood
glucose values and correcting the hyperglycemia (Answer E) with
the existing correction factor will not be sufficient to maintain blood
glucose in the target range because of insulin resistance.
Increasing basal rates within 6 to 12 hours after initiating
prednisone (Answer D)—not immediately (Answer B) (which may
cause hypoglycemia)—is a reasonable first step to ameliorate the
anticipated deterioration of glycemic control. A 30% increase is a
good place to begin, although some patients may require more
aggressive increases in basal rates. If a patient continues to have
significant glycemic excursions after meals, it is reasonable to also
increase meal bolus insulin coverage by intensifying insulin-to-
carbohydrate ratios and even the correction/sensitivity factors.
The insulin management strategy highlighted in this vignette is
specific to patients who are on insulin pump therapy. If the patient
were on multiple daily injections of insulin, the concept of increasing
basal insulin by 30% would still be a valid approach. Yet, depending
on the timing of the “basal” insulin injection, there could be a delay
in increasing the basal insulin amount. The delay could potentially
result in interval hyperglycemia, which could be remedied by
administering frequent (every 3 hours) “correction” bolus doses of
rapid-acting insulin until the adjustment in basal insulin takes effect.

Educational Objective
Anticipate changes needed in the insulin regimen of a patient with
diabetes mellitus to address hyperglycemia associated with
glucocorticoid therapy.

Reference(s)
Tamez-Pérez HE, Quintanilla-Flores DL, Rodríguez-Gutiérrez R, González-González JG,
Tamez-Peña AL. Steroid hyperglycemia: prevalence, early detection and therapeutic
recommendations: a narrative review. World J Diabetes. 2015;6(8):1073-1081. PMID:
26240704
Bevier WC, Zisser HC, Jovanovič L, et al. Use of continuous glucose monitoring to estimate
insulin requirements in patients with type 1 diabetes mellitus during a short course of
prednisone. J Diabetes Sci Technol. 2008;2(4):578-583. PMID: 19885233
26 ANSWER: C) Activating pathogenic variant in the FGFR3
gene
This patient has poor growth, short stature, obesity, macrocephaly,
brachydactyly, and short limbs. She had normal onset and
progression of puberty. These features are indicative of skeletal
dysplasia. Achondroplasia is the most common form of dwarfism,
occuring in 1 in 15,000 to 25,000 individuals. Autosomal dominant
gain-of-function pathogenic variants in the FGFR3 gene (Answer C),
most of them sporadic but with 100% penetrance, are the most
common cause of achondroplasia and related chondrodysplasia
syndromes, including hypochondroplasia, severe achondroplasia
with developmental delay and acanthosis nigricans (SADDAN), and
the lethal form thanatophoric dysplasia.
Obstructive and central sleep apnea are common in individuals
with achondroplasia, and a sleep study is recommended for all
patients with this condition. Other common issues in individuals
with achondroplasia include middle ear dysfunction, lower-
extremity bowing, kyphosis, spinal stenosis, joint laxity, and
arthritis. Thus, regular evaluation with otolaryngology and
orthopedics is indicated for individuals with achondroplasia.
Contact sports, joint hyperextension, trampoline use, diving, and
hanging upside down from knees/feet should all be avoided due to
risk of injury to the spinal cord at the craniocervical junction.
References are available for anthropometric variables in
achondroplasia such as sitting height, leg length, arm span, relative
sitting height (sitting height/height), and foot length.
Short-term and long-term GH therapy in patients with
achondroplasia has led to some height improvement, but a recent
meta-analysis concluded that there are insufficient data on final
adult height and changes in body proportion with GH therapy. C-
type natriuretic peptide analogue has been proposed as a potential
treatment option in clinical trials, as it antagonizes FGFR3 signaling
by down-regulating the mitogen-activated protein kinase pathway,
which is activated as a result of an FGFR3 gain-of-function
pathogenic variant in affected patients.
Maternal uniparental disomy of chromosome 15 (Answer A)
results in Prader-Willi syndrome, which is associated with
hypotonia, hyperphagia after infancy, morbid obesity,
developmental delays, and hypogonadism. However, the patient in
this vignette does not have any of these features.
Although patients with Turner syndrome (Answer B) are also at
high risk for frequent ear infections, macrocephaly and short limbs
are not characteristic of this condition.
Patients with an inactivating pathogenic variant in the maternal
allele of the GNAS gene (Answer D) have Albright hereditary
osteodystrophy with short stature, brachydactyly, and evidence of
hormone resistance such as hypocalcemia due to PTH resistance and
abnormal thyroid function tests. In this patient, the normal
comprehensive metabolic panel, normal thyroid function, and
presence of macrocephaly and short limbs make Albright hereditary
osteodystrophy less likely to be the diagnosis.
Pathogenic variants in the FMR1 gene (Answer E) result in fragile
X syndrome, which is associated with developmental delay, learning
disabilities, mental retardation, and some characteristic dysmorphic
features. FMR1 pathogenic variants are not associated with short
stature and macrocephaly.

Educational Objective
List the physical characteristics of short stature in patients with
achondroplasia and identify pathogenic variants in the FGFR3 gene
as the cause of achondroplasia.

Reference(s)
Ornitz DM, Legeai-Mallet L. Achondroplasia: development, pathogenesis, and therapy. Dev
Dyn. 2017;246(4):291-309. PMID: 27987249
Merker A, Neumeyer L, Hertel NT, Grigelioniene G, Mohnike K, Hagenas L. Development
of body proportions in achondroplasia: Sitting height, leg length, arm span, and foot
length. Am J Med Genet A. 2018;176(9):1819-1829. PMID: 30152086
Miccoli M, Bertelloni S, Massart F. Height outcome of recombinant human growth hormone
treatment in achondroplasia children: a meta-analysis. Horm Res Paediatr. 2016;86(1):27-
34. PMID: 27355624
Trotter TL, Hall JG; American Academy of Pediatrics Committee on Genetics. Health
supervision for children with achondroplasia [published correction appears in
Pediatrics. 2005;116(6):1615]. Pediatrics. 2005;116(3):771-783. PMID: 16140722
27 ANSWER: D) Partial androgen insensitivity syndrome
Hypospadias is one of the most common congenital anomalies in
newborn males, affecting up to 1 in 200 boys. While hypospadias has
been described in more than 200 syndromes, a genetic cause is
identified in only 30% of cases. When associated with other
abnormalities such as cryptorchidism or micropenis, there is a higher
likelihood of androgen deficiency. Heritability appears to vary with
severity. Severe (proximal) hypospadias is also associated with
intrauterine growth restriction, maternal hypertension, and
oligohydramnios, suggesting that placental insufficiency can have a
role, possibly through a decrease in the fetus’s exposure to hCG.
The infant in this scenario has both environmental and genetic
risk factors. Given that he also has microphallus (<2.5 cm),
investigation for an underlying cause is warranted. Although he
most likely has idiopathic hypospadias, of the options given, partial
androgen insensitivity syndrome (PAIS) (Answer D) is most
consistent with his presentation. PAIS is an X-linked recessive
disorder characterized by ambiguous genitalia, with perineoscrotal
hypospadias, microphallus, and bifid scrotum. Testes can be
undescended, palpable, or anywhere in between. Affected infants
usually have a normal neonatal testosterone surge, although this
may be reduced in the setting of complete androgen insensitivity
syndrome. LH and testosterone levels are often, but not always,
elevated. The ability to synthesize testosterone is intact, but there is
loss of negative feedback inhibition of gonadotropins. The diagnosis
is confirmed by molecular genetic testing of the androgen receptor
gene (AR), which would be an appropriate next step for this infant.
The frequency of detecting AR pathogenic variants ranges from 65%
to 95% in individuals with complete androgen insensitivity and from
40% to 45% in those with PAIS.
Individuals with 5α-reductase deficiency (Answer A) have
normal testosterone production but do not undergo normal
virilization due to a defect in converting testosterone to
dihydrotestosterone. While the phenotype ranges from normal
female appearance to nearly normal male appearance of external
genitalia, approximately 75% of affected individuals are identified as
female at birth. A testosterone-to-dihydrotestosterone ratio greater
than 10 is suggestive of this disorder; however, timing of laboratory
studies is critical. Hormone levels should be assessed by gas or
liquid chromatography–mass spectrometry (LC/MS) to avoid cross-
reactivity. This assessment should be performed between 1 week and
6 months of age. In this vignette, LH and testosterone levels are
consistent with the mini-puberty of infancy, so interpretation of
hormone levels without hCG-stimulation testing can be considered
reliable. While this infant’s phenotype is consistent with possible 5α-
reductase deficiency, the testosterone-to-dihydrotestosterone ratio of
3.6 makes this less likely.
Congenital adrenal hyperplasia due to 3β-hydroxysteroid
dehydrogenase deficiency (Answer B) leads to undervirilization in
46,XY individuals due to testosterone deficiency. Undervirilization
can range from a severe presentation, associated with salt-losing
adrenal crises, to a mild presentation with hypospadias and
microphallus. Characteristic laboratory findings include elevated
pregnenolone, 17-hydroxypregnenolone, and DHEA concentrations.
In this vignette, laboratory findings are not consistent with 3β-
hydroxysteroid dehydrogenase deficiency.
Swyer syndrome (Answer C) is one form of partial XY gonadal
dysgenesis, and it can be caused by pathogenic variants in one of
several genes required for sexual differentiation of the fetus. Swyer
syndrome is most often due to pathogenic variants in the SRY gene.
Affected individuals typically have a female external appearance,
thus Swyer syndrome would be an unlikely diagnosis in this
vignette. Pelvic ultrasonography would demonstrate the presence of
a uterus, which helps distinguish this condition from androgen
insensitivity syndrome.
Individuals with pathogenic variants in the LHCGR gene
(luteinizing hormone choriogonadotropin receptor) (Answer E)
demonstrate a phenotype ranging from mild undervirilization to
completely female external genitalia. Testes are typically inguinal or
intraabdominal. While the phenotype in this vignette is consistent
with an LHCGR pathogenic variant, laboratory testing would be
expected to demonstrate a low testosterone level with elevated LH
due to Leydig-cell hypoplasia.

Educational Objective
Evaluate the etiology of undervirilization in an infant with a 46,XY
karyotype.

Reference(s)
Bouty A, Ayers KL, Pask A, Heloury Y, Sinclair AH. The genetic and environmental factors
underlying hypospadias. Sex Dev. 2015;9(5):239-259. PMID: 26613581
28 ANSWER: B) Stop atorvastatin; wait 2 weeks and then
reintroduce atorvastatin and assess for symptoms
Although statins are an effective way of lowering LDL-cholesterol
levels, myopathy remains an important potential adverse effect. This
has been reported predominantly in adults, but it has also been
described in adolescents. This is a risk factor for rhabdomyolysis
when creatine kinase levels are greater than 10 times the upper
normal limit. The reasons for this boy’s muscle soreness and creatine
kinase elevations are unclear. However, his creatine kinase
elevations are mild, which can be observed following vigorous
physical activity. A practice guideline for pediatric statin treatment
states, “Practitioners should be aware that recommended monitoring
of creatine kinase levels is more likely to result in numerous mild to
severe sporadic increases that may not be attributable to the statin.
Vigorous exercise, particularly contact sports or weightlifting, may
cause physiological increases, and it is important that not all
increases are immediately attributed to the statin.” This boy has clear
reasons for being on a statin, with a high pretreatment LDL-
cholesterol level and a family history of early death due to
cardiovascular disease, so continuing adequate treatment for his
dyslipidemia should be part of his care plan.
However, since this is the second event of myalgias, most experts
would recommend stopping the statin, waiting for symptoms to
resolve, and reintroducing either the same or different statin and
assessing for symptoms (Answer B). Continuing atorvastatin and
measuring creatine kinase in 2 days (Answer A) is incorrect. There
are no guidelines for either adult or pediatric myopathies, so clinical
judgment continues to be the best approach to caring for patients
with this symptom. Since the statin was very effective in reducing
this patient’s LDL-cholesterol level and he is in the high-risk
category for cardiovascular events in the future, an attempt should
be made to remain on statin therapy.
This boy’s creatine kinase elevations do not currently require
hospitalization (Answer D) and are not concerning enough to
recommend permanent discontinuation of statin treatment (Answer
C).
MRI (Answer E) would not be expected to reveal any notable
findings, even in the setting of a muscle reaction. Despite the mild
nature of the patient’s pain and degree of creatine kinase elevation,
there remains the possibility that this is an early step in the process
of myocyte damage.
While this boy’s liver transaminase levels were normal, statin use
in adults has been associated with a 0.5% rate of elevated
transaminase levels (>3 times the upper normal limit. In studies of
pediatric patients, there have been similar rates of elevated
transaminase levels among patients taking statins and those taking
placebo, potentially because of higher rates of underlying
nonalcoholic steatohepatitis among treated patients.
Few nonstatin treatments have adequate safety data in children,
and those that do are less effective than statins in reducing LDL-
cholesterol levels. Therefore, they would not be warranted without
adequate suspicion of statin-related myopathy. Nonstatin therapies
include ezetimibe, bile acid sequestrants, PCSK9 inhibitors, and
bempedoic acid.
Ezetimibe lowers LDL-cholesterol levels by inhibiting cholesterol
absorption and up-regulating hepatic LDL receptors. While often
used as an add-on therapy to statin treatment, recent studies using it
as monotherapy in children showed promising results.
Bile acid sequestrants have long been known to reduce
cholesterol levels. Their mechanism of action is to decrease the
absorption of bile acids, which results in increased synthesis of bile
acids from cholesterol leading to a decrease in hepatic cholesterol
and an up-regulation of hepatic LDL receptors. They are not very
well tolerated by children.
PCSK9 monoclonal antibodies lower LDL cholesterol by binding
PCSK9 (regulator of LDL receptors), reducing the breakdown of LDL
receptors, as well as by reducing lipoprotein (a) levels. Often used
when statins are not tolerated, PCSK9 inhibitors are in phase 3
clinical trials in children.
Bempedoic acid lowers LDL cholesterol by inhibiting hepatic ATP
citrate lyase activity, thereby reducing cholesterol synthesis in the
liver and up-regulating LDL receptors. Bempedoic acid is also used
in patients who do not tolerate statins. It has not been studied in
children to date.

Educational Objective
Evaluate the possibility of statin-induced myopathy in an
adolescent.

Reference(s)
Luirink IK, Wiegman A, Kusters DM, et al. 20-year follow-up of statins in children with
familial hypercholesterolemia. N Engl J Med. 2019;381(16):1547-1556. PMID: 31618540
Selva-O’Callaghan A, Alvarado-Cardenas M, Pinal-Fernández I, et al. Statin-induced
myalgia and myositis: an update on pathogenesis and clinical recommendations. Expert
Rev Clin Immunol. 2018;14(3):215-224. PMID: 29473763
Avis HJ, Vissers MN, Stein EA, et al. A systematic review and meta-analysis of statin
therapy in children with familial hypercholesterolemia. Arterioscler Thromb Vasc Biol.
2007;27(8):1803-1810. PMID: 17569881
29 ANSWER: C) Graves disease
A hot nodule (Answer D) is unlikely in this patient, as she had a
diffuse goiter and elevated thyroid-stimulating immunoglobulin. A
thyroid uptake and scan can distinguish between Graves disease
(Answer C) and a hot nodule. In Graves disease, the uptake is
increased throughout the gland, whereas in a hot nodule it is
increased only in the nodule and is low in the rest of the gland.
Given this patient’s elevated thyroid-stimulating immunoglobulin,
which was negative in the past, she most likely has Graves disease,
although she did not have characteristic eye findings or a bruit. A
number of cases of Graves disease have been reported following
Hashimoto thyroiditis, either with or without a painful gland, with
most occurrences in adults.
Hashitoxicosis (Answer A) is characterized by a brief period (a
few weeks or less than 3 months) of hyperthyroidism that presents
early in the course of Hashimoto thyroiditis. It does not present late
in the course, and it occurs without elevated thyroid-stimulating
immunoglobulin.
Silent subacute thyroiditis (Answer B) is a possibility in this
patient, but her weight loss over 5 months and elevated thyroid-
stimulating immunoglobulin make this an unlikely diagnosis. In
subacute thyroiditis, there is decreased iodine uptake during the
transient hyperthyroid phase.
Exogenous ingestion of levothyroxine (Answer E) is incorrect, as
it is not expected to result in such a high total T3 level and it is not
associated with high thyroid-stimulating immunoglobulin.

Educational Objective
Diagnose Graves disease in a patient who has had Hashimoto
thyroiditis for several years.

Reference(s)
Ohye H, Nishihara E, Sasaki I, et al. Four cases of Graves’ disease which developed after
painful Hashimoto’s thyroiditis. Intern Med. 2006;45:385-389. PMID: 16617190
Takasu N, Yamada T, Sato A, et al. Graves’ disease following hypothyroidism due to
Hashimoto’s disease: studies of eight cases. Clin Endocrinol (Oxf). 1990;33(6):687-698.
PMID: 1982861
30 ANSWER: C) GH-stimulation test
Endocrine consequences of radiation therapy for CNS malignancies
depend on the anatomic site of cranial radiation, radiation dose,
fractionation, and age at the time of treatment. Radiation directed to
the hypothalamic-pituitary region, as was the case for the child in
this vignette, is more likely to induce endocrine abnormalities
involving the hypothalamic-pituitary axis. The hypothalamus is
more sensitive than the pituitary gland to radiation. However, the
somatotrophs (the GH-secreting cells of the pituitary gland) are more
sensitive to radiation than the gonadotrophs, thyrotrophs, or
corticotrophs. As a result, GH deficiency is more likely to occur after
irradiation of the hypothalamic-pituitary region even with a lower
dose (such as ≥10 Gy single fraction total body or ≥12 Gy
fractionated total body), intermediate dose (≥18 Gy and ≤30 Gy
cranial radiation), or higher dose (≥30 Gy cranial radiation), while
deficiencies of LH, FSH, TSH, and ACTH are less likely to occur at
lower or intermediate doses, but are more likely to occur after higher
doses of cranial radiation (≥30 Gy). Interestingly, intermediate doses
of cranial radiation (≥18 Gy and ≤30 Gy) may induce precocious
puberty as opposed to LH/FSH deficiency, possibly due to
disruption of hypothalamic physiologic inhibition of pubertal onset.
The child in this vignette has evidence of activation of the
hypothalamic-pituitary-gonadal axis as manifested by testicular
volume consistent with Tanner stage 2 to 3, which, at an age younger
than 9 years in a boy is consistent with central precocious puberty.
Therefore, it is unnecessary to perform a GnRH-stimulation test
(Answer A).
Secondary sexual characteristics, such as pubic hair and penile
enlargement, may be related to adrenarche or to endogenous
testosterone of testicular origin. However, despite the degree of
pubertal development, his growth velocity has remained normal
(prepubertal). The lack of linear growth deceleration may be
confusing, as it would not prompt the clinician to think about GH
deficiency. However, in the presence of precocious puberty, growth
deceleration may not be obvious. IGF-1 levels may not appear low in
the face of precocious puberty and therefore are not a reliable
indicator in this patient. Additionally, based on the history of 20 Gy
of radiation delivered to the hypothalamic-pituitary region, the
possibility of GH deficiency should be entertained. Evaluation for
GH deficiency with a GH-stimulation test (Answer C) is the best
next step.
Brain MRI (Answer D) is not indicated in this patient, as it would
not change his management. His previous tumor is not associated
with central precocious puberty, and the effects of the radiation
would not be seen on MRI.
Testicular ultrasonography (Answer E) could be performed to
look for a peripheral source for testosterone. However, with
measurable gonadotropins, it is not the best option.
A low-dose cosyntropin-stimulation test (Answer B) is not the
best next step, as there are no clinical manifestations described in the
vignette to support central adrenal insufficiency and the patient had
a normal 8-AM cortisol concentration. ACTH deficiency is unlikely
to occur with low or intermediate doses of cranial radiation.

Educational objective
Describe the pitfalls in diagnosing GH deficiency when there is
simultaneous precocious puberty in children receiving radiation
therapy to the hypothalamic-pituitary region.

Reference(s)
Rutter MM, Rose SR. Long-term endocrine sequelae of childhood cancer. Curr Opin Pediatr.
2007;19(4):480-487. PMID: 17630615
Sklar CA, Antal Z, Chemaitilly W, et al. Hypothalamic-pituitary and growth disorders in
survivors of childhood cancer: an Endocrine Society clinical practice guideline. J Clin
Endocrinol Metab. 2018;103(8):2761-2784. PMID: 29982476
31 ANSWER: C) Perform a 99Tc sestamibi scan
This patient has hyperparathyroidism as evidenced by elevated
serum calcium, low phosphate, and inappropriately high-normal
PTH. PTH is exquisitely sensitive to calcium such that any elevation
in the serum calcium concentration should result in PTH
suppression. High PTH levels, even in the mid- to upper-normal
range, in the setting of hypercalcemia establish the diagnosis of
primary hyperparathyroidism. The next step is to determine
whether she has a parathyroid adenoma or hyperplasia. A sestamibi
scan (Answer C) is helpful in making this distinction.
Autosomal dominant inactivating pathogenic variants in the gene
encoding the calcium-sensing receptor (CASR) can present with
hyperparathyroidism. Adolescents or adults with this condition are
generally asymptomatic, but there have been associations with
pancreatitis or chondrocalcinosis. The biochemical profile can be
similar to primary hyperparathyroidism with elevated calcium and
an inappropriately normal or slightly elevated PTH level; however,
the urinary calcium-to-creatinine ratio is usually less than 0.01 and
the phosphate is normal. This disorder is known as familial
hypocalciuric hypercalcemia, and parathyroidectomy is not
recommended in this setting. Conversely, total parathyroidectomy is
recommended for patients with homozygous inactivating
pathogenic variants in CASR, as this typically presents with
profound hypercalcemia, very high PTH levels, and severe bone
disease, including multiple fractures and hypotonia in the neonatal
period. This patient is unlikely to have familial hypocalciuric
hypercalcemia, and genetic testing (Answer A) is not the best next
step.
While thiazide diuretics can cause hypercalcemia due to inability
to excrete calcium in the urine, the physiologic response would be to
suppress PTH. In this vignette, PTH was even higher on the thiazide
diuretic. Thus, stopping the thiazide diuretic and repeating
laboratory tests (Answer E) is inappropriate.
Hyperparathyroidism has a more profound effect on cortical bone
mineral density than on trabecular bone, which may explain, in part,
this patient’s normal lumbar spine bone mineral density. The one-
third distal radius is primarily cortical bone, and DXA (Answer D)
may be useful in assessing the degree of skeletal involvement.
However, it would not be useful in determining the etiology of
hyperparathyroidism. In adult patients, this may guide surgical
decision-making, whereas in younger patients (<50 years)
parathyroidectomy is usually indicated, even in asymptomatic
patients.
The most common cause of hyperparathyroidism is a benign
adenoma (80%). The 2 most common techniques to localize
parathyroid adenomas are parathyroid ultrasonography (Answer B)
or a technetium 99Tc sestamibi scan (Answer C). Single-photon
emission CT (SPECT) has also been used to localize parathyroid
adenomas, but this approach is generally better for ectopic
parathyroid tissue and carries the risk of radiation exposure.
Parathyroid ultrasonography is technician dependent and may be
more challenging in pediatric patients in whom this is a more rare
condition. Technetium 99Tc sestamibi scans have a higher positive
predictive value, although they may miss small adenomas and are
not ideal for multiglandular disease (hyperplasia). This is because
the technique relies on the preferential retention of the radiotracer in
the mitochondrial-rich oxyphil cells of the hyperfunctioning gland. If
all glands are functioning with a similar intensity, they will all
experience similar washout and the overall result of the scan will be
negative. It is important to therefore remember that a 99Tc sestamibi
scan is not used to diagnose hyperparathyroidism, but rather to
determine whether it is caused by a parathyroid adenoma and to
guide the surgical approach.

Educational Objective
Recommend 99Tc sestamibi scanning to determine whether
hyperparathyroidism is secondary to a solitary parathyroid
adenoma and to guide surgical management in young patients.
Reference(s)
Davies JH. Approach to the child with hypercalcaemia. Endocr Dev. 2015;28:101-118. PMID:
26138838
Wermers RA, Kearns AE, Jenkins GD, Melton LJ 3rd. Incidence and clinical spectrum of
thiazide-associated hypercalcemia. Am J Med. 2007;120(10):911.e9-e15. PMID: 17904464
Hughes DT, Sorensen MJ, Miller BS, Cohen MS, Gauger PG. The biochemical severity of
primary hyperparathyroidism correlates with the localization accuracy of sestamibi and
surgeon-performed ultrasound. J Am Coll Surg. 2014;219(5):1010-1009. PMID: 25086814
32 ANSWER: E) Undernutrition
This patient presents with marked weight loss, growth deceleration,
elevated GH, and low IGF-1 levels. Her history is concerning for a
restrictive eating disorder. IGF-1, which is primarily produced in the
liver, is regulated by GH secretion. IGF-1 secretion is also altered by
nutritional status. In states of undernutrition or starvation, GH levels
are normal or elevated in the setting of low IGF-1 levels. This state of
GH resistance may represent an adaptive response to decreased
energy intake, a mechanism to preserve energy during periods of
starvation. In animal models of starvation, low IGF-1 levels are seen
along with decreased GH receptor mRNA and decreased GH
binding, suggesting that down-regulation of GH receptors in the
liver contributes to the GH resistance. Elevated GH levels may be
necessary to mobilize fat stores and maintain euglycemia, and low
IGF-1 levels may lead to decreased energy expenditure for growth.
Therefore, undernutrition (Answer E) is the most likely cause of this
patient’s growth delay.
Craniopharyngiomas (Answer A) are rare intracranial tumors
that account for 6% to 9% of all childhood brain tumors and have a
peak incidence in children aged 5 to 14 years. Affected children may
present with signs of increased intracranial pressure, endocrine
deficits (52%-87%), and vision impairment (62%-84%). The growth
deceleration is this setting is due to GH deficiency, which would be
associated with low IGF-1 and GH levels.
GH insensitivity syndrome (Answer B), also known as Laron
dwarfism, is associated with marked short stature with preservation
of weight, a history of a relatively normal birth size, very low IGF-1
levels, and elevated GH levels. This child had normal growth until
recently, and, unlike individuals with Laron dwarfism, she has had
weight loss and is underweight.
Addison disease (Answer C), or primary adrenal insufficiency,
can be acute or chronic in presentation. Chronic adrenal insufficiency
is characterized by poor weight gain or weight loss, weakness,
fatigue, nausea or vague gastrointestinal complaints, and salt
craving. Skin hyperpigmentation is common and is due to elevated
ACTH binding to the melanocortin 1 receptor. Height growth is
generally normal in persons with Addison disease. Hypotension,
tachycardia, hyponatremia, and hyperkalemia are frequently
present. While Addison disease should be considered in this patient,
her height growth has slowed, she has no hyperpigmentation on
examination and no information is given about her cortisol or ACTH
levels.
This patient has a strong family history of constitutional delay of
maturity (Answer D). She has no breast development on
examination and her bone age is delayed. However, children with
constitutional delay would not be expected to have dramatic weight
loss, growth deceleration, or elevated GH levels.

Educational Objective
Explain how deficiencies in nutrition can cause functional GH
resistance.

Reference(s)
Fazeli PK, Klibanski A. Determinants of GH resistance in malnutrition. J Endocrinol.
2014;220(3):R57-R65. PMID: 24363451
Savage MO, Attie KM, David A, Metherell LA, Clark AJ, Camacho-Hübner C. Endocrine
assessment, molecular characterization and treatment of growth hormone insensitivity
disorders. Nat Clin Pract Endocrinol Metab. 2006;2(7):395-407. PMID: 16932322
33 ANSWER: D) Retained portion of histrelin implant, which is
continuing to suppress puberty
Silver-Russell syndrome is characterized by prenatal and postnatal
growth retardation, feeding difficulties, and recurrent hypoglycemia.
Individuals with Silver-Russell syndrome are at increased risk for
premature adrenarche and early and rapid central puberty, which
can lead to early closure of epiphyses and limited growth.
The patient in this vignette presents with pubertal delay and a
height velocity that has decreased over the past year despite GH
treatment. Effective GH dosages for children with a history of being
small for gestational age are higher than those for children with GH
deficiency; thus, there may be a degree of GH resistance. However,
his IGF-1 value is at the upper end of the reference range for his age,
so it is unlikely that poor adherence to GH treatment (Answer A) is
responsible for slowing his growth rate.
In Silver-Russell syndrome, puberty typically starts at the early
end of the normal range and progresses rapidly. Delayed puberty
has not been reported in Silver-Russell syndrome, and, anecdotally,
it was not observed by a group that studied a large cohort of patients
with Silver-Russell syndrome. In one cohort, puberty began, on
average, 1 year earlier in children with Silver-Russell syndrome vs
children who were small for gestational age but did not have Silver-
Russell syndrome. While there is a history of constitutional growth
delay in this patient’s father, delayed puberty (Answer C) is highly
unlikely in the setting of Silver-Russell syndrome. Gonadotropin
deficiency (Answer B) is possible, but it has not been described as a
feature of Silver-Russell syndrome and is an unlikely explanation for
his lack of pubertal progression.
Children with Silver-Russell syndrome experience acceleration of
bone age advancement, most likely due to premature adrenarche
and early and rapid pubertal progression, as well as other factors not
yet identified. Untreated, this leads to attenuation of the pubertal
growth spurt and compromised adult height. As such, bone age
cannot be used to make accurate adult height predictions in children
with Silver-Russell syndrome. However, bone age can be expected to
correlate with timing of peak growth velocity. A height velocity of 7
to 8 cm/y would be expected with a bone age of 14 years. Thus,
early epiphyseal closure (Answer E) is unlikely to be a factor in this
patient’s growth pattern.
Recommendations in a consensus statement on the diagnosis and
management of Silver-Russell syndrome include considering
treatment with a GnRH agonist for at least 2 years in children with
evidence of central puberty (starting no later than 12 years in girls
and 13 years in boys) to preserve adult height potential. One study
reported a high rate of procedural difficulties with implant removal,
citing implant fracture during removal in 16 of 58 cases (28%).
Implant fracture is more likely in implants that have remained in
place longer. While there are no formal guidelines for management
of this situation, removing retained portions of an implant may not
be practical, and the recommendation is to allow the medication to
wear off over time.
For the patient in this case, the continued suppression of puberty
and length of time the implant remained in place suggest that the
implant may have fractured on removal, with retained histrelin
continuing to suppress puberty (Answer D). This may be
underrecognized clinically and should be considered when puberty
fails to resume with removal of a histrelin implant. Single histrelin
implants have been shown to consistently suppress puberty for at
least 2 years, thus the duration of action of retained fragments of
implant may be significantly longer.

Educational Objective
Identify characteristic variations in timing of pubertal events in
individuals with Silver-Russell syndrome.

Reference(s)
Wakeling EL, Brioude F, Lokulo-Sodipe O, et al. Diagnosis and management of Silver-
Russell syndrome: first international consensus statement. Nat Rev Endocrinol.
2017;13(2):105-124. PMID: 27585961
Davis JS, Alkhoury F, Burnweit C. Surgical and anesthetic considerations in histrelin
capsule implantation for the treatment of precocious puberty. J Pediatr Surg.
2014;49(5):807-810. PMID: 24851775
Lewis KA, Goldyn AK, West KW, Eugster EA. A single histrelin implant is effective for 2
years for treatment of central precocious puberty. J Pediatr. 2013;163(4):1214-1216. PMID:
23809043
34 ANSWER: B) MRI of the pancreas
This case illustrates a rare presentation of an insulinoma.
Insulinomas are extremely rare in children, often diagnosed after a
long duration of symptoms. In various case reports, affected children
are often misdiagnosed as having refractory seizures, insomnia, and
migraine headaches. Insulinoma recurrence is 4 times more common
within 10 years in individuals who have a pathogenic variant in the
MEN1 gene (associated with multiple endocrine neoplasia type 1)
than in those who do not.
Hypoglycemia in infancy can be a presentation of congenital
hyperinsulinism, GH deficiency, or adrenal insufficiency. Insulinoma
should be considered in children presenting with nonketotic
hypoglycemia beyond the age of infancy, as hypopituitarism does
not usually present with isolated hypoglycemia in older children.
GH and cortisol may not increase if hypoglycemia is insidious or if
there are frequent episodes of hypoglycemia as observed in this case.
Inappropriately high insulin and C-peptide in the presence of
nonketotic hypoglycemia indicate a β-islet–cell lesion. Other causes
of nonketotic hypoglycemia are carnitine deficiency or β-oxidation
defects such as MCAD (medium-chain acyl-coenzyme A
dehydrogenase deficiency) presenting with low insulin levels,
hepatomegaly, increased anion gap, hyperuricemia, elevated liver
transaminases, and hyperammonemia.
Localization of an insulinoma is challenging due to the
probability of multiple, diffuse, or extrapancreatic lesions. Up to 10%
of insulinomas are malignant and 6% are associated with multiple
endocrine neoplasia type 1. Different invasive and noninvasive
imaging modalities are used to localize the lesion, although the
sensitivities and specificities are not documented in children.
Invasive modalities include (1) arterial stimulation and venous
sampling, in which different arteries supplying pancreatic tissue are
infused with calcium followed by hepatic venous sampling to detect
a rise in insulin; (2) transhepatic portal venous sampling; and (3)
endoscopic ultrasonography. Peranteau et al reported 8 cases of
insulinoma in children; one had arterial stimulation and venous
sampling and another had transhepatic portal venous sampling.
Both modalities accurately localized the lesion, whereas endoscopic
ultrasonography localized the lesion in 2 of 5 cases.
MRI is a noninvasive modality with high sensitivity and
specificity for detecting insulinoma in adults. Peranteau et al
reported a detection rate of 50% (3/6) while Padidela et al reported
accurate localization in 88% (7/8) with MRI. Octreotide scans have
poor detection rates for insulinoma, as these tumors usually do not
express enough somatostatin receptors. 18F-DOPA is taken up by
neuroendocrine cells and is used to distinguish between focal and
diffuse lesions in congenital hyperinsulinism. The 18F-DOPA PET
scan is inconsistent in localizing insulinoma in children (100% [3/3]
in one series and 50% [2/4] in another). GLP-1 receptor imaging is a
new noninvasive modality that has shown 100% successful
localization in a small study of 6 cases. Radiolabeled peptides have
shown promising results in localizing the lesions in oncology and
endocrinology cases. GLP-1 binds to specific receptors on β-islet cells
to induce insulin release. GLP-1 receptors are expressed on
insulinomas in large quantity. In this patient, MRI of the pancreas
(Answer B) detected the lesion correctly at first presentation, as well
as at recurrence. An octreotide scan performed at first presentation
and PET scan performed at the time of recurrence both detected the
lesion correctly.
Sulfonylureas are used extensively for treating type 2 diabetes.
Sulfonylurea poisoning can produce sustained and profound
hypoglycemia refractory to intravenous dextrose, particularly in
children and elderly patients. A sulfonylurea screen (Answer A)
could be considered, but there is no suggestion in the vignette that
this child has access to a sulfonylurea. The history of rapid weight
gain also favors chronicity of insulin secretion vs an acute elevation
due to ingestion.
One should also consider factitious hypoglycemia due to
exogenous insulin administration. Endogenous insulin is
synthesized as a precursor, proinsulin, which undergoes enzymatic
cleavage to produce insulin and C-peptide in equimolar amounts.
Therefore, factitious hypoglycemia produced by exogenous insulin
administration leads to suppressed C-peptide levels. In addition,
proinsulin levels are poorly suppressible in patients with insulinoma
in contrast to those without insulinoma. In patients with
surreptitious use of insulin or an oral hypoglycemic agent, the
proinsulin level is either normal or decreased.
Diazoxide is a benzothiazine derivative that acts as a potent β-cell
KATP channel opener. The stabilization of open KATP channels
leads to the inhibition of insulin secretion. Diazoxide challenge
(Answer C) would not provide any diagnostic value in this case, but
it could help manage hypoglycemia until surgical intervention.
After a second critical sample, obtaining a third critical sample
(Answer D) is not necessary.
Genetic testing (Answer E) is relevant to establish the diagnosis of
multiple endocrine neoplasia type 1 (MEN1 pathogenic variant).
However, it is not the best next step in establishing the diagnosis of
insulinoma in this case.

Educational Objective
Guide the evaluation to localize an insulinoma in a child.

Reference(s)
Padidela R, Fiest M, Arya V, et al. Insulinoma in childhood: clinical, radiological, molecular
and histological aspects of nine patients. Eur J Endocrinol. 2014;170(5):741-747. PMID:
24599222
Peranteau WH, Palladino AA, Bhatti TR, et al. The surgical management of insulinomas in
children. J Pediatr Surg. 2013;48(12):2517-2524. PMID: 24314196
Kao KT, Simm PJ, Brown J. Childhood insulinoma masquerading as seizure disorder. J
Paediatr Child Health. 2014;50(4):319-322. PMID: 24698060
Gozzi Graf T, Brändle M, Clerici T, l’Allemand D. Insulinoma: only in adults?-case reports
and literature review. Eur J Pediatr. 2014;173(5):567-574. PMID: 23604412
Service FJ, McMahon MM, O’Brien PC, Ballard DJ. Functioning insulinoma—incidence,
recurrence, and long-term survival of patients: a 60-year study. Mayo Clin Proc.
1991;66(7):711-719. PMID: 1677058
Mirallié E, Pattou F, Malvaux P, et al. Value of endoscopic ultrasonography and
somatostatin receptor scintigraphy in the preoperative localization of insulinomas and
gastrinomas. Experience of 54 cases. Gastroenterol Clin Biol. 2002;26(4):360-366. PMID:
12070411
Christ E, Wild D, Forrer F, et al. Glucagon-like peptide-1 receptor imaging for localization of
insulinomas. J Clin Endocrinol Metab. 2009;94(11):4398-4405. PMID: 19820010
Sweet CB, Grayson S, Polak M. Management strategies for neonatal hypoglycemia. J Pediatr
Pharmacol Ther. 2013;18(3):199-208. PMID: 24052783
35 ANSWER: B) Calcium supplementation and calcitriol
This patient has a classic presentation of rickets, including a
prominent forehead, rachitic rosary of the ribs, limb deformities,
widening of the wrists, and poor growth/short stature. In addition
to history and physical examination findings, biochemical and
radiologic findings (see images) are helpful in the diagnosis.
Management of a patient with rickets depends on the underlying
cause (see table). This patient had a low calcium level, low phosphate
level, normal 25-hydroxyvitamin D level, and inappropriately low
normal 1,25-dihydroxyvitamin D level, especially in the setting of
elevated PTH. This pattern is consistent with the diagnosis of 1α-
hydroxylase deficiency leading to vitamin D–dependent rickets type
1A. The appropriate treatment is calcitriol with or without calcium
supplementation (Answer B).

Table. Causes of Calcipenic or Hypophosphatemic Rickets (not all inclusive)

25- 1,25-
Serum Hydroxy- Dihydroxy-
calcium Serum phosphate vitamin D vitamin D Diagnosis Treatment
⇓⇔ ⇓⇔ ⇔ ⇑ Nutritional calcium deficiency Calcium
⇓⇔ ⇓⇔ ⇓ ⇓⇔ Nutritional vitamin D deficiency Calciferol ± calcium
⇓⇔ ⇓⇔ ⇓⇓ ⇓ 25-Hydroxylase deficiency Calcitriol ± calcium
Despite (vitamin D–dependent rickets type 1B)
adequate
calciferol
intake
⇓⇔ ⇓⇔ ⇔ ⇓ or low- 1α-Hydroxylase deficiency Calcitriol ± calcium
normal (vitamin D–dependent rickets type 1A)
⇓⇔ ⇓⇔ ⇓ ⇑ Hereditary 1,25-dihydroxyvitamin D–resistant rickets due to pathogenic High dosages of
variants in the VDR gene (vitamin D–dependent rickets type 2) or to calcitriol ± calcium
overexpression of a vitamin D response element VDRE (vitamin D–
dependent rickets type 2B)
⇑⇔ ⇓ ⇔ ⇑⇔ Nutritional phosphate deficiency Phosphorus
supplementation
Low urine
phosphate
Low or
normal
serum
FGF-23
⇔ ⇓ ⇔ ⇓ FGF-23–dependent hypophosphatemic rickets Calcitriol and
phosphorus
High urine XLH (PHEX pathogenic variant) supplementation or
phosphate ADHR (FGF23 pathogenic variant) burosumab therapy
High ARHR (DMP1 or ENPP1 pathogenic variant)
serum
FGF-23
⇔ ⇓ ⇔ ⇑⇔ FGF-23–independent hypophosphatemic rickets Phosphorus
supplementation
High urine Low or normal FGF-23 level
phosphate Pathogenic variants in SLC34A1 (encoding sodium-
Low or dependent phosphate transport protein 2A [NPT2A])
normal Pathogenic variants in SLC34A3 (encoding NTP2C), causing
serum hereditary hypophosphatemic rickets with hypercalciuria
FGF-23

Note: Images in this vignette provided courtesy of Dr. Sirisha Kusuma, Rainbow Children’s
Hospital, Hyderabad and Vijayawada, India.

Treating with calcium alone (Answer D) or with calcium and


ergocalciferol (Answer A) is not enough in this patient, as he has a
normal vitamin D concentration and cannot convert that to 1,25-
dihydroxyvitamin D given his diagnosis of 1α-hydroxylase
deficiency. Improving the 1,25-dihydroxyvitamin D level by
administering calcitriol is necessary, so he can absorb enteral calcium
intake through diet and supplementation. Calcitriol and phosphorus
supplementation (Answer C) or treatment with the human
monoclonal antibody burosumab (Answer E) would be options for
patients with X-linked hypophosphatemic rickets who have a
normal serum calcium level.

Educational Objective
Diagnose and treat vitamin D–dependent rickets type 1A.

Reference(s)
Carpenter TO, Shaw NJ, Portale AA, Ward LM, Abrams SA, Pettifor JM. Rickets. Nat Rev
Dis Primers. 2017;3:17101. PMID: 29265106
Thacher TD, Levine MA. CYP2R1 mutations causing vitamin D-deficiency rickets. J Steroid
Biochem Mol Biol. 2017;173:333-336. PMID: 27473561
Goldsweig BK, Carpenter TO. Hypophosphatemic rickets: lessons from disrupted FGF23
control of phosphorus homeostasis. Curr Osteoporos Rep. 2015;13(2):88-97. PMID:
25620749
Razali NN, Hwu TT, Thilakavathy K. Phosphate homeostasis and genetic mutations of
familial hypophosphatemic rickets. J Pediatr Endocrinol Metab. 2015;28(9-10):1009-1017.
PMID: 25894638
Munns CF, Shaw N, Kiely M, et al. Global consensus recommendations on prevention and
management of nutritional rickets. J Clin Endocrinol Metab. 2016;101(2):394-415. PMID:
26745253
36 ANSWER: A) Discontinue methimazole and discuss
thyroidectomy
Thyrotoxicosis is a clinical condition caused by increased thyroid
hormone action. Hyperthyroidism is thyrotoxicosis secondary to
increased synthesis of thyroid hormones by the thyroid gland. The
most common cause of hyperthyroidism in children is Graves
disease. Graves disease is an autoimmune thyroid disease. The
pathophysiology of autoimmune thyroid disease involves diffuse
infiltration of T-cell lymphocytes into the thyroid gland, while B-cell
lymphocytes produce different autoantibodies (most commonly
against the TSH receptor (TSHR), thyroglobulin, and TPO. In Graves
disease, the predominant antibodies are directed against the TSHR
(TRAb), with predominant activating-type antibodies.
In children and adolescents, Graves disease can be treated with
methimazole, radioactive iodine therapy, or thyroidectomy.
However, the use of radioactive iodine is not recommended for
children younger than 5 years because of the higher risk of thyroid
cancer in this population after external radiation. Medical treatment
is considered first-line therapy in children and adolescents because
some children go into remission.
Antithyroid drugs used in the United States include methimazole
and propylthiouracil. In Europe, carbimazole, a methimazole pro-
drug, is also available. Antithyroid drugs peak in serum within 1 to 2
hours after ingestion and antithyroid effects last 12 to 24 hours. Their
primary effect is to inhibit thyroid hormone synthesis by blocking
TPO. Propylthiouracil also blocks the peripheral conversion of T4
into T3, and these drugs have also been shown to have
immunomodulatory effects.
Methimazole is the recommended drug in children and
adolescents, while propylthiouracil is contraindicated in this
population because of the high risk of hepatotoxicity (liver failure in
1 in 2000 to 1 in 4000 children). Methimazole can be given once daily,
as it has a longer duration than propylthiouracil, and the dosage
used ranges between 0.1 and 1.0 mg/kg per day, usually 0.2 to 0.5
mg/kg per day. To maintain euthyroidism, the dosage is usually
decreased as levels normalize. The approach of blocking with
methimazole and replacing with levothyroxine is not currently
recommended in the guidelines because this practice has been
associated with higher dosages of methimazole and complications.
Antithyroid medications have mild and severe adverse effects.
Mild adverse effects include rash, arthralgias, nausea, abnormal
sense of taste or smell, and, very rarely, sialadenitis. Rash can be
managed by concurrent administration of antihistamine. Major
complications include agranulocytosis, hepatitis, cholestasis, liver
failure, ANCA-positive vasculitis, and polyarthritis. Very rarely
thrombocytopenia and aplastic anemia and pancreatitis may also
develop.
Most adverse effects of methimazole are dosage dependent;
therefore, lower dosages are recommended except in cases of severe
hyperthyroidism. However, reactions can occur even at lower
dosages and all patients must be warned about adverse effects.
Adverse effects in children have been reported in 6% to 35%,
usually in the first 3 months of treatment, although they can occur at
any time.
Mild granulocytopenia and mild elevation of liver enzymes can
be seen in patients with Graves disease. Therefore, obtaining
baseline levels before starting treatment is a good practice, as
illustrated in this patient.
Complete blood cell count with differential should be ordered
immediately if the patient develops fever, sore throat, arthralgias,
mouth sores, or malaise. Liver enzymes must be measured
immediately if the patient develops abdominal pain, nausea,
vomiting, jaundice, dark urine, light-colored stools, pruritus,
anorexia, or malaise.
Patients and their caregivers should be educated on the adverse
effects of antithyroid drugs, preferably in writing, and they should
be advised to stop the medication immediately and to contact their
physician if they develop any of the symptoms associated with
major complications of antithyroid medications. Antithyroid
medications should be discontinued if transaminase levels (in
asymptomatic or symptomatic patients) are 2 to 3 times the upper
normal limit. After discontinuing the drug, levels must be monitored
weekly until they resolve. If there is no evidence of improvement,
referral to gastroenterology is indicated. This patient’s levels are
already 2 to 3 times above the upper normal limit, and they are
rising. Decreasing the methimazole dosage (Answers C and E) is not
recommended. Although the patient may benefit from seeing a
gastroenterologist/hepatologist (Answers D and E) if liver enzymes
do not improve following discontinuation of methimazole, referring
her to gastroenterology before discontinuing the drug is incorrect.
This patient has developed a severe reaction to methimazole and
the drug must be discontinued. She needs definitive treatment for
Graves disease. In view of her age, total thyroidectomy in the hands
of a high-volume surgeon (Answer A) is the best course of action.
Radioactive iodine (Answer B) is not recommended in children
younger than 5 years. She will need supersaturated potassium
iodine drops (SSKI, 50 mg iodine/drop), 1 to 2 drops 3 times daily
for 10 days before surgery.

Educational Objective
Summarize the medical management of Graves disease with
antithyroid drugs, including pharmacologic actions, dosage,
monitoring, adverse effects, and indications for seeking alternative
treatments.

Reference(s)
Ross DS, Burch HB, Cooper DS, et al. 2016 American Thyroid Association guidelines for
diagnosis and management of hyperthyroidism and other causes of thyrotoxicosis.
Thyroid. 2016;26(10):1343-1421. PMID: 27521067
Rivkees SA. Controversies in the management of Graves’ disease in children. J Endocrinol
Invest. 2016;39(11):1247-1257. PMID: 27153850
Bauer AJ. Approach to the pediatric patient with Graves’ disease: when is definitive therapy
warranted? J Clin Endocrinol Metab. 2011;96(3):580-588. PMID: 21378220
Cooper DS. Antithyroid drugs. N Engl J Med. 2005;352(9):905-917. PMID: 15745981
37 ANSWER: C) Brain and pituitary MRI
Brain and pituitary MRI (Answer C) would be the most useful next
test to determine the position and morphology of the pituitary gland
(both anterior and posterior) and pituitary stalk, as well as presence
or absence of the septum pellucidum, optic nerve appearance,
midline defects, or masses. MRI of the brain and pituitary is the most
sensitive method to image the hypothalamic-pituitary axis. In this
newborn, brain MRI was normal, thus excluding any structural or
space-occupying lesions that may affect the hypothalamic-pituitary
axis.
The diagnosis of secondary adrenal insufficiency is clear in view
of the undetectable serum cortisol and inappropriately normal
plasma ACTH concentrations during a documented hypoglycemic
episode. Therefore, performing either a standard dose (Answer A) or
low-dose (Answer B) cosyntropin-stimulation test is not necessary.
Identification of additional pituitary hormone deficiencies (TSH, GH,
gonadotropin, diabetes insipidus) is important, although this can be
challenging during the neonatal period. In this vignette, the normal
free T4, TSH, and GH levels during a hypoglycemic episode make
deficiencies in TSH and GH unlikely now. Gonadotropin
measurement would be helpful to detect mini puberty and to
exclude congenital hypogonadotropic hypogonadism. The
recommendation is to perform gonadotropin hormone profiling at 4
to 8 weeks of life. Prolactin levels should also be assessed, as some
genetic forms of congenital hypopituitarism are associated with
prolactin deficiency. Central diabetes insipidus should always be
considered, especially in neonates and children with midline defects.
In this child, the lack of polyuria and hypernatremia is reassuring. It
is important to remember that diabetes insipidus can be masked in
patients with ACTH deficiency, as cortisol is essential for water
excretion, and diabetes insipidus may only present clinically
following treatment with hydrocortisone.
Measuring 17-hydroxyprogesterone (Answer D) and performing
ultrasonography of the adrenal glands (Answer E) are unhelpful in
the situation of secondary adrenal insufficiency.
Secondary adrenal insufficiency can be congenital or acquired
and either isolated or combined with other pituitary hormone
deficiencies. Genetic workup should be performed in all patients
with congenital secondary adrenal insufficiency. Recessive
pathogenic variants in the TBX19 gene (formerly TPIT) are the main
cause of congenital isolated ACTH deficiency. In this particular case,
a homozygous frameshift mutation in exon 6 (782delA) was
identified. ACTH deficiency has also been described in other
congenital forms of hypopituitarism associated with pathogenic
variants in the following genes: PROP1, LHX3, FGF8, LHX4, ARNT2,
GLI2, PCSK1, and POMC. Both PCSK1 and POMC are associated
with weight gain and hypopigmentation. With several of these
conditions, such as PROP1 pathogenic variants, ACTH deficiency
can evolve with time, so continuous monitoring and assessment are
required. An important acquired cause of secondary adrenal
insufficiency is the use and subsequent withdrawal of
glucocorticoid-containing drugs. Other causes of secondary adrenal
insufficiency include trauma (prenatal or postnatal), infection or
inflammation (eg meningitis, sarcoidosis, tuberculosis,
autoimmune), infiltrative causes (eg, Langerhans cell histiocytosis,
hemochromatosis), tumors (eg, craniopharyngiomas), or pituitary
surgery or radiotherapy.
Diagnosis of secondary adrenal insufficiency can be challenging,
especially in the first 9 months of life before the establishment of a
circadian rhythm (eg, asymptomatic neonates with midline defects
at risk of adrenal insufficiency). In such circumstances, multiple
paired cortisol and ACTH measurements may point towards a
diagnosis. A standard cosyntropin-stimulation test is safe and easy
to perform. Once a circadian rhythm has been established, the
combination of an 8-AM serum cortisol measurement greater than
6.3 µg/dL (>175 nmol/L) and a 30-minute serum cortisol value
greater than 18.1 µg/dL (>500 nmol/L) on standard cosyntropin-
stimulation testing have a sensitivity of 69% and a specificity of
100% in excluding ACTH deficiency. The low-dose cosyntropin-
stimulation test has been proposed as a more sensitive test for
diagnosing secondary adrenal insufficiency although there is no
consensus and it is not performed universally. The corticotropin-
releasing hormone test (corticotropin-releasing hormone 1 mcg/kg,
maximum dose 100 mcg) has been suggested to distinguish
hypothalamic from pituitary disease in secondary adrenal
insufficiency. However, because of variable responses and undefined
normal ranges, the usefulness of this test is unclear.
Treatment of secondary adrenal insufficiency consists of
glucocorticoid replacement. Endocrine Society guidelines suggest a
starting dosage of 8 mg/m2 per day in 3 to 4 divided doses to be
titrated according to individual need. The American Academy of
Pediatrics suggests a dosage of 9 to 12 mg/m2 per day to
compensate for incomplete intestinal absorption and hepatic
metabolism. Hydrocortisone is preferred in children over other types
of glucocorticoid (prednisolone and dexamethasone) to avoid
adverse effects and to be able to titrate treatment more accurately.
Depending on the cause and associated pituitary deficiency,
additional hormone replacement may be required at varying time
points in childhood. Before thyroid hormone replacement is
initiated, it is important to ensure that glucocorticoid deficiency is
adequately treated. Furthermore, patients with combined pituitary
hormone deficiencies who start glucocorticoid replacement should
be carefully monitored for development of diabetes insipidus. Like
all forms of adrenal insufficiency, primary or secondary emergency
management during illnesses and trauma, along with patient and
family education is critical. Apart from individuals with isolated
ACTH deficiency due to pathogenic variants in the TBX19 gene,
other causes of secondary adrenal insufficiency are likely to require
long-term surveillance and assessment of progressive involvement
of other pituitary hormones.

Educational Objective
Develop an approach to the diagnosis and evaluation of secondary
adrenal insufficiency.

Reference(s)
Patti G, Guzzeti C, Di lorgi N, et al. Central adrenal insufficiency in children and
adolescents. Best Pract Res Clin Endocrinol Metab. 2018;32(4):425-444. PMID: 30086867
Alatzoglou KS, Dattani MT. Genetic forms of hypopituitarism and their manifestation in the
neonatal period. Early Hum Dev. 2009;85(11):705-712. PMID: 19762173
Couture C, Saveanu A, Barlier A, et al. Phenotypic homogeneity and genotypic variability
in a large series of congenital isolated ACTH-deficiency patients with TPIT gene
mutations. J Clin Endocrinol Metab. 2012;97(3):E486-E495. PMID: 22170728
Higham CE, Johannsson G, Shalet SM. Hypopituitarism. Lancet. 2016;388(10058):2403-2415.
PMID: 27041067
Shulman DI, Palmert MR, Kemp SF; Lawson Wilkins Drug and Therapeutics Committee.
Adrenal insufficiency: still a cause of morbidity and death in childhood. Pediatrics.
2007;119(2):e484-e494. PMID: 17242136
Bornstein SR, Allolio B, Arlt W, et al. Diagnosis and treatment of primary adrenal
insufficiency: an Endocrine Society clinical practice guideline. J Clin Endocrinol Metab.
2016;101(2):364-389. PMID: 26760044
38 ANSWER: E) Heterozygous inactivating pathogenic variant
in NPR2 in the patient and homozygous inactivating pathogenic
variants in NPR2 in her brother
The 4-year-old girl in this vignette has mild short stature, normal
bone age, and normal laboratory results. These findings would be
consistent with possible diagnoses of familial short stature (as her
parents are short) or idiopathic short stature. However, her family
history is revealing in that there is parental consanguinity, which
may increase the possibility of autosomal recessive disorders. In fact,
her brother has severe disproportionate short stature with
dysmorphic features consistent with acromesomelic dysplasia
Maroteaux type (short stature with shortening of the distal and
midsegments of the extremities, prominent forehead, wide and
depressed nasal bridge, prominent lips) and normal intellect. This
condition is due to homozygous inactivating pathogenic variants in
the NPR2 gene, which encodes natriuretic peptide receptor B, one of
the receptors for C-natriuretic peptide. C-natriuretic peptide,
through its interaction with natriuretic peptide receptor B, stimulates
growth plate chondrogenesis and, therefore, promotes linear growth.
Homozygous inactivating pathogenic variants in NPR2 are
responsible for acromesomelic dysplasia Maroteaux type, while
heterozygous inactivating pathogenic variants have been described
in individuals with idiopathic short stature without significant
dysmorphism except for slightly short extremities, as manifested by
an arm span that is slightly shorter than the height of this 4-yearold
girl. Thus, Answer E is correct. Both parents have mild-to-moderate
short stature and are obligate carriers of NPR2 inactivating
pathogenic variants. As such, with each pregnancy, they have a 25%
chance of having a child with acromesomelic dysplasia Maroteaux
type (homozygous NPR2 pathogenic variants), a 50% chance of
having a child with isolated short stature of mild-to-moderate
degree (heterozygous NPR2 pathogenic variant), and a 25% chance
of having an unaffected child (normal biallelic NPR2 gene).
Activating pathogenic variants in the NPR2 gene (Answer B) would
be expected to lead to an overgrowth condition. This has been
indirectly demonstrated in animals overexpressing the C-natriuretic
peptide gene.
Inactivating FGFR3 pathogenic variants (Answer C) result in
overgrowth conditions. Achondroplasia (Answer A) results from
heterozygous activating pathogenic variants in the FGFR3 gene.
Achondroplasia is inherited in an autosomal dominant manner or
arises by de novo mutation. The condition is characterized by
rhizomelic (proximal [ie, humeral and femoral]) limb shortening,
macrocephaly, midface hypoplasia, and trident hand configuration,
which are easily recognized at birth. Achondroplasia differs from
acromesomelic dysplasia Maroteaux type in which the limb
segments that appear short are the mid and distal segments and the
shortening is not manifested at birth but becomes evident
postnatally. Pseudoachondroplasia is an unrelated disorder due to
heterozygous (autosomal dominant) pathogenic variants in the
COMP gene (cartilage oligomeric matrix protein), which lead to
accumulation of an abnormal version of this protein in the
endoplasmic reticulum that induces endoplasmic reticulum stress
and death of the chondrocytes. Clinical manifestations include
disproportionate short stature, deformities of the lower extremities,
short fingers, laxity of ligaments, and progressive osteoarthritis.
Although Turner syndrome could be a consideration in this 4-
year-old girl, even in the absence of stigmata suggestive of this
condition, it is unlikely that the severe short stature and multiple
dysmorphic features of the 8-year-old brother are due to SHOX
haploinsufficiency (Answer D), where mesomelic (midsegment) limb
shortening along with Madelung deformity are more characteristic.

Educational Objective
Identify heterozygous inactivating pathogenic variants in the NPR2
gene as a cause of idiopathic short stature and homozygous
pathogenic NPR2 variants as the cause of acromesomelic dysplasia
Maroteaux type.
Reference(s)
Olney R, Bükülmez H, Bartels C, et al. Heterozygous mutations in natriuretic peptide
receptor-B (NPR2) are associated with short stature. J Clin Endocrinol Metab.
2006;91(4):1229-1232. PMID: 16384845
Wang SR, Jacobsen CM, Carmichael, et al. Heterozygous mutations in natriuretic peptide
receptor-B (NPR2) gene as a cause of short stature. Hum Mutat. 2015;36(4):474-481.
PMID: 25703509
Pauli RM. Achondroplasia: a comprehensive clinical review. Orphanet J Rare Dis.
2019;14(1):1-49. PMID: 30606190
39 ANSWER: D) BMI percentile
Bariatric procedures are increasingly used as a means of effectively
inducing weight loss and avoiding complications of extreme obesity
in adolescents. The American Academy of Pediatrics developed
guidelines to help determine which patients are appropriate
candidates for bariatric surgery. The December 2019 policy statement
recommends that pediatric patients with the following findings be
considered for bariatric surgery:

Class 2 obesity, BMI ≥35 kg/m2 or 120% of the 95th


percentile for age and sex (whichever is lower), with
clinically significant comorbidities (including obstructive
sleep apnea [apnea-hypopnea index >5], type 2 diabetes,
idiopathic intracranial hypertension, nonalcoholic
steatohepatitis, Blount disease, slipped capital femoral
epiphysis, gastroesophageal reflux disease, and
hypertension)
OR
Class 3 obesity, BMI ≥40 kg/m2 or 140% of the 95th
percentile for age and sex (whichever is lower), with or
without comorbidities
BMI and BMI percentile (Answer D) are considered one of the
criteria to determine whether a pediatric patient should be
considered for bariatric surgery. This patient’s BMI of 38 kg/m2,
however, does not meet criteria for recommending bariatric surgery,
as he has no qualifying comorbidities.
Dyslipidemia (Answer A) is not considered to be a clinically
significant comorbidity according to the guidelines. Neither family
history of bariatric surgery (Answer C) nor history of participating
in a formal weight-loss program (Answer E) is a consideration for
bariatric surgery eligibility according to the American Academy of
Pediatrics.
Completion of pubertal growth (Answer B) is not necessary in
order to proceed with a bariatric procedure. Therefore, this patient’s
age does not disqualify him from being considered for bariatric
surgery. Previously, clinicians were concerned that the procedure
might accelerate and limit further linear growth in adolescent
patients. In fact, studies, including a prospective study by Olbers et
al, have shown that patients who have the procedure continue to
grow afterwards.

Educational Objective
Identify criteria to recommend bariatric procedures in adolescents.

Reference(s)
Armstrong AC, Bolling CF, Michalsky MP, Reichard KW; Section on Obesity, Section on
Surgery. Pediatric metabolic and bariatric surgery: evidence, barriers, and best practices.
Pediatrics. 2019;144(6):e20193223, PMID: 31656225
Alqahtani A, Elahmedi M, Qahtani AR. Laparoscopic sleeve gastrectomy in children
younger than 14 years: refuting the concerns. Ann Surg. 2016;263(2):312-319. PMID:
26496081
Pratt JSA, Browne A, Browne NT, et al. ASMBS pediatric metabolic and bariatric surgery
guidelines, 2018. Surg Obes Relat Dis. 2018;14(7):882-901. PMID: 30077361
Olbers T, Beamish AJ, Gronowitz E, et al. Laparoscopic Roux-en-Y gastric bypass in
adolescents with severe obesity (AMOS): a prospective, 5-year, Swedish nationwide
study [published correction appears in Lancet Diabetes Endocrinol. 2017;5(5):e3]. Lancet
Diabetes Endocrinol. 2017;5(3):174-183. PMID: 28065734
40 ANSWER: D) Ovarian failure due to follicle depletion
This patient has hypergonadotropic hypogonadism, as indicated by
elevated FSH and LH and low serum estradiol. This most often falls
under the category of primary ovarian insufficiency (POI). POI is
defined as ovarian failure before age 40 years and has an incidence
of approximately 1%. In this vignette, lack of pubertal development
suggests absent ovarian function before onset of central puberty.
In POI, ovarian failure is due to either ovarian follicle dysfunction
or accelerated ovarian depletion, without clear etiology
approximately 90% of the time. As such, ovarian follicle depletion
(Answer D) is the most likely etiology of the options listed. Other
causes of POI include autoimmunity, galactosemia, and fragile X
premutation.
An increased number of CGG triplet repeats in the FMR1 gene
(Answer A) is associated with both fragile X syndrome and fragile X
premutation. Normally, this DNA segment is repeated 5 to 40 times.
In fragile X syndrome, the segment is repeated more than 200 times,
which causes loss of the FMR1 protein. Small CGG repeat
expansions in the range of 55 to 200 repeats, classified as a fragile X
premutation, causes production of a dysfunctional protein. Fragile X
premutations are identified in approximately 5% of patients of POI.
Individuals with fragile X syndrome, however, have the same risk of
POI as the general population.
Excess ovarian androgen production (Answer B) can cause
anovulation, but lack of pubertal development would not be
expected. Individuals with polycystic ovary syndrome have an
elevated LH-to-FSH ratio, but within normal range and not
suggestive of POI, as in this case. This patient lacks other features
associated with polycystic ovary syndrome such as hirsutism, and
hyperandrogenemia would be an unlikely explanation for her
examination and laboratory findings.
Elevated prolactin has a number of causes, including pituitary
adenoma, other hypothalamic and pituitary disorders,
hypothyroidism, renal or liver failure, or autoimmune disorders.
Hyperprolactinemia is a fairly common cause of amenorrhea;
however, the mechanism is due to inhibition of LH pulses, so the
expected hormone profile would show low levels of FSH and LH. As
such, elevated prolactin levels (Answer E) would not be consistent
with this case presentation.
Several chromosomal abnormalities (Answer C) are associated
with POI, including Turner syndrome and tetrasomy X. Short stature
is a nearly universal finding in Turner syndrome. This patient is
growing well along higher percentiles and has no other features
consistent with Turner syndrome. While half of females with
tetrasomy X develop POI, most have other identifiable features,
including dysmorphic facial features, cleft palate, dental
abnormalities, hypotonia, scoliosis, developmental delays, hearing
and vision problems, cardiac defects, and seizures. While a
karyotype would be indicated in any female patient with POI, the
patient in this vignette is unlikely to have a chromosomal
abnormality as an explanation for her hypogonadism.

Educational Objective
Identify the underlying etiology of primary ovarian insufficiency in
an adolescent girl.

Reference(s)
Nelson LM. Clinical practice. Primary ovarian insufficiency. N Engl J Med. 2009;360(6):606-
614. PMID: 19196677
Torrealday S, Kodaman P, Pal L. Premature ovarian insufficiency - an update on recent
advances in understanding and management. F1000Res. 2017;6:2069. PMID: 29225794
41 ANSWER: B) Thyroglobulin measurement and assessment of
thyroglobulin antibodies
In most cases of hyperthyroidism, total T3, total T4, and free T4 are
elevated. Some patients with Graves disease, for example, present
with T3 thyrotoxicosis, but free T4 would not be in the low-normal
range as it is in this patient, and the discrepancy between T4 and T3
would not be as great as it is in this vignette. The clinician must
determine whether this patient has subacute thyroiditis or
exogenous intoxication, most likely from the diet pills.
A complete blood cell count with differential (Answer A) would
not help distinguish between the 2 diagnoses. Measuring calcitonin
(Answer C) is not indicated. Procalcitonin measurement (Answer D)
could help assess whether there is a bacterial infection (which could
also explain an elevated erythrocyte sedimentation rate), but it
would not be helpful in this case, as subacute thyroiditis is often
associated with preceding viral illnesses. Thyroid ultrasonography
(Answer E) may be helpful in imaging a hot nodule, but a focus of
intense uptake would have been visible on the thyroid scan if that
were the case.
The best next step is to measure thyroglobulin (Answer B), which
would be low in the setting of exogenous intoxication and normal in
subacute thyroiditis. Measuring thyroglobulin antibodies would
help ensure that the thyroglobulin level reported is not artificially
low due to an elevated antibody titer. Specialized thyroglobulin
assays (liquid chromatography/tandem mass spectrometry) used to
monitor patients with thyroid cancer can also be used in the setting
of elevated thyroglobulin antibodies.
This patient was indeed receiving exogenous T3 from her diet
pills, which explains the very high T3 and normal free T4. She
obtained the pills while on vacation in Mexico. Such pills are banned
by the US FDA, but they can still be purchased outside the United
States. The diet pills she took each contained 75 mcg of liothyronine,
as well as norpseudoephedrine, atropine, aloin, and diazepam,
which can mask or alter some of the symptoms of thyrotoxicosis.

Educational Objective
Distinguish between subacute thyroiditis and exogenous
intoxication in a patient with hyperthyroidism.

Reference(s)
Graves CL, Newfield RS. Dangerous dieting – Mexican diet pills and T3 thyrotoxicosis. J
Case Rep Images Pediatr. 2018;1:100002Z19CG2018.
Pacifico L, Osborn JF, Natale F, Ferraro F, De Curtis M, Chiesa C. Procalcitonin in pediatrics.
Adv Clin Chem. 2013;59:203-263. PMID: 23461137
42 ANSWER: E) Vitamin D supplementation, 2000 IU daily;
enteral calcium, 50 mg/kg per day; and intravenous calcium
gluconate as needed
In general, infants double their birth weight by 4 months of age. This
infant has failed to do so by 6 months of age, most likely because of
insufficient caloric intake from a primarily plant-based diet.
Individuals who follow a vegan diet are at risk for protein, vitamin
D, and calcium deficiencies in addition to insufficient intake of other
micronutrients and macronutrients. This patient has evidence of
significant malnourishment, as well as calcium and phosphorus
deficiencies secondary to severe vitamin D deficiency. Although his
1,25-dihydroxyvitamin D level is also low, this is most likely
secondary to profound hypovitaminosis D as opposed to an
inhibition of 1α-hydroxylase. Calcitriol supplementation is not
recommended as part of the initial management of hypocalcemia
because endogenous levels of 1,25-dihydroxyvitamin D will rise
substantially within 72 hours. Providing a maintenance dosage of
calcitriol (Answers B and C) can prevent this rise and actually
impede the treatment of hypocalcemia. In addition, normal infant
formula generally has sufficient phosphorus, so additional
supplementation is not necessary. It is also important to recognize
that intravenous calcium gluconate will temporarily increase serum
calcium levels, but the concentration will drop again within 2 to 3
hours if enteral calcium is not provided. Intravenous calcium
(Answer C) can be used as an adjunct to enteral calcium when
symptoms are severe or patients are not able to tolerate enteral
calcium, but it should not be the sole source of calcium
supplementation.
In Answer A, the vitamin D supplementation only provides the
recommended daily allotment (RDA) for infants. This is not
sufficient to adequately replete the vitamin D stores in this patient.
While breastfeeding should be supported when possible, and studies
have shown that maternal high-dosage vitamin D supplementation
of 5000 to 10,000 IU daily (Answer D) results in normal infant serum
25-hydroxyvitamin D levels without direct infant supplementation,
this also would be appropriate for maintaining vitamin D levels, not
increasing them to normal from a deficient state. In addition, the
concentration of both calcium and phosphorus in maternal breast
milk at 6 months postpartum is low.
Severe vitamin D deficiency and hypocalcemia can cause
significant cardiac effects, including prolonged QTc interval (present
in this patient), as well as dilated cardiomyopathy secondary to
ventricular dilatation. In a case series of 16 infants with vitamin D–
associated dilated cardiomyopathy, 6 presented with cardiac arrest
and 10 presented in heart failure. Fifty percent of patients required
mechanical ventilation. Nearly 20% (3/16) of the children died. The
child in this vignette has profound hypocalcemia with
electrocardiographic changes, increasing his risk for dilated
cardiomyopathy. These findings warrant urgent intervention with
intravenous calcium and ongoing monitoring of telemetry. Enteral
calcium must be introduced simultaneously with intravenous
calcium to allow for sustained repletion of calcium stores. High-
dosage vitamin D supplementation is necessary to allow absorption
of enteral calcium. Monitoring the patient’s other electrolytes during
refeeding is also reasonable. Thus, Answer E provides the best
approach for treating the manifestations of severe vitamin D
deficiency in this infant.

Educational Objective
Recognize that severe vitamin D deficiency in infancy can present
with cardiac changes, including prolonged QTc interval, dilated
cardiomyopathy, laryngospasm, and seizures.

Reference(s)
Maiya S, Sullivan I, Allgrove J, et al. Hypocalcaemia and vitamin D deficiency: an
important, but preventable, cause of life-threatening infant heart failure. Heart.
2008;94(5):581-584. PMID: 17690157
Hollis BW, Wagner CL, Howard CR, et al. Maternal versus infant vitamin D
supplementation during lactation: a randomized controlled trial. Pediatrics.
2015;136(4):625-634. PMID: 26416936
43 ANSWER: C) Thyroid C-cell tumors: contraindicated in
patients with a personal or family history of medullary thyroid
cancer or in patients with multiple endocrine neoplasia type 2
A “black box” warning is the strictest warning put in the labeling of
a prescription drug by the US FDA when there is reasonable
evidence of an association of a serious hazard with the drug. First
implemented in 1979, black box warnings highlight serious and
sometimes life-threatening adverse drug reactions within the
labeling of prescription drug products.
Liraglutide is a GLP-1 receptor agonist. GLP-1 is a peptide
hormone that increases insulin secretion and decreases glucagon
secretion from the pancreas in a glucose-dependent manner. GLP-1
receptor agonists provide pharmacologic levels of GLP-1, which
reduce glucose and weight by increasing glucose-dependent insulin
secretion, decreasing glucagon secretion, delaying gastric emptying,
and increasing satiety. All of the GLP-1 receptor agonist agents are
administered as subcutaneous injections.
Liraglutide is indicated as an adjunct to diet and exercise to
improve glycemic control in patients aged 10 years and older with
type 2 diabetes. It reduces the risk of major adverse cardiovascular
events in adults with type 2 diabetes and established cardiovascular
disease.
Liraglutide causes thyroid C-cell tumors at clinically relevant
exposures in both sexes of rats and mice. It is unknown whether
liraglutide causes thyroid C-cell tumors, including medullary
thyroid carcinoma, in humans. The human relevance of liraglutide-
induced rodent thyroid C-cell tumors has not been determined.
However, liraglutide has a black box warning and is contraindicated
in patients with a personal or family history of medullary thyroid
carcinoma or in patients with multiple endocrine neoplasia type 2.
Physicians who prescribe liraglutide should counsel patients
regarding the potential risk of medullary thyroid carcinoma and the
symptoms of thyroid tumors (Answer C).
Other warnings and precautions include:
Pancreatitis (Answer A): Postmarketing reports, including
fatal and nonfatal hemorrhagic or necrotizing pancreatitis.
Discontinue promptly if pancreatitis is suspected. Do not
restart if pancreatitis is confirmed.
Serious hypoglycemia (Answer D): When liraglutide is
used with an insulin secretagogue (eg, a sulfonylurea) or
insulin, consider lowering the dosage of the insulin
secretagogue or insulin to reduce the risk of hypoglycemia.
The risk of hypoglycemia is higher in pediatric patients 10
years and older regardless of concomitant antidiabetes
therapies.
Renal impairment (Answer B): Postmarketing reports,
usually in association with nausea, vomiting, diarrhea, or
dehydration that may sometimes require hemodialysis.
Use caution when initiating or escalating doses of
liraglutide in patients with renal impairment.
Hypersensitivity (Answer E): Postmarketing reports of
serious hypersensitivity reactions (eg, anaphylactic
reactions and angioedema). Liraglutide should be
discontinued and the patient should promptly seek
medical advice.
Acute gallbladder disease: If cholelithiasis or cholecystitis
is suspected, gallbladder studies are indicated.
Liraglutide was evaluated in a 26-week, double-blind, randomized,
parallel group, placebo-controlled, multicenter trial in 134 pediatric
patients with type 2 diabetes aged 10 years and older. Patients were
randomly assigned to liraglutide once daily or placebo once daily in
combination with metformin with or without basal insulin
treatment. All patients were on a metformin dosage of 1000 to 2000
mg daily before randomization. The basal insulin dose was
decreased by 20% at randomization and liraglutide was titrated
weekly by 0.6 mg for 2 to 3 weeks based on tolerability and an
average fasting plasma glucose goal of 110 mg/dL or less (≤6.1
mmol/L). At week 26, treatment with liraglutide was superior to
placebo in reducing hemoglobin A1c from baseline. The estimated
treatment difference in hemoglobin A1c reduction from baseline
between liraglutide and placebo was –1.06% with a 95% confidence
interval of –1.65% to –0.46%.

Educational Objective
Explain the risks associated with use of liraglutide to treat children
with type 2 diabetes mellitus.

Reference(s)
Tamborlane WV, Barrientos-Pérez M, Fainberg U, et al; Ellipse Trial Investigators.
Liraglutide in children and adolescents with type 2 diabetes. N Engl J Med.
2019;381(7):637-546. PMID: 31034184
44 ANSWER: D) LH, high; FSH, high; testosterone, normal or
low; inhibin B, undetectable
This boy has Klinefelter syndrome (47,XXY), which occurs in
approximately 1 in 600 newborn boys. It was first described in 1942
by Harry F. Klinefelter and is characterized by gynecomastia, small
testes, Sertoli-cell dysfunction with absent or reduced
spermatogenesis, normal to moderately reduced Leydig-cell
function, and increased FSH secretion. Klinefelter syndrome is a
frequent genetic cause of infertility in men. Additional findings can
include delayed or incomplete puberty, decreased muscle mass,
reduced body and facial hair, language delay, learning disabilities,
and behavioral issues. However, the clinical features of Klinefelter
syndrome vary widely. Like the patient in this vignette, individuals
with Klinefelter syndrome may have evidence of normal virilization
on examination because of relatively normal Leydig-cell function.
However, testicular size is small due to a decline in germ cells,
hyalinization of the tubules, and degeneration of Sertoli cells.
Inhibin is a glycoprotein hormone made primarily by the gonads.
It is a disulfide-linked dimer with a common α subunit and either a
βA subunit forming inhibin A or a βB subunit forming inhibin B.
Dimers of these β subunits form the activins (activin A βA- βA,
activin B βB- βB, activin AB βA-βB). Inhibins and activins are
members of the transforming growth factor-β (TGF-β) family. Inhibin
A and inhibin B suppress FSH secretion at the pituitary in a classic
feedback loop and may also have paracrine effects in the gonads.
Although activins share a common β subunit with inhibin, they have
the opposite effect, stimulating FSH release. Inhibin B is the
circulating form of inhibin in males. It is primarily produced by the
Sertoli cells of the testis. In normal men, inhibin B levels are
positively correlated with Sertoli-cell function and sperm number
and negatively correlated with FSH levels.
Inhibin A is mainly secreted by the corpus luteum and may serve
as a prognostic factor for predicting the return of ovarian function in
women. During pregnancy, the placenta becomes the major source
for inhibin A secretion, also produced by the corpus luteum.
Measurement of inhibin A has clinical value in women in early
pregnancy with impending abortion or hydatiform mole. Inhibin B is
secreted by antral follicles in response to FSH, and it is the major
marker of follicular growth. Therefore, inhibin B levels in women
may serve as a marker of ovarian function.
Prepubertal boys with Klinefelter syndrome generally have
normal serum levels of FSH, LH, testosterone, and inhibin B until
onset of puberty. During puberty, testosterone levels increase
initially but then plateau and remain in the low-normal range, while
LH and FSH levels rise to hypergonadotropic levels by adulthood.
Inhibin B levels simultaneously decrease. In most adults with
Klinefelter syndrome, serum inhibin B levels are undetectable.
Therefore, the most likely laboratory findings in this patient are
represented in Answer D.

Educational Objective
Explain the effects of the inhibins and activins on gonadotropin
synthesis and secretion.

Reference(s)
Luisi S, Florio P, Reis FM, Petraglia F. Inhibins in female and male reproductive physiology:
role in gametogenesis, conception, implantation and early pregnancy. Hum Reprod
Update. 2005;11(2):123-135. PMID: 15618291
O’Connor AE, De Kretser DM. Inhibins in normal male physiology. Semin Reprod Med.
2004;22(3):177-185. PMID: 15319820
45 ANSWER: E) Treat with 131I 120 mCi (4440 MBq) and
perform posttreatment scan 4 to 7 days later
This patient has a high recurrence risk, as evidenced by regional
extensive disease (N1b), tumor size, and extrathyroidal extension.
She also has an aggressive type of papillary thyroid carcinoma.
Although having the stimulated thyroglobulin level would further
assist in medical decision-making, she would most likely benefit
from radioactive iodine treatment since she has multifocal T3 tumor
with extensive nodal disease and a high-risk variant of thyroid
cancer.
While her diagnostic 123I scan did not identify remnant nodal
disease, some studies show that lymph node metastasis is sometimes
only found on the posttreatment scan. For these reasons, reassuring
the family based on 123I uptake alone (Answers A, B, and C) is not
appropriate.
A recent study provides guidelines for adults regarding use of
whole-body scanning and thyroglobulin measurement to guide the
dose of 131I to decrease recurrence risk in patients with radioactive-
avid differentiated thyroid cancer. Doses of radioactive iodine less
than 50 mCi (1850 MBq) (Answer D) are generally used in adults for
thyroid remnant ablation, when there is a low risk for regional or
distant metastatic disease. Therapeutic doses used in adults at high
recurrence risk are usually 100 to 150 mCi (3700-5180 MBq) or
higher.
In children, radioactive iodine administration is usually
determined based on empiric doses or may be based on whole-body
dosimetry. As nicely reviewed in the management guidelines for
children with thyroid nodules and differentiated thyroid cancer, for
empiric dosing, there are several approaches:

1. 131I may be adjusted according to weight or body surface


area, giving a fraction of the adult dose (eg, correcting for a
70-kg adult with similar extent of disease)
2. The dose may be adjusted based on body weight alone
using a dose between 1.0 and 1.5 mCi/kg (37-56 MBq/kg)
3. A general rule based on age may be used, assuming that a
15-year-old child needs five-sixths of the adult activity, a
10-year-old child needs one-half of the adult activity, and a
5-year-old child may need only one-third of the adult dose
for equivalent disease.

However, current guidelines recommend that all activities of 131I


should be calculated by experts with experience in dosing children.
For children with extensive metastatic disease or diffuse lung uptake
or in whom multiple doses are anticipated or who may have bone
marrow disease, dosimetry is recommended.
This patient would benefit from radioactive iodine treatment at a
therapeutic dose to minimize the need of future exposure to
radioactive iodine. An adult with similar extent of disease would
receive about 150 mCi (5180 MBq). Of the options provided, 120 mCi
(4440 MBq) (Answer E) is the best answer for this 15-year-old
patient, which corresponds to five-sixths of the adult dose.
Her stimulated thyroglobulin level was 43 ng/mL (43 µg/L) with
negative antibodies. She received 120 mCi (4440 MBq) of 131I. A
posttreatment scan showed mild residual activity in the thyroid bed
and uptake in bilateral cervical lymph nodes with no evidence of
distant metastasis. Nonstimulated thyroglobulin levels have
decreased over 6 months to 0.2 ng/mL (athyreotic level <0.1 ng/mL)
and antibodies have remained negative. Findings on neck
ultrasonography have been negative.

Educational Objective
Make recommendations based on the guidelines for medical
management following surgery for thyroid cancer.

Reference(s)
Antonelli A, Miccoli P, Fallahi P, et al. Role of neck ultrasonography in the follow-up of
children operated on for thyroid papillary cancer. Thyroid. 2003;13(5):479-484. PMID:
12855015
Francis GL, Waguespack SG, Bauer AJ, et al. Management guidelines for children with
thyroid nodules and differentiated thyroid cancer. Thyroid. 2015;25(7):716-759. PMID:
25900731
Jarzab B, Handkiewicz-Junak D, Wloch J. Juvenile differentiated thyroid carcinoma and the
role of radioiodine in its treatment: a qualitative review. Endocr Relat Cancer.
2005;12(4):773-803. PMID: 16322322
Jin Y, Ruan M, Cheng L, et al. Radioiodine uptake and thyroglobulin-guided radioiodine
remnant ablation in patients with differentiated thyroid cancer: a prospective,
randomized, open-label, controlled trial. Thyroid. 2019;29(1):101-110. PMID: 30560716
46 ANSWER: A) Redraw a fasting lipid panel in 3 months
This patient is considered to be at risk for cardiovascular disease due
to her very elevated LDL-cholesterol level. However, guidelines
recommend repeating a fasting lipid panel (Answer A) before
diagnosing dyslipidemia.
Cardiovascular disease remains the most common cause of death
in the United States. Beyond discontinuation of cigarette smoking,
statin medications to lower LDL cholesterol have had an important
role in recent improvements in cardiovascular mortality. LDL
cholesterol–related atherosclerosis often has its roots in early
childhood, with autopsy studies revealing strong correlations
between atherosclerotic plaques and LDL-cholesterol levels, even
during early childhood. Genetic conditions such as familial
hypercholesterolemia are relatively common causes of high
cholesterol levels, with a carrier rate of 1 in 500. Heterozygotes may
have LDL-cholesterol levels up to 300 to 400 mg/dL (7.77-10.36
mmol/L), although they often have lower levels. This is the main
basis for current recommendations regarding universal LDL-
cholesterol screening.
In most lipid panel assessments, direct measurements are made of
total cholesterol and HDL cholesterol, while triglycerides are
estimated from direct measurement of serum glycerol levels. LDL
cholesterol is calculated using the Friedewald equation (as long as
triglycerides are <400 mg/dL [<4.52 mmol/L]):

LDL = total cholesterol – HDL – 1/5 triglycerides

Total cholesterol and HDL cholesterol are stable in the serum


regardless of fasting status, while triglycerides can be greatly
elevated after meal ingestion.
Current guidelines that have been in place for more than 25 years
are to treat children 10 years and older with medical therapy if they
have an LDL-cholesterol level greater than 160 mg/dL (>4.14
mmol/L) plus a family history of early heart disease and to treat
children 10 years and older if they have an LDL-cholesterol level
greater than 190 mg/dL (>4.92 mmol/L) but no relevant family
history or unknown family history and if 6 months of dietary and
exercise treatment have not been successful. This patient is 7 years
old and while her lipid levels are suspicious for familial
hypercholesterolemia, she does not meet the high-risk criteria for
initiating a statin (Answer D) with a low-risk family history.
It should be noted that these guidelines are not based on long-
term trials demonstrating efficacy in avoiding future cardiovascular
disease, but instead are based on expert opinion given the strong
relationships between LDL-cholesterol levels and clinically
significant cardiovascular disease in adult cohorts.
In this vignette, initiating the CHILD-2 diet (Answer B) without
first attempting the CHILD-1 diet would be incorrect. The
Cardiovascular Health Integrated Lifestyle Diet (CHILD-1 diet) is
the initial step recommended by nutritionists to achieve a healthy
lifestyle. The recommendations are age-based and vary for very
young to older adolescents. The initial dietary recommendation is
restricting saturated fat to less than 10% of daily caloric intake and
reducing cholesterol consumption to less than 300 mg daily. It is also
recommended to reduce consumption of sugary beverages. If a
patient is unable to achieve the laboratory or BMI goals after a 3-
month trial, the CHILD-2 diet is recommended. It further restricts
saturated fat (<7% of total caloric intake) and cholesterol (<200 mg
daily), and may introduce plant sterol and stanol esters, water-
soluble psyllium fiber, or omega-3 fatty acids to reduce lipid levels.
While fibrates (Answer C) have been used as cholesterol-lowering
agents, their use is not indicated at this time without a repeated lipid
panel and an attempt at nutritional intervention.
Fish oil (Answer E) has been recommended to treat
hypertriglyceridemia; however, experts recommend its use only after
strict lifestyle intervention and for a triglyceride concentration
greater than 500 mg/dL (>5.65 mmol/L).

Educational Objective
Interpret nonfasting lipid measurements and recommend
management for children with elevated LDL-cholesterol levels.

Reference(s)
de Ferranti SD, Steinberger J, Ameduri R, et al. Cardiovascular risk reduction in high-risk
pediatric patients: a scientific statement from the American Heart Association.
Circulation. 2019;139(13):e603-e634. PMID: 30798614
Expert Panel on Integrated Guidelines for Cardiovascular Health and Risk Reduction in
Children and Adolescents; National Heart, Lung, and Blood Institute. Expert panel on
integrated guidelines for cardiovascular health and risk reduction in children and
adolescents: summary report. Pediatrics. 2011;128(Suppl 5):S213-S256. PMID: 22084329
de Ferranti SD. Familial hypercholesterolemia in children and adolescents: a clinical
perspective. J Clin Lipidol. 2015;9(5 Suppl):S11-S19. PMID: 26343208
47 ANSWER: B) C-type natriuretic peptide analogue
Achondroplasia results from heterozygous activating pathogenic
variants in the fibroblast growth factor receptor 3 gene (FGFR3) on
chromosome 4p16.3. It can be inherited in an autosomal dominant
manner or result from de novo mutation. The condition is
characterized by rhizomelic (proximal [ie, humeral and femoral])
limb shortening, macrocephaly, midface hypoplasia, and trident
hand configuration, which are usually easily recognized at birth.
Four fibroblast growth factor receptors are known in humans.
They localize on the cell surface and are involved in cellular
proliferation. Natural ligands for FGFR-3 include several fibroblast
growth factors (FGFs). The net effect of the interaction of these FGFs
with FGFR-3, which involves activation of a tyrosine kinase in the
post receptor cascade, is a decrease in chondrogenesis by limiting the
duration of the proliferative phase and accelerating the terminal
differentiation. A constitutively active gain-of-function pathogenic
variant in FGFR3, therefore, leads to decreased growth plate
chondrogenesis, as seen in achondroplasia.
Acromesomelic dysplasia Maroteaux type is due to homozygous
inactivating pathogenic variants in the NPR2 gene, which encodes
natriuretic peptide receptor B, one of the receptors for C-natriuretic
peptide (CNP). CNP, through its interaction with natriuretic peptide
receptor B, stimulates growth plate chondrogenesis and, therefore,
promotes linear growth. CNP has been postulated to enhance
growth plate chondrogenesis in achondroplasia through a pathway
that does not involve the FGFR-3 protein, which is dysfunctional in
these individuals. CNP analogues (Answer B), through activation of
the natriuretic peptide receptor B pathway, have been shown to
counteract the constitutively activated FGFR-3 signaling. Phase 3
studies are now underway for the management of short stature
associated with achondroplasia.
Although recombinant human GH (Answer D) has been used for
the management of short stature in achondroplasia and is approved
in Japan for this indication, it induces a transient increase in growth
velocity, which rapidly attenuates with prolongation of therapy. It is
estimated that the net effect of GH therapy in individuals with
achondroplasia is a height gain of 1.0 to 1.5 in (2.5-3.8 cm) after
several years of treatment.
GH action is mediated by IGF-1 in the growth plate. While in
vitro studies have shown that IGF-1 prevents apoptosis through
phosphatidylinositol 3 kinase and MAPK and was expected to
improve altered growth plate chondrogenesis in achondroplasia, the
use of recombinant human IGF-1 (Answer A) in the clinical setting is
not supported by the attenuated growth response seen during
prolonged recombinant human GH therapy.
GnRH analogues and aromatase inhibitors (Answers C and E),
although not standard of care for the management of growth
disorders, have been used to slow down growth plate maturation
and delay epiphyseal fusion in peripubertal individuals, while
enhancing growth plate chondrogenesis with recombinant human
GH in peripubertal or pubertal individuals with short stature. These
interventions would not be appropriate for the patient described in
this vignette.

Educational Objective
Describe new developments in the management of severe short
stature associated with achondroplasia to appropriately guide
parents who inquire about potential interventions.

Reference(s)
Pauli RM. Achondroplasia: a comprehensive clinical review. Orphanet J Rare Dis.
2019;14(1):1. PMID: 30606190
Harada D, Namba N, Hamioka Y, et al. Final adult height in long-term growth hormone-
treated achondroplasia patients. Eur J Pediatr. 2017;176(7):873-879. PMID: 28501952
Koike M, Yamanaka Y, Inoue M, Tanaka H, Nishimura R, Seino Y. Insulin-like growth
factor-1 rescues the mutated FGF receptor 3 (G380R) expressing ATDC5 cells from
apoptosis through phosphatidylinositol 3-kinase and MAPK. J Bone Miner Res.
2003;18(11):2043-2051. PMID: 14606518
48 ANSWER: A) Perform a pubertal examination
A pubertal examination (Answer A) is the most immediate course of
action in view of the high documented height velocity at 4 years of
age. Determining bone age (Answer B), measuring 17-
hydroxyprogesterone (Answer C), and increasing the hydrocortisone
dosage (Answer D) all have merit, but pubertal examination is an
essential first step. Reducing the fludrocortisone dosage (Answer E)
would not have any effect on her growth, so it is irrelevant in this
case. Children with congenital adrenal hyperplasia (CAH) are
predisposed to development of central precocious puberty. This is
more common in those who have poor control or are diagnosed late
with virilizing forms of CAH. In this case, the child was
appropriately prepubertal, having been diagnosed with CAH at
birth and treated since that time.

Laboratory test results (sample drawn 2 hours after the morning


dose of hydrocortisone):
Androstenedione = 137.5 ng/dL (0-20.0 ng/dL) (SI: 4.8 nmol/L [0-0.7 nmol/L])
DHEA-S = 14.8 µg/dL (0-18.5 µg/dL) (SI: <0.4 µmol/L [0-0.5 µmol/L])
Testosterone = 17.3 ng/dL (<14.4 ng/dL) (SI: 0.6 nmol/L [<0.5 nmol/L])
17-Hydroxyprogesterone = 10,656 ng/dL (SI: 322.9 nmol/L)
Plasma renin activity = 4.5 ng/mL per h (<2.5 ng/mL per h for age 1-6 year old)
Sodium = 138 mEq/L (133-146 mEq/L) (SI: 138 mmol/L [133-146 mmol/L])
Potassium = 4.0 mEq/L (3.5-5.3 mEq/L) (SI: 4.0 mmol/L [3.5-5.3 mmol/L]).

The high serum 17-hydroxyprogesterone and androstenedione


suggest poor control, either because of nonadherence or the need to
intensify her current treatment. To assess her hydrocortisone dosage
and interval, a 24-hour profile with 2 hourly serum measurements of
cortisol, pre-dose 17-hydroxyprogesterone measurement, and
androstenedione measurement was arranged.
The aim of monitoring in children with CAH is first to ensure
adequate treatment, but not overtreatment, with hydrocortisone to
minimize long-term overexposure to glucocorticoids and to allow
suppression of adrenal androgens. The recommended
hydrocortisone dosage in children with CAH is 10 to 15 mg/m2 per
day. Monitoring CAH treatment varies among different centers and
is often dependent on availability of local services. However,
published guidelines recommend that children with CAH be closely
monitored in the first 3 months of life then every 3 months until 18
months old. After age 18 months, the recommendation is to monitor
every 4 months. Monitoring incorporates regular clinical
assessments and inquiring about history of adrenal crisis or salt-
craving and symptoms of undertreatment or overtreatment. Clinical
assessment includes measurement of growth velocity, weight, and
blood pressure; assessment of pubertal status; assessment for
evidence of virilization; and biochemical measurements to determine
the adequacy of glucocorticoid and mineralocorticoid dosages.
Traditionally, 17-hydroxyprogesterone and androstenedione have
been used as indicators of the adequacy of glucocorticoid treatment,
although alternatives have been suggested such as 21-deoxycortisol
and 11-oxysteroids. When using serum 17-hydroxyprogesterone as a
guide to treatment, it is important to remember that complete
suppression to the normal range is an indication of overtreatment.

Educational Objective
Develop an approach to the monitoring of treatment in children with
congenital adrenal hyperplasia.

Reference(s)
Speiser PW, Azziz R, Baskin LS, et al; Endocrine Society. Congenital adrenal hyperplasia
due to steroid 21-hydroxylase deficiency: an Endocrine Society clinical practice
guideline [published correction appears in J Clin Endocrinol Metab. 2010;95(11):5137]. J
Clin Endocrinol Metab. 2018;103(11):4043-4088. PMID: 20823466
Turcu AF, Mallappa A, Elman MS, et al. 11-Oxygenated androgens are biomarkers of
adrenal volume and testicular adrenal rest tumors in 21-hydroxylase deficiency. J Clin
Endocrinol Metab. 2017;102(8):2701-2710. PMID: 28472487
49 ANSWER: A) Wait for the result of the T3 measurement
before adjusting the methimazole dosage
This patient’s thyroid laboratory results are quite typical of what is
observed after methimazole therapy is initiated. Repeating the
measurement of thyroid-stimulating immunoglobulin (Answer D) so
soon after diagnosis is unlikely to demonstrate a drop and will not
help guide methimazole dosing. Assessing T3 uptake (Answer B)
would help evaluate free T4, but it is not indicated because her total
T4 concentration decreased almost 50% since starting methimazole.
While her T4 level is already normal, one must verify that the T3 has
normalized before reducing the methimazole dosage (thus, Answer
A is correct and Answer C is incorrect). The methimazole dosage
would most likely be reduced to 10 mg once daily. If the patient is
already hypothyroid, the methimazole dosage could be reduced by
50%. As noted in the American Thyroid Association 2016 guidelines:
“Serum T3 should be monitored because the serum free T4 levels may
normalize despite persistent elevation of serum total T3. Serum TSH may
remain suppressed for several months after starting therapy, and it is
therefore not a good parameter for monitoring therapy early in the course.”
Because TSH stays suppressed early in the course, increasing the
methimazole dosage now (Answer E) is incorrect. Methimazole is a
thionamide that acts as an inhibitor of the TPO enzyme, and this
decreases thyroid hormone synthesis. Higher doses may increase
adverse effects and also cause her to become hypothyroid.

Educational Objective
Monitor effectiveness of therapy in a child with newly diagnosed
Graves disease.

Reference(s)
Ross DS, Burch HB, Cooper DS, et al. 2016 American Thyroid Association guidelines for
diagnosis and management of hyperthyroidism and other causes of thyrotoxicosis.
Thyroid. 2016;26(10):1343-1421. PMID: 27521067
Chen JJ, Ladenson PW. Discordant hypothyroxinemia and hypertriiodothyroninemia in
treated patients with hyperthyroid Graves’ disease. J Clin Endocrinol Metab.
1986;63(1):102-106. PMID: 2423547
50 ANSWER: A) Refer to sports medicine for evaluation of
running mechanics
In this case, recognizing and addressing the misinformation
provided to the family is very important. In pediatric patients, the
definition of osteoporosis is based on the presence of low-trauma
vertebral compression fractures (regardless of bone mineral density)
or a bone mineral density Z-score less than –2.0 and 2 pathologic
long bone fractures before the age of 10 years or 3 pathologic long
bone fractures before the age of 19 years. There are no criteria for
osteopenia. This patient’s bone density is within normal limits.
Stress fractures are also not considered to reflect pathologic bone
disease. Rather, they are secondary to excessive mechanical load on
normal bone. While total body and hip DXA scans (Answer B)
would give additional information and are reliable sites for children
in this age group, this information would not aid in the diagnosis or
treatment of this patient. In addition, DXA scans are not
recommended for stress fractures alone. This patient’s vitamin D
level is mildly low, but his alkaline phosphatase concentration is
actually normal for a young man in the midst of a pubertal growth
spurt. Therefore, he is unlikely to have osteomalacia. Large meta-
analyses have failed to show that raising vitamin D levels from the
insufficient to sufficient range (Answer C) has a significant impact
on bone density or fracture risk.
In adults, significant risk factors for fracture (previous fracture,
cigarette smoking history, alcohol use, family history, secondary
osteoporosis) in combination with bone mineral density can be used
to calculate the 10-year probability of fracture (Fracture Risk
Assessment Tool or FRAX) to guide the clinician in the initiation of
prophylactic bisphosphonate therapy. This tool has not been
validated in pediatric patients, and there are no studies showing
efficacy of oral bisphosphonates in preventing fractures in otherwise
healthy children with low bone density. Thus, starting prophylactic
oral bisphosphonate therapy (Answer D) would be incorrect in this
patient who has a normal bone density.
Although high-contact sports such as football, soccer, and lacrosse
may increase risk for traumatic fractures, stress fractures occur from
repetitive overuse. In patients with normal bone density, there is no
reason to restrict physical activity (Answer E). Weight-bearing
activity is important for bone mass accrual during childhood and
maintaining bone density throughout adulthood. Running provides
this mechanical stimulation; however, an evaluation by sports
medicine (Answer A) may be helpful to identify gait abnormalities
or areas of muscle weakness that are causing excessive mechanical
strain on the bones leading to stress fracture. In addition, sports
medicine physicians work closely with physical therapists and
athletic trainers to help a young athlete safely return to sport. Of all
the options, this is most likely to be beneficial in preventing further
injuries both now and potentially during young adulthood as a
military recruit.
On the basis of his age and examination findings, this young man
most likely started puberty at the late end of normal or possibly after
age 14 years. That being said, his current Tanner staging indicates
that he is progressing through puberty. There is a known increase in
fracture risk around the time of the pubertal growth spurt, possibly
due to the lag in mineralization and the need for recorticalization
that occurs as long bones are undergoing rapid elongation at the
growth plate. While there are not recommendations to adjust bone
mineral density Z-scores for pubertal status, it is important to
recognize that most 16-year-old boys have completed their linear
growth and thus are further into the process of peak bone mass
accrual that occurs in the 6 to 12 months after peak linear growth
velocity. Therefore, the family could be further reassured that bone
density is most likely to continue to increase and fracture risk will
continue to decrease over the next year as his linear growth velocity
declines.

Educational Objective
Summarize the physiologic changes that occur during puberty and
explain how the adolescent growth spurt correlates with peak
fracture incidence in childhood and timing of peak bone mass
accrual.

Reference(s)
Bishop N, Arundel P, Clark E, et al; International Society of Clinical Densitometry. Fracture
prediction and the definition of osteoporosis in children and adolescents: the ISCD 2013
Pediatric Official Positions. J Clin Densitom. 2014;17(2):275-280. PMID: 24631254
Rauch F. Bone growth in length and width: the Yin and Yang of bone stability. J
Musculoskelet Neuronal Interact. 2005;5(3):194-201. PMID: 16172510
Baxter-Jones AD, Faulkner RA, Forwood MR, Mirwald RL, Bailey DA. Bone mineral accrual
from 8 to 30 years of age: an estimation of peak bone mass. J Bone Miner Res.
2011;26(8):1729-1739. PMID: 21520276
51 ANSWER: E) Offer reassurance and follow-up in 6 months
Pubertal gynecomastia is common (present in approximately 50% of
boys) and is most often physiologic. Gynecomastia is generally
caused by an imbalance between estrogen and androgen effects on
glandular breast tissue. Estrogen levels rise more rapidly than
testosterone levels during early puberty, which leads to a transiently
elevated estrogen-to-androgen ratio. The patient in this scenario has
evidence of puberty onset on examination without other findings
suggestive of a pathologic cause for gynecomastia. Thus, reassurance
and observation (Answer E) would be most appropriate at this point.
Karyotype analysis (Answer A) should be ordered if there is
suspicion for Klinefelter syndrome in a pubertal or postpubertal boy
(eg, tall stature, small testes, and/or other features). Gynecomastia is
among the more common findings in individuals with Klinefelter
syndrome and is caused by elevated LH, which stimulates estradiol
secretion from Leydig cells out of proportion to testosterone
production. While the patient in this vignette does have excessively
tall stature for family background, the normal LH concentration and
stage 2 testicular development are not consistent with Klinefelter
syndrome and karyotype analysis would most likely not be helpful.
Assessing serum tumor markers (Answer B) would be
appropriate if testicular tumors were suspected. Sertoli-cell tumors
are rare but often present in boys younger than 13 years. In Sertoli-
cell tumors, overexpression of p450 aromatase (CYP19A1) causes an
increase in conversion of androstenedione to estrone, which is
sufficient to advance skeletal maturation, accelerate linear growth,
and cause gynecomastia. Ectopic β-hCG production from certain
tumors (large cell lung carcinoma, gastric carcinoma, renal cell
carcinoma, hepatoma) acts via the Leydig-cell LH receptor to
stimulate testicular production of estrogen out of proportion to
testosterone, which causes gynecomastia. The patient in this vignette
does not have a testicular mass or other concerning findings that
would warrant screening for an hCG-secreting tumor.
Boys can experience significant distress due to gynecomastia and
are often referred to endocrinology for consideration of treatment.
When identified, treatment of an underlying disorder or elimination
of a medication or environmental exposure suspected of causing
gynecomastia can lead to resolution. In the rare case of aromatase
excess, which includes aromatase excess syndrome, Carney complex,
and Peutz-Jeghers syndrome, aromatase inhibitors (Answer C) have
been shown to be beneficial. However, the only randomized, double-
blind, placebo-controlled trial of an aromatase inhibitor for pubertal
gynecomastia showed no benefit of anastrozole over placebo.
It has recently been reported that pubertal boys with
gynecomastia have a significantly higher estradiol-to-testosterone
ratio compared with that of healthy boys with either
pseudogynecomastia or no breast enlargement. While it is not
currently possible to prospectively determine which individuals
have a transient, reversible elevation in the estradiol-to-testosterone
ratio, it is known that pubertal gynecomastia that persists beyond 3
years is unlikely to regress. In this situation, surgery may be a
reasonable option, particularly if there is pain or a significant degree
of distress or embarrassment associated with the gynecomastia.
Given that up to 90% of adolescents with gynecomastia experience
spontaneous regression of breast tissue over a 1- to 3-year period, it
would be premature to recommend surgery (Answer D) in a
younger patient.

Educational Objective
Distinguish between pathologic and physiologic causes of pubertal
gynecomastia and recommend appropriate management.

Reference(s)
Ma NS, Geffner ME. Gynecomastia in prepubertal and pubertal boys. Curr Opin Pediatr.
2008;20(4):465-470. PMID: 18622206
Reinehr T, Kulle A, Barth A, Ackermann J, Lass N, Holterhus P-M. Sex hormone profile in
pubertal boys with gynecomastia and pseudogynecomastia. J Clin Endocrinol Metab.
2020;105(4):1-8. PMID: 31996898
52 ANSWER: D) Pathogenic variant in the MC4R gene
Genetic testing should be considered in pediatric patients who
exhibit early-onset, severe, rapid weight gain, particularly if other
features such as hyperphagia or clinical findings that could indicate
a genetic syndrome are evident. Monogenic obesity results from
pathogenic variants in a single gene. Leptin deficiency, leptin
receptor pathogenic variants, proopiomelanocortin (POMC)
deficiency, and melanocortin 4 receptor (MC4R gene) pathogenic
variants are all examples of monogenic obesity conditions. Most
monogenic causes are located along the leptin-melanocortin
pathway, which is critical for appetite and weight regulation.
Significant hyperphagia is a hallmark of monogenic conditions. Of
these, an MC4R pathogenic variant (Answer D) represents the most
common cause of genetic early-onset obesity and is the most likely
diagnosis for the patient in this vignette. The prevalence of MC4R
pathogenic variants varies between 0.5% and 6%, depending on the
population studied. In most cases, it is an autosomal dominant mode
of transmission. Children with an MC4R pathogenic variant have
hyperphagia that is evident early in life. Additionally, they have
linear growth acceleration and are tall for age, but they do not
demonstrate dysmorphic features and do not have intellectual
impairment.
The other monogenic obesity syndromes listed below all have
distinct phenotypic features that the patient in this vignette does not
demonstrate.
Bardet-Biedl syndrome is a rare syndromic form of obesity
characterized by retinal degeneration, early-onset obesity,
polydactyly, learning problems, kidney abnormalities, and
hypogonadism. There are at least 19 different genes associated with
Bardet-Biedl syndrome, but pathogenic variants in BBS1 (Answer A)
account for about 25% of all cases. These variants lead to defective
structure and function of cilia.
Leptin deficiency is an extremely rare cause of monogenic obesity
with fewer than 100 cases reported worldwide. It is due to
pathogenic variants in the LEP gene (Answer B) and is inherited in
an autosomal recessive fashion. Children with congenital leptin
deficiency have early-onset obesity and hyperphagia and may be
predisposed to increased risk of serious bacterial infections and
hypogonadotropic hypogonadism.
The most common cause of syndromic obesity is Prader-Willi
syndrome, with a prevalence of 1 in 15,000 to 1 in 25,000 births.
Prader-Willi syndrome is an imprinting disorder. Up to 70% of cases
are due to deletions in the paternal copy of the Prader-Willi critical
region, which is located on chromosome 15q11.2-q13 (Answer C).
Features of Prader-Willi syndrome may include severe hypotonia in
infancy, dysmorphic features, intellectual impairment, behavioral
challenges, endocrine dysfunction, and nutritional phases that range
from poor feeding in infancy to hyperphagia and food-seeking in
childhood.
Alstrom syndrome is a rare cause of syndromic obesity that, in
addition to early-onset obesity, is characterized by progressive loss
of vision and hearing, short stature, cardiomyopathy, early-onset
type 2 diabetes, and endocrine dysfunction. It is caused by
pathogenic variants in the ALMS1 gene (Answer E).
Proopiomelanocortin (POMC) deficiency is another very rare
cause of severe, early-onset obesity in childhood. It is inherited in an
autosomal recessive manner and is caused by loss-of-function
pathogenic variants in the POMC gene. In addition to obesity and
hyperphagia, affected patients tend to have fair skin and red hair.
Patients with POMC deficiency are also at risk for secondary adrenal
insufficiency, which requires glucocorticoid treatment to avoid
potential adrenal crisis.

Educational Objective
Differentiate among genetic syndromes that cause early-onset
obesity and diagnose MC4R deficiency on the basis of clinical
presentation.
Reference(s)
Thaker VV. Genetic and epigenetic causes of obesity. Adolesc Med State Art Rev.
2017;28(2):379-405. PMID: 30416642
Mason K, Page L, Balikcioglu PG. Screening for hormonal, monogenic, and syndromic
disorders in obese infants and children. Pediatr Ann. 2014;43(9):e218-e224. PMID:
25198446
Huvenne H, Dubern B, Clément K, Poitou C. Rare genetic forms of obesity: clinical
approach and current treatments in 2016. Obes Facts. 2016;9(3):158-173. PMID: 27241181
53 ANSWER: B) Elevated serum sodium
This newborn has lobar holoprosencephaly and cleft lip and palate.
Had this mother received prenatal care, these findings would most
likely have been noted on fetal ultrasonography. Holoprosencephaly
is a congenital brain malformation that results from failure of the
forebrain to bifurcate into 2 hemispheres, with alobar
holoprosencephaly being the most severe form. Due to the midline
malformation, children with holoprosencephaly commonly have
endocrinopathies from variable abnormalities of the hypothalamus
and pituitary gland. The most common endocrinopathy is diabetes
insipidus. In a study of 117 children with classic holoprosencephaly,
70% had diabetes insipidus. The severity of diabetes insipidus has
been correlated with the severity of the holoprosencephaly, and it is
believed to be due to abnormal development of the hypothalamus.
Diabetes insipidus in this setting can present with severe
hypernatremia and dehydration, but it may also evolve slowly and
can be relatively asymptomatic, with hypernatremia (Answer B)
found incidentally on routine screening. Appropriate fluid
management with or without desmopressin is an effective treatment.
In some cases, there may be abnormal hypothalamic
osmoreception with rare reports of both diabetes insipidus and
syndrome of inappropriate antidiuresis causing hyponatremia
(Answer E) in the same patient. While anterior pituitary dysfunction
can occur, it is less common. In the same series, hypothyroidism
(Answer A) was observed in 11% of patients. Hypocortisolism,
which could contribute to hypoglycemia (Answer D) occurred in 7%
and GH deficiency was observed in 5%. A low IGF-1 level (Answer
C) would be expected in the setting of GH deficiency. Growth delay
is commonly observed in these children, and it may also be
multifactorial and not necessarily a result of GH deficiency. While
anterior endocrine dysfunction is less common, a baseline evaluation
of ACTH, thyroid, and GH status should be undertaken.

Educational Objective
Describe the spectrum of anterior and posterior hormone
deficiencies associated with holoprosencephaly.

Reference(s)
Hahn JS. Holoprosencephaly. Handb Clin Neurol. 2008;87:13-37. PMID: 18809016
Hasegawa Y, Hasegawa T, Yokoyama T, Kotoh S, Tsuchiya Y. Holoprosencephaly associated
with diabetes insipidus and syndrome of inappropriate secretion of antidiuretic
hormone. J Pediatr. 1990;117(5):756-758. PMID: 2231210
54 ANSWER: B) DICER1 syndrome
Thyroid nodules are rare in children, and their evaluation should
include assessment for the possibility of an underlying genetic
syndrome that predisposes to thyroid neoplasia, particularly if the
patient is very young or has multiple thyroid nodules. When a
genetic syndrome is suspected, confirmatory genetic testing should
be performed because most such syndromes predispose to other
benign or malignant tumors for which surveillance may be required.
DICER1 syndrome (Answer B) is associated with tumors of the
lung, thyroid, female reproductive tract, kidney, brain, and other
organs. The syndrome is caused by inactivating pathogenic variants
in the DICER1 gene, which is involved in microRNA regulation of
multiple biologic pathways. Lung tumors arise in infancy or early
childhood and range from simple cysts to malignant
pleuropulmonary blastomas. Thyroid nodules—often multiple—
develop in up to 30% of affected individuals by age 20 years, and
thyroid cancer can occur as early as 8 years. Ovarian Sertoli-Leydig–
cell tumors or embryonal rhabdomyosarcoma of the cervix may arise
in adolescence or early adulthood. Macrocephaly is a common
finding.
This patient’s findings of multiple thyroid nodules, a functional
ovarian tumor, lung cysts, and relative macrocephaly are most
consistent with DICER1 syndrome. Most cases of DICER1 syndrome
are inherited in an autosomal dominant fashion. Consensus
recommendations for the care of individuals with DICER1 syndrome
include screening thyroid ultrasonography beginning at age 8 years
and every 2 to 3 years thereafter. In children previously treated with
chemotherapy for other tumors, thyroid ultrasonography is
recommended annually for 5 years, then every 2 to 3 years thereafter.
McCune-Albright syndrome (Answer D) is caused by activating
pathogenic variants in the GNAS gene, which encodes the
stimulatory α subunit associated with multiple G-protein–coupled
hormone receptors. Constitutive activation leads to the classic triad
of cutaneous café-au-lait macules, gonadotropin-independent
precocious puberty, and fibrous dysplasia of bone. Activation of the
TSH receptor can cause hyperthyroidism or multinodular goiter.
Additional features may include adrenal Cushing syndrome and GH
excess.
Carney complex (Answer A) consists of lentigines, myxomas of
the heart and other tissues, endocrine neoplasia, and other tumors.
Most cases are caused by inactivating pathogenic variants in the
PRKAR1A gene, which encodes the regulatory subunit of protein
kinase A. Skin findings, including lentigines or cutaneous myxomas,
are present in most patients. The most common endocrine
manifestation is primary pigmented nodular adrenal disease, which
causes adrenal Cushing syndrome. Affected patients are at increased
risk for thyroid nodules and thyroid cancer, which is often of the
follicular type. Testicular or ovarian tumors can occur but are
generally not hormone-producing, as was the ovarian tumor in this
patient. Lung cysts are not typical of Carney complex.
PTEN hamartoma tumor syndrome (Answer E) encompasses
several clinical syndromes caused by inactivating pathogenic
variants in the tumor suppressor gene PTEN, including Cowden,
Bannayan-Riley-Ruvalcaba, and Proteus-like syndromes. These
syndromes predispose to benign and malignant tumors of the
thyroid, skin, vascular tissue, breast, and other organs; however,
tumors of the lung or ovary are not commonly observed.
Macrocephaly is common, as is developmental delay or autism
spectrum disorder, which were not present in this patient. Thyroid
nodules can develop in early childhood and are often multiple;
thyroid cancer has been reported in children as young as 6 years.
Familial adenomatous polyposis (Answer C) is characterized by
the development of innumerable colonic polyps and eventual
colonic carcinoma. This syndrome is caused by autosomal dominant
pathogenic variants in the APC gene. Affected individuals are also at
increased risk of developing thyroid nodules and papillary thyroid
cancer, which can occur in adolescence. Tumors of the lung or ovary
are not common, and macrocephaly is not observed.
Educational Objective
Diagnose DICER1 syndrome on the basis of clinical findings.

Reference(s)
Schultz KAP, Williams GM, Kamihara J, et al. DICER1 and associated conditions:
identification of at-risk individuals and recommended surveillance strategies.
Clin Cancer Res. 2018;24(10):2251-2261. PMID: 29343557
Francis GL, Waguespack SG, Bauer AJ, et al; American Thyroid Association Guidelines Task
Force. Management guidelines for children with thyroid nodules and differentiated
thyroid cancer. Thyroid. 2015;25(7):716-759. PMID: 25900731
55 ANSWER: E) Inhibition of 11β-hydroxysteroid
dehydrogenase 1
The action of corticosteroids in peripheral tissues is regulated, in
part, by a “shuttle” mechanism of interconversion of the active
hormone (cortisol = formerly known as compound F) and the
inactive hormone (cortisone = formerly known as compound E),
catalyzed by the 11β-hydroxysteroid dehydrogenase (11β-HSD)
isoenzymes. The enzyme 11β-HSD1 activates cortisone to cortisol in
the liver and adipose tissue, while 11β-HSD2 inactivates cortisol to
cortisone in the kidney.
There is evidence that GH indirectly, through IGF-1, modulates
cortisol metabolism by decreasing 11β-HSD1 activity. This leads to a
decrease in the cortisol-to-cortisone ratio, which, in individuals with
limited cortisol production such as patients with hypopituitarism
and partial ACTH deficiency, can lead to adrenal insufficiency once
GH is administered.
The patient in this vignette was diagnosed with GH deficiency
via an arginine-L-dopa stimulation test, which does not evaluate the
integrity of the ACTH-cortisol (hypothalamic-pituitary-adrenal) axis
as do tests that include insulin or glucagon as one of the stimuli. It is
conceivable that, in addition to GH deficiency, this child has partial
ACTH deficiency that has not rendered major clinical
manifestations. Initiation of GH therapy leads to reduction of an
already limited cortisol reserve, triggering clinical manifestations of
adrenal insufficiency. Therefore, the mechanism that best explains
this patient’s presentation is preexisting, unrecognized ACTH
deficiency with exacerbation of cortisol deficiency by GH-induced
inhibition of 11β-HSD1 (Answer E) (not stimulation of 11β-HSD1
[Answer D]).
Modulation of 11β-HSD2 activity could potentially lead to a
similar scenario via an increase in the activity of this enzyme
(Answer B). However, GH does not decrease the free cortisol-to-free
cortisone ratio in the urine, suggesting that it does not affect renal
11β-HSD2 activity. A decrease in this isoenzyme activity (Answer C)
would lead to enhanced activation of cortisone to cortisol and an
adrenal crisis would not be expected.
Inhibition of 21-hydroxylase (Answer A) could lead to a drop in
cortisol levels and manifestations of an adrenal crisis in individuals
with limited endogenous cortisol production such as those with
partial ACTH deficiency. However, GH is not known to affect 21-
hydroxylase activity.

Educational Objective
Explain the mechanism by which GH decreases cortisol synthesis.

Reference(s)
Stewart PM, Toogood AA, Tomlinson JW. Growth hormone, insulin-like growth factor-I and
the cortisol-cortisone shuttle. Horm Res. 2001;56(Suppl 1):1-6. PMID: 11786677
Gelding SV, Taylor NF, Wood PJ, et al. The effect of growth hormone replacement on
cortisol-cortisone interconversion in hypopituitary adults: evidence for growth
hormone modulation of extrarenal 11-beta-hydroxysteroid dehydrogenase activity. Clin
Endocrinol (Oxf). 1998;48(2):153-162. PMID: 9579226
Walker SB, Weiss ME, Tattoni DS. Systemic reaction to human growth hormone treated with
acute desensitization. Pediatrics. 1992;90(1):108-109. PMID: 1614758
Kim SH, Park MJ. Effects of growth hormone on glucose metabolism and insulin resistance
in human. Ann Pediatr Endocrinol Metab. 2017;22(3):145-152. PMID: 29025199
56 ANSWER: C) DXA scan
Adults with type 1 diabetes mellitus have a significantly increased
fracture risk compared with the risk in the general population.
Recent studies confirm that children and adolescents with type 1
diabetes also have a higher fracture risk. Multiple studies have
demonstrated that children and adolescents with type 1 diabetes
have lower bone mineral density than individuals without diabetes
and that this correlates with future fracture risk. The bone mass
deficit in type 1 diabetes may be present at an early stage after
diagnosis. Suboptimally controlled celiac disease is an additional
independent risk factor for low bone density. Therefore, fracture risk
should contribute to the decision to order a DXA scan (Answer C) to
evaluate this patient’s bone health.
The existence of a low bone-turnover state, characterized by low
circulating levels of both bone-specific alkaline phosphatase and C-
terminal telopeptide of type I collagen (Answer A), is important to
highlight in children with type 1 diabetes, even with optimal
glycemic control. This biochemical picture has also been described in
young adults with type 1 diabetes. However, studies on bone
turnover status of children and adolescents with type 1 diabetes
have produced variable results. These markers vary widely in
growing children, and the values for bone-specific alkaline
phosphatase and C-terminal telopeptide of type I collagen require
adjustment for age and sex.
The low bone turnover state might be a reflection of functional
GH resistance that may exist in persons with diabetes, but
circulating IGF-1 was not particularly low in the studies. There may
be an inverse association between bone formation and
hyperglycemia and between bone resorption and the age at diabetes
diagnosis. The mechanisms that influence bone turnover in type 1
diabetes are possibly mediated through several pathways. Calcium
and vitamin D levels are also usually normal, if not better, in
children with type 1 diabetes compared with levels in age-matched
controls. Thus, measurement of IGF-1, calcium, and 25-
hydroxyvitamin D (Answer B) is incorrect.
Skeletal survey (Answer D) may help to evaluate previously
healed or healing fractures and fibrous dysplasia. However, this
vignette does not suggest any history of fractures or musculoskeletal
symptoms and skeletal survey is therefore not the correct choice.
Recommending no further investigation unless the patient has a
second fracture (Answer E) is not a valid strategy because it would
delay identifying and managing underlying osteopenia.

Educational Objective
Explain the determinants of bone health and fractures in children
with type 1 diabetes mellitus.

Reference(s)
Ching Chen S, Shepherd S, McMillan M, et al. Skeletal fragility and its clinical determinants
in children with type 1 diabetes. J Clin Endocrinol Metab. 2019;104(8):3585-3594. PMID:
30848792
Vestergaard P. Discrepancies in bone mineral density and fracture risk in patients with type
1 and type 2 diabetes—a meta-analysis. Osteoporos Int. 2007;18(4):427-444. PMID:
17068657
Hothersall EJ, Livingstone SJ, Looker HC, et al. Contemporary risk of hip fracture in type 1
and type 2 diabetes: a national registry study from Scotland. J Bone Miner Res.
2014;29(5):1054-1060. PMID: 24155126
Weber DR, Haynes K, Leonard MB, Willi SM, Denburg MR. Type 1 diabetes is associated
with an increased risk of fracture across the life span: a population-based cohort study
using The Health Improvement Network (THIN). Diabetes Care. 2015;38(10):1913-1920.
PMID: 26216874
Vavanikunnel J, Charlier S, Becker C, et al. Association between glycemic control and risk of
fracture in diabetic patients: a nested case-control study. J Clin Endocrinol Metab.
2019;104(5):1645-1654. PMID: 30657918
Weber DR, Schwartz G. Epidemiology of skeletal health in type 1 diabetes. Curr Osteoporos
Rep. 2016;14(6):327-336. PMID: 27744554
57 ANSWER: D) Wean treatment and measure 8-AM serum
cortisol
This child most likely has adrenal suppression due to the long
history of steroid use. Therefore, stopping treatment (Answer A) is
incorrect. Measuring cortisol while being on prednisolone (Answer
B) is not helpful, as it will not be detected. There is no clear evidence
that alternate-day dosing (Answer C) prevents adrenal suppression.
Weaning treatment and measuring serum cortisol (Answer D) is easy
and may obviate the need for a standard cosyntropin-stimulation
test (Answer E).
Glucocorticoid treatment is the mainstay of treatment for many
inflammatory diseases in childhood. Prolonged courses of high-
dosage steroids taken orally, transdermally, or inhaled could all
result in secondary adrenal suppression. Therefore, clinicians should
be cautious and screen at-risk patients for adrenal suppression.
When treating inflammatory bowel diseases, a 10-week course of
systemic glucocorticoids is frequently prescribed to induce
remission, and this treatment can suppress the hypothalamic-
pituitary-adrenal axis for up to 1 year. The rates of adrenal
insufficiency have been reported to be between 20% and 89% in
patients with inflammatory bowel disease. In adults, adrenal
suppression has been reported with hydrocortisone dosages of 15
mg/m2 per day and prednisolone dosages greater than 7.5 mg daily
taken for 3 weeks or longer. In children, treatment courses as brief as
2 weeks may result in transient suppression of endogenous cortisol
production. Four weeks of glucocorticoid therapy suppresses the
hypothalamic-pituitary-adrenal axis for up to 8 weeks after
discontinuation.
Central adrenal insufficiency is associated with low blood cortisol
and ACTH levels. According to a review by Shulman et al and the
Lawson Wilkins Drug and Therapeutics Committee, an 8-AM
cortisol concentration of 3 µg/dL (82.8 nmol/L) is suggestive of the
diagnosis; a value of 18 µg/dL (496.6 nmol/L) essentially eliminates
it. Cutoff values are often debated and may be unit and assay
dependent. Woods et al point out the utility of using a single
morning cortisol measurement as the first screening step for adrenal
suppression in patients with asthma to reduce the need for dynamic
testing. Alternative screening methods are also possible, with a
standard or low-dose cosyntropin-stimulation test being the most
popular.
The timing of testing is important, and different centers have
adopted various time points ranging from the final phase of
treatment (1 to 5 mg prednisolone daily) to months after the course
is completed. Weaning regimens are different depending on the units
used. The length of time off glucocorticoids before a blood test is
performed is also important and is dependent on the half-life of the
glucocorticoid in question.

Educational Objective
Develop an approach to identify and test for adrenal suppression
due to exogenous glucocorticoid use.

Reference(s)
Sidoroff M, Kolho KL. Screening for adrenal suppression in children with inflammatory
bowel disease discontinuing glucocorticoid therapy. BMC Gastroenterology. 2014;14:51.
PMID: 24661924
Ahmet A, Mokashi A, Goldbloom EB, et al. Adrenal suppression from glucocorticoids:
preventing an iatrogenic cause of morbidity and mortality in children. BMJ Paediatr
Open. 2019;3(1):e000569. PMID: 31750407
Shulman DI, Palmert MR, Kemp SF; Lawson Wilkins Drug and Therapeutics Committee.
Adrenal insufficiency: still a cause of morbidity and death in childhood. Pediatrics.
2007;119(2):e484-e494. PMID: 17242136
Goldbloom EB, Mokashi A, Cummings EA, et al. Symptomatic adrenal suppression among
children in Canada. Arch Dis Child. 2017;102(4):340-345. PMID: 28320817
Woods CP, Argese N, Chapman M, et al. Adrenal suppression in patients taking inhaled
glucocorticoids is highly prevalent and management can be guided by morning cortisol.
Eur J Endocrinol. 2015;173(5):633-642. PMID: 26294794
Wood P, Henderson P. Letter: screening for adrenal suppression in paediatric inflammatory
bowel disease. Aliment Pharmacol Ther. 2018;48(8):884-885. PMID: 30281831
58 ANSWER: C) Fetal karyotyping by amniocentesis
Prenatal screening that is universally available to screen for multiple
genetic or chromosomal disorders can also detect fetuses with
Turner syndrome. Such screens include prenatal ultrasonography,
which may detect increased nuchal translucency, cystic hygroma,
congenital heart defects such as coarctation of the aorta and/or left-
sided cardiac defects, brachycephaly, renal anomalies,
polyhydramnios, oligohydramnios, and growth retardation.
Increased nuchal translucency seen in first-trimester
ultrasonography and left-sided heart defects are common in fetuses
with Turner syndrome or autosomal trisomy syndromes. However,
prenatal ultrasonography and triple or quadruple marker first-
trimester screening can be normal in such cases, so normal results
from these assessments cannot be used to reassure patients (Answer
A) when results from cell-free DNA screening are abnormal.
Noninvasive prenatal screening using cell-free DNA extracted
from maternal blood is increasingly used to screen for many genetic
disorders. It performs well in detecting trisomy 21, 18, and 13 and
can also detect sex chromosome aneuploidy. However, currently
available cell-free DNA screening has a very low positive predictive
value (23%) for Turner syndrome and thus positive results should be
confirmed with diagnostic testing prenatally or postnatally. Fetal
karyotyping by chorionic villus sampling (10-13 weeks’ gestation)
(Answer D) or amniocentesis (15-18 weeks’ gestation) (Answer C)
should be offered to confirm the diagnosis before making
irreversible decisions (Answer E) relative to the pregnancy outcome.
Repeating the cell-free DNA screen (Answer B) will not determine
the diagnosis or help with guiding the next management step. Given
the gestational age in this case, amniocentesis is the most
appropriate diagnostic test to recommend. Amniocentesis is
associated with a low risk of complications such as leakage of
amniotic fluid, infection, needle injury to the fetus, and miscarriage.
It is important to note that the patient may choose not to undergo
any further testing, but rather to continue the pregnancy to term and
consider further testing postnatally.
Turner syndrome is a common chromosomal disorder with
varying genotypes and phenotypes. It is characterized by complete
or partial absence of the second sex chromosome with or without
mosaicism in phenotypic females. Common genotypes leading to
Turner syndrome include monosomy 45,X (45%-50% of all patients
with Turner syndrome); varying degrees of 45,X/46,XX mosaicism
(15%-25%); 45,X/46,XY mixed gonadal dysgenesis with a female
phenotype (10%-12%); 46,Xi(Xq) or 46,X,idic(Xp) isochromosome Xq
or isodicentric Xp (10%); and 45,X/47,XXX or 45,X/46,XX/47,XXX
mosaicism with triple X (3%). Patients with small microdeletions of
Xp22.3, Xq24 deletion or isodicentric Xq24, or 45,X/46,XY gonadal
dysgenesis with a male phenotype are not diagnosed with Turner
syndrome.

Educational Objective
Explain the advantages and limitations of various prenatal screening
options for Turner syndrome.

Reference(s)
Gravholt CH, Andersen NH, Conway GS, et al; International Turner Syndrome Consensus
Group. Clinical practice guidelines for the care of girls and women with Turner
syndrome: proceedings from the 2016 Cincinnati International Turner Syndrome
Meeting. Eur J Endocrinol. 2017;177(3):G1-G70. PMID: 28705803
Wang Y, Li S, Wang W, et al. Cell-free DNA screening for sex chromosome aneuploidies by
non-invasive prenatal testing in maternal plasma. Mol Cytogenet. 2020;13:10. PMID:
32190123
Meck JM, Dugan EK, Matyakhina L, et al. Noninvasive prenatal screening for aneuploidy:
positive predictive values based on cytogenetic findings. Am J Obstet Gynecol.
2015;213(2):214.e1-e5. PMID: 25843063
59 ANSWER: C) Free T4, normal; total T3, low; reverse T3, high;
high-output heart failure
The infant in this vignette has the classic presentation of
consumptive hypothyroidism due to excess iodothyronine
deiodinase type 3 enzyme activity, associated with large liver
hemangiomas, which express this enzyme. This condition typically
manifests in early infancy in babies who harbor a large tumor with
high enzyme activity, either with a few large hemangiomas or
multiple smaller hemangiomas that overcome the thyroid gland’s
ability to compensate. TSH levels can range from mildly elevated to
concentrations greater than 500 mIU/L. The mechanism was first
elucidated in 2000, only after the index patient had died and his
hemangioma tissue was analyzed. Type 3 deiodinase is normally
present in the brain and placenta, and it catalyzes the conversion of
T4 to reverse T3 and the conversion of T3 to 3,3′-diiodothyronine,
both of which are biologically inactive.
Heart failure in such cases is high-output failure due to decreased
peripheral vascular resistance that occurs because of large
arteriovenous shunts in the hemangiomas. Affected patients may
also have consumptive coagulopathy depending on the size of the
hemangiomas. As noted in the initial report, cardiac impairment
from hypothyroidism can exacerbate the congestive heart failure
associated with high-flow hemangiomas. Therefore, due to the
deiodinase activity, such patients have elevated reverse T3 and low
or low-normal T4 and T3, depending on the severity of the
hypothyroidism. Thus, the most representative option is Answer C.
Answers B and E list low reverse T3. Answer D lists high total T3
and low-output heart failure. Answer A would be correct as far as
the thyroid hormone profile is concerned; however, it lists low-
output heart failure.
Due to the inactivation of the thyroid hormones T4 and T3, such
patients require higher levothyroxine dosages than would normally
be required to treat congenital or acquired hypothyroidism
(sometimes given twice daily, and sometimes adding liothyronine)
and thus the name consumptive hypothyroidism.

Educational Objective
Anticipate that consumptive hypothyroidism can cause high-output
heart failure.

Reference(s)
Huang SA, Tu HM, Harney JW, et al. Severe hypothyroidism caused by type 3
iodothyronine deiodinase in infantile hemangiomas. N Engl J Med. 2000;343(3):185-189.
PMID: 10900278
Kim EH, Koh KN, Park M, Kim BE, Im HJ, Seo JJ. Clinical features of infantile hepatic
hemangioendothelioma. Korean J Pediatr. 2011;54(6):260-266. PMID: 21949521
Emir S, Ekici F, İkiz MA, Vidinlisan S. The association of consumptive hypothyroidism
secondary to hepatic hemangioma and severe heart failure in infancy. Turk Pediatri Ars.
2016;51(1):52-56. PMID: 27103866
60 ANSWER: A) Increased leptin, increased insulin, suppressed
ghrelin
Hypothalamic obesity is a frequent complication following
craniopharyngioma treatment, occurring in up to 50% of patients. In
addition to craniopharyngiomas, central nervous system trauma,
cranial irradiation, certain genetic conditions, and suprasellar
tumors are other etiologies that can lead to hypothalamic damage
and dysfunction. The hypothalamus has a critical role in energy
homeostasis. Children who develop hypothalamic obesity are
usually hyperphagic with impaired satiety. Even if attempts are
made at caloric restriction, they demonstrate significant weight gain
in the first year after treatment, followed by maintenance of a higher
body weight, which is most likely due to alterations in feeding
regulation and low energy expenditure. There is altered neural
regulation of pancreatic β cells, which leads to increased insulin
secretion. This chronic hyperinsulinemia causes increased
production of leptin and suppression of ghrelin (Answer A). Leptin
levels positively correlate with amount of body fat; thus, individuals
with excess adiposity, regardless of cause, have higher leptin levels
than do lean individuals. However, when corrected for BMI, leptin
levels remain higher in patients with hypothalamic obesity than in
those with simple obesity, indicating more severe leptin resistance in
these patients.
Congenital leptin deficiency causes very severe early-life obesity
characterized by hyperphagia. These patients also have
hypogonadotropic hypogonadism and exhibit insulin resistance, not
insulin sensitivity. Hyperinsulinemia is the presence of elevated
circulating levels of insulin in the blood. Insulin resistance refers to
the diminished capacity of cells to respond to circulating insulin.
Hyperinsulinemia is commonly seen in the setting of insulin
resistance since the pancreas has to produce more insulin in order to
maintain glucose homeostasis; however, this is not always the case.
Hypersecretion of insulin has been demonstrated in hypothalamic
obesity, but many affected patients have normal fasting insulin levels
and have been shown to be insulin sensitive, rather than insulin
resistant. There is a subset of patients who, in addition to their
hypothalamic obesity, also have features of metabolic syndrome,
including insulin resistance. Patients with Prader-Willi syndrome
have hyperleptinemia similar to what is observed in patients with
hypothalamic obesity. Patients with Prader-Willi syndrome show
heightened insulin sensitivity compared with what is observed in
other obesity conditions. Unlike hypothalamic obesity, however,
patients with Prader-Willi have significantly increased ghrelin levels,
both fasting and postprandial.

Educational Objective
Identify the alterations in leptin and insulin that occur in patients
with hypothalamic obesity.

Reference(s)
Abuzzahab MJ, Roth CL, Shoemaker AH. Hypothalamic obesity: prologue and promise.
Horm Res Paediatr. 2019;91(2):128-136. PMID: 30884480
Kim JH, Choi JH. Pathophysiology and clinical characteristics of hypothalamic obesity in
children and adolescents. Ann Pediatr Endocrinol Metab. 2013;18(4):161-167. PMID:
24904871
61 ANSWER: B) Inactivating variant in CASR
This patient most likely has familial hypocalciuric hypercalcemia
secondary to a heterozygous inactivating pathogenic variant in
CASR (Answer B). This autosomal dominant condition most often
presents with asymptomatic hypercalcemia and an inappropriately
elevated or normal PTH, but low urinary calcium excretion.
However, it can also present with symptomatic hypercalcemia in the
neonatal period, most often when inherited from the father, and is
termed neonatal hyperparathyroidism. It is hypothesized that in
utero, the fetus’s calcium-sensing receptors at the parathyroid glands
sense relative hypocalcemia (maternal calcium is actively
transported across the placenta) and increase PTH secretion. Medical
intervention with hyperhydration, bisphosphonates, and/or
cinacalcet is the preferred treatment during infancy, with patients
becoming asymptomatic as they grow older despite a higher serum
calcium concentration (altered set point). Despite hypercalcemia,
affected individuals do not develop nephrocalcinosis because the
calcium-sensing receptor present in the thick ascending loop of
Henle functions independently of PTH to resorb calcium and
maintain the higher serum concentrations. As magnesium is also
reabsorbed at the renal tubule through the action of the calcium-
sensing receptor, hypermagnesemia is another common finding.
Infants with homozygous or compound heterozygous pathogenic
variants in CASR can present with severe neonatal
hyperparathyroidism with calcium levels as high as 30 mg/dL (7.5
mmol/L). Severe bone disease manifesting as rickets and multiple
fractures can also be present. Generally, these patients require total
parathyroidectomy, but often surgical intervention can be delayed
with the above medical management. The management of
postsurgical hypoparathyroidism is slightly different for these
patients, as serum calcium levels can be maintained in the normal
range given the low concern for nephrocalcinosis.
Activating pathogenic variants in PTH1R (Answer A) result in
hypercalcemia with low PTH and hypercalciuria and also have
skeletal manifestations of short-limbed dwarfism (metaphyseal
chondrodysplasia, Jansen type). Ninety percent of patients with
inactivating pathogenic variants in MEN1 (Answer C) develop
hyperparathyroidism, but this is secondary to parathyroid adenomas
and generally does not present until the second or third decade of
life. Inactivating variants in CYP24A1 (Answer D) and deletion of
the ELN gene on chromosome 7 (Answer E) result in hypercalcemia
with suppressed PTH. Deletion of the long arm of chromosome 7,
including the region where the elastin gene (ELN) is located, is the
cause of Williams syndrome. Additionally, patients with Williams
syndrome often have distinctive facial features (broad forehead,
short nose with broad tip, wide mouth with full lips) and
supravalvular aortic stenosis. Inactivation of 24-hydroxylase
(CYP24A1) results in elevated levels of 25-hydroxyvitamin D and
1,25-dihydroxyvitamin D. Affected patients can develop
nephrocalcinosis.

Educational Objective
Identify inactivating pathogenic variants in the CASR gene as a
cause of severe hyperparathyroidism in neonates and describe how
this can resolve with medical treatment.

Reference(s)
Marx SJ, Sinaii N. Neonatal severe hyperparathyroidism: novel insights from calcium, PTH,
and the CASR gene. J Clin Endocrinol Metab. 2020;105(4):1061-1078. PMID: 31778168
Cole DE, Janicic N, Salisbury SR, Hendy GN. Neonatal severe hyperparathyroidism,
secondary hyperparathyroidism, and familial hypocalciuric hypercalcemia: multiple
different phenotypes associated with an inactivating Alu insertion mutation of the
calcium-sensing receptor gene. Am J Med Genet. 1997;71(2):202-210. PMID: 9217223
62 ANSWER: C) Start oral methimazole
This adolescent with Down syndrome (trisomy 21) has
hyperthyroidism due to Graves disease based on his elevated
thyroid-stimulating immunoglobulin. Children with Down
syndrome are at increased risk for autoimmune conditions,
primarily thyroiditis. Hashimoto thyroiditis is more common than
Graves disease in this population. In children without Down
syndrome, Graves disease is more common in females, but this is not
the case in patients with Down syndrome. Pediatric patients with
Down syndrome tend to develop Graves disease at a younger age
than do children in the general pediatric population.
While thyroidectomy (Answer A) and radioactive iodine (Answer
E) are definitive therapies for Graves disease and are valid choices,
they are not the best next step in this setting. The American Thyroid
Association guidelines indicate that children with Graves disease
may be treated with methimazole, radioactive iodine, or
thyroidectomy. In choosing the initial treatment, age, clinical status,
and likelihood of remission should be considered. Because some
patients will go into remission, methimazole (Answer C) is still
considered first-line treatment for most children. Since this child has
no goiter and has mild hyperthyroidism, he would most likely
benefit from methimazole, as recommended for most children with
Graves disease.
This child has no tachycardia, no complaints of palpitations, and
no hypertension, so propranolol (Answer B) is not indicated. When a
β-adrenergic blocker is needed, it should be used in addition to
methimazole. This patient is symptomatic and losing weight, and
Graves disease is unlikely to resolve spontaneously. Therefore,
observing him and repeating thyroid function tests (Answer D) is
incorrect.
Although children and adolescents with Down syndrome are at
higher risk of developing acute leukemia (in adolescents it would be
lymphoblastic leukemia) than children in the general population,
there is no increased risk of leukemia described in patients with
Down syndrome on methimazole. Higher doses of radioactive
iodine, as used to treat well-differentiated thyroid cancer (>100 mCi
in adults), have been associated with an increased risk of leukemia,
but no such risk has been observed with the 131I doses used for
Graves disease. Radioactive iodine use has been reported in children
with Down syndrome and Graves disease (in those older than 5
years just like in the general population, due to possible/theoretical
risk of cancer in children younger than 5 years). As with surgery,
131I is not considered first-line therapy, but it is a valid option.
Older children and adolescents with Down syndrome may have
lymphopenia (both B and T cells), but they are not at higher risk for
agranulocytosis due to methimazole than are patients with Graves
disease who do not have Down syndrome. Therefore, as in other
children and adolescents with Graves disease, methimazole is
usually first-line therapy. One could prescribe a β-adrenergic blocker
such as propranolol, or atenolol (in case of asthma) if the patient
develops palpitations before methimazole is effectively controlling
the hyperthyroidism, which usually takes 1 to 2 months. One
publication (De Luca et al) suggests that children with Down
syndrome are more likely to go into remission than children without
Down syndrome, but another smaller series (Goday-Arno et al) does
not support that finding. Because this patient has mild
hyperthyroidism, a modest methimazole dosage may be started in
the range of 5 to 10 mg twice daily (or given as 10-20 mg once daily,
as used in adults), which would reduce the likelihood of adverse
effects. Once his hyperthyroidism is controlled, the methimazole
dosage can be decreased and administered once daily to achieve
better adherence.

Educational Objective
Identify important considerations in the management of children
with Graves disease and trisomy 21.

Reference(s)
Ross DS, Burch HB, Cooper DS, et al. 2016 American Thyroid Association guidelines for
diagnosis and management of hyperthyroidism and other causes of thyrotoxicosis.
Thyroid. 2016;26(10):1343-1421. PMID: 27521067
De Luca F, Corrias A, Salerno M, et al. Peculiarities of Graves’ disease in children and
adolescents with Down’s syndrome. Eur J Endocrinol. 2010;162(3):591-595. PMID:
19955260
Goday-Arno A, Cerda-Esteva M, Flores-Le-Roux JA, Chillaron-Jordan JJ, Corretger JM,
Cano-Pérez JF. Hyperthyroidism in a population with Down syndrome (DS). Clin
Endocrinol (Oxf). 2009;71(1):110-114. PMID: 18793345
Roberts I, Izraeli S. Haematopoietic development and leukaemia in Down syndrome. Br J
Haematol. 2014;167(5):587-599. PMID: 25155832
63 ANSWER: C) Baseline 17-hydroxyprogesterone measurement
The most likely diagnosis in this child is premature adrenarche,
which is diagnosed by clinical evaluation and exclusion of
alternative adrenal pathologies. Premature adrenarche is due to the
premature activation of the zona reticularis of the adrenal gland,
which results in increased adrenal androgens, most notably DHEA
and DHEA-S. One of the main differential diagnoses is nonclassic
congenital adrenal hyperplasia. A baseline 17-hydroxyprogesterone
value less than 200 ng/dL (<6.1 nmol/L) is reported to have 100%
sensitivity to exclude nonclassic congenital adrenal hyperplasia.
Cutoff values may vary depending on differences in hormonal
assays used at different centers. Measuring 17-hydroxyprogesterone
(Answer C) is the most appropriate next step.
In some circumstances (mild symptoms without progression),
physicians may choose to follow the child clinically with or without
measurement of bone age. In this vignette, there was clinical
progression, so further investigations were warranted. Bone age
measurement can be helpful, and some clinicians recommend
further evaluation if bone age is more than 2 years advanced for a
patient’s given chronologic age. However, advanced bone age is
common in premature adrenarche.
With more frequent use of liquid chromatography–tandem mass
spectrometry, increases in alternative steroids such as circulating 11-
oxygenated androgens have been described in patients with
premature adrenarche. 11-Ketotestosterone may be the dominant
circulating bioactive androgen that results in the clinical features
seen in premature adrenarche.
Urinary steroid profiles (Answer E) are increasingly being used
for the diagnosis of disorders of steroidogenesis, and with growing
availability, this assessment will no doubt have clinical utility in the
exclusion of other disorders of hyperandrogenism.
Adrenal ultrasonography (Answer A) could be used to assist in
the diagnosis of an adrenal tumor, although the study is user
dependent. In this child with mild androgenization, the diagnosis is
unlikely.
The lack of breast development on clinical examination rules out
central precocious puberty, so LH measurement (Answer B) is not
necessary at this point.
Measuring TSH (Answer D) could diagnose severe primary
hypothyroidism, which has been associated with pseudoprecocious
puberty (frequently with delayed bone age) presenting with breast
development, galactorrhea, and menarche. This is also referred to as
Van Wyk-Grumbach syndrome. The mechanism leading to
pseudoprecocious puberty in this setting is not entirely clear,
although it is thought to be due to extremely high TSH acting on the
FSH receptor, which stimulates the gonads. Clinically, this child does
not have breast development; rather, she has advanced bone age and
increased height velocity. Thus, Van Wyk-Grumbach syndrome is an
unlikely diagnosis and TSH measurement is not the best next step.
Although premature adrenarche is considered a benign variant of
maturation that is more common in girls than in boys and is
associated with higher BMI, increased susceptibility to polycystic
ovary syndrome and insulin resistance have been described in
adults. Furthermore, rare disorders can present as premature
adrenarche such as cortisone reductase deficiency due to inactivating
pathogenic variants in the gene encoding the enzyme hexose-6-
phosphate dehydrogenase and cortisone reductase deficiency due to
inactivating pathogenic variants in the HSD11B1 gene. Finally, a
recent study highlighted that premature adrenarche is more common
among black girls than among white girls and other racial groups. In
black patients with premature adrenarche, the age at presentation is
younger and there is a lower incidence of organic pathology. Hence,
the age definition of premature adrenarche (<8 years in girls and <9
years in boys) may not be appropriate for patients of African
American descent, and management may need to be stratified to
account for ethnic background.

Educational Objective
Diagnose premature adrenarche and explain why classic definitions
may not necessarily be appropriate for all ethnic groups.

Reference(s)
Grandone A, Marzuillo P, Luongo C, et al. Basal levels of 17-hydroxyprogesterone can
distinguish children with isolated precocious pubarche. Pediatr Res. 2018;84(4):533-536.
PMID: 29976972
Rege J, Turcu AF, Kasa-Vubu JZ, et al. 11-Ketotestosterone is the dominant circulating
bioactive androgen during normal and premature adrenarche. J Clin Endocrinol Metab.
2018;103(12):4589-4598. PMID: 30137510
Lavery GG, Idkowiak J, Sherlock M, et al. Novel H6PDH mutations in two girls with
premature adrenarche: ‘apparent’ and ‘true’ CRD can be differentiated by urinary
steroid profiling. Eur J Endocrinol. 2013;168(2):K19-K26. PMID: 23132696
DeSalvo DJ, Mehra R, Vaidyanathan P, Kaplowitz PB. In children with premature
adrenarche, bone age advancement by 2 or more years is common and generally
benign. J Pediatr Endocrinol Metab. 2013;26(3-4):215-221. PMID: 23744298
Şiklar Z, Öçal G, Adiyaman P, Ergur A, Berberoğlu M. Functional ovarian
hyperandrogenism and polycystic ovary syndrome in prepubertal girls with obesity
and/or premature pubarche. J Pediatr Endocrinol Metab. 2007;20(4):475-481. PMID:
17550211
Foster C, Diaz-Thomas A, Lahoti A. Low prevalence of organic pathology in a
predominantly black population with premature adrenarche: need to stratify definitions
and screening protocols. Int J Pediatr Endocrinol. 2020;2020:5. PMID: 32165891
64 ANSWER: E) Initiation of cabergoline and counseling for
psychiatric disturbance
This patient has hyperprolactinemia and an area of
hypoenhancement of the pituitary measuring less than 10 mm,
which is most consistent with a microprolactinoma. Prolactin-
secreting tumors account for approximately 40% of all pituitary
adenomas. Prolactinomas are rare in childhood, but the incidence
increases in adolescence. Reproductive disturbances (amenorrhea
and galactorrhea in girls and pubertal delay and hypogonadism in
boys) are the most common presentation.
Dopamine agonist therapy is recommended to lower prolactin
levels, decrease tumor size, and restore gonadal function in patients
with a microprolactinoma and symptoms of hyperprolactinemia,
including hypogonadism (infertility, oligomenorrhea, galactorrhea,
low bone density). Cabergoline is a long-acting dopamine agonist
that is generally recommended over bromocriptine because of better
tolerance and greater efficacy in all endpoints. It should be noted
that safety and efficacy have not been established in the pediatric
population.
Onset or exacerbation of impulse control disorders have been
recognized as an adverse effect of dopamine agonist therapy,
occurring in 17% of patients treated with cabergoline for
prolactinoma in one uncontrolled study of 308 adult patients. A case
study documented psychiatric disturbance in several children with
prolactinoma treated with cabergoline at a median dosage of 0.5 mg
weekly (range 0.25-2.0 mg weekly). In several patients, this
necessitated a reduced cabergoline dosage. Given the potential risk
of psychiatric disturbance with cabergoline treatment, patients
should be counseled regarding this risk when starting treatment
(Answer E).
Diagnosis of hyperprolactinemia in an individual with a
suspected prolactinoma is straightforward. The Endocrine Society
guidelines recommend a single measurement of prolactin obtained
without excessive venipuncture stress, as a single prolactin
measurement is highly sensitive for hyperprolactinemia. Serial
dilution of serum samples is recommended if there is a discrepancy
between a large pituitary tumor and mildly elevated prolactin. This
eliminates artifact with immunoradiometric assays that can lead to a
falsely low prolactin value (“hook effect”). However, dynamic
testing of prolactin secretion (Answer A) has not been shown to be
superior to measurement of a single serum prolactin measurement.
It is reasonable to defer treatment and monitor microadenomas
that are not causing symptoms, as most microadenomas do not
increase in size over time. However, the patient in this case has
oligomenorrhea and low estradiol, so repeating prolactin
measurement and brain MRI without initiating treatment (Answer
C) is incorrect.
Visual field deficits are a concern in patients presenting with
macroadenoma due to compression of the optic chiasm. However,
the presence of visual field defects in individuals with
microadenomas would not be expected, as the tumor does not
extend beyond the sella. As such, a neuro-ophthalmology
examination (Answer B) is not required.
Dopamine agonists are well known to cause fibrotic valvulopathy
in a dose-dependent manner and most microadenomas respond to
cabergoline dosages well under 2 mg weekly. As there is low risk of
cabergoline-associated valvulopathy at dosages used to treat
macroprolactinomas, only patients taking a cabergoline dosage
greater than 2 mg weekly require annual echocardiography (Answer
D). Echocardiography before commencing dopamine agonist
therapy followed by echocardiography at 5 years is recommended
for patients taking a cabergoline dosage of 2 mg weekly or lower.

Educational Objective
Identify potential precautions and adverse effects associated with
dopamine agonist therapy.

Reference(s)
Melmed S, Casanueva FF, Hoffman AR, et al; Endocrine Society. Diagnosis and treatment of
hyperprolactinemia: an Endocrine Society clinical practice guideline. J Clin Endocrinol
Metab. 2011;96(2):273-288. PMID: 21296991
Brichta CM, Wurm M, Krebs A, Schwab KO, van der Werf-Grohmann N. Start low, go
slowly - mental abnormalities in young prolactinoma patients under cabergoline
therapy. J Pediatr Endocrinol Metab. 2019;32(9):969-977. PMID: 31323004
Steeds R, Stiles C, Sharma V, Chambers J, Lloyd G, Drake W. Echocardiography and
monitoring patients receiving dopamine agonist therapy for hyperprolactinaemia: a
joint position statement of the British Society of Echocardiography, the British Heart
Valve Society and the Society for Endocrinology. Clin Endocrinol. 2019;90(5):662-669.
PMID: 30818417
65 ANSWER: A) Brain MRI
This patient has diencephalic syndrome, which is a rare cause of
profound emaciation in infancy characterized by significant absence
of subcutaneous tissue despite normal caloric intake. Affected
patients do not have vomiting, hepatomegaly, or abnormal liver
function tests as observed in patients with loss of subcutaneous
tissue due to generalized lipodystrophy. This condition was first
described by Russell in 1951. Linear growth is usually maintained or
only minimally affected in these children. Other features that can be
observed in some children with diencephalic syndrome include
hyperkinesis, emesis, and ophthalmologic findings such as
nystagmus, optic pallor, or papilledema. Affected children usually
attain developmental milestones appropriate for age. Some have
hydrocephalus. A high index of suspicion should prompt brain MRI
(Answer A), which would demonstrate tumor near the anterior
hypothalamus and occasionally a brain tumor in the posterior fossa.
Treatment considerations are focused on reducing the tumor burden.
Differential diagnosis of failure to thrive during infancy is broad.
However, this child has normal linear growth, no history of
hypoglycemia, normal electrolytes and glucose, normal thyroid
function, and a normal GH concentration. Thus, adrenal
insufficiency or other endocrinopathies are less likely to be the cause
of poor weight gain. Cortisol measurement and cosyntropin-
stimulation testing (Answer B) are not necessary. The baby is happy
and interactive with normal development and no history of
fractures. Therefore, there is no need to obtain a skeletal survey
(Answer C) or bone age x-ray (Answer D). Referral to
gastroenterology (Answer E) will only delay the diagnosis and
treatment and hence is not the best next step in this patient’s care.

Educational Objective
Identify the characteristic clinical features of diencephalic syndrome
as a cause of failure to thrive in young children.
Reference(s)
Fleischman A, Brue C, Poussaint TY, et al. Diencephalic syndrome: a cause of failure to
thrive and a model of partial growth hormone resistance. Pediatrics. 2005;115(6):e742-
e748. PMID: 15930202
Brauner R, Trivin C, Zerah M, et al. Diencephalic syndrome due to hypothalamic tumor: a
model of the relationship between weight and puberty onset. J Clin Endocrinol Metab.
2006;91(7):2467-2473. PMID: 16621905
66 ANSWER: A) Elevated serum reverse T3
This infant presents at 6 weeks of age with primary hypothyroidism
that was not present on newborn screening, suggesting a postnatally
acquired cause. His findings of marked hepatomegaly and a
cutaneous hemangioma suggest that the most likely cause is the
presence of massive hepatic hemangiomas. These vascular lesions
highly express type 3 deiodinase (D3), an enzyme that converts both
T4 and T3 to the inactive metabolites reverse T3 and T2, respectively.
Massive hemangiomas, which occur most often in the liver, can
express sufficient amounts of D3 to produce consumptive
hypothyroidism, which is characterized by elevated levels of reverse
T3 (Answer A), low levels of T3 (not elevated [Answer C]), and,
when severe, low levels of T4. Serum thyroglobulin levels are
increased (not low [Answer B]) due to stimulation by elevated TSH.
Because of the rapid degradation of circulating thyroid hormones,
management consists of levothyroxine at high dosages, sometimes
given twice daily due to its shortened serum half-life. Replacement
of T3 also may be required in severe cases. Hemangiomas can cause
severe complications including cardiac failure, but they usually
regress over time, leading to resolution of consumptive
hypothyroidism.
Exposure to excess iodine—such as from topical iodine
antiseptics, iodinated radiocontrast agents, or maternal iodine
ingestion—can cause acquired hypothyroidism in infancy. In such
cases, the urinary iodine concentration is elevated (Answer E).
However, hepatomegaly is not characteristic of iodine-induced
hypothyroidism, and there is no history of excess iodine exposure in
this case.
Maternal transfer of TSH-receptor antibodies that block TSH-
receptor signaling (Answer D) is a rare cause of congenital
hypothyroidism, which is unlikely in this patient with a normal
newborn screen. Congenital hypothyroidism may be missed on
newborn screening due to problems with the screening sample
(including obtaining the sample after blood transfusion) or in infants
who are preterm, of low birth weight, or are monozygotic twins;
however, none of these risk factors is present in this case. Although a
sample-handling error cannot be excluded based on the information
provided, the physical findings in this patient are not typical of
congenital hypothyroidism.

Educational Objective
Diagnose consumptive hypothyroidism based on clinical findings
and identify the laboratory abnormalities associated with this
diagnosis.

Reference(s)
Huang SA, Tu HM, Harney JW, et al. Severe hypothyroidism caused by type 3
iodothyronine deiodinase in infantile hemangiomas. N Engl J Med. 2000;343(3):185-189.
PMID: 10900278
67 ANSWER: D) Syndrome of inappropriate antidiuresis
This hospitalized child who is undergoing treatment for acute
lymphoblastic leukemia presented with hyponatremia and
bacteremia. His blood pressure and heart rate were normal and there
was no evidence of edema on examination, which together suggest a
euvolemic state. Hyponatremia occurs commonly in hospitalized
patients and has many contributing factors. The kidney’s ability to
generate and excrete free water is the primary way the body defends
against developing hyponatremia. With normal renal function, it is
rare to develop hyponatremia from excess water ingestion, due to
the ability of the kidneys to excrete large volumes of free water.
Similarly, it is rare to develop hyponatremia from large renal losses
alone without an excess of free water ingestion. It is arginine
vasopressin (AVP), released from the posterior pituitary, that
increases water permeability in the collecting tubules of the kidney.
In addition to increases in serum osmolality, AVP release can be
stimulated by both hemodynamic stimuli (eg, volume depletion,
hypotension, congestive heart failure, nephrotic syndrome, adrenal
insufficiency, liver cirrhosis) and nonhemodynamic stimuli (eg, pain,
stress, nausea, vomiting, medications, central nervous system
disease, pulmonary disease, perioperative state, inflammation).
Syndrome of inappropriate antidiuresis (SIAD) is a common
cause of hyponatremia in hospitalized children. Patients with SIAD
have hypotonic hyponatremia caused by impaired free water
excretion due to excess AVP secretion from nonphysiologic stimuli,
in the absence of endocrine or renal dysfunction. There are
numerous causes of SIAD, but in children it most often results from
central nervous system disorders, pulmonary disorders, malignancy,
and medications (most commonly the anticonvulsant drugs
carbamazepine and oxcarbazepine and the chemotherapeutic agents
vincristine and cyclophosphamide). Patients with SIAD are typically
euvolemic or have mild volume expansion. However, volume status
is often difficult to assess accurately; therefore, SIAD is generally a
diagnosis of exclusion. While there is no specific diagnostic test to
confirm SIAD, hypouricemia with an elevated fractional excretion of
urate is highly suggestive of SIAD. Other hallmarks of SIAD include
mild volume expansion with low to normal plasma concentrations of
creatinine, urea, uric acid, and potassium; impaired free water
excretion with normal sodium excretion that reflects sodium intake;
and hyponatremia that is relatively unresponsive to giving sodium
without concomitant fluid restriction. Measurement of AVP or
copeptin is not useful. Useful biochemical markers in adults include
a spot urine sodium concentration greater than 30 mEq/L (>30
mmol/L), fractional excretion of sodium greater than 0.5%,
fractional excretion of urea greater than 55%, fractional excretion of
urate greater than 11%, and plasma uric acid less than 4 mg/dL
(<237.9 µmol/L).
In the workup of hyponatremia, it is important to distinguish true
hypoosmolality from pseudohyponatremia (seen with
hyperproteinemia or severe hyperlipidemia) or translocational
hyponatremia (seen with hyperglycemia, mannitol, hypertonic
radiocontrast). Urine osmolality should be measured to determine
whether there is impaired free water clearance. Diseases causing
decreased circulating volume, renal impairment, adrenal
insufficiency, and hypothyroidism must be excluded. The patient in
this vignette appears to be euvolemic. He has hyponatremia with
true hypoosmolality. His serum uric acid concentration is very low
and his urinary sodium excretion is elevated. Although he has
bacteremia, his vital signs are not suggestive of sepsis. Therefore, he
most likely has SIAD (Answer D) associated with vincristine
chemotherapy.
Severe hypertriglyceridemia causes pseudohyponatremia. While
this child does have an elevated triglyceride concentration (Answer
A), it is not high enough to cause this degree of hyponatremia.
Cerebral/renal salt wasting (Answer B) can be very difficult to
distinguish from SIAD, but it would be a more likely etiology if there
were evidence of circulatory volume depletion.
Hospitalized patients are at risk of developing hyponatremia if
given excessive hypotonic fluids (Answer C). However, this patient
was receiving isotonic fluids at a maintenance rate.
Cortisol insufficiency (Answer E) can be associated with SIAD, as
glucocorticoids have an inhibitory effect on AVP synthesis. Although
this child has a very low cortisol level, this is most likely due to
adrenal suppression from the dexamethasone he is currently
receiving.

Educational Objective
Identify characteristic clinical and laboratory findings in patients
with syndrome of inappropriate antidiuretic hormone secretion.

Reference(s)
Moritz ML. Syndrome of inappropriate antidiuresis. Pediatr Clin North Am. 2019;66(1):209-
226. PMID: 30454744
Cuesta M, Thompson CJ. The syndrome of inappropriate antidiuresis (SIAD). Best Pract Res
Clin Endocrinol Metab. 2016;30(2):175-187. PMID: 27156757
68 ANSWER: C) Start oxandrolone at a dosage of 0.03 mg/kg
daily and maintain the current GH dosage
GH therapy is approved for the management of short stature
associated with Turner syndrome, which is usually not due to GH
deficiency. Early initiation of GH therapy leads to better final height
outcomes than late treatment. Clinical guidelines suggest starting
GH around age 4 to 6 years and certainly before age 12 to 13 years. A
decline in growth velocity in a patient with Turner syndrome may
also be considered as a reason to start treatment at an even earlier
age. The GH dosage recommended for Turner syndrome (45-50
mcg/kg daily or 0.315-0.35 mg/kg weekly) is higher than the dosage
recommended to treat GH deficiency (25-34 mcg/kg daily or 0.18-
0.24 mg/kg weekly). Nevertheless, surveillance of dosage adequacy
with measurement of IGF-1 is recommended to avoid overdosing as
manifested by an elevated IGF-1 SDS (>+2.0). Therefore, increasing
the GH dosage to 0.35 mg/kg weekly (Answer A) is not the
preferred strategy given that this patient already has an IGF-1 SDS
above +2.0 while receiving a GH dosage lower than that which is
recommended for this condition.
Girls with Turner syndrome whose karyotype shows a 45,X
complement without mosaicism also have gonadal dysgenesis with
lack of pubertal development and lack of estrogen secretion. Sex
steroids have a synergistic effect with endogenous GH on linear
growth during pubertal development. However, at the same time,
estrogen advances growth plate fusion, thus limiting the duration of
linear growth. Initiation of estrogen replacement in girls with Turner
syndrome, who do not have spontaneous puberty, is recommended
around age 11 or 12 years with transdermal estradiol being the
preferred route of administration. Starting estrogen at an earlier age
(Answer B) may induce undue advancement in skeletal maturation
with subsequent limitation of growth potential. Initiation of ultra-
low estrogen dosages at an earlier age has been proposed, but this
intervention has not reached the consensus guidelines. However, it
has been shown that delaying the initiation of estrogen replacement
to a later age such as 14 or 16 years (Answer E) does not have an
additional beneficial effect on final height as initially thought. This
may actually have a negative psychosocial impact because of lack of
pubertal changes at this critical age.
Androgens have a positive effect on linear growth during
puberty. While estrogens may have a similar role, they also exert a
negative impact on final height as a result of their effect on growth
plate maturation and fusion. Aromatase inhibitors (Answer D) are
capable of blocking the conversion of androgens into estrogens.
Hence, the use of aromatase inhibitors has been proposed for the
management of short stature in boys in early puberty, but there is no
FDA approval for this indication. Aromatase inhibitors would not
have this effect in a prepubertal girl with Turner syndrome whose
endogenous estrogen production is most likely insignificant.
Oxandrolone (Answer C) is a synthetic anabolic steroid that has
been used for the treatment of constitutional delay of growth and
puberty, as it induces secondary sexual characteristics, including an
increase in height velocity. It is nonaromatizable and, therefore, does
not lead to an increase in estrogen levels that would induce growth
plate maturation and fusion. Oxandrolone has been shown to
increase the final height of girls with Turner syndrome who are
treated with GH. The recommended age of initiation is around 10
years and the recommended dosage is 0.03 mg/kg daily and
maintained below 0.05 mg/kg daily. In some studies, higher dosages
are possibly associated with some degree of virilization.

Educational Objective
List the benefits and caveats of oxandrolone in the management of
short stature in girls with Turner syndrome.

Reference(s)
Gault EJ, Cole TJ, Casey S, et al. Effect of oxandrolone and timing of pubertal induction on
final height in Turner Syndrome: final analysis of the UK randomized placebo-
controlled trial. Arch Dis Child. 2019 [Epub ahead of print] PMID: 31862699
Gravholt CH, Andersen NH, Conway GS, et al; International Turner Syndrome Consensus
Group. Clinical practice guidelines for the care of girls and women with Turner
syndrome: proceedings from the 2016 Cincinnati International Turner Syndrome
Meeting. Eur J Endocrinol. 2017;177(3):G1-G70. PMID: 28705803
Klein KO, Rosenfield R, Santen RJ, et al. Estrogen replacement in Turner syndrome:
literature review and practical considerations. J Clin Endocrinol Metab. 2018;103(5):1790-
1803. PMID: 29438552
Ross JL, Quigley CA, Cao D, et al. Growth hormone plus childhood low-dose estrogen in
Turner syndrome. N Engl J Med. 2011;364:1230-1242. PMID: 21449786
Hasegawa Y, Itonaga T, Ikegawa K, et al. Ultra-low-dose estrogen therapy for female
hypogonadism. Clin Pediatr Endocrinol. 2020;29(2):49-53. PMID: 32313372
69 ANSWER: C) Measurement of ALT
Nonalcoholic fatty liver disease (NAFLD) is a chronic liver disease
due to excess fat accumulation in the liver. It has become the most
common liver disease in children in western countries. The
prevalence of pediatric NAFLD in the United States ranges from 29%
to 38% in select studies. Male sex, older age, and ethnicity (Hispanic
children have the highest risk) are all risk factors for NAFLD.
Although data on the long-term complications resulting from
pediatric NAFLD are limited, disease in children appears to be more
severe than in adults. It is often asymptomatic and thus screening for
NAFLD in at-risk patients is prudent, so that the condition can be
recognized and treated before the onset of end-stage liver disease.
According to the most recent pediatric guidelines, screening should
be considered for all children with obesity aged 9 to 11 years and for
children who are overweight who present with other risk factors
such as insulin resistance, prediabetes/diabetes, dyslipidemia, or a
family history of NAFLD/NASH. However, it may be prudent to
screen younger patients who present with severe obesity or those
with a positive family history of NAFLD/nonalcoholic
steatohepatitis.
While ALT measurement (Answer C) has limitations, including
the fact that optimal cutoffs have not been well established, it is
inexpensive and is widely available as a screening tool. It has also
been shown to correlate with the presence of hepatic steatosis.
Recent standards have been proposed for normal cutoff values of
ALT (≤26 U/L [≤0.43 µkat/L] for boys and ≤22 U/L [≤0.37 µkat/L]
for girls). ALT values greater than 2 times the upper normal limit
(ALT ≥50 U/L [≥0.84 µkat/L] for boys and ≥44 U/L [≥0.73 µkat/L]
for girls) in overweight/obese children aged 10 years and older have
a sensitivity of 88% for diagnosing NAFLD.
AST (Answer B), as well as γ-glutamyl transferase, has not been
tested independently for the purpose of screening in pediatric
patients. Additionally, elevated AST or γ-glutamyl transferase may
represent liver conditions other than NAFLD, particularly if the ALT
concentration is normal.
Abdominal ultrasonography (Answer A) is poor at detecting
steatosis. Referral to gastroenterology (Answer D) could be
considered in the face of an abnormal ALT value, but screening with
ALT is a more appropriate starting place. Abdominal MRI (Answer
E) has been validated and shown to be accurate for detection and
quantification of hepatic steatosis in both adults and children;
however, cost, lack of availability, and lack of validated cutoffs limit
its current usefulness as a screening tool. Newer noninvasive
diagnostic tools, such acoustic radiation force impact, transient
elastography, and magnetic resonance elastography, are increasingly
being used for the detection of hepatic steatosis and fibrosis,
although these still need to be validated in pediatric patients.

Educational Objective
Identify nonalcoholic fatty liver disease as the most common liver
disease in children living in the United States and recommend ALT
measurement as the preferred first-line screening test.

Reference(s)
Draijer L, Benninga M, Koot B. Pediatric NAFLD: an overview and recent developments in
diagnostics and treatment. Expert Rev Gastroenterol Hepatol. 2019;13(5):447-461. PMID:
30875479
Vos MB, Abrams SH, Barlow SE, et al. NASPGHAN clinical practice guideline for the
diagnosis and treatment of nonalcoholic fatty liver disease in children:
recommendations from the expert committee on NAFLD (ECON) and the North
American Society of Pediatric Gastroenterology, Hepatology and Nutrition
(NASPGHAN). J Pediatr Gastroenterol Nutr. 2017;64(2):319-334. PMID: 28107283
Styne DM, Arslanian SA, Connor EL, et al. Pediatric obesity—assessment, treatment, and
prevention: an Endocrine Society clinical practice guideline. J Clin Endocrinol Metab.
2017;102(3):709-757. PMID: 28359099
70 ANSWER: C) Lumbar spine DXA and vertebral fracture
analysis by DXA
Atraumatic vertebral compression fractures alone are sufficient to
diagnose pediatric osteoporosis. While many adult guidelines define
chronic steroid exposure as at least 3 months or more of greater than
7.5 mg daily of prednisone equivalents, work from the Steroid-
Induced Osteoporosis in the Pediatric Population program identified
that pediatric patients may present with vertebral compression
fractures within 30 days of starting glucocorticoid therapy. Risk
factors include greater cumulative glucocorticoid exposure,
increased BMI (likely secondary to the cushingoid effect from high-
dosage glucocorticoids), and lower bone mineral density. This
patient is at high risk for vertebral compression fractures. Strict bed
rest (Answer A) can further compromise skeletal health by leading
to muscle deconditioning and bone loss from immobility. In fact,
physical therapy is still recommended for patients with vertebral
compression fractures to strengthen core muscles.
Imaging can help guide the treatment plan, as the presence of 1
atraumatic vertebral compression fracture is sufficient for the
diagnosis of pediatric osteoporosis. Bone density assessment by
DXA now is important to assess the patient’s current bone health, as
well as to monitor response to treatment interventions over time.
Importantly, vertebral compression fractures can occur even with a
normal bone mineral density Z-score (thus, Answer B is incorrect).
One also must be cautious interpreting a lumbar spine bone mineral
density Z-score result alone without review of the DXA image itself,
as a vertebral compression fracture could result in false elevation of
bone mineral density. While DXA images can suggest the presence of
a vertebral compression fracture, dedicated spine imaging is
recommended.
The most recent pediatric position statement developed by the
International Society for Clinical Densitometry in 2019 supports the
validity of vertebral fracture analysis via DXA (Answer C) to detect
vertebral compression fractures in at-risk children when images are
interpreted by a clinician with experience in pediatric vertebral
development. Anterior wedging may be present as part of normal
physiologic development, especially in the midthoracic region where
most vertebral compression fractures occur. Reliance on DXA
manufacturer software would thus potentially overdiagnose
vertebral deformities in children. Vertebral fracture analysis has
similar sensitivity and specificity to that of standard spine
radiographs to detect moderate to severe vertebral compression
fractures (>25%), but requires less than half of the amount of
radiation. Vertebral fracture analysis by DXA does have limited
ability to detect mild vertebral compression fractures (20%-25%
height loss), especially in the upper thoracic spine (T4-T7); however,
standard radiographs also have this limitation secondary to the
overlying soft tissue and other organs in that region. Therefore, the
new guidelines are that vertebral fracture analysis by DXA is a
reasonable first-line assessment for patients at high risk for vertebral
compression fractures, followed by spine radiograph (Answer D) or
MRI only if the vertebrae are unable to be measured accurately. In
centers without this expertise, it may be appropriate to obtain
standard radiographs initially.
While this child complained of back pain, it is important to be
aware that nearly half of vertebral compression fractures in at-risk
populations are asymptomatic. Therefore, monitoring an objective
measure of spine health should be considered when patients are at
increased risk for vertebral compression. Children with sufficient
remaining linear growth have the ability to recover the vertebral
height loss without the use of antiresorptive agents such as
bisphosphonates (Answer E) if risk factors for osteoporosis are
mitigated (ie, cessation of glucocorticoid therapy). It is unknown
how much growth potential must remain, but given the fact that this
child is prepubertal, bisphosphonate therapy may not be necessary if
she is able to discontinue the high-dosage glucocorticoid therapy
and undergo remission.

Educational Objective
Recommend vertebral fracture analysis by DXA as a reliable method
to diagnose vertebral compression fractures if scans are read and
interpreted by an experienced clinician.

Reference(s)
Crabtree NJ, Arabi A, Bachrach LK, et al. Dual-energy X-ray absorptiometry interpretation
and reporting in children and adolescents: the revised 2013 ISCD Pediatric Official
Positions. J Clin Densitom. 2014;17(2):225-242. PMID: 24690232
Halton J, Gaboury I, Grant R, et al; Canadian STOPP Consortium. Advanced vertebral
fracture among newly diagnosed children with acute lymphoblastic leukemia: results of
the Canadian Steroid-Associated Osteoporosis in the Pediatric Population (STOPP)
research program. J Bone Miner Res. 2009;24(7):1326-1334. PMID: 19210218
Ward LM, Ma J, Lang B, et al; Steroid-Associated Osteoporosis in the Pediatric Population
(STOPP) Consortium. Bone morbidity and recovery in children with acute
lymphoblastic leukemia: results of a six-year prospective cohort study. J Bone Miner Res.
2018;33(8):1435-1443. PMID: 29786884
Weber DR, Boyce A, Gordon C, et al. The utility of DXA assessment at the forearm,
proximal femur, and lateral distal femur, and vertebral fracture assessment in the
pediatric population: 2019 ISCD Official Position. J Clin Densitom. 2019;22(4):567-589.
PMID: 31421951
71 ANSWER: D) Assess thyroid function now and aim for a TSH
concentration 0.5-1.0 mIU/L; measure thyroglobulin in 2 to 3
months while on levothyroxine
The recent 2015 pediatric guidelines from the American Thyroid
Association classify the risk of having persistent cervical disease
and/or distant metastases after thyroidectomy with/without
cervical lymph node dissection. The mortality risk from thyroid
cancer is very low in pediatric patients. The risk assessment is used
to guide the management and monitoring of patients with
nonmedullary differentiated thyroid cancer and is geared primarily
toward papillary thyroid cancer or the follicular variant of papillary
thyroid cancer. The American Thyroid Association pediatric low-risk
designation is reserved for thyroid cancer grossly confined to the
thyroid with N0/Nx (no metastatic lymph node/s found, or no
lymph node/s checked) or incidental N1a (microscopic metastasis to
a small number of central neck [level 6] lymph nodes). The American
Thyroid Association pediatric intermediate-risk designation defines
those with extensive N1a or minimal N1b (lateral neck), whereas the
high-risk designation defines those with regionally extensive disease
(extensive N1b) or locally invasive disease (T4 tumors per the
American Joint Committee on Cancer [AJCC] Tumor, Node,
Metastasis [TNM] classification), with or without distant metastasis
(typically pulmonary). Both intermediate- and high-risk cancers
require stimulated thyroglobulin measurement and whole-body 123I
scan to better stage them postoperatively and decide whether 131I
radioablation and/or additional surgery is required. Thyroglobulin
antibodies should be confirmed to be undetectable to ensure that the
thyroglobulin level in a routine panel is reliable.
Although this patient’s large tumor size would make it more
likely for her to be in a higher risk group, her tumor does not
manifest any aggressive features such as invasion of the capsule or
lymphatic or vascular invasion. She is in the N0/Nx category and is
therefore in the low-risk group. As a result, this patient’s goal TSH
concentration is 0.5 to 1.0 mIU/L, and nonstimulated (basal)
thyroglobulin should be measured within 12 weeks of surgery
(Answer D).
Patients in the high-risk group and some in the intermediate-risk
group who require 131I (Answer A) receive it after review of the
results of the stimulated thyroglobulin measurement and 123I
whole-body scan. The TSH cutoffs in Answer B (TSH cutoff is for
high risk) and Answer C (TSH cutoff is for intermediate risk) are
incorrect. Stimulated thyroglobulin and a whole-body scan (Answer
E) are not recommended for patients at low risk.

Educational Objective
Guide the management of low-risk papillary thyroid cancer in an
adolescent.

Reference(s)
Edge SB, Byrd DR, Compton CC, eds. AJCC Cancer Staging Manual. 7th ed. Springer
International Publishing; 2010.
Francis GL, Waguespack SG, Bauer AJ, et al; American Thyroid Association Guidelines Task
Force. Management guidelines for children with thyroid nodules and differentiated
thyroid cancer. Thyroid. 2015;25(7):716-759. PMID: 25900731
72 ANSWER: D) Hypophosphatasia
The most notable finding in this child is the low serum alkaline
phosphatase concentration. While the reported value is just outside
of the reference range, it is important to remember that children have
significantly higher levels of alkaline phosphatase than adults, and
levels less than 100 U/L (<1.67 μkat/L) should raise concern when
there are physical signs of rickets. Hypophosphatasia (Answer D)
occurs in 1 in 100,000 live births and is due to loss-of-function
pathogenic variants in the ALPL gene, which encodes tissue
nonspecific alkaline phosphatase. Inadequate action of tissue
nonspecific alkaline phosphatase results in accumulation of the
substrate inorganic pyrophosphate, which is a potent inhibitor of
mineralization. The spectrum of disease includes perinatal lethal,
infantile-onset, childhood-onset, and adult-onset forms, as well as
odontohypophosphatasia (premature tooth loss alone without a
skeletal phenotype). The phenotype is related to the residual
function of the enzyme. Skeletal manifestations include
craniosynostosis, rickets, bone and joint pain, fractures or
pseudofractures, and the hallmark premature loss (before age 5
years) of primary teeth with the root intact. Radiographs show
“radiolucent tongues” extending from the growth plate into the
metaphysis. The infantile form presents with respiratory issues due
to an underdeveloped thoracic cage, hypercalcemia and
hyperphosphatemia due to inability to deposit mineral into bone,
nephrocalcinosis, and seizures responsive to vitamin B6. Some
children present with weakness, delayed walking, and a waddling
gait. Adult forms can present with recurrent metatarsal stress
fractures, loss of adult dentition, osteomalacia, and pseudogout.
Ongoing data collection suggests that bisphosphonate use in adults
with hypophosphatasia may result in an increased risk for atypical
femur fractures. In 2015, the enzyme replacement therapy asfotase
alfa became clinically available to treat this disorder. Treatment with
asfotase alfa results in improved skeletal mineralization, respiratory
function, growth, and cognitive and motor function for the infantile
and childhood forms, as well as improved bone mineralization,
motor function, muscle strength, and patient-reported function in
adolescents and adults. While the mother’s recurrent stress fractures
may be secondary to her training for marathons, it would be
reasonable to measure her alkaline phosphatase level. It is also
important to ensure age-specific reference ranges are reported, as
alkaline phosphatase levels are significantly higher in children than
in adults.
The differential diagnosis of low serum alkaline phosphatase is
broad and includes vitamin D intoxication, hypoparathyroidism,
zinc deficiency, magnesium deficiency, celiac disease, Wilson disease,
Cushing syndrome, hypothyroidism, pernicious anemia,
osteogenesis imperfecta type 2, and cleidocranial dysostosis. Vitamin
D intoxication (Answer A) leads to hypercalcemia,
hyperphosphatemia, and low serum alkaline phosphatase; however,
serum 25-hydroxyvitamin D values are generally above 100 ng/dL
(>249.6 nmol/L). Lower-extremity bowing deformities are not part
of the clinical presentation. This patient’s PTH was low-normal with
a mildly elevated phosphate concentration and normal serum
calcium concentration appropriate for the 25-hydroxyvitamin D
level. Thus, he does not have hypoparathyroidism (Answer B).
Risk factors for rickets of prematurity (Answer E) include
gestational age less than 27 weeks, birth weight less than 2 lb 3 oz
(<1000 g), prolonged use of total parental nutrition (>4 weeks),
history of necrotizing enterocolitis, and prolonged postnatal steroid
use. Biochemical changes of low serum phosphate and elevated
alkaline phosphatase present between 3 and 14 weeks of life due to
the inability to replicate the extensive skeletal uptake of calcium by
the fetus during the third trimester. While physical changes are rare,
some cases can manifest with fractures. Although this child was at
risk for rickets of prematurity, his symptoms are progressing,
whereas rickets of prematurity is self-resolving and there is little
evidence for long-term consequences on bone health.
Physiologic bowing (Answer C) is certainly a possibility;
however, the alkaline phosphatase concentration is frankly low.
Educational Objective
Diagnose hypophosphatasia in a child with rachitic changes in the
setting of low serum alkaline phosphatase.

Reference(s)
Whyte MP. Hypophosphatasia - aetiology, nosology, pathogenesis, diagnosis and treatment.
Nat Rev Endocrinol. 2016;12(4):233-246. PMID: 26893260
73 ANSWER: A) Inhibin B = 17.7 pg/mL (SI: 17.7 ng/L);
antimullerian hormone = 41 ng/mL (SI: 293 pmol/L); FSH = 0.33
mIU/mL (SI: 0.33 IU/L)
The patient in this vignette has a testicular volume less than 3 mL at
15 years of age. The differential diagnosis includes constitutional
delay of growth and puberty vs primary or hypogonadotropic
hypogonadism. The history of cryptorchidism, hyposmia, and
hypogonadism is most consistent with a diagnosis of Kallmann
syndrome, which results from a defect in formation of the olfactory
placode during embryonic development. This causes loss of
olfactory nerve fibers and failure of GnRH neuron migration to the
hypothalamus. Low inhibin B, antimullerian hormone, and FSH
levels (Answer A) are consistently found in patients with
hypogonadotropic hypogonadism due to Kallmann syndrome.
Distinguishing hypogonadotropic hypogonadism from
constitutional delay is challenging but can be based on certain
features, including absent or diminished sense of smell and the
finding of a hypoplastic olfactory bulb on brain MRI. Other findings
that are suggestive of Kallmann syndrome include micropenis,
cryptorchidism, and synkinesia (mirror movements of the hands).
While more than 25 genes involved in Kallmann syndrome have
been identified, a genetic cause is not found in nearly half of cases.
In children with hypogonadotropic hypogonadism, inadequate
low FSH levels and consequent failure of Sertoli-cell proliferation
result in decreased production and secretion of peptide hormones,
including inhibin B and antimullerian hormone. As such, inhibin B
and antimullerian hormone levels are helpful in distinguishing
constitutional delay of growth and puberty from hypogonadotropic
hypogonadism. Children with constitutional delay of growth and
puberty have normal inhibin B and antimullerian hormone levels
before onset of puberty. In contrast, both inhibin B and antimullerian
hormone are low in children with hypogonadotropic hypogonadism.
Antimullerian hormone levels peak in the first year of life and
stay elevated throughout childhood. Antimullerian hormone levels
decrease in puberty due to inhibition by testosterone. Pubertal-aged
individuals with hypogonadotropic hypogonadism have
antimullerian hormone levels that are low when compared with the
normal prepubertal range due to FSH deficiency but are high for age
due to lack of testosterone inhibition.
Inhibin B levels remain detectable throughout childhood and then
rise in puberty along with FSH levels. Inhibin B concentrations are
significantly lower in individuals with hypogonadotropic
hypogonadism than in those with constitutional delay of growth and
puberty; a cutoff of 61 pg/mL (61 ng/L) has 90% sensitivity and 83%
specificity for detecting hypogonadotropic hypogonadism, which
makes inhibin B a useful marker for detection.
The following graphs contain data used to establish reference
intervals for antimullerian hormone and inhibin B in children and
adolescents (see figures).
Pediatric antimullerian hormone measurement: male (n = 469) and female (n =
235). Reference intervals established using the automated Beckman Coulter
Access AMH assay. Reprinted from Jopling H, Yates A, Burgoyne N, Hayden K,
Chaloner C, Tetlow L. Paediatric anti-müllerian hormone measurement: male
and female reference intervals established using the automated Beckman
Coulter Access AMH assay. Endocrinol Diabetes Metab. 2018;1(4):e00021.
Pediatric inhibin B measurement in 366 boys aged 0 to 18 years. The thick line
shows the median for each yearly age group, plotted at the midpoint of each
year. Reprinted from Crofton PM, Evans AEM, Groome NP, Taylor MRH, Holland
CV, Kelnar CJH. Inhibin B in boys from birth to adulthood: relationship with age,
pubertal stage, FSH and testosterone. Clin Endocrinol (Oxf). 2002;56(2):215-
221.

Educational Objective
Identify hormone profiles consistent with Kallmann syndrome.

Reference(s)
Condorelli RA, Cannarella R, Calogero AE, La Vignera S. Evaluation of testicular function
in prepubertal children. Endocrine. 2018;62(2):274-280. PMID: 29982874
Crofton PM, Evans AEM, Groome NP, Taylor MRH, Holland CV, Kelnar CJH. Inhibin B in
boys from birth to adulthood: relationship with age, pubertal stage, FSH and
testosterone. Clin Endocrinol (Oxf). 2002;56(2):215-221. PMID: 11874413
Grinspon RP, Urrutia M, Rey RA. Male central hypogonadism in paediatrics – the relevance
of follicle-stimulating hormone and Sertoli cell markers. Eur Endocrinol. 2018;14(2):67-
71. PMID: 30349597
Jopling H, Yates A, Burgoyne N, Hayden K, Chaloner C, Tetlow L. Paediatric anti-müllerian
hormone measurement: male and female reference intervals established using the
automated Beckman Coulter Access AMH assay. Endocrinol Diabetes Metab.
2018;1(4):e00021. PMID: 30815559
74 ANSWER: C) Decrease the GH dosage
In June 2000, GH therapy was approved by the US FDA for the
management of growth failure associated with Prader-Willi
syndrome based on studies showing an improvement in linear
growth and body composition (decrease in fat mass and increase in
lean mass) in these children.
IGF-1 is a useful indicator of GH dosage adequacy in children
with idiopathic GH deficiency. Higher IGF-1 levels have been
observed in patients with Prader-Willi syndrome in response to
standard dosages of GH when compared with IGF-1 levels in
children without Prader-Willis syndrome. The possibility of higher
sensitivity to GH therapy is thought to be a possible mechanism. An
alternative mechanism has been proposed for this finding. In
children with Prader-Willi syndrome treated with GH, most of the
IGF-1 is sequestered in the 150-kDa complex with IGFBP-3 and acid-
labile subunit, decreasing the proportion of bioactive IGF-1. This has
led to the proposal that elevated IGF-1 levels are not a reason to
decrease the GH dosage in these individuals because there is no
good correlation between serum immunoreactive IGF-1 and
bioactive IGF-1 in children with Prader-Willi syndrome treated with
GH. However, the clinical consensus guidelines still recommend
guiding the dosing of GH in patients with Prader-Willi based on
clinical response and recommend maintaining IGF-1 levels within
the physiological range. This child’s IGF-1 is elevated (SDS >+2);
therefore, a decrease in the GH dosage (Answer C) is the best next
step. Continuation of the current GH dosage (Answer A) is not the
preferred next step despite the reported good response to therapy.
The recommended initial dosage of GH in children with Prader-
Willi syndrome is 0.5 mg/m2 daily with dosage titration to 1 mg/m2
daily based on clinical response and IGF-1 measurements.
High levels of IGF-1 are associated with lymphoid hyperplasia,
which can worsen obstructive sleep apnea. On the basis of several
reports of sudden death occurring shortly after initiation of GH
therapy in children with Prader-Willi syndrome, a warning was
added to the prescribing recommendations, specifically in patients
with preexisting significant obesity and severe respiratory
impairment. Polysomnography (sleep study) and otolaryngology
evaluation of the upper airway are recommended before initiation of
GH therapy. Some authors also recommend a repeated sleep study
approximately 6 months after therapy initiation.
While GH, an insulin counterregulatory hormone, increases
insulin resistance and the risk of dysglycemia/hyperglycemia,
especially in individuals with obesity, the screening laboratory tests
in this vignette (random glucose and hemoglobin A1c) are normal
and not suggestive of an alteration of carbohydrate metabolism at
this time. Thus, an oral glucose tolerance test (Answer D) is not the
best next step.
Children with Prader-Willi syndrome may have hypothalamic
dysfunction, which can be associated with central hypothyroidism.
This child’s thyroid function is normal and does not warrant thyroid
hormone replacement now (Answer B).
This patient’s vitamin D level is normal, so there is no indication
for vitamin D therapy (Answer E). Nevertheless, it is advised to
ensure adequate maintenance of vitamin D levels through feeds or
supplementation.

Educational Objective
Guide the management of GH therapy in children with Prader-Willi
syndrome.

Reference(s)
Bakker NE, van Doorn J, Renes JS, Donker GH, Hokken-Koelega AC. IGF-1 levels, complex
formation, and IGF bioactivity in growth hormone-treated children with Prader-Willi
syndrome. J Clin Endocrinol Metab. 2015;100(8):3041-3049. PMID: 26050733
Whitman B, Carrel A, Bekx T, Weber C, Allen D, Myers S. Growth hormone improves body
composition and motor development in infants with Prader-Willi syndrome after six
months. J Pediatr Endocrinol Metab. 2004;17(4):591-600. PMID: 15198290
Barbera J, Voloh I, Berall G, Shapiro CM. Sleep abnormalities and Prader-Willi syndrome.
In: Pandi-Perumal, Kramer M, eds. Sleep and Mental Illness. Cambridge University Press;
2010.
Miller JL, Shuster J, Theriaque D, Driscoll DJ, Wagner M. Sleep disordered breathing in
infants with Prader-Willi syndrome during the first 6 weeks of growth hormone
therapy: a pilot study. J Clin Sleep Med. 2009;5(5):448-453. PMID: 19961030
Eiholzer U. Deaths in children with Prader-Willi syndrome. Horm Res. 2005;63(1):33-39.
PMID: 15604598
Carrel AL, Myers SE, Whitman BY, Eickhoff J, Allen DB. Long-term growth hormone
therapy changes the natural history of body composition and motor function in
children with Prader-Willi syndrome. J Clin Endocrinol Metab. 2010;95(3):1131-1136.
PMID: 20061431
Chernausek SD, Backeljauw PF, Frane J, Kuntze, Underwood LE; GH insensitivity
syndrome Collaborative Group. J Clin Endocrinol Metab. 2007;92(3):902-910. PMID:
17192294
Feigerlová E, Diene G, Oliver I, et al. Elevated insulin-like growth factor-I values in children
with Prader-Willi syndrome compared with growth hormone (GH) deficiency children
over two years of GH treatment. J Clin Endocrinol Metab. 2010;95(10):4600-4608. PMID:
20926543
Deal C, Tony M, Höybye C, et al; 2011 Growth Hormone in Prader-Willi Syndrome Clinical
Care Guidelines Workshop Participants. Growth Hormone Research Society workshop
summary: consensus guidelines for recombinant human growth hormone therapy in
Prader-Willi syndrome. J Clin Endocrinol Metab. 2013;98(6):E1072-E1087. PMID: 23543664
75 ANSWER: B) VHL (von Hippel-Lindau tumor suppressor)
Pheochromocytomas and paragangliomas caused by pathogenic
variants in the VHL gene (Answer B), associated with von Hippel–
Lindau syndrome, have a noradrenergic biochemical profile, unlike
tumors associated with RET (Answer A), TMEM127 (Answer C), and
NF1 (Answer D), which show adrenergic predominance. Pathogenic
variants in the SDHC gene (Answer E) can be associated with
tumors that have a noradrenergic biochemical profile, but these
variants tend to cause solitary head and neck paragangliomas. The
Endocrine Society guidelines for the evaluation and treatment of
pheochromocytomas and paragangliomas advocate that genetic
testing be considered in all patients independent of a clear family
history. This is consistent with other practice guidelines and
consensus statements from expert groups, as 40% to 50% of
pheochromocytomas and paragangliomas are associated with
pathogenic variants in known causative genes. More than 12 genes
are associated with pheochromocytomas and paragangliomas (see
table). Some of these are associated with distinct clinical syndromes
and predominant biochemical profiles. The most commonly
associated genes in pediatric patients are VHL and SDHB.
Table. Genes Associated With Pheochromocytomas and Paragangliomas

Frequency
of
Gene (symbol, location, and pathogenic Biochemical
inheritance pattern) variants Syndrome profile
NF1 3% Neurofibromatosis Adrenergic
17q11.2 (germline), type 1 predominance
Autosomal dominant 20-25%
(somatic)
VHL 7%-10% von Hippel–Lindau Noradrenergic
3p25-p26 (germline or disease
Autosomal dominant somatic)
RET 5%-6% Multiple endocrine Adrenergic
10q11.2 (germline or neoplasia type 2 predominance
Autosomal dominant somatic)
SDHx genes SDHB Hereditary Noradrenergic,
(8%-10%, paraganglioma- possibly
Autosomal dominant except SDHD, germline) pheochromocytoma dopaminergic,
which is autosomal dominant and SDHD syndrome possibly
paternally inherited (due to maternal (5%-7%, nonsecreting
imprinting) germline)
SDHC
SDHB (1p36.13) (1%-2%,
SDHD (11q23) germline)
SDHC (1q21) SDHA
SDHAF2 (11q13.1) (<1%,
germline or
somatic)
SDHAF2
(<1%)
TMEM127 1%-2% Hereditary Adrenergic
2q11.2 (germline) pheochromocytoma predominance
Autosomal dominant and paraganglioma
MAX 1%-2% Hereditary Adrenergic
14q23.3 (germline or pheochromocytoma predominance
Autosomal dominant/paternally somatic) and paraganglioma
inherited
EPAS1 (also known as HIF2A) 6%-12% Polycythemia Noradrenergic
2p21-p16 (mosaic or paraganglioma
Mosaic/somatic somatic) syndrome
FH 1%-2% Hereditary Noradrenergic
(germline) leiomyomatosis and
renal cell cancer
syndrome
MDH2 <2% Hereditary Noradrenergic
(germline) pheochromocytoma
and paraganglioma

Table adapted from references (Fishbein 2019, Toledo et al 2017, Favier et al 2015).
Additional genes include TP53 <5% (somatic), MET <2% (germline) or <2%-10% (somatic),
MERTK <2% (germline), KMT2D <2% (germline or somatic), KIF1B <5% (germline or
somatic), IDH2 <0.5% (somatic), H3F3A <2% (mosaic), FGFR1 approximately 1%
(somatic), EGLN1/PHD2 <1% (germline or somatic), CDKN2A <2% (somatic), BRAF <2%
(somatic), ATRX <5% (somatic). Additional variants in other genes (SCL25A11, GOT2,
DNMT3A, and DLST) have been reported but causality is unclear.

The presence of additional features can often help determine which


candidate gene to test: for example, retinal angiomas in von Hippel–
Lindau syndrome; a thyroid mass in multiple endocrine neoplasia
type 2; cafe-au-lait spots, axillary and inguinal freckling, Lisch
nodules on the iris, and subcutaneous neurofibromas in
neurofibromatosis type 1; and a neck mass in paraganglioma
syndromes. A number of algorithms for genetic testing have been
developed depending on whether the genetic testing is undertaken
by candidate gene Sanger sequencing, gene panel, or next-
generation sequencing. In the absence of syndromic features, the
pathways account for the presence or absence of metastasis, location
of paragangliomas, extraadrenal or adrenal disease,
unilateral/bilateral disease, and secretory biochemical phenotypes.
In the case of isolated adrenal disease with a noradrenergic
secretory phenotype, VHL genetic testing is recommended.
Increasingly, the diagnosis of pathogenic variants is through use of
gene panels and next-generation sequencing. A consensus statement
on next-generation sequencing in pheochromocytomas and
paragangliomas was published in 2017, and it recommends the use
of a validated targeted gene panel for the clinical genetic diagnosis
of hereditary pheochromocytomas and paragangliomas. Although
next-generation sequencing is increasingly used in research, there
are concerns that without a rigorous pipeline to interpret the data,
high-throughput sequencing methods may lead to false-positive
results with many variants of unknown significance reported as
causative. This could have severe consequences for patients and
their relatives who would undergo surveillance/screening
accordingly.

Educational Objective
Explain the genetics of pheochromocytomas and paragangliomas
and describe the stratification of genetic diagnosis dependent on
clinical phenotype, tumor location, and biochemical phenotype.

Reference(s)
Fishbein L. Pheochromocytoma/paraganglioma: is this a genetic disorder? Curr Cardiol Rep.
2019;21(9):104. PMID: 31367972
Lenders JW, Duh QY, Eisenhofer G, et al; Endocrine Society. Pheochromocytoma and
paraganglioma: an Endocrine Society clinical practice guideline. J Clin Endocrinol Metab.
2014;99(6):1915-1942. PMID: 24893135
Hampel H, Bennett RL, Buchanan A, Pearlman R, Wiesner GL, Guideline Development
Group, American College of Medical Genetics and Genomics Professional Practice and
Guidelines Committee and National Society of Genetic Counselors Practice Guidelines
Committee. A practice guideline from the American College of Medical Genetics and
Genomics and the National Society of Genetic Counselors: referral indications for
cancer predisposition assessment. Genet Med. 2015;17(1):70-87. PMID: 25394175
NGS in PPGL (NGSnPPGL) Study Group, Toledo RA, Burnichon N, et al. Consensus
statement on next-generation-sequencing-based diagnostic testing of hereditary
phaeochromocytomas and paragangliomas. Nat Rev Endocrinol. 2017;13(4):233-247.
PMID: 27857127
Favier J, Amar L, Gimenez-Roqueplo AP. Paraganglioma and phaeochromocytoma: from
genetics to personalized medicine. Nat Rev Endocrinol. 2015;11(2):1010-111. PMID:
25385035
76 ANSWER: D) Ophthalmologist
Elevated hemoglobin A1c levels in the double digits are associated
with long-term microvascular complications, as demonstrated in the
Diabetes Control and Complications Trial and in many studies since
then. Recent data in adult populations suggest that the glycemic
excursions are as concerning as elevated hemoglobin A1c levels.
Even with the same hemoglobin A1c level, patients with much wider
glycemic fluctuations have an increased risk of complications.
This vignette is a good example of glycemic variability that can be
simply defined as the degree to which the patient’s blood glucose
level fluctuates between high (peaks) and low (nadir) levels. These
patients are at greater risk of developing microvascular
complications in the short- and long-term. These complications
include, but are not limited to, retinopathy, nephropathy, and
neuropathy.
The risk of retinopathy is far greater than any other complications
at this stage; therefore, the first referral should be to an
ophthalmologist (Answer D). In a published study, the retinopathy
risk doubled when glycemic variation worsened.
Referring this patient to a cardiologist (Answer A) could be
considered because cardiac autonomic neuropathy, defined as
measurement of heart rate variability assessed by 10-minute
continuous electrocardiography, is a potential complication.
However, the patient is not reporting any symptoms and is able to
tolerate exercise.
He is indeed at increased risk of microalbuminuria and
nephropathy, which may require consultation with a nephrologist
(Answer B). However, his urine albumin is detectable but still within
the normal range.
Referral to a neurologist (Answer C) could be considered if he
were experiencing any symptoms consistent with neuropathy, which
he is not.
Podiatrist referral (Answer E) for annual foot examination is also
important, especially for this patient who is an avid soccer player,
but it is not the first priority for children with type 1 diabetes
without relevant clinical concerns.

Educational Objective
Explain the importance of early referral to appropriate subspecialty
care to address potential microvascular complications in an
adolescent with suboptimally controlled type 1 diabetes mellitus.

Reference(s)
Virk SA, Donaghue KC, Cho YH, et al. Association between HbA1c variability and risk of
microvascular complications in adolescents with type 1 diabetes. J Clin Endocrinol
Metab. 2016;101(9):3257-3263. PMID: 27186858
77 ANSWER: B) Cell-surface receptor with intrinsic tyrosine
kinase activity
This patient has classic features of IGF-1 resistance, including poor
prenatal and postnatal growth, short stature, microcephaly, midface
hypoplasia, and global developmental delay. Furthermore, results of
the whole-exome sequencing confirmed a heterozygous pathogenic
variant in the IGF1R gene. The IGF-1 receptor is a heterotetrameric
transmembrane glycoprotein encoded by IGF1R. It consists of an α-
subunit, involved in ligand binding, and a β-subunit, accounting for
the intrinsic tyrosine kinase activity (Answer B). IGF1R is located on
the distal part of the long arm of chromosome 15 (15q26.3). Patients
with pathogenic variants in the IGF1R gene and those who have 15q
deletions that include IGF1R may exhibit clinical signs and
symptoms of IGF-1 resistance. The serum IGF-1 level in these
patients can be normal or high, and the phenotype is variable. GH
therapy has been shown to have moderate effects when used to treat
short stature in these patients.
IGF-1 is one of the important regulators of prenatal growth,
whereas GH does not have a vital role in intrauterine growth.
Table. Hormones Involved in Growth and Mechanisms of Action

Receptor Receptor
Hormone location type Mechanism of action
GHRH Transmembrane G-protein– Increases intracellular cAMP
coupled
receptor
GH Transmembrane Class I JAK2-STAT activation
cytokine
receptor
IGF-1 Transmembrane Tyrosine Tyrosine kinase–mediated receptor
kinase autophosphorylation and phosphorylation of
receptor multiple substrates (IRS 1/2, SHC)
TSH Transmembrane G-protein– Increases intracellular cAMP
coupled
receptor
Thyroid Nucleus Nuclear Regulates gene expression
Hormone receptor

Educational Objective
Describe the physiology of IGF-1 receptor activity and clinical
features of patients with IGF-1 resistance.

Reference(s)
Walenkamp MJE, Robers JML, Wit JM, et al. Phenotypic features and response to GH
treatment of patients with a molecular defect of the IGF-1 receptor. J Clin Endocrinol
Metab. 2019;104(8):3157-3171. PMID: 30848790
Hakuno F, Takahashi S-I. IGF1 receptor signaling pathways. J Mol Endocrinol.
2018;61(1):T69-T86. PMID: 29535161
78 ANSWER: E) Urinary iodine
This infant presents several weeks after birth with severe primary
hypothyroidism that was not documented on newborn screening,
suggesting a postnatally acquired cause. The patient’s mother
reports consuming a significant amount of a traditional Korean soup,
which may contain large amounts of iodine from seaweed that can
be transmitted to the infant through breastmilk. Exposure of infants
to excess iodine, such as from topical iodine antiseptics, iodinated
radiocontrast agents, or maternal iodine ingestion, can cause
acquired hypothyroidism in infancy. In such cases, the urinary
iodine concentration is elevated (Answer E). Excess maternal iodine
ingestion from seaweed soup has been reported to cause
hypothyroidism in term and preterm infants and is particularly
common in Asia. In this case, the infant’s urinary iodine
concentration was 1830 mg/L (regional median 197 mg/L), and the
mother’s breastmilk iodine concentration was 3023 mg/L (regional
median 45-155 mg/L).
Management consists of discontinuing the source of excess
maternal iodine ingestion. There is minimal evidence regarding the
time required to normalize the breastmilk iodine concentration, but
in one reported case, the maternal urinary iodine concentration was
normal 4 weeks after stopping seaweed ingestion. If the infant’s
hypothyroidism is overt (as in this case), temporary levothyroxine
treatment may be necessary.
Increased serum reverse T3 (Answer A) and low serum T3
(Answer C) are observed in consumptive hypothyroidism, which is
caused by massive hepatic hemangiomas that overexpress the type 3
deiodinase enzyme. This infant has no clinical evidence of such a
process, such as hepatomegaly or cutaneous hemangiomas.
Maternal transfer of TSH-receptor antibodies (Answer D) that
block TSH-receptor signaling is a rare cause of congenital
hypothyroidism, which is unlikely in this patient with a normal
newborn screen. In addition, such antibodies generally result in a
small (not normal-sized) thyroid gland by blocking the trophic effect
of TSH on thyroid follicular cells.
TPO antibodies (Answer B) are diagnostic of autoimmune thyroid
disease, which is the most common cause of acquired
hypothyroidism in childhood and adolescence. Autoimmune thyroid
disease generally does not arise in the neonatal period and is
unlikely to be the cause of this patient’s hypothyroidism. Although
maternal TPO antibodies may be transmitted to a fetus across the
placenta, these antibodies are nonfunctional and do not cause
hypothyroidism.

Educational Objective
Identify and diagnose hypothyroidism caused by excessive maternal
iodine intake.

Reference(s)
Shumer DE, Mehringer JE, Braverman LE, Dauber A. Acquired hypothyroidism in an infant
related to excessive maternal iodine intake: food for thought. Endocr Pract.
2013;19(4):729-731. PMID: 23512394
Chung HR, Shin CH, Yang SW, Choi CW, Kim BI. Subclinical hypothyroidism in Korean
preterm infants associated with high levels of iodine in breast milk. J Clin Endocrinol
Metab. 2009;94(11):4444-4447. PMID: 19808851
Emder P, Jack MM. Iodine-induced neonatal hypothyroidism secondary to maternal
seaweed consumption: a common practice in some Asian cultures to promote breast
milk supply. J Pediatr Child Health. 2011;47(10):750-752. PMID: 21276114
79 ANSWER: A) Screen for adrenal insufficiency
Autoimmune polyglandular syndrome (APS) type 1 is a rare
autosomal recessive condition due to loss of function of the
autoimmune regulator gene (AIRE), which results in inappropriate
expression of organ-specific T cells from the thymus. Classic
manifestations include mucocutaneous candidiasis,
hypoparathyroidism (80% with mean age of presentation at 9 years),
and primary adrenal insufficiency (90% with mean age of
presentation of 14 years). Additional endocrine comorbidities may
include oophoritis (70%), testicular failure (30%), type 1 diabetes
mellitus (30%), hypothyroidism (30%), and hypophysitis (30%).
Given her age, weight loss, and symptoms, the most important next
step in this patient’s management is screening for adrenal
insufficiency (Answer A). While the pathophysiology of
hypercalcemia in adrenal insufficiency is unclear, it is reported in 5%
to 6% of new diagnoses. Proposed mechanisms include decreased
glomerular filtration of calcium due to hypovolemia and loss of
glucocorticoid inhibition of 1α-hydroxylase leading to enhanced
enteral calcium absorption.
The pathogenesis of hypoparathyroidism in autoimmune
polyglandular syndrome type 1 is thought to be secondary to
development of antibodies against PTH, the calcium-sensing
receptor, or the NACHT leucinerich repeats present in the
parathyroid gland. Management of hypoparathyroidism can be
especially challenging in patients with gastrointestinal
comorbidities. Classic treatment approaches have focused on
multiple daily doses of enteral calcium and active vitamin D. In
recent years, recombinant PTH has been used to treat adult
hypoparathyroidism and bypass the need for enteral absorption.
Once-daily injections of recombinant 1-84 PTH demonstrate mean
peak serum calcium levels 10 to 12 hours after the dose with
sustained increases above baseline for 24 hours despite the half-life
of the medication being only 3 hours. Recombinant 1-84 PTH is
detected in the serum assay and was already low at the time of
hypercalcemia. Reducing her dosage (Answer D) or discontinuing
calcium carbonate (Answer E), are potential options to address the
hypercalcemia, but neither option addresses the need to rule out
adrenal insufficiency. In addition, given the half-life of calcium
carbonate, this afternoon dose is unlikely to be affecting the morning
serum calcium levels.
While patients with APS type 1 are at increased risk for primary
ovarian insufficiency, this patient has started puberty and is
progressing at a typical tempo given her Tanner stage 4 breast
development. Thus, screening for ovarian insufficiency (Answer B) is
not needed now. Hyperthyroidism is a potential cause of
hypercalcemia and can present with weight loss, fatigue, and
diarrhea; however, the patient does not have tachycardia nor
hypertension. In addition, hyperthyroidism is a feature of APS type
2, not APS type 1. Therefore, thyroid function tests (Answer C) are
not the best next step.

Educational Objective
Describe the presentation of adrenal insufficiency in autoimmune
polyglandular syndrome type 1 and explain how hypercalcemia can
be a manifestation.

Reference(s)
Winer KK, Kelly A, Johns A, et al. Long-term parathyroid hormone 1-34 replacement
therapy in children with hypoparathyroidism. J Pediatr. 2018;203:391-399.e1. PMID:
30470382
Tuli G, Buganza R, Tessaris D, Einaudi S, Matarazzo P, de Sanctis L. Teriparatide (rhPTH 1-
34) treatment in the pediatric age: long-term efficacy and safety data in a cohort with
genetic hypoparathyroidism. Endocrine. 2020;67(2):457-465. PMID: 31705387
Winer KK. Advances in the treatment of hypoparathyroidism with PTH 1-34. Bone.
2019;120:535-541. PMID: 30243992
80 ANSWER: B) Increase in weight percentile without
appreciable changes in appetite or calories
Prader-Willi syndrome (PWS) is a complex imprinted condition with
the majority of cases due to deletions in the 15q11.2-q13 region of the
paternally inherited chromosome. Obesity, intellectual disability,
hypotonia, behavioral challenges, and neuroendocrine dysfunction
have been commonly described. In the past, it was thought that there
were only 2 nutritional phases associated with PWS (stage 1 with
hypotonia and poor feeding in infancy, followed by stage 2 with
hyperphagia and rapid weight gain in childhood). However, the
nutritional stages are now known to be much more complex than
previously recognized. Five main stages have been described with
subphases in phase 1 and 2.
Infants with PWS are characteristically hypotonic and have
difficulty feeding (Answer E). This phase (phase 1a) can occur
between birth and 15 months of age. Infants in this stage may have
failure to thrive as a result of feeding difficulties. In late infancy
(between 9 and 15 months), there is phase 1b in which infants track
along their growth percentile without excess weight gain (Answer
D).
Toddlers (median age 2.08 years; age quartiles 20-31 months),
such as the patient in this vignette, are most typically described as
being in phase 2a in which there is a weight increase that occurs
without a change in appetite or calories consumed (Answer B).
Whether this weight gain is due to a change in metabolic rate and/or
other factors is not known. To avoid excess weight gain at this stage,
it is thought that calories consumed must be restricted to 50% to 80%
of the recommended daily allowance for age.
In phase 2b, which occurs at a median age of 4.5 years, there is an
increasing interest in food, coupled with increased caloric intake and
weight gain (Answer C).
Hyperphagia with food-seeking and lack of satiety (Answer A)
occurs later than previously thought at a median age of 8 years
(phase 3).
It is important to note that not all children with PWS go through
all 5 stages, but patients and families should be informed of the
possible stages and counseled accordingly regarding appropriate
management.
Table. Nutritional Phases Associated With Prader-Willi Syndrome

5 nutritional phases (with subphases)


associated with Prader-Willi syndrome Expected clinical course
Phase 0 (in utero) Decreased fetal movement, lower birth
weight
Phase 1a (infancy) Hypotonia, difficulty feeding, failure to thrive
Phase 1b (late infancy) Maintenance of weight percentile along the
growth curve
Phase 2a (median age 2.08 years; age Increase in weight percentile without
quartiles 20-31 months) change in appetite or calories consumed
Phase 2b (median age of 4.5 years) Increase in weight percentile associated
with an increased interest in food
Phase 3 (median age 8 years) Hyperphagia with food-seeking and lack of
satiety
Phase 4 (adulthood) Improvement in appetite control, not as
preoccupied with food

Educational Objective
Explain the major nutritional phases associated with Prader-Willi
syndrome.

Reference(s)
Miller JL, Lynn CH, Driscoll DC, et al. Nutritional phases in Prader-Willi syndrome. Am J
Med Genet A. 2011;155A(5):1040-1049. PMID: 21465655
Goldstone AP, Holland AJ, Hauffa BP, Hokken-Koelega AC, Tauber M; speakers
contributors at the Second Expert Meeting of the Comprehensive Care of Patients with
PWS. Recommendations for the diagnosis and management of Prader-Willi syndrome. J
Clin Endocrinol Metab. 2008;93(11):4183-4197. PMID: 18697869
81 ANSWER: A) Arginine inhibits somatostatin
Many secretagogues can be used for provocative GH-stimulation
testing, including insulin, arginine, glucagon, propranolol, clonidine,
and levodopa. GHRH and exercise can also stimulate GH release.
Provocative testing is by nature nonphysiologic and the precise
mechanisms by which these agents stimulate GH are not fully
understood. However, many affect GH release by suppressing
somatostatin or stimulating GHRH release.
Pulsatile GH secretion is in part regulated by the hypothalamic
regulatory peptides GHRH and somatostatin (somatotropin release–
inhibiting factor). GHRH release is stimulated by α2 adrenergic
agonists, including clonidine, dopaminergic agonists (eg, levodopa),
and exercise.
Somatostatin secretion is stimulated by GHRH, dopaminergic, β-
adrenergic stimulation, and GH and IGF-1 feedback. Somatostatin is
inhibited by arginine, cholinergic GABAergic stimulation, and
insulin-induced hypoglycemia.
Arginine appears to cause GH release by inhibiting somatostatin
and thereby increasing GHRH activity (Answer A). Glucagon
(Answer B) may work by inducing a relative hypoglycemia
following hyperglycemia, although the exact mechanism is not fully
understood. Propranolol (Answer C) is a β-adrenergic antagonist,
which would be expected to inhibit somatostatin. Clonidine (Answer
D) stimulates α2 adrenergic receptors, which would increase GHRH.
Levodopa (Answer E) most often increases GHRH.

Educational Objective
Explain the effects and mechanism of action of levodopa, clonidine,
glucagon, propranolol, and arginine hydrochloride during testing
for GH secretory capacity.

Reference(s)
Melmed S, Polonsky SK, Larense PR, Kronenberg HM, eds. Williams Textbook of
Endocrinology. 13th ed. Elsevier; 2016.
Rosenfeld RG, Albertsson-Wikland K, Cassorla F, et al. Diagnostic controversy: the
diagnosis of childhood growth hormone deficiency revisited. J Clin Endocrinol Metab.
1995;80(5):1532-1540. PMID: 7538145
82 ANSWER: A) SGPL1 (sphingosine-1-phosphate lyase 1)
Pathogenic variants in the SGPL1 gene (Answer A) have recently
been identified as the cause of a new syndrome of primary adrenal
insufficiency associated with nephrotic syndrome and multiple
endocrinopathies. Pathogenic variants in MRAP (Answer B), POR
(Answer C), AIRE (Answer D), and AAAS (Answer E) can all give
rise to primary adrenal insufficiency, but they are not associated with
nephrotic syndrome.
Since the first description in 2017, more than 35 patients carrying
a pathogenic variant in the SGPL1 gene in the homozygous or
compound heterozygous state have been reported worldwide. The
clinical presentation and phenotypes are variable. Most affected
patients have steroid-resistant nephrotic syndrome presenting from
birth to 19 years of age. Just over half of all those with SGPL1
pathogenic variants have glucocorticoid deficiency (with or without
mineralocorticoid deficiency). Primary adrenal insufficiency can
present from infancy to age 11 years. Some patients have adrenal
calcification on imaging. Associated features reported include
ichthyosis, neurologic involvement, and lymphopenia. Neurologic
involvement is diverse and ranges from early developmental delay,
microcephaly, seizures, and sensorineural deafness to late
presentation with abnormal gait, peripheral neuropathy, and
progressive neurologic deficit. In addition to primary adrenal
insufficiency, multiple endocrinopathies such as primary
hypothyroidism and gonadal dysfunction have also been described.
Dysmorphic features and bone defects have been reported in a few
patients. Fetal demise and hydrops fetalis have also been noted in
some families.
Deficiency of the SGPL1 protein represents a novel form of
sphingolipidosis. Other forms include Fabry disease, Gaucher
disease, and Niemann-Pick disease. The SGPL1 protein is an
intracellular enzyme that catalyses the breakdown of sphingolipids
by cleaving substrate, sphingosine-1-phosphate (S1P), in the final
step in the sphingolipid degradation pathway. S1P is a bioactive
sphingolipid metabolite that acts on the family of 5 G-protein–
coupled receptors to mediate diverse biologic activities, including
angiogenesis, immune-cell trafficking, stem-cell differentiation, and
programmed cell death. Inactive SGPL1 leads to an accumulation of
sphingolipids and ceramides. Accumulation of ceramides induces
apoptosis, while sphingosine is a ligand for steroidogenic factor 1,
inhibiting steroidogenesis by maintaining steroidogenic factor 1 in
an inactive conformation. More recently, SGPL1 deficiency has been
shown to be associated with mitochondrial dysfunction, which may
contribute to the pathophysiology of this disorder.

Educational Objective
Identify a new form of syndromic primary adrenal insufficiency due
to pathogenic variants in the SGPL1 gene.

Reference(s)
Prasad R, Hadjidemetriou I, Maharaj A, et al. Sphingosine-1-phosphate lyase mutations
cause primary adrenal insufficiency and steroid-resistant nephrotic syndrome. J Clin
Invest. 2017;127(3):942-953. PMID: 28165343
Lovric S, Goncalves S, Gee HY, et al. Mutations in sphingosine-1-phosphate lyase cause
nephrosis with ichthyosis and adrenal insufficiency. J Clin Invest. 2017;127(3):912-928.
PMID: 28165339
Maharaj A, Theodorou D, Banerjee II, Metherell LA, Prasad R, Wallace D. A sphingosine-1-
phosphate lyase mutation associated with congenital nephrotic syndrome and multiple
endocrinopathy. Front Pediatr. 2020;8:151. PMID: 32322566
Lucki NC, Sewer MB. Nuclear sphingolipid metabolism. Annu Rev Physiol. 2012;74:131-151.
PMID: 21888508
Maharaj A, Williams J, Brashaw T, et al. Sphingosine-1-phosphate lyase (SGPL1) deficiency
is associated with mitochondrial dysfunction. J Steroid Biochem Mol Biol.
2020;202:105730. PMID: 32682944
83 ANSWER: E) Methylation analysis of chromosome 11p15
This child has significant short stature with a history of prenatal
growth restriction. Target height (midparental height) is at the 25th
percentile. This, and the severity of his short stature, makes familial
short stature an unlikely cause of his growth pattern.
Hypothyroidism, inflammatory bowel disease, and celiac disease are
also unlikely given his normal thyroid function, normal erythrocyte
sedimentation rate, and negative celiac screen. His IGF-1
concentration is in the midpoint of the reference range for age, and
his IGFBP-3 concentration is in the upper half of the reference range,
which is not consistent with GH deficiency. Therefore, an arginine-
insulin tolerance test (Answer A) for diagnostic purposes is not the
best option. Additionally, this child was small-for-gestational-age
and did not display “catch-up” growth during the first 4 years of
life. He is a candidate for GH therapy based on the diagnosis of
being small-for-gestational-age without catch-up growth. A GH-
stimulation test would not be required to receive GH therapy.
Although this child is described as having café-au-lait spots,
which can be seen in neurofibromatosis type 1 (Answer B), the other
clinical manifestations in this case, particularly the significant short
stature, are not typical of this condition.
Prenatal and postnatal growth restriction have also been
described in individuals with defects in the gene encoding the IGF-1
receptor (Answer D). Elevated IGF-1 levels are described in these
individuals and, although not present in all, some dysmorphic
features such as microcephaly (as opposed to relative macrocephaly
described in this vignette) are seen. Therefore, sequencing this gene
is incorrect.
The child in this vignette has clinical manifestations consistent
with Silver-Russell syndrome, including prenatal and postnatal
growth retardation, prominent forehead, triangular face, asymmetric
growth of the extremities, and café-au-lait spots. Multiple types of
Silver-Russell syndrome have been reported to date. Approximately
50% of affected children carry an epimutation consisting of
hypomethylation of the imprinting control region 1 (ICR1) at the
IGF2/H19 locus on chromosome 11p15 (SRS type 1), while about
10% of affected children have maternal uniparental disomy of
chromosome 7 (SRS type 2). A smaller proportion of cases are due to
structural aberrations of the short arm of chromosome 11 (SRS type
3). Other less common types of Silver-Russell syndrome have been
described with defects on chromosome 8q12.1 (SRS type 4) and
12q14.3 (SRS type 5). Even an X-linked form has been described.
Children with SRS type 1 (caused by hypomethylation of the
ICR1 on chromosome 11p15) seem to have the most severe
phenotype with smaller size at birth and no significant change in
height SDS postnatally when compared with children with SRS type
2 (caused by maternal uniparental disomy of chromosome 7) who
have a longer birth length but show a decline in height SDS
postnatally. Additionally, children with SRS type 1 (11p15
epimutation) have significantly higher levels of IGF-1 and IGFBP-3
(usually within normal range for age) and these levels increase
significantly after GH treatment. This suggests a certain degree of
IGF-1 resistance. Children with SRS type 2 (uniparental disomy of
chromosome 7) typically have IGF-1 and IGFBP-3 levels that are low
for age. The child in this vignette has IGF-1 and IGFBP-3 levels in the
normal range, making hypomethylation of the ICR1 on chromosome
11p15 (Answer E) more likely than maternal uniparental disomy of
chromosome 7 (Answer C).

Educational objective
Identify the clinical manifestations of Silver-Russell syndrome and
recommend testing to confirm the diagnosis.

Reference(s)
Price SM, Stanhope R, Garrett C, Preece MA, Trembath RC. The spectrum of Silver-Russell
syndrome: a clinical and molecular genetic study and new diagnostic criteria. J Med
Genet. 1999;36(11):837-842. PMID: 10544228
Wakeling EL. Silver-Russell syndrome. Arch Dis Child. 2011;96(12):1156-1161. PMID:
21349887
Binder G, Seidel AK, Martin DD, et al. The endocrine phenotype in Silver-Russell syndrome
is defined by the underlying epigenetic alteration. J Clin Endocrinol Metab.
2008;93(4):1402-1407. PMID: 18230663
Abuzzahab MJ, Schneider A, Goddard A, et al. IGF-I receptor mutations resulting in
intrauterine and postnatal growth retardation. N Engl J Med. 2003;349(23):2211-2222.
PMID: 14657428
84 ANSWER: E) MT-TL1 genetic testing
The case presented in this vignette is characterized by a maternal
inheritance pattern and neurosensory deafness that is consistent
with maternally inherited (mitochondrial) diabetes and deafness
(MIDD). The diagnosis could be confirmed with MT-TL1 genetic
testing (Answer E). The most common MT-TL1 pathogenic variant
associated with this condition is 3243 A>G, but other mitochondrial
DNA variants have been linked with a diabetic phenotype
suggestive of MIDD.
Wolfram syndrome can present with diabetes as one of its earliest
manifestations. Also known as DIDMOAD (diabetes insipidus,
diabetes mellitus, optic atrophy, and deafness), Wolfram syndrome is
characterized by early-onset diabetes mellitus and optic atrophy
(<16 years) with variable presence of neuropsychiatric features,
sensorineural hearing loss, and other endocrine features such as
diabetes insipidus, hypogonadism, short stature, and/or
hypothyroidism. It is caused by autosomal recessive pathogenic
variants in the WFS1 gene (Answer D), which is not consistent with
the details in the vignette in terms of inheritance pattern or
presenting clinical features.
Monogenic diabetes (maturity-onset diabetes of the young
[MODY]) type 3 or type 1, due to pathogenic variants in the HNF1A
and HNF4A genes (Answer C), respectively, could be suspected with
the autosomal dominant inheritance pattern in the vignette.
However, neurosensory hearing loss and learning disability are not
associated features of HNF1A or HNF4A pathogenic variants.
All the major type 1 diabetes autoantigens are related to the
secretory apparatus of the β cell. Assessing zinc tranporter-8
antibodies (Answer A) could be a logical next step in addition to the
3 antibodies tested in the vignette (all undetectable), but this will not
lead to the correct diagnosis since the patient has additional clinical
clues that should make the learner consider an alternative diagnosis
beyond type 1 diabetes. HLA typing (Answer B) is neither cost-
effective nor useful in establishing the diagnosis of type 1 diabetes
and should not be pursued.

Educational Objective
Consider rare forms of genetic diabetes in the differential diagnosis
of diabetes mellitus.

Reference(s)
Bingley PJ. Clinical applications of diabetes antibody testing. J Clin Endocrinol Metab.
2010;95(1):25-33. PMID: 19875480
Yeung RO, Hanna-Shmouni F, Niederhoffer K, Walker MA. Not quite type 1 or type 2, what
now? Review of monogenic, mitochondrial, and syndromic diabetes. Rev Endocr Metab
Disord. 2018;19(1):35-52. PMID: 29777474
Mazzaccara C, Iafusco D, Liguori R, et al. Mitochondrial diabetes in children: seek and you
will find it. PLoS One. 2012;7(4):e34956. PMID: 22536343
85 ANSWER: C) Initiate levothyroxine to maintain TSH <0.1
mIU/L; 2 to 3 months after surgery, measure TSH-stimulated
thyroglobulin and perform diagnostic whole-body 123I scan
The key to answering this question is determining the patient’s risk
category and recognizing the importance of postoperative staging in
guiding further management. The patient’s risk category is based on
the 2015 American Thyroid Association (ATA) Management
Guidelines for Children with Thyroid Nodules and Differentiated
Thyroid Cancer, which primarily address papillary thyroid cancer.
The risk is defined as the likelihood of having persistent cervical
disease and/or distant metastases after initial total thyroidectomy
with or without lymph node dissection by a high-volume surgeon.
The degree of risk is determined by the spread of cancer to cervical
lymph nodes (N1a in the central neck compartment, or N1b in the
lateral neck) or local spread beyond the thyroid (locally invasive T4
tumors), with or without distant metastases.
Once the risk category is determined, one can follow the
guidelines for recommended management and monitoring. This
patient had a central and right lateral neck dissection based on
preoperative imaging and was found to have extensive N1b disease.
Therefore, she has an ATA pediatric high-risk tumor. While it is true
that many pediatric patients with high-risk papillary thyroid cancer
require 131I, not all do. Postoperative staging includes both TSH-
stimulated thyroglobulin measurement and a 123I whole-body scan,
as detailed in the pediatric guidelines.
Whole-body scanning helps determine whether there is any
cervical uptake outside of the thyroid bed (metastatic
lymphadenopathy) or any distant metastases. Whole-body scanning
may miss pulmonary micrometastases, which only show up on post-
131I scan. When a high-volume thyroid surgeon performs the
thyroidectomy, the odds of any significant thyroid remnant are
reduced. Documenting a stimulated thyroglobulin value is helpful in
patients without cervical or distant metastases on whole-body
scanning.
A low stimulated thyroglobulin value (<2 ng/mL [<2 µg/L],
assuming negative thyroglobulin antibodies) helps identify which
patients do not require 131I. Those with a thyroglobulin
concentration greater than 10 ng/mL (>10 µg/L) should have 131I
ablation, followed by a posttreatment scan. Those with an in-
between thyroglobulin value (2-10 ng/mL [2-10 µg/L]) may benefit
from 131I ablation, especially if the tumor has aggressive features
such as vascular invasion or is a locally invasive (T4) tumor. Patients
with distant metastases require 131I radioablation, but it may be
delayed if resectable cervical disease remains.
Chest CT (Answer A), with its associated radiation, is not
recommended for all patients at high risk. Furthermore, it may miss
pulmonary micrometastases. While it is recommended to maintain
TSH less than 0.1 mIU/L in patients at high risk, as one cannot rely
solely on unstimulated thyroglobulin measurement at this stage, a
whole-body scan is also necessary (thus, Answer B is incorrect).
Holding off on levothyroxine replacement right after surgery and
administering 131I once the TSH concentration is greater than 30
mIU/L is incorrect, as it assumes that all patients at high risk should
undergo radioablation, without postoperative staging, which is not
the case. This approach could fail to identify some resectable disease,
which may render the 131I less effective in ablating any distant
metastases. Furthermore, thyroid cancer is slow-growing, and there
is no rush to ablate 2 to 3 weeks after thyroidectomy when the
surgical site may be still healing (thus, Answer D is incorrect).
Initiating levothyroxine to maintain TSH between 0.1 and 0.5
mIU/L and performing postoperative 123I whole-body scan
(Answer E) is incorrect as well, as it does not include a stimulated
thyroglobulin measurement as part of the postoperative staging.
While the recommended TSH suppressive level for patients at high
risk is less than 0.1 mIU/L, some endocrinologists choose to
temporarily keep their patients’ TSH concentration less than 0.5
mIU/L ahead of the TSH-stimulated thyroglobulin measurement
and whole-body scan, and thus shorten their levothyroxine
withdrawal time. After 131I is given, it is important to maintain the
TSH concentration less than 0.1 mIU/L.
Initiating levothyroxine to maintain the TSH concentration below
0.1 mIU/L and measuring TSH-stimulated thyroglobulin and
performing a diagnostic whole-body 123I scan 2 to 3 months after
surgery (Answer C) is the correct management. This approach
encompasses the proper postoperative staging and the suppressive
TSH level recommended by the pediatric guidelines and does not
assume that 131I should be given to all patients at high risk. The use
of 131I should be guided by postoperative staging.

Educational Objective
Explain the importance of postoperative staging in high-risk
papillary thyroid cancer in an adolescent.

Reference(s)
Francis GL, Waguespack SG, Bauer AJ, et al; American Thyroid Association Guidelines Task
Force. Management guidelines for children with thyroid nodules and differentiated
thyroid cancer. Thyroid. 2015;25(7):716-759. PMID: 25900731
86 ANSWER: C) Start GH replacement therapy at 0.2 mg/kg per
week
This patient’s height is more affected than his weight, and he has a
normal caloric intake. He has no signs or symptoms of
malabsorption. His laboratory test results are concerning for possible
GH deficiency. Thus, it is not appropriate to provide reassurance
(Answer A) or wait until gastrointestinal evaluation is completed
(Answer B). He had normal thyroid function and adrenal function
when tested, so it is not necessary to repeat these labs again (Answer
E). His clinical picture of normal prenatal growth, poor postnatal
growth, episodes of hypoglycemia, proportional short stature
without dysmorphic features, and delayed bone age plus his
laboratory results (low-normal IGF-1, low stimulated GH
concentration, otherwise normal pituitary workup) indicate that he
has isolated GH deficiency. Patients with isolated GH deficiency are
generally started on GH treatment at a dosage of 0.18 to 0.25 mg/kg
per week (to minimize the risk of adverse effects), which is then
titrated based on growth response. This dosage is lower than the
starting dosage for other indications such as idiopathic short stature,
small-for-gestational-age with poor catch-up growth, or conditions
such as Turner syndrome or Noonan syndrome. The starting dosage
of 0.35 mg/kg per week (Answer D) is too high. There is a family
history of short stature on the paternal side (the father’s first cousin).
Thus, it is prudent to consider genetic testing to evaluate for genetic
causes of GH deficiency, to assess whether this patient is at risk for
developing multiple pituitary hormone deficiency in the future, and
to understand the need to test other family members. Therefore, the
best strategy is to start GH replacement therapy at a dosage of 0.2
mg/kg per week and consider genetic testing (Answer C).
Several patients with short stature inherited in an autosomal
dominant manner have been shown to have a missense pathogenic
variant (p.A204E) resulting in complete loss of constitutive activity
of the GH secretagogue receptor. The GH secretagogue receptor is a
7-transmembrane G-protein–coupled receptor that is highly
expressed in the pituitary and hypothalamus (also known as the
ghrelin receptor, as it binds to and is activated by ghrelin). This
patient was reported to have compound heterozygous pathogenic
variants in the gene encoding the GH secretagogue receptor (GHSR).
Patients with GHSR pathogenic variants are known to have delayed
puberty as well (as reported in his father). The patient responded
well to GH therapy and had delayed pubertal onset at age 15 years.
Although he continued to have 1 to 2 episodes of vomiting and
abdominal pain every year, he did not have further episodes of
hypoglycemia while on GH therapy.

Educational Objective
Identify the diagnostic challenges in a child with short stature and
recall the recommended GH dosage for treating patients with GH
deficiency.

Reference(s)
Grimberg A, DiVall SA, Polychronakos C, et al; Drug and Therapeutics Committee and
Ethics Committee of the Pediatric Endocrine Society. Guidelines for growth hormone
and insulin-like growth factor-I treatment in children and adolescents: growth hormone
deficiency, idiopathic short stature, and primary insulin-like growth factor-I deficiency.
Horm Res Paediatr. 2016;86(6):361-397. PMID: 27884013
87 ANSWER: E) Hypothalamic hamartoma
Precocious puberty, defined as breast development before age 8
years in girls, has been increasing in prevalence since age cutoffs
were established in the 1960s. The percentage of girls with
precocious puberty is now approximately 10%, with central
precocious puberty accounting for 90% to 95% of cases. The
prevalence of central nervous system abnormalities is much lower in
girls with central precocious puberty than in boys and decreases
with age. In one recent meta-analysis, the pooled prevalence of
intracranial lesions was 25% in girls younger than 6 years (n = 173)
and 3% in girls aged 6 to 8 years (n = 514). In this pool of 1853
individuals, the incidence of tumors was 1.6%.
Hypothalamic hamartomas are considered to be the most
common cause of central precocious puberty among girls with
central nervous system lesions. These are rare, congenital, benign
lesions located in the ventral hypothalamus that are highly
associated with gelastic seizures and treatment-resistant epilepsy.
The girl in this vignette has central precocious puberty as indicated
by the presence of Tanner stage 3 breast development and pubertal
gonadotropins. She has a history suspicious for gelastic seizures,
which are primarily manifested by sudden outbursts of laughter or
crying for no apparent reason and lasting less than a minute. Given
her presentation of central precocious puberty with probable gelastic
seizures, she most likely has central precocious puberty due to a
hypothalamic hamartoma (Answer E).
While idiopathic central precocious puberty is more common
than all other causes of central precocious puberty at this age, the
presence of gelastic seizures makes idiopathic (Answer C) or genetic
causes of central precocious puberty less likely. Many cases of central
precocious puberty previously considered idiopathic now have
known genetic etiologies, with variants in the MKRN3 gene (Answer
A) being the most common. MKRN3 encodes the Makorin ring
finger protein 3, a protein that normally functions to inhibit the
release of hypothalamic GnRH. Variants in the MKRN3 gene result
in loss of GnRH inhibition in childhood, which leads to central
precocious puberty. Variants in the GNAS gene (Answer B) causing
constitutive activation of the G-protein subunit Gsα result in
gonadotropin-independent precocious puberty as part of McCune-
Albright syndrome. As the patient in this vignette has central
precocious puberty, McCune-Albright syndrome is an unlikely
diagnosis.
Craniopharyngiomas (Answer D) are also associated with central
precocious puberty. As they expand, craniopharyngiomas can
disrupt pituitary function and cause central precocious puberty.
However, they most commonly affect children between 5 and 14
years of age and are typically associated with findings such as visual
field deficits, nausea and vomiting, headaches, growth failure, and
diabetes insipidus.
Subsequent workup was notable for a 10-mm lesion consistent
with a hypothalamic hamartoma found on MRI and
electroencephalography findings of occasional bursts (1-2 seconds)
of very high-voltage, semirhythmic delta activity consistent with
gelastic seizures, which have been well controlled with
anticonvulsant medication. While hypothalamic hamartomas are
rare (1:200,000 to 1:600,000), gelastic seizures and precocious puberty
are common manifestations. Other endocrine manifestations,
including GH deficiency and Cushing syndrome, are rarely reported
in this setting. Surgery is not usually required; increased appetite
and obesity are common long-term postoperative complications in
individuals who require surgical resection.

Educational Objective
Identify gelastic seizures as part of the presentation of central
precocious puberty due to a hypothalamic hamartoma.

Reference(s)
Cantas-Orsdemir S, Garb JL, Allen HF. Prevalence of cranial MRI findings in girls with
central precocious puberty: a systematic review and meta-analysis. J Pediatr Endocrinol
Metab. 2018;31(7):701-710. PMID: 29902155
Harrison VS, Oatman O, Kerrigan JF. Hypothalamic hamartoma with epilepsy: review of
endocrine comorbidity. Epilepsia. 2017;58(Suppl 2):50-59. PMID: 28591479
88 ANSWER: A) Start GH therapy
The child described in this vignette fulfills the criteria to be classified
as being small-for-gestational-age based on both weight and length
at birth. The etiology of this problem includes placental, fetal,
maternal, and environmental factors. Nevertheless, no clear etiology
is found in approximately 40% of the cases. Most children born
small-for-gestational-age (~90%) undergo linear growth acceleration
in the first year of life, so-called catch-up growth that is mostly
completed by 2 years of age. However, approximately 10% of
children born small-for-gestational-age do not display such
acceleration and fail to catch up. GH therapy (Answer A) has been
approved by the US FDA for the management of short stature in
children born small-for-gestational-age when no catch-up growth
has occurred by 2 years of age. The European Medicines Agency has
also approved the use of GH therapy in these children starting at 4
years of age. The response to GH therapy in these children is clearly
superior, not only in terms of growth velocity but in terms of adult
height, when started early (2-4 years of age). Thus, the best course of
action is to start GH therapy for this patient.
Data from large-scale international databases for surveillance of
efficacy and safety of GH treatment in children with growth
disorders show that, on average, physicians initiate GH therapy in
children born small-for-gestational-age without catch-up growth at a
much older than optimal age (around 8-9 years of age) due to
multiple reasons, including parental and primary care physician
biases. Late initiation of GH therapy in these children would lead to
suboptimal outcomes and be unfavorable from the cost-effective
analysis perspective given that larger dosages are needed for larger
children. Thus, simply following this patient clinically until age 5
years (Answer E) is incorrect.
A marginal TSH elevation in the presence of normal free T4 is
unlikely to explain the degree of short stature in this child.
Therefore, assessing thyroid autoantibodies (Answer B) is not
warranted. Additionally, autoimmune thyroid disease (Hashimoto
thyroiditis) usually leads to acquired hypothyroidism and the
impact on linear growth would not be manifested since birth.
Classic GH deficiency is rare in children born small-for-
gestational-age. However, IGF-1 and IGFBP-3 are somewhat reduced
in children born small-for-gestational-age (by about 1 SD), as is the
case with the patient in this vignette. These levels are not suggestive
of GH deficiency. In addition, in the absence of clinical or
biochemical data suggestive of GH deficiency, the performance of a
definitive test to exclude this condition (GH-stimulation test)
(Answer C) is not required to initiate therapy in these children.
Although this patient’s linear growth did not show the expected
acceleration seen in approximately 90% of children born small-for-
gestational-age, the ponderal growth was not unreasonable, as it
progressed from well below the 3rd percentile at birth to about the
8th percentile by 35 months of age. The linear growth failure is not
linked to poor weight gain and a referral to gastroenterology
(Answer D) is not the best answer.

Educational Objective
List the indications for GH therapy in children born small-for-
gestational-age who do not display catch-up growth in the first 2 to 4
years of life.

Reference(s)
Houk CP, Lee PA. Early diagnosis and treatment referral of children born small for
gestational age without catch-up growth are critical for optimal outcomes. Int J Pediatr
Endocrinol. 2012(1):11. PMID: 22559301
Clayton PE, Cianfarani S, Czernichow P, Johannsson G, Rapaport R, Rogol A. Management
of the child born small for gestational age through adulthood: a consensus statement of
the International Societies of Pediatric Endocrinology and the Growth Hormone
Research Society. J Clin Endocrinol Metab. 2007;92(3):804-810. PMID: 17200164
89 ANSWER: C) Severity of hypothyroidism at diagnosis
Hearing impairment is more common in children with congenital
hypothyroidism than in the general population. One population-
based study reported the presence of hearing impairment in 9.5% of
patients with congenital hypothyroidism. In this study, hearing
impairment in patients with congenital hypothyroidism was most
often diagnosed between 3 and 19 years of age (median 7 years).
Hearing deficits may be detected on neonatal hearing screening (if
performed), but not all cases will be detected in this way. Hearing
impairment in congenital hypothyroidism is mild to moderate in
most cases, and severe in about 10%. For these reasons, long-term
management of patients with congenital hypothyroidism should
include careful attention to speech and language development,
delays in which may indicate underlying hearing deficits. Formal
hearing testing should be recommended for any patient in whom
hearing impairment is suspected.
The severity of congenital hypothyroidism at diagnosis (Answer
C) is a risk factor for associated hearing impairment. This may be
related to the key role of thyroid hormone in cochlear development
in the prenatal period. Patients with ectopic thyroid have a lower
risk of hearing impairment than those with thyroid agenesis or a
eutopic thyroid gland, perhaps because patients with thyroid ectopy
generally have milder hypothyroidism. Postnatal treatment factors,
including timing of treatment initiation (Answer A), rate of thyroid
function normalization (Answer B), or treatment adequacy during
childhood (Answer D), do not appear to be related to the risk of
hearing impairment. This risk is also unrelated to sex (Answer E).
Sensorineural hearing loss in a patient with congenital
hypothyroidism and a eutopic thyroid gland can be observed in
Pendred syndrome, a rare recessive disorder caused by inactivating
pathogenic variants in the pendrin gene (SLC26A4). Most patients
with Pendred syndrome have goiter, which may not be evident until
adolescence or adulthood, and are euthyroid or have mild
hypothyroidism, unlike the patient in this vignette. However, it is
important to recognize that patients with congenital hypothyroidism
of any etiology, particularly if severe, are at increased risk for
hearing deficits and require close monitoring and prompt evaluation
for hearing concerns.

Educational Objective
Identify risk factors and appropriate screening for hearing
impairment in patients with congenital hypothyroidism.

Reference(s)
Lichtenberger-Geslin L, Dos Santos S, Hassani Y, Ecosse E, Van Den Abbeele T, Leger J.
Factors associated with hearing impairment in patients with congenital hypothyroidism
treated since the neonatal period: a national population-based study. J Clin Endocrinol
Metab. 2013;98(9):3644-3652. PMID: 23780375
Léger J, Olivieri A, Donaldson M, et al. European Society for Paediatric Endocrinology
consensus guidelines on screening, diagnosis, and management of congenital
hypothyroidism. J Clin Endocrinol Metab. 2014;99(2):363-384. PMID: 24446653
90 ANSWER: E) TSH-producing adenoma
This girl presents with signs and symptoms of hyperthyroidism,
goiter, elevated free T4 and total T3 levels, and an inappropriately
elevated TSH level. When approaching the evaluation of
inappropriate TSH secretion and elevated thyroid hormone levels, it
is important to rule out genetic causes and interference from
medications such as estrogen. Once medication or other laboratory
interference has been ruled out, measurement of thyroid function in
first-degree relatives is important. This girl is taking an oral
contraceptive pill, which most likely contains estrogen. While total
T4 levels may be elevated due to an increase in thyroxine-binding
globulin seen with estrogen, the free T4 level would be normal, as
would the TSH level. In addition, this patient appears clinically
hyperthyroid. Therefore, medication interference causing abnormal
laboratory findings (Answer A) is incorrect.
Graves disease (Answer D) is a common cause of
hyperthyroidism and, in fact, this patient’s mother had Graves
disease. Measurement of thyroid-stimulating immunoglobulins and
TSH-binding inhibitory immunoglobulins can be helpful. However,
in this setting, the TSH level is expected to be suppressed, so Graves
disease is incorrect.
The syndrome of familial dysalbuminemic hyperthyroxinemia
(Answer C) presents with elevated total and normal or elevated free
T4 serum values in clinically euthyroid patients with normal TSH
levels (euthyroid hyperthyroxinemia). This is an inherited
abnormality (autosomal dominant) characterized by the presence of
a variant serum albumin that has preferential affinity for T4. The
hyperthyroxinemia can be confused with hyperthyroidism. This girl
is clinically hyperthyroid, has a goiter, and has elevated TSH.
Therefore, familial dysalbuminemic hyperthyroxinemia is incorrect.
Both TSH-secreting adenomas (Answer E) and resistance to
thyroid hormone β (Answer B) have elevated thyroid hormones in
common and can present with goiter, but the peripheral response to
the hyperthyroxinemia is different. Resistance to thyroid hormone β
is due to abnormalities in the thyroid hormone receptor β gene
(TRHB) and is generally inherited in an autosomal dominant pattern.
It can be difficult to distinguish between these 2 entities, highlighting
the importance of assessing thyroid function in first-degree relatives.
TSH-secreting pituitary adenomas are a very rare cause of
hyperthyroidism with inappropriate TSH elevations and are even
rarer in the pediatric population. In most cases, these adenomas are
located in the pituitary, but a few instances of ectopic TSH
production have been reported in the suprasellar region and
nasopharynx. TSH-secreting pituitary adenomas are the rarest of the
pituitary adenomas and can be either microadenomas or
macroadenomas and may be found incidentally on imaging for other
causes. In patients with TSH-secreting pituitary adenomas,
cosecretion of GH and prolactin is common. Interestingly, although
no values are given for GH or prolactin levels in this vignette, the
patient is much taller than her midparental height, suggesting the
possibility of excess GH secretion. Measurement of other pituitary
function is essential. MRI would be particularly important to
perform in this individual.
A thyrotropin-releasing hormone test may be helpful to
distinguish between a TSH-secreting pituitary adenoma and
resistance to thyroid hormones. TSH rises appropriately after
thyrotropin-releasing hormone stimulation in patients with
resistance to thyroid hormone but not in most patients with TSH-
secreting pituitary adenomas. Measurement of serum α-subunit may
also be a helpful diagnostic tool and is elevated in approximately
30% of individuals with TSH-secreting pituitary adenomas.

Educational Objective
Diagnose a TSH-secreting pituitary adenoma and explain the
physiologic effects of TSH.

Reference(s)
Tjörnstrand A, Nyström HF. Diagnosis of endocrine disease: diagnostic approach to TSH-
producing pituitary adenoma. Eur J Endocrinol. 2017;177(4):R183-R197. PMID: 28566440
Stanley JM, Najjar SS. Hyperthyroidism secondary to a TSH-secreting pituitary adenoma in
a 15-year-old male. Clin Pediatr (Phila). 1991;30(2):109-111. PMID: 2007301
91 ANSWER: A) Prefilled syringe, ready-to-inject glucagon, 0.5
mg
Hypoglycemia is a common adverse effect in patients with diabetes
mellitus who are receiving insulin therapy. Insulin-induced
hypoglycemia emergency is treated with glucagon, which is
recommended by the American Diabetes Association and the
European Association for the Study of Diabetes for all patients with
diabetes on insulin therapy. Glucagon administration can be handled
by caregivers after proper education. Until recently, a glucagon kit
required mixing of the diluent with the active ingredient to
reconstitute 1 mg per mL concentration. The usual adult dose is 1
mg. For children weighing less than 44 lb (<20 kg), one-half the adult
dose (0.5 mg) is suggested. The patient in this vignette weighs 33 lb
(15 kg) and requires the 0.5-mg dose, not 1 mg (Answer E).
Glucagon is available in a single dose, ready-to-inject liquid
preparation that can be used for children older than 2 years. The
patient in this vignette is 3 years old and requires the 0.5-mg
glucagon injection prefilled syringe (Answer A), not 1 mg (Answer
B).
A dry powder inhaler is also available, which can be
administered via nasal mucosa at a standard dose of 3 mg (Answer
C) to any patient older than 4 years. Since this patient is 3 years old,
this option is incorrect. There is no half dose (Answer D), since the
applicator administers the standard dose of 3 mg in 1 push.

Educational Objective
Determine the dosage and best way to administer glucagon for
hypoglycemia emergencies.

Reference(s)
Carson MJ, Koch R. Clinical studies with glucagon in children. J Pediatr. 1955;47(2):161-170.
PMID: 13243215
Aman J, Wranne L. Hypoglycemia in childhood diabetes. II: Effect of subcutaneous or
intramuscular injection of different doses of glucagon. Acta Paediatr Scand.
1988;77(4):548-553. PMID: 3394508
Sherr JL, Ruedy KJ, Foster NC, et al; T1D Exchange Intranasal Glucagon Investigators.
Glucagon nasal powder: a promising alternative to intramuscular glucagon in youth
with type 1 diabetes. Diabetes Care. 2016;39(4):5555-62. PMID: 26884472
92 ANSWER: B) Start supplemental vitamin D and calcium
The child in this vignette fulfills criteria for pediatric osteoporosis in
that she has a bone mineral density less than –2.0 and has sustained
2 low-trauma fractures by age 10 years. Potential risk factors for
impaired bone health include altered mechanical stimulation due to
inadequate weight-bearing activity and nutrient deficiency. The
effect of vitamin D supplementation on bone density in children is
conflicting, but the best evidence suggests a small beneficial effect in
prepubertal girls who receive vitamin D supplementation in
addition to adequate calcium intake (Answer B). Review of this
child’s dietary intake revealed that she is getting only 2 servings of
calcium each day, which is unlikely to provide the recommended
daily allotment of 1300 mg of calcium. Therefore, supplementing
vitamin D alone (Answer A) would not correct the calcium
deficiency. While vitamin D and calcium supplementation as
primary prevention for falls and fractures is not supported by
evidence (US Preventive Services Task Force Recommendation
Statement), this strategy is still recommended for patients with
vitamin D deficiency and those with an increased risk for
osteoporosis. Of the options available, starting supplemental vitamin
D and calcium is the best treatment choice for this child.
Anticonvulsant medications have historically been associated
with an increased risk for impaired bone mineral density and
fractures; however, the evidence for this is primarily with older
anticonvulsant medications such as phenytoin and valproic acid
through mechanisms of impaired vitamin D metabolism. Newer
anticonvulsant medications such as levetiracetam may be associated
with low vitamin D, but they have not been shown to have negative
effects on bone density. In addition, because seizures increase the
potential for falls and trauma-related fractures, stopping this
medication could result in increased fracture risk (Answer E).
The patient is of pubertal age, and estrogen exposure is clearly
beneficial for improving bone density. However, the incidence of
fractures increases around the time of the pubertal growth spurt;
thus, induction of puberty (Answer D) may increase her risk for
subsequent fractures (albeit temporarily). In addition, it is normal for
girls to be prepubertal at this age, so there is no indication to
commence puberty even with a low bone density.
Bisphosphonate therapy (Answer C) has been shown to improve
bone mineral density in secondary osteoporosis; however, the
reduction in fracture risk is unknown. While this patient meets
criteria for pediatric osteoporosis, the benefit in fracture risk
reduction is likely to be minimal given that her bone mineral density
Z-score is just below the normal range. In addition, bisphosphonate
therapy is not recommended in the setting of low vitamin D due to
the increased risk for severe hypocalcemia.

Educational Objective
Identify risk factors for fractures and falls and manage pediatric
osteoporosis.

Reference(s)
Vestergaard P. Effects of antiepileptic drugs on bone health and growth potential in children
with epilepsy. Paediatr Drugs. 2015;17(2):141-150. PMID: 25567416
Weaver CM, Gordon CM, Janz KF, et al. The National Osteoporosis Foundation’s position
statement on peak bone mass development and lifestyle factors: a systematic review
and implementation recommendations [published correction appears in Osteoporos Int.
2016;27(4):1387]. Osteoporos Int. 2016;27(4):1281-1386. PMID: 26935424
Weaver CM, Alexander DD, Boushey CJ, et al. Calcium plus vitamin D supplementation
and risk of fractures: an updated meta-analysis from the National Osteoporosis
Foundation. Osteoporos Int. 2016;27(8):2643-2646. PMID: 26510847
US Preventive Services Task Force, Grossman DC, Curry SJ, et al. Vitamin D, calcium, or
combined supplementation for the primary prevention of fractures in community-
dwelling adults: US Preventive Services Task Force recommendation statement. JAMA.
2018;319(15):1592-1599. PMID: 29677309
93 ANSWER: B) Puberty
This child was found to have increased cortisol clearance, most likely
due to the onset of puberty (Answer B). Alterations in cortisol
dynamics may occur in puberty for many reasons, including changes
in 11β-hydroxysteroid dehydrogenase activity (altering
cortisol/cortisone conversion), estradiol-related increase in cortisol-
binding globulin (altering free cortisol), and GH secretion leading to
increased glomerular filtration. Collectively, this leads to reduced
circulating cortisol.
The glucocorticoid of choice for the treatment of children with
congenital adrenal hyperplasia (CAH) due to 21-hydroxylase
deficiency is hydrocortisone given orally 3 times daily. In the case of
increased cortisol clearance in puberty, an increase in dosing
frequency may be necessary rather than increasing the dosage. A
recent publication reported improved cortisol concentrations on a
regimen of 4 daily doses.
Obesity (Answer D) increases cortisol clearance due to insulin
insensitivity. In this case, the patient had been growing along the
same percentiles for weight and height. Other factors, including fatty
liver disease (increased cortisol clearance), hepatic metabolism
(particularly the cytochrome P450 system), 11β-hydroxysteroid
dehydrogenase enzyme activity, 5α-reductase system activity,
thyroid hormone levels, cortisol-binding globulin concentration, and
numerous medications can all alter cortisol metabolism. All of these
factors should be considered when assessing a patient with
suboptimally controlled CAH.
Gastritis (Answer C) could lead to poor absorption; however, the
child in this vignette was given hydrocortisone intravenously, which
would bypass any gut issues.
Drugs interactions (Answer E) should always be considered.
Erythromycin is a known inhibitor of CYP3A4 enzyme activity
(involved in the elimination of glucocorticoids). Inhibition of
CYP3A4 leads to increased glucocorticoid availability. Vitamin D
supplementation has no effect on cortisol metabolism. In contrast,
CYP3A4 inducers, such as carbamazepine, phenytoin, and
phenobarbitone, lead to increased clearance and reduced efficacy of
coadministered glucocorticoids and increased risk of adrenal crisis.
Poor regimen adherence (Answer A) is often used to explain
suboptimal control of CAH during puberty. However, physiologic
changes in puberty that can alter cortisol clearance should always be
considered before concluding that poor regimen adherence is the
cause.

Educational Objective
Explain how changes in puberty can alter cortisol dynamics and
control of congenital adrenal hyperplasia.

Reference(s)
Bryan SM, Honour JW, Hindmarsh PC. Management of altered hydrocortisone
pharmacokinetics in a boy with congenital adrenal hyperplasia using a continuous
subcutaneous hydrocortisone infusion. J Clin Endocrinol Metab. 2009;94(9):3477-3480.
PMID: 19567522
Speiser PW, Arlt W, Auchus RJ, et al. Congenital adrenal hyperplasia due to steroid 21-
hydroxylase deficiency: an Endocrine Society clinical practice guideline. J
Clin Endocrinol Metab. 2018;103(11):4043-4088. PMID: 30272171
Charmandari E, Hindmarsh PC, Johnston A, Brook CG. Congenital adrenal hyperplasia due
to 21-hydroxylase deficiency: alterations in cortisol pharmacokinetics at puberty. J Clin
Endocrinol Metab. 2001;86(6):2701-2708. PMID: 11397874
Melin J, Parra-Guillen ZP, Michelet R, et al. Pharmacokinetic/pharmacodynamic evaluation
of hydrocortisone therapy in pediatric patients with congenital adrenal hyperplasia. J
Clin Endocrinol Metab. 2020;105(3):dgaa071. PMID: 32052005
Dineen R, Stewart PM, Sherlock M. Factors impacting on the action of glucocorticoids in
patients receiving glucocorticoid therapy. Clin Endocrinol (Oxf). 2019;90(1):3-14. PMID:
30120786
94 ANSWER: E) BRAF V600E pathogenic variant
Thyroid cancer is the most prevalent endocrine malignancy and is
the third most frequent solid tumor in pediatric patients. The most
common histologic type is papillary thyroid carcinoma, accounting
for about two-thirds of nonmedullary thyroid cancer.
As noted in the vignette, the histopathologic examination
revealed classic papillary thyroid carcinoma. In both adult and
pediatric patients, the single most common genetic abnormality
found in classic papillary thyroid cancer is the BRAF V600E
pathogenic variant (Answer E), accounting for about 30% to 40% of
cases. However, the reality is a bit more complex. Although BRAF
V600E is the most common pathogenic variant in pediatric patients
with classic papillary thyroid cancer, it is observed more often in
older adolescents (typically >15 years). RET/PTC fusions seem to
occur in younger patients and have been associated equally or more
frequently with other types of papillary thyroid cancer in the
pediatric population, including the diffuse variant of papillary
thyroid carcinoma.
RET/PTC fusions (Answer B) (most common RET/PTC1) and
NTRK fusions (Answer D) (most common NTRK3/ETV6), as well as
BRAF pathogenic variants (almost exclusively BRAF V600E) all lead
to papillary thyroid cancer via activation of the mitogen-activated
protein (MAP) kinase pathway. RET/PTC and NTRK fusions are
much more common in pediatric patients with papillary thyroid
cancer than in adults, and gene fusions combined are more common
in pediatric patients than BRAF V600E mutation alone. However,
each fusion group on its own is usually not the most common
genetic alteration. Furthermore, the BRAF V600E pathogenic variant
has recently been documented to be the most common genetic
alteration in classic papillary thyroid carcinoma in children and
adolescents.
The PAX8/PPARG fusion (Answer C) is observed in follicular
variants of papillary thyroid cancer, not in classic papillary thyroid
cancer.
Pathogenic variants in the RET proto-oncogene (Answer A) cause
multiple endocrine neoplasia type 2. RET M918T is the pathogenic
variant that accounts for most multiple endocrine neoplasia type 2B
cases, while a number of other RET variants account for multiple
endocrine neoplasia type 2A, with good correlation between
genotype and aggressiveness of the medullary thyroid cancer.
However, the patient in this vignette does not have medullary
thyroid cancer. Although medullary thyroid cancer nodules can have
microcalcifications noted on ultrasonography, when medullary
thyroid cancer has spread and is associated with cervical lymph
node metastases, the calcitonin concentration is elevated. This
patient’s calcitonin level was normal. As this patient does not have
medullary thyroid cancer, a pathogenic variant in the RET gene is
incorrect.
Another type of differentiated thyroid cancer is follicular thyroid
cancer. Patients with follicular thyroid cancer do not commonly
present with cervical lymph node metastases. Cervical lymph node
metastases are common in papillary thyroid cancer (more so in
pediatric patients), as are psammoma bodies on histopathologic
examination and microcalcifications on ultrasonography. RAS
variants have been found in follicular thyroid cancer and follicular
variants of papillary thyroid cancer.

Educational Objective
Describe the most common genetic alterations found in children and
adolescents with papillary thyroid cancer vs follicular or medullary
thyroid cancer.

Reference(s)
Nikita ME, Jiang W, Cheng SM, et al. Mutational analysis in pediatric thyroid cancer and
correlations with age, ethnicity, and clinical presentation. Thyroid. 2016;26(2):227-234.
PMID: 26649796
Cordioli MIC, Moraes L, Bastos AU, et al. Fusion oncogenes are the main genetic events
found in sporadic papillary thyroid carcinomas from children. Thyroid. 2017;27(2):182-
188. PMID: 27849443
Vanden Borre P, Schrock AB, Anderson PM, et al. Pediatric, adolescent, and young adult
thyroid carcinoma harbors frequent and diverse targetable genomic alterations,
including kinase fusions. Oncologist. 2017;22(3):255-263. PMID: 28209747
Bauer AJ. Molecular genetics of thyroid cancer in children and adolescents. Endocrinol Metab
Clin N Am. 2017; 46 (2): 389-403. PMID: 28476228
Cancer Genome Atlas Research Network. Integrated genomic characterization of papillary
thyroid carcinoma. Cell. 2014;159(3):676-690. PMID: 25417114
Franco AT, Labourier E, Ablordeppey KK, et al. miRNA expression can classify pediatric
thyroid lesions and increases the diagnostic yield of mutation testing. Pediatr Blood
Cancer. 2020;67(6):e28276. doi:10.1002/pbc.28276. PMID: 32196952
95 ANSWER: E) ACAN pathogenic variant
This patient has proportional short stature as evidenced by her
normal sitting height to standing height ratio and normal difference
between arm span and height. Thus, she does not have features
typical of an FGFR3 pathogenic variant (Answer D), which results in
achondroplasia. Classic features of achondroplasia include
macrocephaly, short limbs (rhizomelic dwarfism), and significant
short stature. Although FGFR3 pathogenic variants are inherited in
an autosomal dominant fashion, most affected patients (about 80%)
do not have a family history of achondroplasia but rather have a de
novo variant.
Turner syndrome (45,X karyotype) (Answer C) is one of the
common causes of short stature in girls. However, this patient does
not have features suggestive of Turner syndrome such as a webbed
neck, lymphedema, or a congenital heart defect. Also, the history of
short stature in her father and paternal cousins suggests the likely
presence of an inherited cause of short stature other than Turner
syndrome.
Pathogenic variants in the GNAS gene (Answer B) result in a rare
condition known as Albright hereditary osteodystrophy. Albright
hereditary osteodystrophy is inherited in an autosomal dominant
manner. Patients with Albright hereditary osteodystrophy may have
a wide range of signs and symptoms, including short stature,
obesity, brachydactyly, round face, and subcutaneous ossifications.
In addition, patients who inherit the disorder from their mother also
have evidence of resistance to 1 or more hormones, especially PTH
(pseudohypoparathyroidism type 1a). However, patients who
inherit the disorder from their father have the Albright hereditary
osteodystrophy phenotype, but do not have any hormone resistance
(pseudopseudohypoparathyroidism). This patient does not have the
classic phenotype of Albright hereditary osteodystrophy such as
obesity, round face, or subcutaneous ossifications. Also, there is no
evidence of hormone resistance.
Familial short stature (Answer A) is a nonpathologic variant of
growth and thus it cannot be considered the cause of short stature in
a child with advanced bone age and significant family history of
early-onset joint disease. It is important to note that family members
with short stature, in this case the patient’s father, may have
undiagnosed pathologic conditions leading to poor growth.
Her family history of short stature and early-onset lumbar disk
herniation and osteoarthritis of the knees is highly indicative of an
ACAN pathogenic variant (Answer E), which affects aggrecan—a
major proteoglycan component of the extracellular matrix in the
growth plate and articular cartilage. Patients with ACAN pathogenic
variants may have mild to moderate dysmorphic features, including
macrocephaly, brachydactyly, flat nasal bridge, and midface
hypoplasia. Many children, but not all, with ACAN pathogenic
variants have advanced bone age (as seen in this case), even in
prepubertal years, and have a shortened pubertal growth spurt
resulting in proportional short stature.

Educational Objective
Identify ACAN pathogenic variants as the cause of aggrecanopathies
with specific phenotypic features, including short stature in affected
patients and their family members.

Reference(s)
Stavber L, Hovnik T, Kotnik P, et al. High frequency of pathogenic ACAN variants
including an intragenic deletion in selected individuals with short stature. Eur
J Endocrinol. 2020;182(3):243-253. PMID: 31841439
Gkourogianni A, Andrew M, Tyzinski L, et al. Clinical characterization of patients with
autosomal dominant short stature due to aggrecan mutations. J Clin Endocrinol Metab.
2017;102(2):460-469. PMID: 27870580
Sentchordi-Montané L, Aza-Carmona M, Benito-Sanz S, et al. Heterozygous aggrecan
variants are associated with short stature and brachydactyly: description of 16 probands
and a review of the literature. Clin Endocrinol (Oxf). 2018;88(6):820-829. PMID: 29464738
96 ANSWER: E) Orlistat
A number of weight-loss medications have been approved by the US
FDA. Such medications can be considered for individuals with a
BMI of 30 kg/m2 or greater or those with a BMI of 27 kg/m2 or
greater who have at least 1 obesity-related comorbidity (eg,
dyslipidemia). Most weight-loss medications are only approved for
patients who are 16 years old or older. Intensive lifestyle
modifications that are age-appropriate and family-based are still
first-line therapy. However, pharmacotherapy could be considered
for a patient with obesity who has not lost weight despite
participation in a formal lifestyle modification program, or in an
effort to decrease comorbidities. Pharmacotherapy should only be
prescribed by clinicians who have experience using such therapies
and are familiar with potential adverse risks. Off-label use of obesity
medications in adolescents younger than 16 years is not
recommended because of lack of FDA approval, limited efficacy and
safety data in children and adolescents, and risks for potential
adverse effects.
Orlistat (Answer E) is the only FDA-approved medication for
obesity treatment of adolescents who are between the ages of 12 to
16 years. Orlistat can reduce fat absorption by up to 30% through
inhibition of gastrointestinal lipases, and it reduces BMI by 0.7 to 1.7
kg/m2. However, it has significant gastrointestinal adverse effects,
including fatty stools, fecal incontinence, and increased defecation,
which lead to its discontinuation in many patients. A multivitamin
should be recommended for all patients taking this medication since
it can impede the absorption of fat-soluble vitamins.
Phentermine (Answer A) is only approved for short-term weight
loss in adults. Some of its potential adverse effects include
paresthesias, dizziness, insomnia, and constipation. Octreotide
(Answer B) may have a potential role in the treatment of
hypothalamic obesity, but it is not an FDA-approved obesity
medication. Metformin (Answer C) is not FDA-approved as an
obesity medication and its use results in limited reductions in BMI. It
is approved for patients 10 years or older for treatment of type 2
diabetes. Liraglutide (Answer D) is a GLP-1 analogue, which is
approved for long-term weight loss in adults. It is FDA-approved for
the treatment of pediatric patients 10 years or older with type 2
diabetes, but not for weight loss. It can cause gastrointestinal
distress, headaches, hypoglycemia, and dizziness, as well as
increased risk for possible pancreatitis.

Educational Objective
Identify orlistat as the only medication that is FDA-approved to treat
obesity among adolescents younger than 16 years and recognize that
intensive lifestyle interventions continue to be the mainstay of
treatment for weight management in children and adolescents.

Reference(s)
Styne DM, Arslanian SA, Connor EL, et al. Pediatric obesity—assessment, treatment, and
prevention: an Endocrine Society clinical practice guideline. J Clin Endocrinol Metab.
2017;102(3):709-757. PMID: 28359099
Kelly AS, Fox CK. Pharmacotherapy in the management of pediatric obesity. Curr Diab Rep.
2017;17(8):55. PMID: 28646356
97 ANSWER: A) Decreased energy consumption
Oxytocin is a nonapeptide hormone synthesized by the
magnocellular neurons in the paraventricular and supraoptic nuclei
of the hypothalamus. A single axon from each magnocellular neuron
projects to the posterior pituitary gland where oxytocin is released.
Oxytocin is well known to be involved in the regulation of labor,
where it stimulates ripening of the cervix and facilitates uterine
contraction and clotting post partum. It also stimulates the let-down
reflex in lactation. More recently, knowledge of the role of oxytocin
in regulation of other physiologic processes has been expanding.
Oxytocin has been implicated in psychological and social behaviors
such as cognitive empathy, trust, attachment, and anxiolysis.
Knockout mice deficient in oxytocin or the oxytocin receptor are
obese and have glucose intolerance. Oxytocin administration in
these animals reduces food intake (decreases energy consumption
[Answer A]), increases energy expenditure [Answer D], and
improves glucose homeostasis. However, the increase in energy
expenditure seen with oxytocin administration in animals has not
been confirmed in humans.
Obese animals are more sensitive than lean animals to the effects
of oxytocin administration. In humans, obesity increases plasma
leptin levels and promotes leptin resistance. Conversely, obesity
decreases plasma oxytocin levels and increases oxytocin receptor
expression in adipose tissue and the central nervous system.
Exogenous administration of oxytocin decreases food intake and fat
mass in animals and humans with severe obesity. Since oxytocin
decreases food intake and fat mass in leptin-resistant animal models,
it may improve (decrease) leptin resistance (thus, Answer B is
incorrect). In addition, oxytocin promotes pancreatic β-cell response
to glucose in obese states and improves insulin secretion (thus,
Answer E is incorrect). While it may improve fatty liver (Answer C),
this would not be the most likely mechanism for weight loss.
In a case report of an 8-year-old boy with craniopharyngioma and
evidence of damage to the region of the paraventricular nuclei and
supraoptic nuclei, treatment with intranasal oxytocin resulted in a
decrease in hunger and hyperphagia. However, the patient
continued to have impulsive hedonic food-seeking behaviors. It was
postulated that these continued behaviors may have been due to
damage to other neuronal brain circuitry involved in the brain
reward system. The precise role of oxytocin in the regulation of food
intake continues to evolve. Studies evaluating the role of intranasal
oxytocin as a therapy for obesity, including in Prader-Willi
syndrome, are ongoing.

Educational Objective
Describe the physiologic and pharmacologic effects of oxytocin.

Reference(s)
Hsu EA, Miller JL, Perez FA, Roth CL. Oxytocin and naltrexone successfully treat
hypothalamic obesity in a boy post-craniopharyngioma resection. J Clin
Endocrinol Metab. 2018;103(2):370-375. PMID: 29220529
Olszewski PK, Klockars A, Levine AS. Oxytocin and potential benefits for obesity
treatment. Curr Opin Endocrinol Diabetes Obes. 2017;24(5):320-325. PMID: 28590323
Maejima Y, Yokota S, Nishimori K, Shimomura K. The anorexigenic neural pathways of
oxytocin and their clinical implication. Neuroendocrinology. 2018;107(1):91-104. PMID:
29660735
Bhargava R, Daughters KL, Rees A. Oxytocin therapy in hypopituitarism: challenges and
opportunities. Clin Endocrinol (Oxf). 2019;90(2):257-264. PMID: 30506703
Spetter MS, Hallschmid M. Current findings on the role of oxytocin in the regulation of food
intake. Physiol Behav. 2017;176:31-39. PMID: 28284882
98 ANSWER: D) FNA of the thyroid nodule
This adolescent patient with Graves disease has not achieved
remission of hyperthyroidism and has evidence of active disease
(positive thyroid-stimulating immunoglobulin) after more than 2
years of treatment with antithyroid medication. Therefore, it is
reasonable to consider all options for ongoing therapy, including
continuing antithyroid medication or pursuing definitive therapy
with radioactive iodine (131I) or thyroidectomy. Management should
be guided by an informed discussion with the patient of the risks
and benefits of each approach. This older adolescent desires
definitive therapy and has no contraindication to either radioactive
iodine (Answer B) or thyroidectomy (Answer E), so proceeding with
either approach would have been reasonable, depending on patient
preference, in the absence of a thyroid nodule.
In this case, however, an abnormality on thyroid examination led
to discovery of a thyroid nodule. Thyroid enlargement in patients
with Graves disease is usually symmetric: significant thyroid
asymmetry or palpation of a discrete nodule should prompt further
evaluation with thyroid ultrasonography. Thyroid scintigraphy
(Answer C) is recommended for the initial evaluation of thyroid
nodules in patients with a low TSH concentration to identify an
autonomous nodule. Scintigraphy would not be useful in this patient
with a normal TSH value; even if methimazole were withdrawn to
achieve a low TSH level, scintigraphy generally is not effective for
identifying autonomous nodules in patients with active Graves
disease due to the background of diffuse thyroid gland autonomy.
The prevalence of thyroid cancer in children with Graves disease
may be as high as 1.8%, and a diagnosis of thyroid cancer would
affect management decisions about definitive therapy. Therefore,
thyroid nodules with concerning features should be evaluated by
ultrasound-guided FNA (Answer D) before any decision about
definitive therapy for Graves disease. Thyroid ultrasonography
should also include evaluation of cervical lymph nodes for potential
evidence of metastatic thyroid carcinoma that may alter
management. The sonographic features of this thyroid nodule (solid,
hypoechoic, calcifications) are concerning for malignancy; therefore,
waiting to reevaluate the nodule in 3 months (Answer A) is not
appropriate.

Educational Objective
Evaluate thyroid nodules in a patient with Graves disease and
include assessment for thyroid nodules in the evaluation for
definitive treatment of Graves disease.

Reference(s)
Ross DS, Burch HB, Cooper DS, et al. 2016 American Thyroid Association guidelines for
diagnosis and management of hyperthyroidism and other causes of thyrotoxicosis.
Thyroid. 2016;26(10):1343-1421. PMID: 27521067
Francis GL, Waguespack SG, Bauer AJ, et al; American Thyroid Association Guidelines Task
Force. Management guidelines for children with thyroid nodules and differentiated
thyroid cancer. Thyroid. 2015;25(7):716-759. PMID: 25900731
MacFarland SP, Bauer AJ, Adzick NS, et al. Disease burden and outcome in children and
young adults with concurrent Graves disease and differentiated thyroid carcinoma. J
Clin Endocrinol Metab. 2018;103(8):2918-2925. PMID: 29788090
99 ANSWER: B) Start a GnRH analogue to prevent further
pubertal changes
Endocrine Society clinical practice guidelines updated in 2017 made
a number of recommendations regarding the treatment of gender
dysphoric/gender incongruent adolescents, which included the
following:

Only mental health providers who meet specific criteria


should diagnose gender dysphoria/gender incongruence.
This includes knowledge of criteria for puberty-blocking
and gender-affirming hormone treatment in adolescents.
Decisions regarding the social transition of prepubertal
youths with gender dysphoria/gender incongruence
should be made with the assistance of a mental health
professional or another experienced professional.
Adolescents who meet diagnostic criteria for gender
dysphoria/gender incongruence, fulfill criteria for
treatment, and are requesting treatment should initially
undergo treatment to suppress pubertal development.
Clinicians should begin pubertal hormone suppression
after signs of puberty develop. Where indicated, GnRH
analogues should be used to suppress pubertal hormones.
Initiation of sex hormone treatment (partly irreversible) is
recommended after a multidisciplinary team of medical
and mental health providers has confirmed the persistence
of gender dysphoria/gender incongruence and sufficient
mental capacity exists to give informed consent, which
most adolescents have by age 16 years.

The patient in this vignette has been followed by a qualified mental


health professional, has met criteria for gender dysphoria, is in
puberty, and has requested treatment to suppress male development.
According to the guidelines, she meets criteria to start treatment
with a GnRH analogue (Answer B) to prevent further pubertal
changes.
Early puberty is an ideal time to start puberty blockers in children
with gender dysphoria to prevent or delay the development of
undesired secondary sexual characteristics. Pubertal suppression is
reversible and enables full pubertal development in the natal gender
after cessation of treatment if desired. For those who ultimately
choose to proceed with irreversible treatments, early suppression of
puberty can help the individual avoid the need for future surgeries
and is associated with better physical outcomes. It has also been
shown that early access to pubertal suppression is linked to better
psychological outcomes and lower suicide risk for transgender
patients.
Medroxyprogesterone acetate (Answer C) can be used as an
alternative to GnRH analogues to suppress testosterone production.
However, this medication is not as effective as GnRH analogues at
lowering endogenous androgen production and is associated with
adverse effects, including fatigue, weight gain, and increased risk of
blood clots.
Starting low-dosage estradiol (Answer A) is not indicated at this
age. Guidelines from the Endocrine Society and the World
Professional Association for Transgender Health recommend against
gender-affirming hormone treatment until approximately 16 years of
age. Aside from lack of data studying the effects of this treatment in
a younger age group, guidelines require that before starting gender-
affirming hormone treatment, an individual must have sufficient
mental capacity and maturity to understand the consequences of
partly irreversible treatment (which include potential loss of
fertility).
Treatment to block pubertal progression is considered reversible
and not likely to affect future fertility. Current guidelines do not
explicitly recommend counseling on fertility (Answer D) until an
individual is ready to consider gender-affirming hormone treatment.
This patient’s gender dysphoria may be transient. However,
allowing puberty to progress over time (Answer E) may lead to
detrimental outcomes for the patient in terms of masculinization,
which can increase complexity of gender-affirming treatment and
confer a greater risk of psychological harm.

Educational Objective
Provide appropriate guidance for management of gender dysphoria
in early puberty.

Reference(s)
Hembree WC, Cohen-Kettenis PT, Gooren L, et al. Endocrine treatment of gender-
dysphoric/gender-incongruent persons: an Endocrine Society clinical practice
guideline. J Clin Endocrinol Metab. 2017;102(11):3869-3903. PMID: 28945902
100 ANSWER: B) L-asparaginase–induced hypoglycemia
This patient has nonketotic hypoglycemia with detectable insulin
levels during a hypoglycemic episode. The suppressed β-
hydroxybutyrate and free fatty acid concentrations confirm the
relative hyperinsulinemic state. He has no history of hypoglycemia
and did not experience any blood glucose abnormality until day 18
of induction chemotherapy. The repeated severe hypoglycemia
experienced by the patient in this vignette is not a normal response
to chemotherapy (Answer E).
The patient is on prednisolone therapy, most likely with a
supraphysiologic dosage on day 18 of therapy. With the high steroid
dosages used in chemotherapy induction, patients are expected to
develop insulin resistance and even hyperglycemia. Hypoglycemia
secondary to adrenal insufficiency (Answer A) is a ketotic form of
hypoglycemia, which is not the case in this vignette.
Intrathecal injections can cause pituitary dysfunction and disrupt
GH secretion; however, GH deficiency–related hypoglycemia
(Answer D) is also a ketotic form of hypoglycemia and is not
consistent with the patient’s presentation or laboratory values.
Hyperemesis induced by chemotherapy can certainly impact
nutritional intake (Answer C) and influence glucose levels, but this
is not the case in this vignette. Ketotic hypoglycemia is also expected
if there is inadequate nutritional intake with suppressed insulin
levels.
For more than 30 years, the enzyme L-asparaginase has been
commonly used to treat childhood acute lymphoblastic leukemia.
Two preparations of L-asparaginase are approved by the US FDA:
Escherichia coli L-asparaginase (native L-asparaginase) and
pegaspargase (a polyethylene glycol-conjugated form of the E coli L-
asparaginase). Many adverse effects of L-asparaginase have been
documented over the years, such as coagulopathy, acute pancreatitis,
allergic reaction, hyperlipidemia, hyperammonemia, hepatotoxicity,
and hyperglycemia. The case in this vignette is an unusual
presentation of hypoglycemia associated with pegylated L-
asparaginase-induced hyperinsulinism (Answer B). There are
increasing case reports of this phenomena that are directly associated
with the timing of L-asparaginase use. The mechanism of elevated
insulin level is not well understood. The episodes can last weeks
after use, but they resolve with time.

Educational Objective
Determine the etiology of hypoglycemia in a child undergoing
induction chemotherapy for acute lymphoblastic leukemia.

Reference(s)
Tanaka R, Osumi T, Miharu M, et al. Hypoglycemia associated with L-asparaginase in acute
lymphoblastic leukemia treatment: a case report. Exp Hematology Oncol. 2012;1(1):8.
PMID: 23211036
Panigrahi M, Swain TR, Jena RK, Panigrahi A. L-asparaginase-induced abnormality in
plasma glucose level in patients of acute lymphoblastic leukemia admitted to a tertiary
care hospital of Odisha. Indian J Pharmacol. 2016;48(5):595-598. PMID: 27721550
Question-Mapping Index
PEDIATRIC ENDOCRINE SELF-
ASSESSMENT PROGRAM 2021-2022
Part III
This question-mapping index groups question topics according to
the 7 umbrella sections of Pediatric ESAP (Adrenal, Bone,
Carbohydrate and Lipid Metabolism/Obesity, Growth, Pituitary,
Reproductive System, and Thyroid). Relevant question numbers
follow each topic.

ADRENAL
Adrenal adenoma: 21
Adrenal insufficiency: 1, 37, 82
Atypical genitalia: 48
Catecholamine-secreting paraganglioma: 10
Congenital adrenal hyperplasia: 48, 93
Cortisol clearance: 93
Familial glucocorticoid deficiency: 1
Glucocorticoid-related adrenal suppression: 57
21-Hydroxylase deficiency: 48
Hypokalemia: 21
Pheochromocytoma: 10, 75
Premature adrenarche: 63
Primary aldosteronism: 21
SGPL1 gene: 82
VHL gene: 75
von Hippel–Lindau syndrome: 75

BONE
Adrenal insufficiency: 79
AIRE gene: 79
Alkaline phosphatase: 72
Autoimmune polyglandular syndrome type 1: 79
CASR gene: 31, 61
Craniosynostosis: 2
Familial hypocalciuric hypercalcemia: 61
Fracture: 50, 92
Glucocorticoid therapy: 70
Hypercalcemia: 79
Hyperparathyroidism: 31, 61
Hypophosphatasia: 72
Magnesium deficiency: 15
Osteoporosis: 50, 70, 92
Parenteral nutrition: 15
Rachitic changes: 72
SPINT2 gene: 15
99Tc sestamibi scan: 31
Trichohepatoenteric syndrome: 15
Vertebral fracture analysis: 70
Vitamin D deficiency: 42
X-linked hypophosphatemic rickets: 2

CARBOHYDRATE AND LIPID METABOLISM/OBESITY


Acute lymphoblastic leukemia: 100
Apolipoprotein B: 16
Bariatric surgery: 39
Bone fracture: 56
Cardiovascular disease: 46
Craniopharyngioma: 60
Diabetic ketoacidosis: 91
Exercise and insulin therapy: 19
Glucocorticoid therapy: 25
Glucagon: 91
Glycogen debrancher deficiency: 7
Glycogen storage disease: 7
Hepatosplenomegaly: 7
Hypercholesterolemia: 28, 46
Hypoglycemia: 34, 91, 100
Induction chemotherapy: 100
Insulin therapy: 3
Insulinoma: 34
L-asparaginase–induced hypoglycemia: 100
Leptin: 60
Liraglutide: 43
Maternally inherited diabetes and deafness: 84
MC4R gene: 52
Metabolic syndrome: 16
MT-TL1 gene: 84
Myopathy: 28
Neurosensory deafness: 84
Nonalcoholic fatty liver disease: 69
Obesity: 39, 52, 60, 69, 80, 96
Obesity, hypothalamic: 60
Orlistat: 96
Polycystic ovary syndrome: 16
Prader-Willi syndrome: 80
Retinopathy: 76
Statin therapy: 28
Type 1 diabetes mellitus: 3, 19, 25, 56, 76
Type 2 diabetes mellitus: 43
Weight-loss medications: 96

GROWTH
ACAN gene: 95
Achondroplasia: 26, 47
Acromesomelic dysplasia Maroteaux type: 38
Aggrecanopathy: 95
Albright hereditary osteodystrophy: 20
Chronic myeloid leukemia: 22
C-type natriuretic peptide analogue: 47
Diencephalic syndrome: 65
Failure to thrive: 65
FGFR3 gene: 26, 47
Glycogen storage disease: 12
Growth hormone deficiency: 6, 30, 55, 86
Growth hormone therapy: 55, 68, 74, 86, 88
11β-Hydroxysteroid dehydrogenase 1: 55
Hypoglycemia: 12
Hypopituitarism: 6
IGF1R gene: 77
IGF-1 resistance: 77
Lumbar disk herniation: 95
Medulloblastoma: 6
NPR2 gene: 38
Obesity: 74
Osteoarthritis: 95
Oxandrolone: 68
PAPPA2 gene: 8
Prader-Willi syndrome: 74
Precocious puberty: 30
Prenatal growth restriction: 83
Prenatal screening: 58
Pseudohypoparathyroidism type 1A: 20
Radiation therapy: 30
Rickets, vitamin D–dependent type 1A: 35
Short stature: 6, 8, 12, 20, 22, 26, 35, 38, 47, 55, 58, 68, 77, 83, 86, 88, 95
Silver-Russell syndrome: 83
Skeletal dysplasia: 26
Small-for-gestational-age: 88
Turner syndrome: 58, 68
Tyrosine kinase inhibitors: 22

PITUITARY
Adipsia: 24
Craniopharyngioma: 24, 97
Galactorrhea: 11
Growth delay: 32
Growth hormone/prolactin-secreting pituitary adenoma: 11
Growth hormone resistance: 32
Growth hormone–stimulation testing: 81
Holoprosencephaly: 53
Hyponatremia: 67
Hypopituitarism: 97
Hypothalamic obesity: 97
Klinefelter syndrome: 44
Oxytocin: 97
Precocious puberty: 14
Prolactinoma: 11
Short stature: 14, 81
Somatostatin analogues: 11
Syndrome of inappropriate antidiuretic hormone secretion: 67
TSH-secreting pituitary adenoma: 90
Undernutrition: 32
Van Wyk-Grumbach syndrome: 14

REPRODUCTIVE SYSTEM
Amenorrhea, primary: 40
Amenorrhea, secondary: 18, 64
Androgen insensitivity syndrome: 27
Antipsychotic agents: 18
Dopamine agonist therapy: 64
Galactorrhea: 5
Gelastic seizures: 87
Gender dysphoria: 99
GnRH analogue: 99
Gynecomastia: 5, 51
Histrelin implant: 33
Hypergonadotropic hypogonadism: 40, 73
Hyperprolactinemia: 18, 64
Hypospadias: 27
Hypothalamic hamartoma: 87
Kallmann syndrome: 73
Precocious puberty: 87
Premature ovarian insufficiency: 40
Prolactinoma: 64
Pubertal delay: 33, 73
Short stature: 33
Silver-Russell syndrome: 33

THYROID
BRAF gene: 94
Consumptive hypothyroidism: 59, 66
DICER1 syndrome: 54
Down syndrome: 62
Goiter: 36
Graves disease: 29, 36, 49, 62, 98
Exogenous intoxication: 41
Hashimoto thyroiditis: 29
Hearing impairment: 89
Heart failure: 59
Hyperthyroidism: 29, 36, 41, 49, 62, 98
Hypothyroidism: 4, 9, 23, 29, 59, 66, 78, 89
Liver hemangiomas: 59, 66
Macro-TSH: 9
Maternal iodine intake: 78
Medullary thyroid cancer: 13
Multiple endocrine neoplasia type 2A: 13
Newborn screening: 9, 23
Papillary thyroid cancer: 4, 45, 71, 94
RET gene: 13
Thyroid nodules: 17, 54, 94, 98
Thyrotoxicosis: 36
Ultrasonography: 17, 54, 94, 98

You might also like